Вы находитесь на странице: 1из 370

Анна Бураго

Дневник математического кружка:


первый год занятий

Перевод с английского А. В. Абакумова

Москва
Издательство МЦНМО
2017
УДК 51(07)
ББК 22.1
Б91

Бураго А. Г.
Б91 Дневник математического кружка: первый год занятий /
Перевод с английского А. В. Абакумова. — М.: МЦНМО, 2017. —
368 с.
ISBN 978-5-4439-1096-3
Книга содержит весь необходимый материал для проведения математи­
ческого кружка в 5—7 классах в течение всего учебного года. Приводятся
подробно изложенные темы для обсуждения в классе, наборы задач с ре­
шениями, математические игры и конкурсы. Автор — преподаватель мате­
матических кружков с многолетним стажем — делится профессиональными
навыками ведения кружка. Читатель найдет в книге советы, как организо­
вать занятие, преподнести материал и избежать типичных ошибок.
Книга адресована учителям и руководителям математических кружков.
Также она будет интересна школьникам, увлекающимся математикой, и их
родителям.
ББК 22.1
This work was originally published in English by the American Mathematical
Society under the title Mathematical Circle Diaries, Year1.Complete Curriculum
for Grades 5 to 7 by Anna Burago.

Учебно-методическое издание
Анна Геннадьевна Бураго
ДНЕВНИК МАТЕМАТИЧЕСКОГО КРУЖКА: ПЕРВЫЙ год ЗАНЯТИЙ

Художники Е. Грабарь, С. Васильева


Подписано в печать 21.11.2016 г. Формат 60 х 90 1/16. Бумага офсетная.
Печать офсетная. Печ. л. 23. Тираж 2000 экз. Заказ № ВЗК-05405-16.
Издательство Московского центра
непрерывного математического образования.
119002, Москва, Большой Власьевский пер., д. 11. Тел. (499) 241-08-04.
Отпечатано в АО «Первая Образцовая типография»,
филиал «Дом печати — ВЯТКА»
в полном соответствии с качеством предоставленных оригиналов.
610033, г. Киров, ул. Московская, 122. Факс (8332) 53-53-80, 62-10-36.
http://www.gipp.kirov.ru e-mail: order@gipp.kirov.ru
Книги издательства МЦНМО можно приобрести в магазине «Математическая книга»,
Москва, Большой Власьевский пер., д. 11. Тел. (495) 745-80-31.
E-mail: biblio@mccme.ru

© Бураго А. Г., 2013, 2017.


ISBN 978-5-4439-1096-3 © МЦНМО, 2017.
Содержание
Предисловие 6
Благодарности 13

Раздел 1. Планы занятий


Введение 17
Занятие 1. Как решить задачу 19
Занятие 2. Рыцари и лжецы 31
Занятие 3. Как превратить ложь в правду 39
Занятие 4. Математический аукцион I 46
Занятие 5. Текстовые задачи и здравый смысл 52
Занятие 6. Ещё текстовые задачи 59
Занятие 7. Чётные и нечётные числа I. Волшебные бумажные стакан­
чики 65
Занятие 8. Чётные и нечётные числа II. Определения и свойства . . . 72
Занятие 9. Математический хоккей I 82
Занятие 10. Чётные и нечётные числа III. Чередование 88
Занятие 11. Взвешивания и фальшивые монеты 96
Занятие 12. Математическая олимпиада I 103
Занятие 13. Знакомство с кубом 109
Занятие 14. Сечения 118
Занятие 15. Математический аукцион II 126
Занятие 16. Комбинаторика I 131
Занятие 17. Комбинаторика II 143
Занятие 18. Математический хоккей II 153
Занятие 19. Числовые ребусы I. Потерянные цифры 158
Занятие 20. Числовые ребусы II. Зашифрованные примеры 164
Занятие 21. Математическая олимпиада II 169
Занятие 22. Делимость I. Определение и свойства 174
Занятие 23. Делимость II. Простые числа и разложение на простые мно­
жители 184
Занятие 24. Математический аукцион III 193
Занятие 25. Делимость III. Признаки делимости 196
Занятие 26. Делимость IV. Взаимно простые числа 204
Занятие 27. Математические игры со стратегией I 209
4 Содержание

Занятие 28. Математические игры со стратегией II 218


Занятие 29. Математическая олимпиада III 224

Раздел 2. Математические турниры


Математические соревнования 231
Математический аукцион 232
Математический хоккей 238
Математические олимпиады 241
Короткие увлекательные игры 246

Раздел 3. Еще несколько советов преподавателю


Как стать первоклассным преподавателем математического кружка . 251
Будни математического кружка 254
Где найти ответы на вопросы 261

Раздел 4. Решения
Занятие 1. Как решить задачу 265
Занятие 2. Рыцари и лжецы 268
Занятие 3. Как превратить ложь в правду 271
Занятие 4. Математический аукцион I 274
Занятие 5. Текстовые задачи и здравый смысл 275
Занятие 6. Ещё текстовые задачи 278
Занятие 7. Чётные и нечётные числа I. Волшебные бумажные стакан­
чики 282
Занятие 8. Чётные и нечётные числа II. Определения и свойства . . . 285
Занятие 9. Математический хоккей I 288
Занятие 10. Чётные и нечётные числа III. Чередование 291
Занятие 11. Взвешивания и фальшивые монеты 294
Занятие 12. Математическая олимпиада I 298
Занятие 13. Знакомство с кубом 303
Занятие 14. Сечения 306
Занятие 15. Математический аукцион II 309
Занятие 16. Комбинаторика I 312
Занятие 17. Комбинаторика II 317
Занятие 18. Математический хоккей I I 320
Занятие 19. Числовые ребусы I. Потерянные цифры 323
Занятие 20. Числовые ребусы II. Зашифрованные примеры 326
Содержание 5

Занятие 21. Математическая олимпиада II 329


Занятие 22. Делимость I. Определение и свойства 335
Занятие 23. Делимость II. Простые числа и разложение на простые мно­
жители 339
Занятие 24. Математический аукцион III 343
Занятие 25. Делимость III. Признаки делимости 345
Занятие 26. Делимость IV. Взаимно простые числа 349
Занятие 27. Математические игры со стратегией I 352
Занятие 28. Математические игры со стратегией II 356
Занятие 29. Математическая олимпиада III 359
Библиография 367
Предисловие
Что такое математический кружок?
Когда о математике говорят в повседневной жизни, чаще всего
имеется в виду арифметика. Подобным образом и родители, оцени­
вая успехи своих детей в математике, часто исходят из того, насколь­
ко хорошо они справляются с решением типичных задач, встречаю­
щихся на экзаменах. Но в математических кружках занимаются со­
вершенно другими вещами.
В математическом кружке не учат считать и не готовят к те­
стам. Там занимаются той математикой, которая развивает логи­
ческое мышление, творческие способности, умение анализировать
и решать сложные задачи. Занятия в математическом кружке дают
ребёнку гораздо больше, чем просто набор новых математических
знаний. Кружок помогает детям обнаружить и развить свои способ­
ности; он вселяет в них интерес к математике и смежным с ней
наукам, который сохраняется на всю жизнь.
Интересные и увлекательные темы, которыми занимаются в ма­
тематическом кружке, обычно выходят за рамки школьной про­
граммы. Немалая часть занятия посвящается решению нестандарт­
ных задач. И задачи, и темы занятий подбираются так, чтобы сти­
мулировать творческие способности и развивать математическое
мышление. Нередко теория приходит к ученикам естественным пу­
тём, через задачи, которые они решают. Важную часть программы
составляют математические игры, соревнования, олимпиады.
И ещё математический кружок — это такое замечательное ме­
сто, где дети, увлекающиеся математикой, могут познакомиться со
сверстниками со сходным складом ума. Кружок становится группой
по интересам, где его участники находят себе друзей.
Кружки предназначены для любознательных детей, которые ин­
тересуются математикой и точными науками, получают удоволь­
ствие от решения трудных задач и готовы изучать математику за
пределами школьной программы. Весьма вероятно, что большая
часть участников математического кружка не станут профессио­
нальными математиками, а проявят себя в совершенно других об­
ластях. Но математика останется для них добрым помощником как
в школьные и университетские годы, так и в дальнейшей жизни.
Математическая культура, способности к решению задач и навыки
Предисловие 7

анализа, которые развиваются в математических кружках, послу­


жат надёжной опорой во многих профессиях

Об этой книге
Эта книга была изначально написана на английском языке, для
того чтобы помочь зарубежным читателям познакомиться с культу­
рой математических кружков.
Долгие годы такие кружки существовали только в России и в стра­
нах Восточной Европы. Но в последние годы внеклассные формы
обучения математике, в том числе кружки, становятся всё более по­
пулярными во многих странах мира.
Инициатива исходит и от родителей, и от учебных заведений.
Многие родители видят в таких занятиях возможность развить у де­
тей творческое мышление, повысить интерес к предмету и вооб­
ще к точным наукам. Другие стремятся ускорить развитие матема­
тических способностей, хотят, чтобы дети участвовали в матема­
тических соревнованиях и олимпиадах. Помимо этого, всё больше
университетов рассматривают устройство математических кружков
как свой вклад в образование школьников.
Таким образом, среди родителей и учителей математики в са­
мых разных концах земного шара растёт интерес к математическим
кружкам и к методике преподавания в них.
К сожалению, самообразование в этой области может занять
немало времени: количество литературы и других ресурсов пока
ещё не очень велико. Обычно будущему учителю приходится вни­
мательно изучать несколько книг и много придумывать самому, для
того чтобы сформировать преподавательский подход и разработать
программу для кружка.
Данная книга призвана ускорить этот процесс. В ней есть всё,
что необходимо для ведения кружка в течение целого года. Книга
разделена на 29 занятий; приводятся подробно изложенные темы
для обсуждения в классе, наборы задач, типичные детские ошибки,
решения, математические игры и конкурсы. Ещё одна цель книги —
поделиться профессиональными навыками ведения кружка. Чита­
тель найдёт в ней советы о том, как организовать занятия, препод­
нести материал и избежать типичных ошибок.
Этой книгой можно пользоваться по-разному:
• взять её за основу при составлении программы кружка;
8 Предисловие
• по мере необходимости брать из неё теоретический материал
и задачи по темам;
• пользоваться ей как сборником задач, конкурсов и игр.
В течение последних десяти лет автор этой книги активно участ­
вовала в проведении математических кружков в США. Эта книга
основана на её опыте и на материалах этих кружков.
Книга рассчитана на учеников 5—7 классов. Школьники этого
возраста необыкновенно восприимчивы к обучению. Они любо­
знательны, легко впитывают всё новое, искренне гордятся неожи­
данным открытием или победой над трудной задачей. Показывать
юным ученикам красоту этой науки, наблюдать за их ростом и раз­
витием чрезвычайно увлекательно. А умение учиться и думать, при­
обретённое детьми в этом возрасте, сохранится на всю жизнь и ока­
жет неоценимую помощь в последующем образовании.

Об авторе
Меня зовут Анна Бураго. Я выросла в Санкт-Петербурге — од­
ном из центров математической культуры. В пятом классе я впер­
вые пришла в математический кружок. С тех пор математика стала
увлечением всей моей жизни. Математический кружок —одно из
самых приятных воспоминаний моих школьных лет. Я приобрела
не только знания, но и новых друзей. Замечательные преподаватели
и в школе, и в кружке всегда вызывали у меня уважение и вос­
хищение. Окончив физико-математическую школу, я продолжила
изучение математики в Санкт-Петербургском государственном уни­
верситете. На всём протяжении учёбы в университете я сама с увле­
чением вела математический кружок.
После того как мы с мужем переехали в США, я училась в аспи­
рантуре по специальности «математическая физика», а потом мно­
го лет занималась разработкой компьютерных программ. (Кстати,
опыт решения нестандартных задач очень помогает при прохожде­
нии собеседований на работу.) Идея математического кружка в пер­
вый раз возникла в нашей семье, когда наш старший сын перешёл
в пятый класс. Мы стали искать для него хорошую программу по вне­
школьной математике, но оказалось, что в наших краях (мы живём
в Сиэтле) были только центры подготовки к контрольным работам.
Мой муж Андрей — выпускник математической школы и боль­
шой любитель внешкольной математики. Немного подумав, мы ре-
Предисловие 9

шили, что сами откроем математический кружок! Этому решению


сильно помогло то, что нам было легко найти единомышленников.
Многие из наших друзей, живших неподалёку, тоже прошли через
математические школы и кружки и хотели того же для своих детей.
Мы собрались с духом, установили в гостиной классную доску
и объявили об открытии первого математического кружка города
Сиэтла. К нам пришли дети наших друзей, друзья наших детей, де­
ти друзей наших друзей и друзья друзей наших детей. К нашему
удивлению и гордости, этот первый кружок просуществовал восемь
лет—до тех пор пока большинство участников не окончили школу
и не разъехались по университетам. Некоторые из наших выпуск­
ников теперь сами преподают в кружках.
Через три года после начала работы первого кружка был органи­
зован второй, потом третий, четвёртый, пятый... Движущая сила во
всех этих кружках—добровольцы (родители и не только), которые
тратят своё время и усилия на обучение подрастающего поколения
премудростям математической науки. Не могу не упомянуть имена
наших замечательных преподавателей. Александр Гиль —один из
руководителей нашего первого кружка и неотъемлемый участник
всех остальных. Люба и Сергей Малкины — замечательные препо­
даватели второго кружка, неизменные помощники во всех осталь­
ных и руководители многочисленных англоязычных кружков. Алек­
сандр Ващилло и Дмитрий Василевский — руководители и движу­
щая сила для нескольких последующих поколений русскоязычных
кружков.
Сейчас город Сиэтл может гордиться своими разнообразными ма­
тематическими кружкам. Есть наша система русскоязычных кружков,
для которой мы придумали весёлое имя — «Северо-Западная акаде­
мия наук» (Northwest Academy of Sciences). Также есть сеть англо­
язычных кружков Prime Factor, которую я основала, когда оконча­
тельно решила сменить профессию программиста на преподавате­
ля. Есть математические кружки при нашем университете, в кото­
рых успешно учили детей наши старшие выпускники, и есть ещё
много мест, где дети могут заниматься интересной и полезной ма­
тематикой.
Мы уверены, что наше дело далеко от завершения. Мы не ста­
ли дожидаться, пока наша младшая дочь дорастёт до пятого класса.
Вместо этого, когда ей было семь лет, мы открыли кружок для детей
её возраста. Теперь мы осваиваем науку ведения математических
10 Предисловие

кружков для младшеклассников. Впереди у нас долгий путь с откры­


тиями, приключениями и сюрпризами.

Как возникла эта книга


Если относиться к математическому кружку серьёзно, он требует
от преподавателя огромных затрат времени. Надо выбрать подхо­
дящую программу. Для каждого занятия нужно продумать форму
подачи материала, подобрать задачи и сделать многое другое.
Конечно, по мере накопления опыта всё это становится проще.
Но первый кружок, который мы вели в США, потребовал от нас мас­
сы времени и усилий, хотя все мы преподаватели со стажем и в на­
шем распоряжении было множество материалов — помимо англо­
язычной литературы по этой теме, обширная коллекция книг на рус­
ском языке.
Оказалось, однако, что большинство книг на английском язы­
ке рассчитано на более высокий уровень математической подготов­
ки, чем пятый класс. Книгами на русском языке также нужно было
пользоваться с осторожностью, так как оказалось, что не все темы
из них подходят для американских кружков. Дело в том, что школь­
ные программы по математике в США и России существенно раз­
личаются: американские школьники знают меньше своих русских
сверстников. Поэтому некоторые темы пришлось либо дополнить
изучением материала, который в России входит в школьную про­
грамму, либо вовсе исключить.
Неожиданной для нас оказалась проблема мотивации и вовле­
чённости участников. Вспоминая детство, мы все можем сказать:
математические кружки были одним из главных дел в нашей жизни.
Во многом это определялось средой — математическими олимпиа­
дами, лагерями, школами. Участники наших первых американских
кружков находились в другой ситуации. Это были способные и лю­
бознательные дети, но математический кружок для них был одним
из многих внешкольных занятий.
Поэтому часть нашей работы состояла в том, чтобы убедить де­
тей: математика —это увлекательно.
Самого по себе очарования необычных, интересных задач было
недостаточно: приходилось прибегать к специальным приёмам для
оживления занятий. Например, мы старались придумывать для за­
дач занимательные сюжеты. Мы также проводили множество раз-
Предисловие 11

влекательных математических мероприятий: конкурсы команд, иг­


ры, олимпиады.
К чести наших участников, эти усилия оправдали себя: в каж­
дом из кружков сформировалась группа детей, для которых обуче­
ние в кружке стало очень важным и интересным.
Долгая жизнь нашего первого кружка объясняется двумя при­
чинами: наш небольшой коллектив преподавателей всегда работал
вместе, и мы все видели, насколько полезен кружок для его участни­
ков. Вести следующие кружки было значительно проще благодаря
накоплению опыта и учебных материалов.
Наш опыт оказался ценным не только для нас. На фоне растуще­
го интереса к математическим кружкам Американское математи­
ческое общество (AMS) и Институт математических исследований
в Беркли (MSRI) заинтересовались нашим опытом. Эта книга на­
писана по заказу AMS и MSRI, её цель — подробно рассказать и по­
казать, как начинать математический кружок и чему учить детей
в первый год обучения.
Книга задумана и написана как ресурс, максимально облегчаю­
щий жизнь начинающего преподавателя. Она шаг за шагом прово­
дит читателя через уроки, подробно рассказывая, советуя и указы­
вая на типичные ошибки. Этот формат оказался очень удачным для
страны, в которой нет давних традиций, но есть большой интерес
к внешкольной математике.
Я очень благодарна издательству МЦНМО за возможность напе­
чатать эту книгу на русском языке. У российских читателей, конеч­
но же, гораздо больше опыта и больше возможностей для самообра­
зования в области математических кружков. Но я надеюсь, что эта
книга найдёт своих читателей, которым она поможет сориентиро­
ваться в мире математики, даст им материал для размышлений и,
наконец, облегчит жизнь и сэкономит время при подготовке и про­
ведении занятий.
Возраст участников
Эта книга предназначена для работы с учениками 5—7 классов1,
не обязательно имеющими опыт занятий в математическом круж­
ке. Некоторые темы и задачи можно также включить в программу
кружка для детей помладше или постарше.
1
Возраст пятиклассников в США (на начало учебного года) —от 10 до 11 лет.—
Прим. перев.
12 Предисловие

Представленные в книге темы интересны и доступны участни­


кам с разным уровнем подготовки. Подборки задач тоже составлены
так, чтобы и начинающим, и опытным участникам было над чем по­
работать. Некоторые задачи решаются с помощью знаний, получен­
ных в кружке, а для других достаточно логики и здравого смысла.
В каждую подборку включено несколько дополнительных задач для
групп с более высоким уровнем подготовки.
Благодарности

Начинать эту книгу, конечно, следует с длинного списка благо­


дарностей. Трудно перечислить всех тех замечательных людей, ко­
торые прямо или косвенно помогали мне в работе: одни — советом,
опытом и одобрением, другие —тем, что гуляли с моими детьми
и помогали готовить обеды.
Возглавляет этот список моя семья — муж, мама и дети. Без их
поддержки и помощи я никогда не смогла бы уделить так много
времени работе над этой книгой. Моя замечательная семья стой­
ко терпит все мои странности, увлечения и круглосуточную работу.
Я благодарна им за поощрение, за готовность подставить плечо и за
поддержку в быту и в жизни.
Особая благодарность причитается моему мужу, Андрею Бураго.
С самого начала он был тем самым человеком, который верил, что
я могу довести этот проект до конца. Пока я работала над книгой, он
совмещал должности болельщика, редактора, технического консуль­
танта, эксперта в области математики, а также громоотвода: именно
на него выливались все мои жалобы, расстройства и сомнения.
Мне очень повезло, что в моей жизни есть замечательная по­
друга Нелли Ткач — филолог, знаток русского и английского язы­
ка, и человек, активно заботящийся о благе человечества. Главу за
главой, она редактировала со мной эту книгу, при этом терпеливо
обучая меня навыкам писательского мастерства и искусству работы
с текстом. Если бы не её вежливые и настойчивые и вопросы о том,
когда же будет готова следующая глава, то книга была бы закончена
намного позже.
Издательство AMS позаботилось о том, чтобы у этой книги был
замечательный технический редактор, профессор Сильвио Леви,
MSRI. Я благодарю его за все поправки, комментарии, предложе­
ния и улучшения. Его энциклопедические знания и способность
плодотворно работать 24 часа в сутки вызывают у меня зависть
и восхищение.
Я очень признательна Юлии Бродской, моей коллеге и энтузи­
асту математических кружков, человеку, с которым хорошо и ин­
тересно общаться. Идея писать эту книгу оказалась неожиданным
результатов одной из наших бесед.
14 Благодарности

Большое спасибо всем, кто помогал мне советами и редактиро­


вал мои тексты: Коле Малкину, Ольге Радко, Любе Малкиной, Алек­
сандру Гилю, Сергею Генкину и многим другим.
Я признательна Давиду Окли, Татьяне Шубиной, Сергею Гельфанду
и совету серии «Библиотека математического кружка/MSRI». Они уви­
дели потенциал в первых главах, поддержали идею и оказывали по­
мощь и поддержку на всех этапах подготовки и публикации книги.
Последними в этом списке, но первыми по важности идут заме­
чательные люди, которые создали, поддерживают и развивают куль­
туру математических кружков. Эта книга является результатом опыта
работы многих поколений преподавателей, профессионалов и масте­
ров своего дела, которые находили замечательные темы, придумыва­
ли интересные задачи и щедро делились своим знаниями.

Благодарности к русскому изданию


Я безмерно рада, что моя книга будет напечатана в издательстве
МЦНМО. Трудно преувеличить вклад МЦНМО в поддержку и разви­
тие математической культуры: это полезные книги, интересные ме­
роприятия, многочисленные классы, обширная сетевая база задач
и математической литературы и многое другое.
Отдельная благодарность моему редактору Ольге Васильевой —
человеку высокой квалификации и замечательному мастеру своего
дела. Работая над книгой, она находила такие хитрые ошибки и опе­
чатки, которые ускользнули от взгляда автора, от редактора первого
издания, а также от самых дотошных читателей.
РАЗДЕЛ 1

Планы занятий
Введение

Эта книга — об искусстве преподавания в математическом круж­


ке. Наряду с обширными учебными материалами, мы обсуждаем
профессиональные приёмы, накопленные за долгие годы работы
в кружках.
Книга состоит из четырёх частей. Первая часть —это учебная
программа математического кружка для 5—7 классов, рассчитан­
ная на год. В этой части двадцать девять глав — по числу занятий.
В каждой главе приводится подробный конспект занятия и даётся
подборка задач для самостоятельного решения. Мы рассказываем
о том, как лучше изложить тему, каких типичных ошибок следует
избегать и какие вопросы обычно задают дети. Для тех занятий,
на которые запланированы математические развлечения (турниры,
игры, олимпиады), мы даём советы по их подготовке и проведению.
Вторая часть посвящена математическим развлечениям. В ней
подробно рассказывается о конкурсах, олимпиадах и играх, кото­
рые мы любим проводить на занятиях.
В третьей части рассматриваются принципы преподавания в круж­
ке и некоторые вопросы организации занятий.
Наконец, четвёртая часть — это подсказки, ответы и решения ко
всем задачам в книге.
Программа этой книги была опробована и отлажена на четы­
рёх математических кружках: в «Северо-Западной академии наук»,
в двух кружках центра Prime Factor и в центре Робинсон для юных
исследователей при университете штата Вашингтон. Все четыре
кружка служили источником вдохновения и одновременно испыта­
тельным полигоном. Опыт работы в каждом из кружков внёс свой
вклад в материал, включённый в книгу. Например, благодаря паре
очень способных участников из центра Робинсон в книгу попала
серия более трудных задач.
Один из этих четырёх кружков стал основой сюжета книги: имен­
но из него взята большая часть высказываний, вопросов и замеча­
ний участников. Этот кружок был очень неоднородным: в нём бы­
ло несколько «матёрых математических волков», которые посеща­
ли в своих школах математические клубы, участвовали в конкурсах
и получали призы. Остальные были новичками и пришли потому,
что хотели «попробовать что-то новое».
18 Раздел 1. Планы занятий

В группе, о которой идёт речь, занимались пятиклассники, ше­


стиклассники и один талантливый четвероклассник. Занятия про­
ходили раз в неделю и продолжались два часа. В течение года состав
кружка изменялся: кто-то из участников уходил, приходили новые.
По большей части на занятиях было около десяти детей. Посколь­
ку в тексте книги встречаются замечания и реплики участников,
приведу здесь некоторые имена: Тара, Соннет, Кейси, Сэм, Эндрю,
Эмма М., Эмма К., Кевин, Брейди, Колин, Ришав и Дэвид.

О подборках задач
К каждому занятию в этой книге прилагается подборка задач,
содержащая от пяти до восьми задач. Чтобы сделать задание более
полезным и интересным, в подборку обычно включаются задачи по
разным темам.
Задачи также различаются по уровню сложности: самые трудные
отмечены звёздочкой. Дополнительные задачи, которые преподава­
тель может давать по своему усмотрению, отделены от остальных
задач горизонтальной линией.
Ко всем задачам в книге приведены решения. Они часто со­
провождаются разбором типичных ошибок, подсказками для участ­
ников и преподавателей и рекомендуемыми способами доходчиво
объяснить задачу. Поэтому прочитать решения будет полезно даже
опытному преподавателю.
Занятие 1

Как решить задачу

Сначала несколько слов о том, как пользоваться этой книгой.


Если предстоящее занятие математического кружка — первое в ва­
шей жизни, сначала прочитайте главу «Как стать первоклассным пре­
подавателем математического кружка» (с. 251). В ней вы найдёте
принципы преподавания в кружке и профессиональные секреты, кото­
рые помогут сделать кружок действительно интересным. Затем реко­
мендую просмотреть главу «Будни математического кружка» (с. 254),
где предлагаются идеи по организации и структуре занятий.

Подготовка к занятию
Вот что нужно сделать для подготовки к первому занятию.
• Завести журнал кружка. Он предназначен для хранения учеб­
ных материалов — подборок задач и записей к занятиям, а также
для учёта решённых задач.
• Составить список участников с информацией о каждом: класс,
возраст, адреса электронной почты и контактные телефоны роди­
телей и т.п. Как собрать эти сведения? Во-первых, можно полу­
чить их от родителей по электронной почте. Другой способ — формы
интернет-регистрации. (Такую форму нетрудно создать с помощью
интернет-служб. Например, в Google Forms можно не только созда­
вать опросы, но и хранить их результаты.)
• Подготовить информационный листок для раздачи участни­
кам. На этом листке — то, что нужно знать участникам и их родите­
лям: свой контактный телефон и адрес электронной почты, время
и место проведения занятий, список необходимых принадлежно­
стей и другие важные сведения.
• Распечатать в достаточном количестве экземпляров подборку
задач для первого занятия (она приведена в конце этой главы).
• Купить несколько коробок счётных палочек или спичек. Они
пригодятся для решения одной из задач первого занятия.
20 Раздел 1. Планы занятий

План занятия
На первом занятии предстоит сделать много разных вещей. Луч­
ше записать их, чтобы ничего не пропустить.
1. Знакомство
2. Математическая разминка
3. Рассказ о математических кружках
4. Обсуждение темы занятия: «Что значит решить задачу?»
5. Решение задач
Знакомство и математическая разминка
Я представляюсь и знакомлюсь с участниками. Затем перехожу
к математической разминке. Разминка — это чаще всего несколько
несложных занимательных задач. Отличный способ занять детей,
пока подтягиваются опоздавшие.
Перед тем как дать задачи для разминки, я объясняю, что они
не очень трудные, но требуют сообразительности. Чтобы их решить,
важно мыслить нестандартно и выдвигать неожиданные идеи.
Задача для разминки 1. В семье пять братьев. У каждого брата
одна сестра. Сколько всего детей в этой семье?
Задача для разминки 2. Некто положил две монеты в два ко­
шелька так, что в одном кошельке оказалось в два раза больше мо­
нет, чем в другом. Как он это сделал?
Сначала дети предлагают ничего не класть в один кошелёк, а в дру­
гой положить две монеты. Я объясняю, что тогда в одном кошельке
будет не в два раза больше монет, чем в другом, а на две монеты
больше. Следующая идея — оставить оба кошелька пустыми (в два
раза больше нуля — это всё равно нуль). Я отвечаю, что обе монеты
должны оказаться в кошельках. Наконец, я предлагаю участникам
представить себе большие, мягкие кожаные кошельки. С помощью
этой подсказки Эндрю находит верное решение.
Для преподавателей. Ответы к задачам для разминки и решения всех за­
дач приведены в части «Ответы и решения» (с. 265).
Даже такая простая вещь, как разминка, не обходится без профес­
сиональных секретов. В главе «Будни математического кружка» (с. 254)
я рассказываю о разминке более подробно.
В первый раз я допустила ошибку: начала занятие не с разминки,
а с рассказа о математическом кружке. Пока я объясняла детям важные
организационные вопросы, подходили опоздавшие участники. В итоге
пришлось рассказывать всё сначала.
Занятие 1. Как решить задачу 21

Что такое математический кружок?


Поскольку большинство участников пришли на математический
кружок впервые, мне пришлось вкратце объяснить, чего можно ожи­
дать от наших занятий. Не так-то просто рассказать за пять минут,
что такое математический кружок. Я подготовила короткое выступ­
ление, которым более или менее довольна.
«Сегодня первое занятие нашего математического кружка. Да­
вайте поговорим о том, что же это такое. Я скажу странную вещь:
на математических кружках не изучают математику. Я имею в ви­
ду, что мы не будем заниматься той математикой, которую вы изу­
чаете в школе.
Мы будем гораздо меньше считать, а вместо этого будем ре­
шать интересные и необычные задачи. Они будут самыми раз­
нообразными: логические задачи, головоломки, задачи, требую­
щие творческого мышления и строгих доказательств. Могу обе­
щать, что вы не раз испытаете ни с чем не сравнимое ощущение:
„Нашёл!"
Время от времени мы будем проводить математические игры
и конкурсы. Другие занятия будут посвящены различным спосо­
бам решения задач и элементам математической теории. А ещё
мы будем учиться объяснять решения. Для большинства задач,
с которыми мы будем иметь дело, недостаточно дать ответ: надо
ещё обосновать его.
У этого кружка есть ещё одна особенность. Я — ваш препо­
даватель — хочу, чтобы вы как можно больше разговаривали на
занятиях: прерывали меня вопросами, выдвигали идеи, угады­
вали ответы и даже спорили со мной. Учиться будет не скучно.
Если кто-то даст неверный ответ или выдвинет странную идею —
замечательно: это часть процесса обучения. Странная на первый
взгляд идея может оказаться наилучшим шагом на пути к верно­
му решению».
На этом этапе я не касаюсь организации отдельных занятий: это
будет проясняться по мере необходимости.
Я напоминаю детям, что понадобится приносить с собой на за­
нятия: папку для хранения всех принадлежностей, тетрадь в клетку,
бумагу для черновиков, ручки, простые карандаши, пару цветных
карандашей, линейку. Калькуляторы не нужны.
Наконец, раздаю экземпляры информационного листка.
22 Раздел 1. Планы занятий

Тема занятия:
«Что значит решить задачу?»
Одной из наших основных целей на первые несколько месяцев
будет развитие у участников кружка культуры логических рассуж­
дений. Мы узнаем, что решить задачу—значит вывести ответ, а не
угадать его. А объяснить решение — значит не просто дать ответ.
Решение должно содержать чёткое логическое обоснование всех его
этапов, с формулировкой предположений и выводов.
Поэтому весь остаток занятия мы говорим о том, как решать
задачу и как объяснять решение. Работаем над задачами вместе
(всей группой). Для этого занятия подобраны разнородные зада­
чи, не связанные с конкретной темой. Их объединяет общая цель:
дать участникам попробовать задачи математического кружка «на
вкус» и одновременно показать на примерах, что значит «объяс­
нить решение». Первые задачи довольно простые: они помогают
преодолеть психологический барьер. Ближе к концу задачи услож­
няются.
Для преподавателей. 1. Давайте детям задачи одну за другой. Призывайте
участников выдвигать идеи. Подытоживайте эти идеи, а в конце обсуж­
дения каждой задачи рассказывайте полное решение.
2. Не забывайте, что у детей разный уровень математической под­
готовки. Старайтесь, чтобы каждый внёс вклад в решение задачи. Убе­
дитесь в том, что каждый участник понял все этапы решения.
3. Подробнее об особенностях преподавания в кружке говорится
в главе «Как стать первоклассным преподавателем математического
кружка» (с. 251).

Пример 1. Девочка и мальчик залезли на дерево, чтобы собрать


вишни для пирога. Девочка была старше и поэтому собрала в два ра­
за больше вишен, чем мальчик. Вместе они принесли домой 72 виш­
ни. Сколько вишен собрал каждый из детей в отдельности?
РАЗБОР ПРИМЕРА 1. Начинаем с базового понятия: уточняем, что
означает «в два раза больше» (в одний из моих групп возникла пута­
ница с понятием «на два больше»). Затем обсуждаем, как предста­
вить «в два раза больше» наглядно. В группе рождается идея: в два
раза больше — это две кучи по сравнению с одной кучей. Затем по­
является ещё один вариант: две корзины по сравнению с одной кор­
зиной.
Занятие 1. Как решить задачу 23

Мы останавливаемся на корзинах, поскольку нарисовать их про­


ще, чем кучи. Итак, я рисую на доске три корзины:

Сколько всего вишен в этих трёх корзинах? ОТВЕТ: 72.


Сколько вишен в каждой корзине? ОТВЕТ: 72:3 = 24.
Значит, мальчик собрал 24 вишни, а девочка 48.
Пример 2. Когда Женя получает пятёрку по русскому языку, она
всегда приходит домой в отличном настроении. Сегодня Женя при­
шла домой в отличном настроении. Значит ли это, что она получила
пятёрку по русскому языку?
РАЗБОР ПРИМЕРА 2. Это тоже сравнительно простая задача. Об­
суждая её, мы вводим понятия причины и следствия. Эти понятия
получат дальнейшее развитие при разборе следующей задачи и на
следующем занятии.
Решение. В условии не сказано, что пятёрка по русскому — един­
ственная возможная причина отличного настроения Жени. Могут
быть и другие причины: хорошая погода, пятёрка по математике
или что-нибудь ещё. Поэтому Женя могла получить отличную оцен­
ку по русскому, а могла и не получить.
Можно нарисовать схему причин и следствий:

Следствие: Отличное настроение

Возможные причины: Хорошая погода Пятёрка по


русскому языку

На первый взгляд, для объяснения такой простой задачи можно


обойтись без схемы. Однако эта схема послужит удобной отправной
точкой при разборе следующей задачи.
Пример 3. У капитана Кука есть попугай Полли, который на­
кануне бури всегда чихает. Полли только что чихнул. Капитан Кук
говорит: «Мой попугай чихнул, значит, завтра будет буря». Верно
ли умозаключение капитана? Обязательно ли завтра будет буря?
24 Раздел 1. Планы занятий

РАЗБОР ПРИМЕРА 3.
Эта задача кажется труднее предыдущей, хотя
основная идея та же самая. Трудность вызвана тем, что на этот раз
причина (буря) наступает позже следствия (чихание).
Сначала участники заявляют, что капитан Кук прав: завтра точ­
но будет буря.
Поэтому мы переходим к обсуждению возможных причин чи­
хания попугаев. То, что Полли сегодня чихает, может быть объяс­
нено разными причинами. Надвигающаяся буря — только одна из
них; попугай может также чихнуть, например, из-за простуды или
аллергии на пыль.
Для наглядности мы рисуем схему причин и следствий и вместе
заполняем её.

Следствие: Полли чихает

Возможные причины: Завтрашняя буря Простуда Аллергия


на пыль

Я подчёркиваю, что эта задача похожа на задачу про Женю и пя­


тёрку по русскому языку. В обеих задачах есть следствие, которое
может быть вызвано разными причинами. Если мы наблюдаем след­
ствие, мы не можем сделать вывод, что оно вызвано какой-то кон­
кретной причиной.
Итак: завтра может быть буря, но её может и не быть. Поэтому
умозаключение капитана неверно.
Пример 4. В абсолютно тёмной комнате стоит стеклянная ваза,
в которой 10 чёрных и 12 белых шаров. Какое наименьшее число
шаров надо вынуть из сосуда, чтобы можно было с уверенностью
сказать следующее:
а) среди вынутых шаров есть пара шаров одного цвета;
б) среди вынутых шаров есть пара чёрных шаров?
РАЗБОР ПРИМЕРА 4. Это отличный пример задачи, в которой нетруд­
но догадаться до правильного ответа, а вот строго обосновать его зна­
чительно сложнее.
а) Участники сразу выдают ответ: 3. Я отвечаю, что меня надо
убедить в его правильности.
Как доказать, что 3 шара —действительно «наименьшее возмож­
ное число»?
Занятие 1. Как решить задачу 25

Сначала надо показать, что если взять меньше трёх шаров, то


среди них может не оказаться пары одного цвета. Другими сло­
вами, число, меньшее чем 3, не может быть правильным ответом.
Затем нужно объяснить, что если взять 3 шара, то среди них обяза­
тельно найдётся пара одинаковых шаров.
Как доказать, что если взять меньше трёх шаров, то среди них
может не оказаться пары одного цвета? Достаточно привести при­
мер. Если мы вынули 2 шара, то, возможно, один из них чёрный,
а другой белый. В таком наборе нет пары шаров одного цвета. Сле­
довательно, меньше чем тремя шарами нам не обойтись.
Как объяснить, что среди трёх шаров обязательно найдётся пара
одинаковых? Это чуть-чуть сложнее. Предположим, что среди вынутых
шаров не оказалось ни одной одноцветной пары. Значит, среди них са­
мое большее 1 чёрный шар и самое большее 1 белый, т. е. всего самое
большее 2 шара. Но этого не может быть: ведь мы вынули 3 шара.
Можно объяснить эту часть решения по-другому, просто пере­
числив все возможные сочетания трёх шаров: 3 чёрных; 2 чёрных
и 1 белый; 1 чёрный и 2 белых; 3 белых. В каждом из этих сочетаний
есть пара шаров одного цвета.
б) Несколько человек дают правильный ответ: 14. Однако я на­
поминаю: надо убедительно объяснить, что 14 —действительно наи­
меньшее возможное число.
Сначала докажем, что если взять меньше чем 14 шаров, то сре­
ди них может не оказаться чёрной пары. Действительно, возьмём
13 шаров. Может случиться так, что из них 12 белых и 1 чёрный.
В таком наборе нет чёрной пары. Поэтому для полной уверенности
в том, что у нас есть пара чёрных шаров, меньше чем четырнадца­
тью шарами не обойтись.
Теперь докажем, что если взять 14 шаров, то среди них обязатель­
но найдётся чёрная пара. В сосуде только 12 белых шаров. Следова­
тельно, в любом наборе из 14 шаров самое большее 12 белых. Значит,
в любом наборе из 14 шаров самое меньшее 2 чёрных шара (пара).
Для преподавателей. Обратите внимание участников на то, что рассуж­
дения типа «из 14 шаров 12 будут белыми и 2 чёрными» неверны. На
самом деле среди 14 шаров не обязательно ровно 12 белых и ровно 2 чёр­
ных: возможны другие сочетания.
Верными будут доказательства, в которых употребляются выраже­
ния «самое большее 12» и «самое меньшее 2» (или «не больше 12» и «не
меньше 2»).
26 Раздел 1. Планы занятий

Для преподавателей. В этой задаче, как и во многих других, требуется


найти наименьшее (или наибольшее) число, при котором точно про­
изойдёт некоторое событие. Чтобы решить задачу такого типа, нуж­
но обосновать две вещи: необходимость и достаточность. Доказать
«необходимость» — значит показать, что правильный ответ не может
быть меньше (или больше) предложенного участником. Доказать «до­
статочность» — значит объяснить, что предложенный ответ в любом
случае гарантирует требуемый результат.
Идея «необходимости и достаточности» закрепляется в следующей
задаче и в подборке задач к этому занятию.
Пример 5. В абсолютно тёмной комнате стоит сундук с сокро­
вищами, в котором 25 рубинов, 15 алмазов и 4 сапфира. Какое наи­
меньшее число драгоценных камней надо вынуть из сундука, чтобы
можно было с уверенностью сказать следующее:
а) среди вынутых камней есть пара камней одного вида;
б) среди вынутых камней есть пара алмазов?
РАЗБОР ПРИМЕРА 5. а) ОТВЕТ: 4.
Сначала докажем, что если взять меньше четырёх камней, то
среди них может не оказаться пары. Пример: среди трёх камней
может быть один рубин, один сапфир и один алмаз. В таком наборе
нет ни одной пары. Поэтому надо взять хотя бы 4 камня.
Теперь надо доказать, что если взять 4 камня, то пара обяза­
тельно найдётся. Если среди взятых камней нет ни одной пары,
значит, мы взяли самое большее 1 рубин, самое большее 1 сапфир
и самое большее 1 алмаз. Поэтому всего мы взяли самое большее
3 камня. Это противоречит предположению, что мы взяли 4 камня.
б) ОТВЕТ: 31.
Сначала докажем, что если взять меньше 31 камня, то пары
алмазов среди них может не найтись. Пример. Возьмём 30 камней.
Возможно, среди них 25 рубинов, 4 сапфира и 1 алмаз. В таком на­
боре нет пары алмазов.
Теперь объясним, что если взять 31 камень, то пара алмазов
обязательно найдётся. Среди 31 камня самое большее 25 руби­
нов и самое большее 4 сапфира. Значит, всего рубинов и сапфиров
самое большее 29. Следовательно, среди 31 камня самое меньшее
2 алмаза.
Для преподавателей. Когда дети объясняют достаточность ответа в таких
задачах, им часто кажется удобным рассмотреть «худший случай». На­
пример, «в худшем случае мы достанем один рубин, один сапфир и один
Занятие 1. Как решить задачу 27

алмаз» или «в худшем случае у нас 12 чёрных шаров и 2 белых». Это


не вполне удовлетворительное рассуждение, поскольку «худший слу­
чае—довольно расплывчатое понятие. Наша цель — научить детей бо­
лее строгим объяснениям: «если нет пары, значит, у нас самое большее
по одному камню каждого вида» или «у нас самое большее 12 чёрных
шаров, значит, самое меньшее 2 чёрных».
Однако нужно учитывать возраст детей и то, что они пришли на
математический кружок в первый раз. К тому же им одновременно
предлагаются две новые идеи: доказательство из двух частей (необхо­
димость и достаточность) и обоснование достаточности. Для некоторых
это слишком много.
Поэтому можно порекомендовать временный компромисс: препода­
ватель подчёркнуто придерживается строгих рассуждений, когда объяс­
няет решение сам, и требует того же от самых способных участников.
А остальным детям на этом этапе вполне позволительно рассматривать
«худший случай».

Пример 6. Покажите, как разрезать показан­


ную на рисунке фигуру на две части, а затем, не на­
кладывая эти части друг на друга, составить из них
квадрат. Разрез должен проходить по линиям сетки,
но не обязательно должен быть прямой линией.
РАЗБОР ПРИМЕРА 6. Эту задачу можно решать по-
разному, в том числе методом проб и ошибок, пробуя все возмож­
ные разрезы, до тех пор пока не наткнёмся на решение. Рано или
поздно решение найдётся, но этот способ может оказаться долгим.
Попробуем применить метод проб и ошибок «с умом». Вычислим
размеры квадрата. Поскольку фигура на рисунке имеет площадь 16,
площадь квадрата тоже должна быть равна 16. Поэтому квадрат дол­
жен иметь сторону 4. Теперь, зная длину стороны квадрата, попро­
буем мысленно наложить его на исходную фигуру. Первая идея, ко­
торая приходит в голову: возможно, нижний край фигуры (длина
которого тоже равна 4) совпадает с нижней стороной квадрата. Эта
догадка приводит к верному решению. Правильный разрез приве­
дён в разделе «Решения» (с. 265).
Пример 7*. На дне рождения Робина Гуда его подруга Мэри­
ан решила узнать, сколько ему лет. Малютка Джон сказал ей, что
Робину не меньше 25 лет. Брат Тук заявил, что Робину не меньше
чем 24. Сколько лет Робину Гуду, если известно, что ровно один из
двух друзей солгал?
28 Раздел 1. Планы занятий

РАЗБОР ПРИМЕРА 7. На занятии с новичками приходится подска­


зывать все основные этапы решения. Зато к концу года, набравшись
опыта в рассуждениях про рыцарей и лжецов, дети решают такие
задачи без труда.
Решение. Мы знаем, что ровно один из друзей солгал. Значит,
либо Малютка Джон солгал, а Брат Тук сказал правду, либо Малютка
Джон сказал правду, а Брат Тук солгал.
Предположим, что Брат Тук солгал, а Малютка Джон сказал прав­
ду. Это значит, что утверждение «Робину не меньше 24 лет» ложно.
Другими словами, Робину меньше 24 лет (23 года или меньше). Од­
нако по словам Малютки Джона, который сказал правду, Робину не
меньше 25. Но Робин не может быть одновременно моложе 24 и не
моложе 25. Следовательно, наше предположение, что Брат Тук со­
лгал, неверно.
Значит, Брат Тук сказал правду, а Малютка Джон солгал. Малют­
ка Джон утверждает, что Робину не меньше 25 лет (25 или больше).
Поскольку это ложь, Робину на самом деле 24 года или меньше. По
словам Брата Тука, сказавшего правду, Робину не меньше 24 лет. Из
этих двух утверждений следует, что Робину ровно 24 года.
Наконец, наступает время решать задачи самостоятельно.

О подборках задач и домашнем задании


Я объясняю участникам, что на каждом занятии они будут полу­
чать подборку задач для самостоятельного решения. Вряд ли кто-то
успеет решить все задачи до конца занятия. Нерешённые задачи —
это домашнее задание к следующему разу.
Некоторые из этих задач довольно хитрые, и вполне возможно,
что решить какие-то из них не получится. Но попробовать всегда
стоит. Кому-то поможет обсуждение задач с другими участниками
кружка. Это интересные задачи, и родители тоже могут увлечься их
решением.
Полностью записывать текст решения не обязательно. Доста­
точно записать ровно столько, чтобы по этим записям можно было
вспомнить решение и рассказать его на занятии.
После этого вступления я раздаю задачи, при этом придерживаясь
рекомендаций из раздела «Как организовать решение задач» (с. 259).
Весь остаток занятия участники работают над задачами. Я пе­
риодически напоминаю им, что охотно отвечу на любые вопросы,
Занятие 1. Как решить задачу 29

и предлагаю рассказывать решения (подними руку; расскажи мне,


как ты решил задачу).

Подборка задач
Предварительные замечания. Головоломка с рыбкой (задача 2). Получив
в руки спички, дети сразу начнут решать головоломку и перестанут
слушать преподавателя. Поэтому спички раздаются в последнюю оче­
редь — после листков с задачами.
Задачи 7 и 8. Это «дополнительные» задачи. Они включаются в подбор­
ку, если для некоторых участников остальные задачи слишком простые.
Задача 1*. а) Бобёр распилил элек­
тропилой бревно на поленья. Он сделал
25 распилов. Сколько получилось поле­
ньев?
б) Бобёр распилил два бревна на
поленья. Всего он сделал 40 распилов.
Сколько получилось поленьев? (Приме­
чание. Бобёр никогда не пилит два брев­
на одновременно. Он не обязательно распиливает бревна на одина­
ковое количество поленьев. Поленья могут быть разной длины.)
в) Тот же вопрос, если брёвен было три, а распилов 50.
Задача 2. Переместив только три спички,
сделайте так, чтобы рыбка плыла вправо.
Задача 3. В ящике лежат 14 красных и
10 синих шаров. Ящик находится в абсолют­
но тёмной комнате. Какое наименьшее число
шаров надо вынуть из ящика, чтобы можно
было с уверенностью сказать следующее:
а) среди вынутых шаров есть три шара одного цвета;
б) среди вынутых шаров есть три синих шара?
(Обосновывая оба ответа, помните, что рассуждение должно
состоять из двух частей. Сначала докажите, что если взять мень­
шее число шаров, то нужного сочетания шаров может не полу­
читься. Затем докажите, что ваш ответ в любом случае гаран­
тирует нужное сочетание.)
Задача 4. В коробке лежат 39 носков. Среди любых двух из этих
носков есть хотя бы один синий. Хотя бы один носок в коробке крас­
ный. Сколько красных носков в коробке?
30 Раздел 1. Планы занятий

Задача 5. Четыре участницы ма­


тематических кружков из разных
городов — Москвы, Санкт-Петербур­
га, Самары и Челябинска — провели
вместе две недели в летнем матема­
тическом лагере. Их имена: Маша,
Даша, Ксюша и Наташа. Вот что про
них удалось узнать.
1. Маша и девочка из Челябинска
жили в одной комнате.
2. Маша никогда не была ни в Москве, ни в Самаре.
3. Однажды в лагере провели футбольный матч. Ксюша играла
в одной команде с девочкой из Москвы. Девочка из Челябинска бы­
ла в другой команде.
4. Наташа часто играла в шахматы с девочкой из Москвы.
Определите, в каком городе живёт каждая девочка.
Задача 6. Самолёт вылетает из Мурманска и летит точно на се­
вер. Пролетев 300 километров, он поворачивает и летит ещё 300 ки­
лометров точно на запад. Затем снова поворачивает и летит 300 ки­
лометров точно на юг и ещё 300 километров точно на восток. После
этого самолёт приземляется. Окажется ли он в том же месте, из ко­
торого вылетел?
(Подсказка. Решить эту задачу поможет глобус.)
Задача 7. В ящике 30 пар оранжевых перчаток и 20 пар чёр­
ных перчаток. Какое наименьшее число перчаток надо (не глядя)
достать из ящика, чтобы можно было с уверенностью сказать следу­
ющее:
а) среди вынутых перчаток есть пара перчаток одного цвета;
б) среди вынутых перчаток есть пара чёрных перчаток?
(Учтите, что в каждой паре перчатки разные: одна для левой
и одна для правой руки.)
Задача 8. Сколько слонов и верблюдов в зоопарке города Урю-
пинска, если известно, что всего у этих животных 22 уха, а горбов
в 9 раз больше, чем хоботов? (Все верблюды в этом зоопарке дву­
горбые.)
Занятие 2

Рыцари и лжецы

На этом занятии продолжается разговор о важности точно сфор­


мулированных решений. Для этого подобраны занимательные зада­
чи, требующие чёткости и аккуратности в рассуждениях.
Первая подходящая тема возникает сама собой: на предыдущем
занятии несколько человек запутались в задаче про попугая Полли.
У меня нет уверенности, что они усвоили идею причины и след­
ствия, и я решаю уделить время задачам такого типа.
Но одной этой идеи недостаточно, чтобы поддерживать внима­
ние участников до конца занятия. Поэтому я предлагаю несколько
задач по другой интересной теме, неизменно пользующейся успе­
хом у детей.
Занятие начинается с разминки. После неё мы разбираем зада­
чи из предыдущей подборки, а затем обсуждаем новый материал.
Оставшееся время дети работают над новой подборкой задач.

Что принести на урок


• Глобус или надувной шарик. Он понадобится для разъяснения
задачи про самолёт из предыдущего занятия (см. с. 30).
• Счётные палочки или спички (для задачи про совок с мусором
в новой подборке).

Математическая разминка
Задача для разминки 1. Гномик пошёл ловить ершей. Он пой­
мал 5 ершей плюс половину всего улова. Сколько ершей он поймал?
Задача для разминки 2. Бегемот тяжелее жирафа, а жираф лег­
че носорога. Кто тяжелее: бегемот или носорог?
Задачи для разминки 3. Передвиньте две спички на
рисунке справа так, чтобы три превратилось в шесть.
Ломать спички нельзя!
32 Раздел 1. Планы занятий

Подборка задач
На прошлом занятии участники получили подборку задач для са­
мостоятельной работы. Часть из них дети решили в классе, а осталь­
ные стали домашним заданием. Я даже напомнила родителям о до­
машнем задании по электронной почте.
На сегодняшнем занятии мы будем разбирать решения этих за­
дач в начале урока: это поможет сориентироваться тем участникам,
которые сегодня пришли в первый раз.
Для преподавателей. Разбирать задачи из предыдущей подборки можно
и в конце занятия. О разных способах организации учебного времени
подробнее говорится в главе «Будни математического кружка» (с. 254).
Разбор каждой задачи начинается с того, что кто-то из детей по же­
ланию рассказывает своё решение у доски. По ходу рассказа я коммен­
тирую решение, подчеркивая и повторяя его основные идеи.
Многие участники ещё не решаются выходить к доске, но всегда
находятся те, кто рад выступить перед всей группой.

Тема занятия:
«Часть 1. Задачи на причину и следствие»
Я напоминаю участникам: на прошлом занятии мы начали ра­
ботать над задачами, в которых нужно разобраться с причинами
и следствиями (попугай Полли и буря). Сегодня мы продолжим эту
тему.
Пример 1. Мама сказала Свете, Оле и Саше: «Все зебры полоса­
тые». Каждый сделал из маминого утверждения свой вывод.
Саша: «Если животное полосатое, то это зебра».
Оля: «Если животное полосатое, то это, возможно, зебра».
Света: «Если животное не полосатое, то это точно не зебра».
Кто сделал правильные выводы и почему?
РАЗБОР ПРИМЕРА 1. Условие этой задачи длинное, и его трудно
воспринять на слух. Поэтому лучше записать все утверждения на
доске.
Дети сразу начинают выкрикивать ответы. Не комментируя каж­
дый ответ в отдельности, я веду «подсчёт голосов». Мы имеем пол­
ный набор мнений: Саша прав, Оля права, Саша не прав, Света не
права... Какие же ответы правильные?
Занятие 2. Рыцари и лжецы 33

Я рисую схему:
полосатые животные

Зачем нам эта схема и что мы напишем в пустых ячейках? Дети


отвечают, что в них должны быть названия полосатых животных.
Как связать эту схему с маминым утверждением? Мама сказала,
что зебра — полосатое животное, поэтому в первой ячейке мы за­
писываем слово «зебра», а для двух остальных придумываем другие
примеры полосатых животных:

полосатые животные

зебра тигр попугаи

(Попугай попал в эту компанию не случайно: как оказалось,


у Ришава дома живёт полосатый попугай.)
Схема помогает понять, что Сашино утверждение «Если живот­
ное полосатое, то это зебра» неверно. Полосатое животное может
быть зеброй, но может быть и другим животным. Значит, Саша сде­
лал неверный вывод. А вот утверждение Оли «Если животное поло­
сатое, то это, возможно, зебра» верно.
Теперь нарисуем другую схему, помогающую понять утвержде­
ние Светы. Поскольку Света говорила о неполосатых животных, мы
записываем в нижних ячейках животных без полос:

животные без полос

лев попугаи

(Бывают попугаи без полос. Эмма настаивает на том, что таких


попугаев тоже надо отметить.)
Света утверждает, что если животное оказалось в этой схеме (не
имеет полос), то это не зебра. Действительно, мама сказала, что все
зебры полосатые, поэтому их точно не может быть в этой схеме.
Значит, Света права.
34 Раздел 1. Планы занятий

Тема занятия:
«Часть 2. Задачи про рыцарей и лжецов»
Пришло время познакомиться с задачами
ещё одного типа: про рыцарей и лжецов.
Сначала я рассказываю детям о далёком
острове с необычными обитателями. Некото­
рые из них никогда не лгут, а остальные нико­
гда не говорят правду. Те, кто никогда не лгут,
называют себя рыцарями. Те, кто лгут всегда,
называются лжецами. Несмотря на такое раз­
личие в характере, рыцарей невозможно отличить от лжецов по
внешнему виду: они одинаково одеваются и носят одинаковые при­
чёски.
Остров могут посещать туристы. Это обычные люди: иногда они
лгут, а иногда говорят правду. Следующие несколько задач — о жи­
телях этого острова.
Сначала мы просто учимся понимать, как думают и разговари­
вают местные жители и туристы.
Я приглашаю добровольцев. Один мальчик соглашается побыть
лжецом, одна девочка — рыцарем. Ещё один мальчик готов сыграть
роль туриста. Мы будем задавать им вопросы, а они должны отве­
чать согласно своей роли.
Я спрашиваю лжеца: «Ты мальчик?» Поскольку он должен лгать,
после некоторых сомнений он отвечает: «Нет». Затем мы спрашива­
ем нашего рыцаря: «Сколько участников кружка в этой комнате?»
Она пересчитывает детей и сообщает правильный ответ. Мы инте­
ресуемся у лжеца, есть ли у него внука. Он отвечает «Да», так как не
может сказать правду. Мы задаём тот же вопрос туристу. Он решает
солгать и говорит: «Да». Я обращаю внимание детей на то, что он,
будучи туристом, мог также сказать «Нет».
Наконец, я задаю всем троим один и тот же вопрос: «Есть ли лже­
цы в этой комнате?» Рыцарь отвечает: «Да». Лжец говорит: «Нет».
Турист снова выбирает позицию лжеца и говорит: «Нет». Я опять
замечаю, что он мог сказать и «Да».
Теперь участники знают всё необходимое для выживания на этом
острове. Я объявляю, что они готовы к решению задач.
Пример 2. На острове рыцарей и лжецов вам встретился маль­
чик, который говорит, что он лжец. Это житель острова или турист?
Занятие 2. Рыцари и лжецы 35

РАЗБОР ПРИМЕРА 2. Если бы этот мальчик был рыцарем, он сказал


бы, что он рыцарь. Если бы он был лжецом, он тоже сказал бы, что
он рыцарь. Итак, любой житель острова сказал бы, что он рыцарь.
Но мальчик сказал, что он лжец. Значит, он турист.
Пример 3. Вы знакомитесь с двумя мальчиками, живущими на
острове: Федей и Петей. Федя говорит: «Хотя бы один из нас лжец».
Можно ли определить, кто они?
РАЗБОР ПРИМЕРА 3. Сначала нужно убедиться в том, что все участ­
ники правильно понимают выражение «хотя бы один». Вместе мы
находим более точную формулировку: «один или больше».
Утверждение Феди можно переформулировать так: либо один из
мальчиков лжец, либо они оба лжецы.
Поскольку больше нам ничего не известно, приходится делать
предположения.
Сначала предположим, что Федя —лжец. Тогда высказывание
«Хотя бы один из нас лжец» верно. Но поскольку Федя—лжец, он
не мог произнести верного утверждения. Значит, наше предполо­
жение, что Федя—лжец, ошибочно. Итак, Федя —рыцарь.
Раз Федя —рыцарь, он сказал правду. Значит, хотя бы один из
двух мальчиков—лжец. Это, конечно, Петя.
Пример 4. На острове рыцарей и лжецов я знакомлюсь с мест­
ным рыцарем. Я задаю ему один и тот же вопрос два раза и получаю
разные ответы. Что это за вопрос?
РАЗБОР ПРИМЕРА 4. Эта задача вносит оживление. Участники со­
ревнуются в изобретательности и предлагают множество забавных
вариантов. Кейси говорит, что надо подойти к рыцарю на берегу
моря и спросить его: «Ты сухой?», а потом столкнуть его в воду и ещё
раз задать тот же вопрос.
Я хвалю все эти предложения, но замечаю, что хорошо бы найти
универсальный вопрос, который можно задать в любой ситуации,
а не только на берегу. Нужна какая-то величина, которая точно из­
менится между двумя вопросами. Ришав кричит: «Надо спросить,
сколько сейчас времени!»
Ещё один хороший вопрос: «Сколько вопросов я тебе задал?»
Пример 5. Гуляя по острову, вы встретили компанию из трёх
местных жителей, которых зовут Серёжа, Егор и Настя. Вы задаёте
каждому один и тот же вопрос: «Сколько среди вас рыцарей?» Сере-
36 Раздел 1. Планы занятий

жа отвечает: «Ни одного». Егор говорит: «Один». Можно ли опреде­


лить, кто Настя и что она ответит?
РАЗБОР ПРИМЕРА5. Прежде всего заметим, что Серёжа не может
быть рыцарем, иначе он не мог бы сказать, что в их компании нет
ни одного рыцаря. Следовательно, Серёжа—лжец.
Зная, что Серёжа —лжец, мы заключаем, что его высказывание
«Среди нас нет ни одного рыцаря» неверно. Значит, среди них есть
хотя бы один рыцарь. Кто же он? Сначала подумаем, кем может
быть Егор.
Предположим, что Егор —лжец. Тогда рыцарь — Настя. Вспом­
ним, что ответил Егор. Он сказал: «Среди нас один рыцарь». Но это
правда: ведь единственный рыцарь — Настя. Будучи лжецом, Егор
не мог сказать правду. Значит, наше предположение ошибочно.
Теперь мы знаем, что Егор — рыцарь. Поэтому его ответ правдив,
и в этой компании ровно один рыцарь. Следовательно, Настя—лжец.
Мы доказали, что в этой задаче возможен только один вариант:
Серёжа и Настя—лжецы, а Егор —рыцарь. Теперь подумаем, что
может ответить Настя на заданный вопрос. Настя—лжец, поэтому
она не скажет правду, что рыцарь один. Но она может выбрать
любой другой ответ: «Ни одного», «Два», «Три» или, например, «Сто
пятьдесят».
После небольшого перерыва я раздаю новую подборку задач, за­
читываю задачи вслух и отвечаю на вопросы.
До конца занятия участника работают над задачами. Я напоми­
наю им правила.
• Задачи можно решать в любом порядке.
• Вопросы по задачам приветствуются.
• Рассказ решения также приветствуется.
Для преподавателей. Задачи про рыцарей и лжецов обширно представлены
в математической литературе, особенно в [2, 3, 14, 17, 18, 25]. Пользуясь
этими материалами, следите за тем, чтобы в словах рыцарей и лжецов не
было сложных логических высказываний (когда два или несколько утвер­
ждений соединены союзом «и» или «или»). Давать такие задачи рано: сна­
чала дети должны научиться правильно понимать сложные высказывания.

Подборка задач
Предварительные замечания. Задача 4 со спичками вызовет больше ин­
тереса, если выдать участникам палочки или спички. Но помните: по-
Занятие 2. Рыцари и лжецы 37

лучив в руки палочки, дети обо всём забудут и начнут решать голово­
ломку. Поэтому палочки выдаются после того, как прочитаны условия
всех задач.
Задача 1. Мама сказала Максу: «Все
рыбы любят плавать». Макс отвечает:
«Я люблю плавать, значит, я рыба». Прав
ли Макс? Обоснуйте ваш ответ.
Задача 2. Фома и Ерёма — близнецы. Фома всегда говорит прав­
ду, а Ерёма всегда лжёт. Какой вопрос надо задать каждому из них,
чтобы получить один и тот же ответ? (Отличить Фому от Ерёмы
невозможно.)
Задача 3. Золотой слиток весит две трети самого себя и ещё
пять килограммов. Сколько весит золотой слиток?
Задача 4. На рисунке справа изображён совок с му­
сором. Как «убрать мусор из совка», переложив всего
две спички? Форма совка должна остаться той же. (Это
пример задачи, требующей нестандартного подхода.
Может показаться, что она не имеет решения, но это
не так!)
Задача 5. Сумчатый медведь коала забирается на
эвкалипт высотой 20 метров. Он начинает с земли и каждый день
поднимается на 5 метров, а каждую ночь сползает вниз на 4 метра.
Через сколько дней и ночей коала окажется на верхушке дерева?

Задача 6. Гуляя по острову рыцарей и лжецов, вы попадаете


в прекрасный сад, где три жителя острова — Миша, Боря и Слава —
любуются закатом. Вы спрашиваете Мишу: «Ты рыцарь или лжец?»
38 Раздел 1. Планы занятий

Миша очень застенчив и отвечает так тихо, что ничего невозмож­


но понять. Тогда вы спрашиваете Борю: «Что сказал Миша?» Боря
отвечает: «Он сказал, что он лжец». «Не верьте Боре! Боря лжец!» —
кричит Слава. Можно ли определить, кто Боря и Слава —рыцари
или лжецы?

ДОПОЛНИТЕЛЬНЫЕ ЗАДАЧИ

Задача 7*. На волшебной совиной почте работают совы трёх ви­


дов: ушастые, болотные и полярные. В это воскресенье почти все
совы улетели разносить письма. Тимофей и Кузьма зашли на по­
чту и осмотрелись. Тимофей обратил внимание на то, что все совы,
кроме двух, ушастые. Одновременно Кузьма заметил, что все совы,
кроме двух, болотные. Сколько сов каждого вида было на почте?
(У этой задачи несколько ответов. Постарайтесь найти все.)
Задача 8*. Пятиклассник Илья решил сделать кубик Рубика.
Для этого он взял деревянный куб размерами 3 х 3 х 3 и покрасил
шесть его граней в разные цвета. Потом он распилил этот куб на
27 кубиков 1 x 1 x 1 . (Для этого ему пришлось сделать 6 разрезов:
два по горизонтали, два по вертикали параллельно передней грани
и два по вертикали параллельно боковым граням.)
а) Сколько кубиков имеют ровно три окрашенные грани? Ровно
две? Ровно одну? У скольких кубиков не окрашена ни одна грань?
б) Антон—друг Ильи — считает, что Илье надо было ставить ча­
сти куба друг на друга, когда он их пилил. Тогда можно было бы
обойтись меньшим количеством разрезов. Прав ли Антон? Покажи­
те, как получить 27 кубиков меньше чем за 6 разрезов, или объяс­
ните, почему Антон неправ.
Занятие 3

Как превратить ложь в правду

На прошлом занятии мы познакомились с математической ло­


гикой. Мы решали задачи про рыцарей, которые никогда не лгут,
и лжецов, которые никогда не говорят правду.
При этом выявился важный момент: детям трудно извлекать вер­
ную информацию из высказываний лжецов. Например, если лжец го­
ворит: «Этот предмет чёрный», участники сразу же решают, что он бе­
лый. На самом деле предмет может быть любого цвета, кроме чёрно­
го. Правильно понимать сложные утверждения лжецов ещё труднее.
Пришлось расстаться ещё с одной иллюзией. У меня не было со­
мнений, что дети отлично ориентируются в таких выражениях, как
«не больше 100», «хотя бы 3» или «меньше чем 47». Но оказалось, что
некоторые ученики быстро запутываются, встречаясь с подобными
оборотами речи, а то и вовсе не понимают их точного значения.
Поэтому цель сегодняшнего занятия — научить участников пра­
вильно понимать ложные логические высказывания. По ходу дела
мы заодно обсудим смысл фраз «не меньше», «больше чем» и им
подобных.
Кроме того, мы меняем форму проведения занятий, точнее —
способ разбора домашнего задания. Раньше мы обсуждали задачи
всей группой, слушая выступления отдельных участников у доски.
Но очень важно дать каждому участнику кружка возможность рас­
сказывать решения преподавателю один на один. Во время такой
беседы преподаватель по мере необходимости мягко направляет ход
рассуждений ученика, формируя навыки математического мышле­
ния. Кроме того, для ученика перспектива личной беседы с пре­
подавателем служит дополнительным стимулом к решению задач,
а для преподавателя это отличная возможность узнать о сильных
и слабых сторонах каждого участника кружка.
За предыдущие занятия дети уже привыкли друг к другу и к пре­
подавателю и меньше стесняются. Поэтому с сегодняшнего дня до­
машнее задание будет разбираться в индивидуальном порядке: каж­
дый участник будет рассказывать свои решения мне.
40 Раздел 1. Планы занятий

Обычно я провожу эти беседы в конце занятия — после обсужде­


ния новой темы и раздачи очередной подборки задач. Но сегодня
я не знаю, как пойдут дела. Что, если я не успею поговорить с каж­
дым? Поэтому я решаю нарушить традицию и планирую индивиду­
альные беседы сразу после разминки.
Вскоре выясняется, что эта идея была не самой удачной. Дети по
очереди рассказывают свои решения, и тем, кто уже освободился,
нечем заняться. Они стараются вести себя хорошо, но им явно скуч­
но. Выдать им очередную подборку задач? Но тогда будет сложно
переключить их внимание на новый материал. На беседы со всеми
восемью участниками уходит тридцать минут. За это время я узнаю
много нового о каждом из них, но последние десять минут проходят
в нарастающем напряжении.
Урок на будущее. Если индивидуальное обсуждение домашнего
задания планируется на начало занятия, надо подготовить пару ин­
тересных задач, которыми можно занять освободившихся детей.

Математическая разминка
Задача для разминки 1. Два ската крыши наклонены под раз­
ными углами: 60 и 45 градусов. Петух снёс яйцо на коньке крыши.
В какую сторону покатится яйцо?
Задача для разминки 2. На острове рыцарей и лжецов живут
два брата-близнеца: Сеня и Веня. Каждый из них либо рыцарь, либо
лжец. В день, когда им исполнилось 10 лет, Сеня сказал, что ему не
меньше девяти лет, а Веня — что ему меньше десяти. Кто Сеня и кто
Веня — рыцари или лжецы?
Задача для разминки 3. Эта задача служит подготовкой к об­
суждению темы сегодняшнего занятия.
Я пишу последовательность чисел: 10,11,12,13,14,... — и пред­
лагаю описать её как можно большим количеством способов. Опи­
сание должно содержать выражения «меньше», «больше», «не мень­
ше», «не больше» и т. п.
Участники предлагают свои варианты, а я записываю их. В итоге
мы имеем несколько способов описания этой последовательности,
среди которых попадаются неожиданные.
«Числа, которые не меньше 10» (Эмма).
«Числа, больше или равные 10» (Кейси).
«Числа, которые больше 9» (Сэм).
Занятие 3. Как превратить ложь в правду 41

«Числа, в которых больше одной цифры» (Эндрю).


«Начинается с 10, и каждое число на 1 больше предыдущего»
(Тара).
На сегодняшнем занятии часто будут звучать слова «больше чем»,
«не меньше» и т.п. Чтобы упростить себе жизнь, мы составляем
«шпаргалку», записывая её в редко используемом углу доски.
«больше 10» 11,12,13, ...
«не больше 10» 10, 9, 8, ...
«меньше 10» 9, 8, 7, ...
«не меньше 10» 10, 11, 12, ...
«по крайней мере 10» 10, 11, 12, ...
«самое большее 10» 10, 9, 8, ...

Тема занятия:
«Отрицание логических высказываний»
В прошлый раз нам впервые встретились задачи про рыцарей
и лжецов. Сегодня мы продолжим изучение этой темы. Как извест­
но, рыцари никогда не лгут: всё, что мы узнали от рыцаря, можно,
не задумываясь, использовать для решения задачи. Лжецы всегда
лгут: если лжец что-то сказал, нужно внимательно обдумать его сло­
ва, чтобы узнать, как обстоят дела в действительности. Сейчас мы
потренируемся в этом.
Сначала — несколько простых задач, которые помогут отточить
навыки поиска истины.
Упражнение 1. Лжец утверждает, что его друг Артём говорит
правду. Что означает это высказывание?
Несколько участников отвечают, что Артём лжёт.
Упражнение 2. Лжец заявляет, что этот кусок мрамора чёрный.
Что это означает?
Сэм сразу отвечает, что мрамор должен быть белым. «А почему
не красным?» — возражает Тара. После короткого обсуждения мы
приходим к выводу, что определить настоящий цвет мрамора невоз­
можно. Мы точно знаем об этом куске мрамора только одно: он не
чёрный.
Упражнение 3. Лжец утверждает, что в комнате нет ни одной
девочки. Каким будет истинное высказывание?
42 Раздел 1. Планы занятий

Соннет предлагает ответ: «В комнате одна девочка». Ришав утвер­


ждает: «Все, кто находится в этой комнате,—девочки». Мы разби­
раем эту задачу и приходим к выводу, что определить количество
девочек в комнате невозможно: точно известно только то, что там
есть хотя бы одна. В комнате может быть две, три или любое другое
количество девочек. Истинное высказывание можно сформулиро­
вать по-разному, например так: «В комнате есть девочки». Или так:
«В комнате есть хотя бы одна девочка». Или «Количество девочек
в комнате больше нуля».
На этом этапе удобно ввести новое определение. Напомните участ­
никам, что в каждом упражнении мы некоторым образом «обращали»
высказывание: из ложного утверждения получали истинное.
Для этой операции существует специальный математический
термин: отрицание. Отрицание высказывания — это другое выска­
зывание, противоположное первому по смыслу.
Пришло время решать задачи на отрицание высказываний. Я за­
читываю задачи по одной, и мы решаем их вместе.
Пример 1. Лжец говорит: «Моему дедушке самое меньшее сто
лет». Каким будет истинное высказывание о возрасте дедушки?
РАЗБОР ПРИМЕРА 1. Возможные ответы: «Дедушке меньше ста
лет», «Дедушке не больше 99 лет» или любое другое высказывание,
описывающее набор чисел (99, 98, 97,...}.
Пример 2. Волк заявляет: «В этом доме не меньше 10 поросят».
Известно, что этот волк—лжец. Каким будет истинное высказыва­
ние о количестве поросят в доме?
РАЗБОР ПРИМЕРА 2. Возможные ответы: «В доме не больше чем
9 поросят», «В доме меньше 10 поросят» или любое другое высказы­
вание, описывающее набор чисел (9, 8, 7,...}.
Пример 3. Волк жалуется: «Поросё­
нок, которого я съел вчера, весил мень­
ше 10 килограммов!» Если известно, что
волк лжёт, каким будет истинное выска­
зывание? (Все поросята весят целое чис­
ло килограммов.)
РАЗБОР ПРИМЕРА 3. Возможные отве­
ты: «Поросёнок весил самое меньшее 10 килограммов», «Поросёнок
весил больше чем 9 килограммов» или любое другое высказывание,
описывающее набор чисел (10,11,12,...}.
Занятие 3. Как превратить ложь в правду 43

Пример 4. Участник кружка говорит, что в его кружке занима­


ются только мальчики. Если известно, что этот участник лжёт, что
можно сказать о составе этого кружка?
РАЗБОР ПРИМЕРА 4. Возможные ответы: «Не все участники этого
кружка — мальчики», «В этом кружке есть девочки» или любое дру­
гое высказывание, означающее, что в этом кружке есть хотя бы одна
девочка.
После разбора этой задачи мы делаем небольшой перерыв. За­
тем я раздаю новую подборку задач и предлагаю решать их.

Подборка задач
Предварительные замечания. Можно дать усложнённый вариант зада­
чи 6: сначала был ковёр 9 х 12, у длинного края сгорела полоска 1x8,
после ремонта получился ковёр 10 х 10.
Задача 1. Маша сказала, что её старший
брат Саша съел самое меньшее 12 конфет, но
сам Саша утверждает, что он съел не боль­
ше 13. Известно, что Саша съел в два ра­
за больше конфет, чем Маша, и что они оба
всегда говорят правду. Сколько конфет съел
Саша?
Задача 2. Планета 55Е в системе Рака, на­
зываемая также Крэбби, населена рыцарями
и лжецами (рыцари никогда не лгут, а лжецы никогда не говорят
правду). Кроме них, на этой планете живут странные и необычные
животные.
Однажды рыцарь Тао с планеты Крэбби сказал: «Все трёхголовые
животные на нашей планете очень умные». Исходя из слов Тао опре­
делите, верны ли следующие утверждения.
— «У всех умных животных на планете Крэбби три головы».
— «Если животное с планеты Крэбби глупое, то у него точно не
три головы».
— «Если животное не трёхголовое, то оно не умное».
Задача 3. Во время путешествия по острову рыцарей и лжецов
вы встречаете двух живущих на острове братьев — Ивана и Пет­
ра. Иван говорит вам, что хотя бы один из них—лжец. Кто Иван
и Пётр--рыцари или лжецы?
44 Раздел 1. Планы занятий

Задача 4. Тридцать детей пришли на праздник. Среди каждых


12 из них есть хотя бы один мальчик. Среди каждых 20 из них
есть хотя бы одна девочка. Сколько мальчиков и сколько девочек
на празднике?
Задача 5. На острове рыцарей и лжецов вы оказались в комна­
те, где находятся несколько человек. Каждый из них говорит вам,
что в этой комнате есть лжецы. Что происходит на самом деле?
Сколько в комнате рыцарей и сколько лжецов? (Не забывайте, что
в комнате могут быть и туристы. Туристы иногда говорят правду,
а иногда лгут.)
Задача 6. У Ивана-Царевича есть ковёр-самолёт, размер которо­
го 5 х 8 метров. Во время битвы со Змеем Горынычем полоска вдоль
длинного края ковра полностью сгорела. Сгоревшая полоска —это
прямоугольник 1 x 4 метра.

Ковры-самолёты должны иметь прямоугольную форму, иначе


они теряют устойчивость. Поэтому Иван-Царевич попросил Васи­
лису Премудрую отрезать от повреждённой стороны оставшуюся
полоску 1 x 4 метра, чтобы ковёр снова стал прямоугольным. Но Ва­
силиса Премудрая занимается в математическом кружке, поэтому
она поступила иначе: разрезала ковёр на три прямоугольные части
и сшила их так, что получился квадратный ковёр 6 x 6 метров. Как
она это сделала?
Задача 7*. На острове рыцарей и лжецов есть две деревни. В од­
ной из них живут только рыцари, а в другой — только лжецы. Жите­
ли этих деревень часто ходят друг к другу в гости.
На днях островная газета объявила конкурс: надо придумать
один-единственный вопрос (требующий ответа «да» или «нет»), с по­
мощью которого любой турист может определить, находится ли
он в деревне рыцарей или в деревне лжецов. Вопрос должен быть
таким, чтобы его можно было задать первому встречному. Заранее
Занятие 3. Как превратить ложь в правду 45

неизвестно, кто этот первый встречный — рыцарь или лжец. Неиз­


вестно также, живёт ли он в этой деревне или пришёл в гости.
Дима, занимающийся в математическом кружке, выиграл кон­
курс. Какой вопрос он предложил?

ДОПОЛНИТЕЛЬНЫЕ ЗАДАЧИ

Задача 8. Маша и Даша живут на острове рыцарей и лжецов.


Одна из них—лжец, а другая —рыцарь. Маша утверждает, что 2 ру­
бина дороже, чем 3 сапфира. Даша говорит, что 3 рубина дороже,
чем 4 сапфира. Верно ли, что 12 рубинов дороже, чем 18 сапфиров?
Задача 9. Антон, Боря, Витя и Гриша живут на острове рыцарей
и лжецов. Антон утверждает, что Боря—лжец. Гриша говорит, что
Антон—лжец. Витя заявляет, что Антон и Боря —два лжеца. Витя
также говорит, что и Гриша—лжец. Кто на самом деле рыцари и кто
лжецы? Обоснуйте ответ.
Задача 10*. Тридевятое царство состоит из нескольких обла­
стей. Там есть несколько политических партий. Однажды за ужином
собралась компания политиков из Тридевятого царства. Известно,
что среди них есть жители разных областей и члены разных партий.
Докажите, что в этой компании найдутся два политика, которые
живут в разных областях и состоят в разных партиях.
Занятие 4

Математический аукцион I

Развлекательные мероприятия — важная часть программы мате­


матического кружка. Хорошо подготовленный математический кон­
курс служит нескольким целям: это не только обучение и мотива­
ция участников, но и возможность лучше узнать друг друга в ходе
командной игры.
Сегодня первое математическое соревнование в нашем кружке.
Для такого случая я выбрала математический аукцион—увлека­
тельный конкурс, правила которого легко усвоить. Задачи матема­
тического аукциона успешно решают даже начинающие. Поэтому
я надеюсь, что сегодняшнее занятие понравится детям.
Что принести на урок
• Подарки и призы для участников соревнования. Это, конечно,
не обязательно. Но дети будут рады скромным наградам—леден­
цам или шоколадкам.

Математическая разминка
Задача для разминки 1. В доме десять этажей. На первом этаже
живёт один человек, на втором—два, на третьем —три и так далее.
На каком этаже лифт останавливается чаще всего?
Задача для разминки 2. Лифт поднимается с первого этажа до
третьего за 3 секунды. За какое время он поднимется с первого эта­
жа до девятого?
Событие дня:
игра «Математический аукцион»
Математический аукцион — это увлекательное командное сорев­
нование. В начале игры отводится время на решение задач, а за-
Занятие 4. Математический аукцион I 47
тем начинается купля-продажа решений. Каждая команда добива­
ется права рассказывать свои решения, делая ставки, обдумывая
план действий и вырабатывая стратегию. В наших математических
кружках аукционы пользуются неизменной популярностью. Прави­
ла аукциона изложены на с. 232.
Для преподавателей. Хорошо запомните правила соревнования. В ходе
аукциона возможны неожиданные повороты, а растеряться в процессе
игры —никуда не годится! Внимательно прочитайте советы для препо­
давателей на с. 236. Они помогут провести игру без досадных задержек.

Планирование времени
Математическое соревнование обычно занимает больше време­
ни, чем типичная лекция. К тому же дети очень расстроятся, если
аукцион придётся прервать на середине. Поэтому тщательное пла­
нирование времени в этом случае особенно важно.
Для сегодняшнего аукциона я отвожу 20 минут на объяснение
правил и формирование команд, 30 минут на решение задач и ещё
30 минут на сам аукцион. Итак, общая продолжительность игры
приближается к полутора часам.
В связи с этим обычный распорядок изменяется. Вначале я рас­
сказываю решения задач прошлой недели, не предлагая сделать это
участникам. Кроме того, я не планирую в конце занятия время для
самостоятельного решения задач. Если аукцион закончится немно­
го раньше, можно будет поработать над задачами из новой подбор­
ки; если нет —все они останутся на дом.
Для преподавателей. Если занятие длится меньше полутора часов, можно
обойтись без разминки, а разбор задач из предыдущей подборки отло­
жить до следующего раза.
Если времени всё равно не хватает, можно отвести на решение за­
дач меньше тридцати минут. Задачи математического аукциона име­
ют несколько решений, причём найти какое-нибудь решение не так уж
трудно. Поэтому даже если у команд мало времени на решение, им бу­
дет что выставить на аукцион.

Правила игры
Это первый аукцион в нашем кружке, и поэтому я не жалею
времени и усилий на объяснение правил. Они доходчиво изложены
в разделе «Правила математического аукциона» на с. 232. Там же (на
с. 235) приведён пример игры. Как показано на этом примере, одна
48 Раздел 1. Планы занятий

и та же задача может быть продана несколько раз, и отстающая ко­


манда может выйти в лидеры, если улучшит решение выставленной
на аукцион задачи.

Соревнование
После объяснения правил и деления участников на две коман­
ды начинается игра. Я раздаю задачи, зачитываю их и отвечаю на
вопросы.
На соревнованиях дорога каждая минута, поэтому нужно поль­
зоваться любой возможностью сэкономить время. Например, что­
бы рассказать решения задач, часто нужны сложные рисунки: шах­
матные доски, геометрические фигуры или длинные строки цифр.
Дети обычно рисуют на доске медленно, и их рисунки не вполне со­
вершенны. Рисунки и записи, сделанные преподавателем заранее,
помогут сократить время выступлений.
Итак, пока участники работают над задачами, я подготавливаю
доску.
Первая задача сегодняшнего аукциона —о составлении разных
чисел из двоек и знаков арифметических действий. Поэтому я со­
ставляю столбик из шаблонов, которые участники будут заполнять
нужными знаками. (Если записей не хватит, список можно увели­
чивать прямо по ходу выступления.)

Во второй задаче речь идёт о разбиении круга, и я рисую в дру­


гой части доски два больших круга.
Кроме того, вам стоит нарисовать таблицу, чтобы вести счёт оч­
ков каждой команде (п. 5 на с. 236).
И вот время для решения задач заканчивается. Участники объ­
являют названия команд и представляют капитанов. Я записываю
названия команд вверху счётной таблицы.
Мы ещё раз вспоминаем основные правила игры.
• Одна и та же задача выставляется на торги несколько раз,
пока есть команда, которая готова предложить лучшее решение.
Занятие 4. Математический аукцион I 49

При этом не возбраняется продажа задачи одной и той же команде


несколько раз подряд.
• Стоимость каждой задачи достаётся одной команде — той, ко­
торая предложила наилучшее решение. Все команды (включая по­
бедившую в этом раунде) теряют свои ставки.
• Делать ставки могут только капитаны. Другим членам коман­
ды это запрещено.
• Один и тот же член команды не может рассказывать решения
разных задач, но может рассказывать разные решения одной и той
же задачи, если она повторно выставляется на аукцион. (Это прави­
ло не действует, если команда очень малочисленна.)
Наконец, начинается сам аукцион.
Наш аукцион проходит как нельзя лучше. Обе команды полны
энтузиазма и активно участвуют в торгах. Первая задача продаётся
три раза, вторая — два. Побеждает команда «Пицца», заработавшая
350 шмолларов (см. с. 232). В конце игры победители и проиграв­
шие получают шоколадки.
Для преподавателей. Математический аукцион — это соревнование. Кро­
ме того, предлагаемые на аукционе задачи не имеют «окончательного»
решения. Поэтому в этой книге не публикуются решения задач аукциона.

Задачи для математического аукциона


Каждая задача стоит 100 шмолларов (см. с. 232).
Задача 1. Используя цифру 2 пять раз, получите как можно боль­
ше идущих подряд натуральных чисел начиная с 1. Можно пользо­
ваться действиями арифметики «+», «—», «•», «:» и скобками. Одно
и то же действие может встречаться несколько раз. Использовать
все четыре действия не обязательно. Примеры:
1 = ( 2 - 2 : 2) 2 : 2, 2 = 2 2 - 2 2 + 2.
Решение считается лучше предыдущего, если команда может
продолжить список с того числа, на котором остановилась высту­
павшая перед этим команда.
Задача 2. Разрежьте круг семью прямыми линиями так, что­
бы получилось как можно больше треугольников. «Треугольники»
с кривыми сторонами не считаются. Треугольники с разрезами внут­
ри (состоящие из меньших треугольников или многоугольников)
тоже не считаются.
50 Раздел 1. Планы занятий

Решение считается лучше предыдущего, если команда может по­


казать рисунок, на котором больше треугольников.
Задача 3. Сделав как можно меньше разрезов, разделите семь
круглых пицц на дольки так, чтобы их можно было поровну поде­
лить между восемью людьми. (Разрез — это прямая линия от края
до края пиццы, проходящая через её центр. Все должны получить
равные порции, но не обязательно одинаковое количество долек.
Лишних долек остаться не должно.)
Решение считается лучше предыдущего, если команда может
разделить пиццы меньшим количеством разрезов.
Задача 4. У купца Али есть золотая цепь из 20 звеньев. (Цепь не
замкнута в кольцо.) Али отправляется в дальнее путешествие и на­
нимает проводника для 20-дневного перехода через пустыню. Про­
водник требует в качестве оплаты по одному звену цепи за каждый
день и выставляет ещё одно условие: Али должен рассчитываться
с ним в конце каждого дня.
Для расчёта с проводником купцу придётся распилить часть зо­
лотых звеньев. Это требует дополнительных расходов, поэтому Али
хочет распилить как можно меньше звеньев. К счастью, проводник
соглашается давать сдачу: например, Али может в первый день рас­
платиться одним звеном, а во второй отдать проводнику кусок цепи
из двух звеньев и получить обратно вчерашнее звено.
Какое наименьшее число звеньев должен распилить Али, чтобы
выполнить условие проводника?
Решение считается лучше предыдущего, если команда может по­
казать способ расплатиться, распилив меньше звеньев.

Подборка задач
Задача 1. На острове рыцарей и лжецов вам встретился мест­
ный мальчик. Что нужно у него спросить, чтобы понять, кто он —
рыцарь или лжец?
Пример 2. 6 акул весят меньше,
чем 5 китов, но больше, чем 10 дель­
финов. Все акулы имеют одинаковый
вес, так же как и все дельфины. Кто
тяжелее: 2 акулы или 3 дельфина?
Занятие 4. Математический аукцион I 51

Задача 3. Можно ли разрезать квадрат на 7 квадратов меньшего


(не обязательно одинакового) размера? Если можно, покажите, как
это сделать. Если нельзя, объясните почему. Квадраты с разрезами
внутри (состоящие из более маленьких квадратов) не считаются.
Задача 4. У Макса живут три гово­
рящих попугая: Полли, Долли и Арчи.
Известно, что один из попугаев Мак­
са никогда не говорит правду, а дру­
гой никогда не лжёт. Третий попугай —
обычный обманщик: иногда он лжёт,
а иногда говорит правду.
В гости к Максу приходит Нина. Она
спрашивает каждого попугая: «Кто ты?»
«Я лжец!» — заявляет Полли.
«Я обманщица!» — говорит Долли.
«Я абсолютно честный!» —утверждает Арчи.
Помогите Нине определить, кто лжец и кто обманщик.
Задача 5. В Заколдованной стране есть министерство магии,
где работают 21 ведьма, 15 шарлатанов и 19 настоящих колдунов.
Мы собираемся пригласить в наш кружок делегацию из этого мини­
стерства. Сколько человек должно быть в делегации, чтобы к нам
точно приехали хотя бы три настоящих колдуна?
Занятие 5

Текстовые задачи и здравый смысл

Сегодняшнее и следующее занятие посвящаются алгебраическим


текстовым задачам. Цель этих занятий — продемонстрировать, что
здравый смысл и умение рассуждать часто позволяют решить зада­
чу значительно проще, чем с помощью алгебраических уравнений.

Математическая разминка
Задача для разминки 1. В корзине 6 яблок. Можно ли поделить
их между шестью детьми так, чтобы каждый ребёнок получил по
яблоку, а одно яблоко осталось в корзине?
Задача для разминки 2. Два брата —Али и Абу—спорили, чей
ишак медлительнее. Наконец они решили устроить гонки на иша­
ках до ближайшей деревни. Чей ишак придёт последним, тот выиг­
рает спор.
Братья сели на своих ишаков... Угадайте, что произошло потом?
Правильно: никто не двинулся с места. Наверное, они сидели бы так
до сих пор, если бы эту картину не заметил работавший неподалё­
ку крестьянин. Заинтересовавшись, он закричал братьям: «Что случи­
лось?» Те слезли с ишаков и подошли к нему, чтобы объяснить ситуа­
цию. Выслушав братьев, крестьянин чуточку подумал и что-то сказал
им. В ту же секунду Али и Абу ринулись к ишакам, вскочили на них
и помчались в сторону деревни. Какой совет дал братьям крестьянин?

Тема занятия:
«Часть 1. Рисунок помогает решить задачу»
В этом разделе представлены разнообразные текстовые задачи,
которые объединяет одно: их гораздо легче понять и решить, сделав
наглядные рисунки.
Пример 1. На двух полках стоят 79 книг. На одной полке на 3
книги больше, чем на другой. Сколько книг на каждой полке?
Занятие 5. Текстовые задачи и здравый смысл 53

РАЗБОР ПРИМЕРА 1. Это не очень трудная задача. Эндрю сразу вы­


даёт ответ: «38 и 41». Я отвечаю, что эта пара чисел действитель­
но является ответом, но нужно ещё объяснить его. Иначе как мы
узнаем, что наш ответ единственно возможный? Участники начи­
нают предлагать разные способы обосновать ответ Эндрю. Можно
проверить все числа от 1 до 79. Ещё одно предложение — составить
уравнение и решить его.
Какой способ выбрать? Проверка всех чисел подряд, безусловно,
является обоснованием ответа, но она будет долгой и утомитель­
ной. А если бы книг было 365? Такую проверку ни у кого не хватило
бы терпения довести до конца. Мы решаем, что этот метод нам не
подходит.
Составить уравнение — хороший способ решения таких задач.
Но в нашем кружке для пятого и шестого классов ещё не все умеют
решать уравнения.
Итак, мы выбираем другой метод: понять задачу нам поможет
рисунок. Нарисуем два ряда книг один под другим. Мы ещё не зна­
ем, сколько книг в каждом ряду, но знаем, что в длинном ряду их
на 3 больше, чем в коротком (рисунок слева).
Что будет, если убрать с полки эти лишние 3 книги? Мы получим
два одинаковых ряда, причём всего в обоих рядах 79 — 3 = 76 книг
(см. правую часть рисунка).
первая полка:
вторая полка:
79 книг 76 книг
Поскольку ряды одинаковые, в каждом из них 76: 2 = 38 книг.
Следовательно, на одной полке первоначально было 38 книг, а на
другой — 41.
Для преподавателей. Этот пример иллюстрирует общий принцип реше­
ния задач такого типа. Он заключается в следующем. Сначала мы пыта­
емся определить общую часть, которая содержится во всех неизвестных
величинах. Затем убираем элементы, которые обуславливают различие
этих величин. Наконец вычисляем величину общей части и, зная её,
находим все неизвестные величины. Выделить общее и различное в рас­
сматриваемых величинах нам помогают рисунки.
Для преподавателей. Не упускайте случая напомнить участникам, что
в задачах такого типа недостаточно просто дать ответ, особенно если
54 Раздел 1. Планы занятий

он получен методом проб и ошибок. Ответ без обоснования не гаран­


тирует, что найдены все возможные решения.
Для преподавателей. Более «образованные» участники, уже умеющие ре­
шать уравнения, могут слегка пренебрежительно отнестись к рисункам.
Поэтому стоит особо отметить преимущества наглядного представле­
ния задач. Рисунок часто помогает решить задачу быстрее, чем уравне­
ние, а на математических соревнованиях время дорого. Но важнее дру­
гое: рисунок — это эффективный инструмент математического исследо­
вания. Он тренирует необходимую будущим учёным способность по­
смотреть на задачу с разных точек зрения и формирует полезную при­
вычку к поиску геометрической интерпретации задач.
Наглядное представление — это схема, отражающая существенные
взаимосвязи между элементами задачи. В только что разобранной за­
даче рисунок, по сути, является скрытой формой уравнения. Ведь урав­
нение — это просто другой (алгебраический) способ выразить те же вза­
имосвязи между элементами. Когда дети научатся представлять задачи
в виде рисунков, им будет значительно проще освоить формулирование
взаимосвязей на алгебраическом языке.
Пример 2. Если сложить возраст брата с возрастом сестры, по­
лучится 53 года. Брат на 9 лет старше сестры. Сколько лет брату?
РАЗБОР ПРИМЕРА 2. Поскольку брат старше, общая часть двух
неизвестных возрастов — это возраст сестры:
сестра: 9 лет
брат:
53 года 44 года
Две общие части в сумме дают 53 — 9 = 44. Следовательно, общая
часть равна 4 4 : 2 = 22. Значит, брату 22 + 9 = 31 год.
Пример 3. Сумма трёх последовательных чисел равна 165. Что
это за числа?
РАЗБОР ПРИМЕРА3. На первый взгляд эта задача кажется более
трудной. Но в основе её решения лежит тот же метод: выделить об­
щее, сделать рисунок и убрать лишнее.
первое число:
второе число:
третье число:

165 162
Занятие 5. Текстовые задачи и здравый смысл 55

Верхний прямоугольник означает первое неизвестное число. Вто­


рое число на одну единицу «длиннее» первого, а третье на две еди­
ницы «длиннее» первого. Убрав эти лишние единицы, мы получим
три прямоугольника одинаковой длины. Вместе они составляют
165 — 3 = 162 единицы; значит, длина каждого тёмного прямоуголь­
ника равна 54. ОТВЕТ: 54, 55 и 56.
Пример 4. У горного гнома есть коллекция драгоценных кам­
ней. Он мечтает о том, чтобы камней в его коллекции стало в три
раза больше. Гном заметил, что его мечта сбудется, если он добавит
к коллекции 18 камней. Сколько камней у него сейчас?
РАЗБОР ПРИМЕРА 4. Спросите участников, какие величины было
бы удобно показать на рисунке. Постарайтесь совместными усили­
ями получить примерно такой рисунок:
реальность

мечта
Если мечта гнома осуществится, он станет в три раза богаче. По­
этому на нашем рисунке нижний прямоугольник в три раза длиннее
верхнего. Разница между ними равна удвоенному верхнему прямо­
угольнику. В то же время мы знаем, что эта разница равна 18. Сле­
довательно, длина верхнего прямоугольника равна 9.
Пример 5. Макс и Ксюша потратили в парке аттракционов одну
и ту же сумму денег. Макс три раза катался на американских горках
и один раз на карусели, а Ксюша—два раза на американских горках
и четыре раза на карусели. Во сколько раз билет на американские
горки дороже билета на карусель?
РАЗБОР ПРИМЕРА 5. Вот рисунок к этой задаче:

Макс:
Ксюша:
Тёмные прямоугольники означают стоимость одного катания на
американских горках, а светлые — одного катания на карусели. Из
рисунка следует, что одно катание на американских горках стоит
столько же, сколько три катания на карусели.
56 Раздел 1. Планы занятий

Тема занятия:
«Часть 2. Вместе быстрее!»

Мы переходим к другой категории текстовых задач, которые лег­


ко решаются с помощью особого приёма.
Пример 6. Макс съедает банку клубничного варенья за 15 дней,
а Ксюша съедает такую же банку за 10 дней. Сколько дней продер­
жится банка варенья, если из неё будут есть и Макс, и Ксюша?
РАЗБОР ПРИМЕРА 6. Сначала посчитаем, сколько банок варенья
съел бы каждый из детей за 30 дней. Это просто: Макс съел бы 2 бан­
ки, а Ксюша 3. Значит, за 30 дней Макс и Ксюша вместе съели бы
5 банок. Зная это, легко сосчитать, за сколько времени они вдвоём
прикончат одну банку: 30:5 = 6 дней.
Обсудим этот способ решения. Почему мы выбрали именно 30
дней? Это число оказалось таким удобным, потому что оно делится
и на 10, и на 15. Другими словами, 30 — это общее кратное чисел 10
и 15. Выбрав общее кратное в качестве периода времени, легко под­
считать вклад каждого в поедание варенья за этот период.
Но 30 — не единственное общее кратное чисел 10 и 15. Есть мно­
го других: 60, 90,120 и так далее. Можно было бы выбрать любое из
этих чисел и точно так же решить задачу. Но 30 меньше всех других
общих кратных и поэтому удобнее для расчётов. Самое малое из
всех общих кратных двух чисел называется их наименьшим общим
кратным (сокращённо — НОК).
Мы только что ввели два новых термина: общее кратное и наи­
меньшее общее кратное пары целых чисел. Эти два понятия ещё не
раз пригодятся нам. А сейчас решим с их помощью ещё одну похо­
жую задачу.
Пример 7. В прошлом году бабушка Этель за 3 недели связала
синие шарфики для всех своих внуков. В этом году она хочет пода­
рить им красные шарфики, и ей будет помогать её дочь Мэри. Если
бы Мэри работала одна, она связала бы полный набор шарфиков за
11 недель. В какой срок они закончат работу вдвоём? (Количество
бабушкиных внуков за год не изменилось.)
РАЗБОР ПРИМЕРА 7. Сначала найдём наименьшее общее кратное
чисел 3 и 11. Это 33. За 33 недели бабушка связала бы 11 полных на­
боров шарфиков, а Мэри — 3 набора. Значит, вместе они за 33 неде-
Занятие 5. Текстовые задачи и здравый смысл 57

ли связали бы 14 наборов. Следовательно, бабушка и Мэри свяжут


33
один набор за JT недели, т. е. за две недели и два с половиной дня.
На этом заканчивается обсуждение сегодняшней темы. Весь оста­
ток занятия участники решают задачи из новой подборки.

Подборка задач
Задача 1. а) Соедините четыре точки на левом рисунке тремя
прямыми линиями, не отрывая карандаш от бумаги и закончив
в той же точке, в которой вы начали.

б) Соедините девять точек на правом рисунке четырьмя прямы­


ми линиями, не отрывая карандаш от бумаги. На этот раз возвра­
щаться в исходную точку не обязательно.
Задача 2. У Ксюши и Макса
было одинаковое количество кон­
фет. Макс подарил Ксюше две свои
конфеты.
а) На сколько конфет теперь
у Ксюши больше, чем у Макса?
б) Если у Макса теперь в два
раза меньше конфет, чем у Ксю­
ши, сколько конфет было у каждо­
го из них вначале?
Задача 3. На следующий день после Праздника Конфет у Саши
и Гриши разболелся живот, и их привезли в больницу. Их мама ска­
зала доктору, что она нашла в комнате у мальчиков 141 фантик.
Врач говорит что каждому, кто съел больше 65 конфет, нужно де­
лать промывание желудка. Саша говорит, что ему не нужно промы­
вание, потому что он съел на 7 конфет меньше, чем Гриша. Сделают
ли ему промывание? А Грише?
Задача 4. На острове рыцарей и лжецов вам встретились пять
местных девочек: Алла, Белла, Вика, Глаша и Даша.
58 Раздел 1. Планы занятий

• Алла говорит, что если сложить два чётных числа, то обяза­


тельно получится чётное число.
• Белла утверждает, что можно найти два нечётных числа, сумма
которых — чётное число.
• Вика заявляет, что сумма двух чётных чисел не может быть
нечётной.
• Глаша объявляет, что все девочки в этой компании — рыцари.
• Даша сообщает, что если сложить два нечётных числа, то обя­
зательно получится нечётное число.
Определите, что из девочек рыцари и кто лжецы. Обоснуйте от­
вет.
Задача 5. Четыре голодных камнееда — Филимон, Парамон, Ка-
питон и Архип —устроили конкурс по поеданию камней. Филимон
съел на один камень меньше Парамона, Парамон — на один камень
меньше Капитона, а Капитон —на один камень меньше Архипа.
Сколько камней съел каждый камнеед, если известно, что вместе
они съели 2010 камней?
Задача 6. Чтобы приготовить все подарки к Новому Году, Деду
Морозу нужно 12 месяцев. Снегурочка готовила бы их 20 месяцев.
За сколько времени приготовит подарки Дед Мороз вместе со Сне­
гурочкой?
Задача 7. Три пони и один гном весят столько же, сколько два
пони и четыре гнома. Во сколько раз пони тяжелее гнома?
Задача 8. Лошадь съедает один тюк сена за 2 дня, корова — за
3 дня, а коза — за 6 дней. За сколько дней они съедят тюк сена втро­
ём?
Задача 9. В математическом кружке занимаются 25 детей. Сре­
ди любых 10 участников есть хотя бы одна девочка. Среди любых
17 участников есть хотя бы один мальчик. Сколько девочек и сколь­
ко мальчиков в этом кружке?
Задача 10. Бобриха Бренда нашл в лесу кучу брёвен, оставлен­
ных дровосеком, и решила сделать из них плотину. Но брёвна ока­
зались слишком длинными, и бобриха несколько дней распиливала
их на короткие куски. К тому моменту, когда длинных брёвен не
осталось, она сделал 83 распила. В итоге получилось 109 коротких
брёвен. Сколько брёвен бобриха нашла в лесу?
Занятие 6

Ещё текстовые задачи

Сегодня, как и на прошлом занятии, предлагаются задачи, ко­


торые удобнее решать без составления уравнений — путём простых
рассуждений или с помощью рисунков. Освоить этот нетрадицион­
ный метод будет полезно как начинающим, так и опытным участ­
никам математических кружков.
Для преподавателей. В книге эти два занятия следуют одно за другим,
но в кружке, участники которого уже умеют решать текстовые зада­
чи с помощью уравнений, лучше не проводить их подряд. Таким участ­
никам предлагаемый метод может показаться слишком примитивным,
несмотря на его реальные преимущества (см. с. 54). Поэтому материал
к этому занятию подобран так, чтобы его можно было проводить неза­
висимо от предыдущего.
Тем участникам, которые ещё не очень хорошо знакомы с уравнени­
ями, эта тема принесёт двойную пользу: дети не только освоят простой
способ решать текстовые задачи определённых типов, но и будут лучше
подготовлены к пониманию смысла уравнений. Поэтому в кружке для
начинающих не помешают даже два занятия по материалам этой главы.
А в кружке с высоким уровнем подготовки можно, наоборот, провести
одно занятие на основе занятий 5 и 6.

Математическая разминка
Задача для разминки 1. Шерлок Холмс и доктор Ватсон беседу­
ют у камина. Часы бьют полночь. За окном дождь.
— Какая скверная погода! —говорит Ватсон.— Как вы думаете,
Холмс, через 168 часов будет, наконец, солнечно?
— Это исключено, — отвечает Холмс.
Откуда у Холмса такая уверенность?
Задача для разминки 2. В магазине «Товары для дома» покупа­
тель спрашивает:
— Сколько стоит 1?
—10 рублей, — отвечает продавец.
Другой покупатель:
— Сколько стоит 10?
60 Раздел 1. Планы занятий

— 20 рублей.
Третий покупатель:
— Сколько стоит 100?
— 30 рублей.
Что они покупали, если речь идёт об одном и том же товаре?

Тема занятия:
«О поросятах и цыплятах»
Пример 1. Прасковья Терентьевна — большая любительница ко­
шек: их у неё 20, считая котят! Каждое воскресенье она выдаёт сво­
им любимцам 92 кошачьи конфеты. Каждая взрослая кошка полу­
чает 5 конфет, а каждый котёнок — 3 конфеты. Сколько у Прасковьи
Терентьевны взрослых кошек и сколько котят?

РАЗБОР ПРИМЕРА 1. Эту задачу можно решить с помощью двух


линейных уравнений, но в нашем кружке поощряются решения на
основе простых логических рассуждений. Исходя из условия зада­
чи мы пытаемся ответить на вопрос: что общего между кошками
и котятами и в чём различие между ними? Общее состоит в том,
что каждая взрослая кошка и каждый котёнок получают по три кон­
феты. Различие — в том, что каждая взрослая кошка получает две
дополнительные конфеты.
Будем считать, что Прасковья Терентьевна утром выдаёт всем
по три конфеты, а дополнительные конфеты для взрослых кошек
оставляет на вечер. Значит, утром она раздаёт 20 • 3 = 60 конфет,
а вечером 92 — 60 = 32 конфеты. Вечерние конфеты достаются толь­
ко взрослым кошкам, причём каждая из них получает по 2 конфеты.
Поэтому взрослых кошек у Прасковьи Терентьевны 32:2 = 16, а то­
гда котят 20 —16 = 4.
Занятие 6. Ещё текстовые задачи 61

Пример 2. Дедушка Захар разводит на своей ферме только по­


росят и цыплят. Всего на ферме 32 животных. Вместе у них 104 но­
ги. Сколько поросят и сколько цыплят живёт на ферме?
РАЗБОР ПРИМЕРА 2. И на этот раз мы обходимся без уравнений,
выявляя существенные для нашей задачи различия между порося­
тами и цыплятами. Такое различие одно: у поросёнка на две но­
ги больше, чем у цыплёнка. Вообразим на минуту, что у поросят
столько же ног, сколько у цыплят: представим себе, что они ходят
на задних ногах, так что передние можно не учитывать. Поскольку
всего поросят и цыплят 32, в этом случае у них вместе было бы
64 ноги.
На самом деле у них 104 ноги, т. е. на 40 ног больше, чем в нашей
воображаемой ситуации. Это различие в 40 ног — не что иное, как
количество передних ног у поросят, которые мы временно не учи­
тывали. У каждого поросёнка две передние ноги; значит, на ферме
20 поросят, а тогда цыплят 32 - 20 = 12.
Следующая задача похожа на две предыдущие, но немного слож­
нее.
Пример 3. В магазине волшебных пряников и конфет Миша
и Маша заметили, что пряник в два раза дороже конфеты. Миша ку­
пил шесть пряников и три конфеты, а Маша — три пряника и шесть
конфет. Миша заплатил на 18 рублей больше, чем Маша. Сколько
стоит пряник и сколько — конфета?
РАЗБОР ПРИМЕРА 3. Что общего между покупками Миши и Ма­
ши? В чём различие? Общая часть этих двух наборов — три пряника
и три конфеты. Различие — три пряника Миши и три конфеты Ма­
ши. Значит, 18 рублей —это разница между стоимостью трёх пря­
ников и трёх конфет.
Теперь найдём цену каждой сладости. Один пряник стоит столько
же, сколько две конфеты, значит, три пряника — столько же, сколько
шесть конфет. Поэтому разница в стоимости трёх пряников и трёх
конфет (18 рублей) —это стоимость трёх конфет. Следовательно,
конфета стоит 18:3 = 6 рублей, а пряник 12 рублей.
Следующие две задачи дети решают самостоятельно.
Пример 4. В зоопарк города Урюпинска поступила новая пар­
тия зебр и страусов. Всего у вновь прибывших 30 голов и 84 ноги.
Сколько зебр и сколько страусов прибыло в зоопарк?
62 Раздел 1. Планы занятий

Пример 5. На дне рождения 30 детей угостили мороженым.


Беспечные дети съели по 8 порций, а осторожные — по 3. Всего было
съедено 115 порций мороженого. На следующий день все осторожные
дети пришли в школу, а у всех беспечных разболелись животы, и они
остались дома. Сколько беспечных детей было на дне рождения?

Подборка задач
Задача 1. Настя хочет купить 9 шоколадок, но ей не хватает
20 рублей. Если бы она купила 5 шоколадок, у неё осталось бы
40 рублей. Сколько стоит шоколадка?
Задача 2. Сколько квадратов можно нарисовать
так, чтобы все их вершины были в отмеченных точ­
ках (см. рисунок)?
Задача 3. В школе волшебства состоялся экза­
мен по левитации. Мальчиков, сдавших экзамен,
оказалось столько же, сколько девочек, не сдавших его. Кого больше
в этой школе: девочек или сдавших экзамен по левитации?
Задача 4. В гонке участвуют две чере­
пахи. Первая черепаха за 9 часов пропол­
зает 4 метра, а вторая за 11 часов пропол­
зает 5 метров. Кто из них выиграет гонку?
Задача 5. В Стране драконов живут
двуглавые и трёхглавые драконы. У каж­
дого дракона один хвост. Гуляя по главной площади Драконограда,
Миша из математического кружка считал головы и хвосты драко­
нов. Он насчитал 38 хвостов и 93 головы. Сколько драконов каждого
вида было на площади?
Задача 6. В начальных классах математической академии учатся
30 рассеянных мальчиков и девочек. В октябре каждая рассеянная де­
вочка потеряла три кофточки, а каждый рассеянный мальчик— семь
свитеров. Всего за октябрь рассеянные дети потеряли 134 предмета
одежды. Сколько рассеянных девочек учится в академии?
Задача 7. Маша купила в зоомагазине шесть хорьков и двух по­
пугаев. На другой день её сестра Катя купила в том же магазине двух
хорьков и шесть попугаев. Катя заплатила на 4080 рублей больше,
чем Маша. Попугай в три раза дороже хорька. Сколько стоит хорёк
и сколько — попугай?
Занятие 6. Ещё текстовые задачи 63

Задача 8. Саша хотел сложить на калькуляторе два числа. Наби­


рая второе число, он случайно нажал в конце лишний нуль. Поэтому
вместо 2331 у него получилось 7641. Какие числа хотел сложить
Саша?
Задача 9. На секретном совещании мафии присутствовало пятна­
дцать крупных и мелких мафиози. Во время совещания они курили
сигары. Все мелкие мафиози выкурили одинаковое количество сигар,
а каждый крупный мафиози выкурил на четыре сигары больше, чем
каждый мелкий. После совещания в комнату, где оно проводилось,
проник секретный агент 00Х. Он собрал 99 сигарных окурков. Сколь­
ко крупных и сколько мелких мафиози было на совещании?

Д О П О Л Н И Т Е Л Ь Н Ы Е ЗАДАЧИ

Задача 10. Шайка из двадцати разбойников украла 109 золотых


монет. Старшие разбойники поделили добычу: всем другим членам
шайки дали по три монеты, а оставшиеся монеты поделили поровну
между собой. Сколько старших разбойников в этой шайке? Укажите
все возможные варианты ответа и объясните, почему нет других.
Для преподавателей. Три части следующей задачи —это по сути отдель­
ные задачи возрастающей сложности, объединённые общей идеей. Здесь
они для удобства приведены вместе, но с педагогической точки зрения
их лучше давать последовательно, на разных занятиях.
Задача 11*. а) Волшебник Макс укрепляет свой замок, имею­
щий форму квадрата с четырьмя башнями по углам. У Макса есть
21 талисман с волшебным свойством: если в двух соседних башнях
замка спрятать в сумме больше десяти талисманов, то стена, соеди­
няющая эти две башни, становится непробиваемой. Докажите, что,
как бы Макс ни спрятал все свои талисманы в башнях, непробивае­
мыми станут ровно две стены замка.
б) Волшебница Белла украшает свой замок, имеющий форму ше­
стиугольника с шестью башнями по углам. У Беллы есть 31 вол­
шебный фонарик для развешивания на башнях. Чтобы подсчитать
волшебную силу каждой башни, надо сложить число фонариков на
этой башне с суммарным числом фонариков на обеих соседних баш­
нях. Если волшебная сила башни больше 15, эта башня светится
волшебным светом. Докажите, что, как бы Белла ни развесила все
свои фонарики на башнях, светиться волшебным светом будут ров­
но три башни.
64 Раздел 1. Планы занятий

в) Новый замок волшебника Ильи имеет форму многоугольни­


ка с десятью башнями. У Ильи есть 101 волшебный амулет. Любые
пять расположенных подряд башен, в которых всего спрятано боль­
ше чем 50 амулетов, придают замку одну защитную силу. Докажите,
что, как бы Илья ни спрятал все свои амулеты в башнях, замок по­
лучит ровно пять защитных сил.
Занятие 7

Чётные и нечётные числа I.


Волшебные бумажные стаканчики

Мы начинаем изучение важной темы, к которой ещё не раз бу­


дем возвращаться, — чётных и нечётных чисел. Этот материал сам
по себе не очень сложен, но служит источником идей для решения
задач любой степени трудности. Теме чётности будет посвящено
три занятия — 7, 8 и 10.
Сегодняшнее занятие будет познавательным, творческим и увле­
кательным. Мы познакомимся с задачами нового типа. Они реша­
ются с помощью инвариантов (признаков, которые не изменяются
при заданных условиях). В данном случае инвариантом служит чёт­
ность (свойство числа быть чётным или нечётным).
Задача, предлагаемая на этом занятии, — не из тех, что решают­
ся без труда. И всё же я предпочитаю начинать тему чётности имен­
но с неё: она увлекательна и полностью завладевает вниманием де­
тей. Участники получают задание, которое на первый взгляд кажется
несложным, но почему-то оказывается невыполнимым. Сначала дети
усердно стараются выполнить задание, а потом пытаются понять за­
гадочную причину неудачи. Ключом к разгадке становится чётность.
К концу занятия дети переполнены новыми знаниями. Они по­
знакомились с некоторыми свойствами чётных и нечётных чисел
и увидели, как простая идея чётности помогает решить трудную за­
дачу. Они узнали новый, очень действенный способ решения задач.

Что принести на урок


• Спички для одной из разминочных задач.
• Бумажные стаканчики для основной темы занятия.

Математическая разминка
Задача для разминки 1. В прямоугольной комнате стоит не­
сколько стульев.
66 Раздел 1. Планы занятий

Можно ли расставить четыре стула так, чтобы у каждой стены


стоял стул?
Можно ли сделать то же самое с тремя стульями?
Можно ли расставить 12 стульев так, чтобы у каждой стены сто­
яло 4 стула?
Задача для разминки 2. Участникам раздаётся по три спички.
Надо расположить эти три спички на столе так, чтобы головки спи­
чек не касались стола.
Для преподавателей. Не стоит предлагать вместо спичек авторучки или
карандаши. Они круглые и будут раскатываться.
Задача для разминки 3. В стране три города: А, В и С. Жите­
ли города А никогда не лгут, жители города В лгут всегда, а жите­
ли города С лгут через раз. Однажды кто-то позвонил на пожарную
станцию:
— В нашем городе пожар!
— Где вы живёте?—уточнил дежурный.
— В городе В, — был ответ.
В какой город должны ехать пожарные?

Тема занятия:
«Чётные и нечётные числа»
Обсуждение начинается с игры. Участники разбиваются на пары
или тройки, и каждая группа получает семь бумажных стаканчиков.
(От пластиковых стаканчиков, как показывает опыт, слишком мно­
го шума.)
Я предлагаю каждой группе поставить стаканчики на стол, как по­
казано на рисунке: четыре вверх дном и три вниз дном. В каком по­
рядке стоят стаканчики — на самом деле неважно, но во избежание
ненужных вопросов я прошу расположить их точно как на рисунке.

Мы будем переворачивать стаканчики по два. За каждый ход иг­


ры надо выбрать два стаканчика и перевернуть их одновременно.
• Если оба стаканчика стояли вверх дном, они будут стоять вниз
дном.
• Если оба стаканчика стояли вниз дном, они будут стоять вверх
дном.
Занятие 7. Чётные и нечётные числа I 67

• Если стаканчики стояли в разных положениях, тот из них, ко­


торый стоял вверх дном, будет стоять вниз дном, и наоборот.
Правилами игры запрещается переворачивать стаканчики по од­
ному, а также одновременно переворачивать больше двух.
Цель игры — перевести все стаканчики в положение вверх дном.
Дети работают над задачей, увлечённо обсуждая варианты оче­
редного хода. Но через некоторое время начинают раздаваться голо­
са, что перевернуть все стаканчики вверх дном невозможно. Выбрав
подходящий момент, я прошу объяснить, почему эта задача невы­
полнима. Дети в затруднении. Тогда я предлагаю после каждого хо­
да записывать, сколько стаканчиков стоит вверх дном.
Вскоре то в одной, то в другой группе замечают закономерность:
вверх дном всегда стоит 0, 2, 4 или 6 стаканчиков.
Участники заявляют, что объяснение найдено: раз в этом списке
нет числа 7, все семь стаканчиков не могут оказаться вверх дном.
Но утверждение, что вверх дном может стоять только 0, 2, 4 или
6 стаканчиков, — это всего лишь гипотеза, выдвинутая из наблюде­
ний. Вдруг кто-нибудь изобретёт хитрую последовательность ходов,
до которой мы до сих пор не додумались, и поставит вверх дном дру­
гое количество стаканчиков? Надо не просто заметить, а доказать,
что вверх дном всегда будет стоять 0, 2, 4 или 6 стаканчиков. Только
тогда мы можем быть уверены, что перевернуть все 7 стаканчиков
вверх дном невозможно.
Я подсказываю, что доказать гипотезу будет проще, если сначала
обобщить её. Какое общее свойство у чисел 0, 2,4, 6? Да, все эти
числа чётные.
Теперь осталось объяснить, почему после любой последователь­
ности ходов количество стаканчиков, стоящих вверх дном, обяза­
тельно останется чётным.
Сначала выясним, как может измениться количество стоящих
вверх дном стаканчиков за один ход. Когда мы выбираем пару ста­
канчиков, у нас есть три варианта: оба стаканчика стоят вверх дном,
оба стаканчика стоят вниз дном и стаканчики стоят по-разному.
• Если перевернуть два стаканчика, стоящих вверх дном, то ко­
личество таких стаканчиков уменьшится на два.
• Если перевернуть два стаканчика, стоящих вниз дном, то ко­
личество стаканчиков, стоящих вверх дном, увеличится на два.
• Если перевернуть два стаканчика, стоящих по-разному, то ко­
личество стаканчиков, стоящих вверх дном, не изменится.
68 Раздел 1. Планы занятий

Итак, после каждого хода количество стаканчиков, стоящих вверх


дном, либо остаётся прежним, либо уменьшается на два, либо увели­
чивается на два.
Давайте последим за количеством стаканчиков, стоящих вверх
дном. Сначала таких стаканчиков было 4, а 4 —чётное число. Если
прибавить к чётному числу 2 или вычесть из него 2, опять получится
чётное число. Поэтому, какую бы пару мы ни выбрали на первом
ходе, количество стоящих вверх дном стаканчиков останется чёт­
ным. Это же относится и ко второму ходу: изменив чётное число
на 2, мы опять получим чётное число. Рассуждая точно так же для
каждого хода, приходим к выводу: количество стоящих вверх дном
стаканчиков всегда остаётся чётным.
Теперь наша гипотеза доказана: количество стаканчиков, стоя­
щих вверх дном, обязательно останется чётным, как бы мы ни пе­
реворачивали их. Теперь мы точно знаем, что таких стаканчиков
всегда будет 0, 2, 4 или 6. Значит, мы никогда не поставим вверх
дном 7 стаканчиков.
Для преподавателей. Это трудная задача, в которой промежуточные выво­
ды служат основой для дальнейших рассуждений. Доказательство опира­
ется на несколько идей, новых для большинства участников. Потому в кон­
це разбора желательно подвести итог, повторив важнейшие этапы доказа­
тельства. Ещё лучше сформулировать эти этапы с помощью участников.

Основные этапы
1. Замечаем, что количество стоящих вверх дном стаканчиков
остаётся чётным. Выдвигаем гипотезу: количество стоящих вверх
дном стаканчиков всегда будет чётным.
2. Доказываем, что при каждом переворачивании количество
стоящих вверх дном стаканчиков либо остаётся прежним, либо из­
меняется на 2.
3. Замечаем, что в начале игры вверх дном стоит чётное число
стаканчиков. Поскольку на каждом ходе количество таких стакан­
чиков может увеличиться или уменьшиться только на 2, оно всегда
остаётся чётным.
4. Делаем вывод: все семь стаканчиков невозможно поставить
вверх дном, так как 7 — нечётное число.
Чтобы закрепить этот метод, в подборки для сегодняшнего и сле­
дующего занятия включены похожие задачи. Скорее всего, некото­
рым участникам будет трудно воспроизвести весь ход рассуждений
Занятие 7. Чётные и нечётные числа I 69

самостоятельно. Поэтому аналогичная задача из сегодняшней под­


борки разбита на части, соответствующие основным этапам дока­
зательства.

Подборка задач
Задача 1. Как разделить круглую пиццу на
4, 5, 6 и 7 частей тремя прямыми разрезами?
Каждый разрез должен начинаться и заканчи­
ваться на наружном крае пиццы.
Задача 2. Юный хакер Вася перепрограм­
мировал лифт в 100-этажном здании Института создания и решения
проблем. Теперь в лифте работают только две кнопки: одна отправ­
ляет его на 8 этажей вверх, а другая — на 6 этажей вниз. (Если кноп­
ка посылает лифт выше сотого или ниже первого этажа, он остаётся
на месте.)
а) Пока Вася работал над кнопками, директор института пил ко­
фе на первом этаже. Сможет ли он вернуться в свой кабинет на
95-м этаже, пользуясь только лифтом? Если да, то как? Если нет, то
почему?
б) Тот же вопрос, если кабинет директора находится на 96-м этаже.
Задача 3. Юля возвращается домой из школы. Она выходит на 5
минут раньше Пети, который живёт в том же доме. Петя идёт в пол­
тора раза быстрее Юли: он хочет вернуть ей мобильный телефон,
который она забыла в школе. Когда Юля получит свой телефон? (Так
как Юля и Петя живут далеко от школы, Петя успеет догнать Юлю,
прежде чем она дойдёт до дома.)
Задача 4. В Лукоморье живут 9 счастливых и
9 несчастных принцесс. Волшебник Шмерлин только
что выучил три новых заклинания. С помощью пер­
вого он может сделать любых двух несчастных прин­
цесс (по своему выбору) счастливыми. С помощью
второго — сделать любых двух счастливых принцесс
несчастными. Третье заклинание позволяет ему сде­
лать одну несчастную принцессу счастливой, но одна
счастливая принцесса при этом станет несчастной.
Шмерлин хочет сделать всех принцесс счастливы­
ми. Докажите, что трёх его заклинаний для этого
недостаточно.
70 Раздел 1. Планы занятий

а) Как влияет каждое заклинание Шмерлина на количество не­


счастных принцесс?
б) Сейчас несчастных принцесс нечётное число. Шмерлин произ­
носит одно из своих заклинаний. Докажите, что количество несчаст­
ных принцесс остаётся нечётным.
в) Шмерлин произносит несколько заклинаний подряд. Докажи­
те, что количество несчастных принцесс остаётся нечётным.
г) Может ли количество несчастных принцесс стать равным ну­
лю? Если да, то как? Если нет, то почему?
Задача 5. На волшебном дереве в центре тридесятого королев­
ства растут волшебные плоды трёх видов: яблоки мудрости, груши
храбрости и сливы доброты. Время от времени с этого дерева со­
бирают урожай для блага королевства. На месте собранных плодов
сразу вырастают новые.
• Если снять с дерева один плод, то на его месте вырастает плод
того же вида.
• Если одновременно снять 2 яблока, то вырастают 4 груши.
• Если одновременно снять 2 груши, то вырастают 4 сливы.
• Если одновременно снять 2 сливы, то вырастают 4 яблока.
• Если одновременно снять 2 плода разных видов, то ничего не
вырастает.
Сейчас на дереве 11 яблок, 10 груш и 8 слив. Злая колдунья хочет
лишить королевство силы, украв все волшебные плоды. Она соби­
рается каждое утро забираться на дерево и срывать с него один или
два плода. Может ли она оставить дерево без плодов? Если да, то
как? Если нет, то почему?

ДОПОЛНИТЕЛЬНЫЕ ЗАДАЧИ

Задача 6. В те дни, когда Робин-Бобин-Барабек ходит в школу,


он съедает по два пирога на каждой перемене. В прошлом году Ро­
бин побывал в школе на 499 уроках и съел 798 пирогов. Сколько
дней в прошлом году Робин провёл в школе? (Количество уроков
может меняться день ото дня. Например, может быть день с одним
уроком, а может быть день с 7 уроками.)
Задача 7*. Коля выложил в ряд полный набор домино (28 костя­
шек) по правилам игры в домино (см. ниже). На левой половинке
самой левой костяшки 6 точек. Сколько точек на правой половинке
самой правой костяшки?
Занятие 7. Чётные и нечётные числа I 71

Справка. Костяшка домино — это прямоугольная плитка, разде­


лённая на два квадратика. На каждом квадратике отмечено несколь­
ко точек —от 0 до 6. Полный набор состоит из 28 костяшек со все­
ми возможными сочетаниями точек: [0—0], [0—1], [0—2], [0—3],
[0—4], [0—5], [0—6], [1—1], [1—2], ..., [6—6]. Если половинки раз­
ных костяшек соприкасаются, по правилам игры на этих половин­
ках должно быть одинаковое количество точек.
Занятие 8

Чётные и нечётные числа П.


Определения и свойства

В прошлый раз мы затронули понятие чётных и нечётных чисел.


Сегодня мы рассмотрим их подробнее: сформулируем разные опре­
деления таких чисел, узнаем несколько полезных свойств и научим­
ся применять их при решении задач.

Математическая разминка
Задача для разминки 1. Вам нужно заказать форму для выпеч­
ки торта. Это индивидуальный заказ, и вы можете выбрать любую,
даже самую необычную форму.
а) Придумайте такую форму, чтобы торт можно было разрезать
на четыре части (не обязательно одинаковые) одним прямым раз­
резом ножа.
б) Возможно ли это, если части должны быть одинаковыми?
Задача для разминки 2. Два отца и два сына позавтракали яй­
цами. У них было всего три яйца, но каждый съел по целому яйцу.
Как такое могло случиться?

Тема занятия:
«Как определить чётные и нечётные числа»
Сначала я предлагаю участникам дать определение чётных чи­
сел. Дети в затруднении: им не очень понятно, что от них требу­
ется. Тогда я задаю тот же вопрос менее формально: «Как бы вы
объяснили другу, что такое чётное число?» Начинается оживлённое
обсуждение.
• Брейди: «Число, оканчивающееся на 0, 2,4, 6, 8».
• Тара: «Число, которое делится на 2».
Занятие 8. Чётные и нечётные числа II 73

Эмма сразу реагирует на предложение Тары: «Надо, чтобы при


делении получилось целое число. Иначе такое объяснение будет
неправильным. Например, 7 можно разделить на 2, но это нечётное
число».
Желательно, чтобы дети сформулировали более наглядные опре­
деления. Поэтому я предлагаю представить себе такую ситуацию:
нужно объяснить первокласснику, что такое чётные числа.
В результате появляются ещё два определения.
• Число, которое можно разделить на две равные части без ос­
татка.
• Число, которое можно разбить на пары без остатка.
Участники выдвигают идеи, а я записываю их на доске. Когда по­
ток предложений иссякает, я спрашиваю: «Теперь у нас есть несколь­
ко определений, описывающих одно и то же понятие. Какое из них
лучше? Каким мы будем пользоваться?» Как и следовало ожидать,
дети озадачены и не знают, что ответить.
Я отвечаю на собственный вопрос: все предложенные описания
чётных чисел совершенно верные, и нельзя сказать, что какое-то
из них лучше остальных. Для разных задач, в которых встречаются
чётные и нечётные числа, более удобными могут оказаться разные
определения.
Пожалуй, самые простые определения — «число, которое можно
разделить на две равные части» и «число, которое можно разбить на
пары без остатка». Решая задачи и изучая свойства чётных и нечёт­
ных чисел, мы чаще всего будем пользоваться этими определения­
ми. Но и остальные нам пригодятся.
Мы переходим к обсуждению нечётных чисел. Участники напере­
бой предлагают определения нечётного числа. На доске появляется
новый список. Хотя он очень похож на предыдущий, я приведу и его.
• Число, оканчивающееся цифрой 1, 3, 5, 7 или 9.
• Число, которое не делится на 2.
• Число, которое можно разделить на две равные части с остат­
ком 1.
• Число, которое можно разбить на пары с остатком 1.

Для преподавателей. Характерно, что участники придумывали ответы на


этот вопрос с не меньшим воодушевлением, чем на предыдущий. Это
ещё раз подтверждает, что предлагать сходные задачи — эффективный
74 Раздел 1. Планы занятий

педагогический приём. Детям очень нравится заново открывать только


что усвоенные идеи. Для них это ещё одно достижение.
Для преподавателей. Если в кружке есть более продвинутые школьни­
ки, то можно немного поговорить о буквенном обозначении чётных
и нечётных чисел в математике. Здесь я излагаю обсуждение этой темы
в нашем кружке.
Сначала я спрашиваю, как обозначить неизвестное целое число.
Большинство участников уже знакомо с понятием переменной. Поэто­
му они предлагают буквенные обозначения (например, хиу). Я согла­
шаюсь и добавляю, что неизвестные целые числа традиционно принято
обозначать такими буквами, как п, т и к.
Затем я задаю ещё один вопрос: как обозначить неизвестное целое
число, чтобы было видно, что оно чётное? Дети затрудняются ответить.
Тогда я напоминаю, что любое чётное число — это произведение числа 2
и какого-то целого числа. (Ведь все чётные числа делятся на 2.) Это на­
водит на мысль, что неизвестное четное число п можно записать в виде
2 • к, где к — другое неизвестное целое число.
Аналогично неизвестное нечётное число п можно записать в виде
2-fc + l.
2-к и 2 • /с + 1 — стандартные обозначения чётных и нечётных чисел
в математике. Но для самых младших из наших кружковцев они слиш­
ком абстрактны, поэтому в течение первого года обучения мы не будем
активно пользоваться ими. Более продвинутые кружковцы уловили идею,
а её сложность вызвала у них дополнительный интерес. Позднее, когда мы
вспоминали способы описания чётных и нечётных чисел, кто-то предло­
жил вариант: «Числа, которые можно записать как 2 • к и 2 • к +1».
В конце обсуждения мы вводим удобное определение, которое поз­
волит нам говорить о чётных и нечётных числах короткими предложе­
ниями.
Определение. Чётность — это свойство числа быть чётным или
нечётным 1 .
Например, высказывание «Если количество перевёрнутых ста­
канчиков было чётным, оно остаётся чётным, а если было нечёт­
ным — остаётся нечётным» теперь можно заменить более коротким:
«Чётность количества перевёрнутых стаканчиков не изменяется».
1
Примечание для русского издания. В английском языке термин «четность» обо­
значается редко употребляемым словом «parity», которое не родственно словам «чет­
ный» и «нечетный» («odd» и «even»). Термин этот в школе практически не употребля­
ется, и он гораздо менее интуитивен, чем его русский аналог. Поэтому англоязыч­
ным детям гораздо сложнее освоить употребление термина «parity», чем русскоязыч­
ным — слова «четность».
Занятие 8. Чётные и нечётные числа II 75

Чётность суммы и разности


Затем мы обсуждаем основные свойства сложения и вычитания
чётных и нечётных чисел.
чётное + чётное = чётное
нечётное + нечётное = чётное
нечётное + чётное = нечётное
чётное — чётное = чётное
нечётное — нечётное = чётное
нечётное — чётное = нечётное
чётное — нечётное = нечётное
Большая часть этих свойств уже знакома участникам: они встре­
чались в задачах из прошлой подборки. Но тогда мы не пытались
объяснить их: они были просто наблюдениями.
Теперь мы изучаем чётные и нечётные числа, поэтому не можем
обойтись без объяснений. Хотя интуитивно эти свойства очевидны,
их разбор послужит отличной практикой строгих математических
рассуждений.
Я прошу участников объяснить, почему при сложении двух чёт­
ных чисел обязательно получается чётное число. Кто-то предлага­
ет проверить все возможные суммы последней цифры первого чис­
ла (0,2,4,6, 8) и последней цифры второго (0,2, 4,6, 8) и убедить­
ся, что каждый раз получается число с последней цифрой из этого
же набора. Я отвечаю, что такое объяснение будет правильным, но
потребует долгого и утомительного перебора вариантов. Поэтому
стоит вспомнить, какие ещё у нас есть определения чётных чисел.
Может быть, какое-то из этих определений поможет получить более
простое доказательство.
Мы вместе выбираем самое подходящее определение: «Чётное
число можно разбить на пары без остатка». Получается такое объ­
яснение: каждое из двух чётных чисел можно без остатка разбить
на несколько пар. Следовательно, и их сумма будет состоять из пар.
Значит, эта сумма чётная. Как обычно, хороший рисунок стоит ты­
сячи слов. Вот что я рисую на доске:

четное четное четное


76 Раздел 1. Планы занятий

Теперь наша задача—доказать, что сумма двух нечётных чи­


сел всегда нечётная. Дети выбирают определение: «Нечётное число
можно разбить на пары с остатком 1». Если сложить два нечётных
числа, то два остатка 1 можно объединить в пару. Поэтому сумма
будет чётной:

нечётное нечётное чётное


Чтобы попрактиковаться ещё, я предлагаю объяснить, почему
сумма чётного и нечётного чисел будет нечётной.
Затем мы обсуждаем свойства разности двух чисел. Я спраши­
ваю, как доказать первое правило вычитания (чётное — чётное =
= чётное). Брейди предлагает воспользоваться тем же способом,
что и для сложения. Я отвечаю, что есть другой путь: логически
вывести правила вычитания из правил сложения. Предположим,
что разность двух чётных чисел может быть нечётной. Перенося
вычитаемое в другую часть равенства, получаем, что чётное число
равно сумме нечётного и чётного. Но такая сумма должна быть
нечётной по правилу
нечётное + чётное = нечётное.
Значит, разность двух чётных чисел обязательно чётная. Таким же
образом правило
нечётное — нечётное = чётное
выводится из правила
нечётное + нечётное = чётное,
а правило
чётное — нечётное = нечётное
— из правила
чётное + нечётное = нечётное.
Следующая часть занятия посвящена сумме и разности несколь­
ких чисел. Здесь я опускаю подробности обсуждения и ограничива­
юсь выводами.
Занятие 8. Чётные и нечётные числа II 77

— Сумма нескольких чётных чисел всегда чётная. Например,


18 + 6 + 22 = 46 —чётное число, как и 18 + 6 + 22 + 4 = 50.
Это свойство нетрудно объяснить, опираясь на уже известный
факт: сумма двух чётных чисел чётная. Начнём постепенно сводить
сумму нескольких чётных чисел к сумме двух чётных чисел. Для это­
го повторим несколько раз одно и то же действие: заменим первые
два слагаемых их суммой. При этом количество слагаемых умень­
шится на 1 и все слагаемые останутся чётными. В конце концов
слагаемых останется только два. Значит, сумма чётная. Например,
(18 + 6)+22 + 4 = 24 + 22 + 4 = ( 2 2 + 24)+4 = 46 + 4 = 50.
— Если в сумме нескольких чётных чисел некоторые знаки «плюс»
заменить знаками «минус», результат останется чётным.
Это свойство можно объяснить тем же способом, что и в случае
суммы нескольких чётных чисел. Будем постепенно сводить выра­
жение к сумме (или разности) двух чётных чисел. На каждом шаге
мы пользуемся одним из двух свойств: сумма двух чётных чисел чёт­
ная; разность двух чётных чисел чётная. Например,

1 8 - 6 + 2 0 - 4 + 8 = ( 1 8 - 6 ) + 2 0 - 4 + 8 = 12 + 2 0 - 4 + 8 =
= (12 + 2 0 ) - 4 + 8 = 3 2 - 4 + 8 = ( 3 2 - 4 ) + 8 = 28 + 8 = 36.
— Чётность суммы нескольких нечётных чисел зависит от коли­
чества слагаемых. В этом случае чётность чередуется: сумма нечёт­
ного числа слагаемых нечётная, а сумма чётного числа слагаемых
чётная.
Это правило не так очевидно. Поэтому прежде чем сформулиро­
вать его, мы рассматриваем несколько примеров: вычисляем сум­
му трёх нечётных слагаемых (5 + 3 + 11 = 19 —нечётная), четырёх
нечётных слагаемых (5 + 3 + 11 + 7 = 26 —чётная), пяти нечётных
слагаемых (5 + 3 + 11 + 7 + 9 = 35 — нечётная).
Затем переходим к обоснованию. Сначала объясняем, почему
сумма трёх нечётных чисел нечётная. Затем доказываем, что сумма
четырёх нечётных чисел чётная, и так далее. Переходя к новой сум­
ме, мы каждый раз определяем её чётность на основе предыдущего
результата. Например, нам уже известно, что сумма двух нечётных
чисел чётная. Поэтому сумму трёх нечётных чисел можно пред­
ставить как сумму чётного числа, которое получается сложением
первых двух слагаемых, и нечётного числа (третьего слагаемого).
78 Раздел 1. Планы занятий

Значит, такая сумма нечётная. Например,


5 + 3 + 11 = (5 + 3)+ 11 = 8 +11 = 19.
Теперь участники готовы сами сформулировать правило: сумма
чётного числа нечётных слагаемых чётная; сумма нечётного числа
нечётных слагаемых нечётная.
— Если в сумме нескольких чётных чисел некоторые знаки «плюс»
заменить знаками «минус», чётность результата не изменится. Та­
кое выражение будет нечётным, если в нём нечётное количество
членов, и чётным, если их чётное количество.
Например, выражения 5 + 3 + 11 + 7 + 9 и 5 - 3 + 1 1 - 7 - 9 име­
ют одинаковую чётность.
Теперь решим несколько задач с помощью чётности.
Пример 1. Два горных тролля Боб и Тоб родились с интервалом
ровно в один год. Сегодня они оба празднуют день рождения. Боб
заявляет, что им в сумме 1128 лет. Но хоббит Филипп уверен, что
Боб ошибается. На чём основана его уверенность?
РАЗБОР ПРИМЕРА 1. Хотя обосновать ответ можно по-разному,
предложите участникам придумать объяснение на основе чётных
и нечётных чисел. Предполагается примерно такое рассуждение: из
двух последовательных чисел одно всегда чётное, а другое — нечёт­
ное. Поэтому сумма двух последовательных чисел всегда нечётная.
Следовательно, такая сумма не может быть равна 1128.
Для преподавателей. Вероятно, потебуется пояснить термин «последова­
тельные числа»1. Дети склонны понимать эти слова как «написанные од­
но за другим», и в подходящем контексте такое значение может оказать­
ся правильным! Но в большинстве случаев, когда у нас нет записанных
подряд чисел, этот термин означает «соседние числа в ряду натуральных
чисел». Например, 3 и 4 —два последовательных числа, а 156, 157, 158
и 159 —четыре последовательных числа.
Пример 2. В Стране сладостей король Пончик обещал достой­
ную награду первому, кто сможет разделить 100 конфет между тре­
мя детьми. Каждый ребёнок должен получить нечётное число кон­
фет (не обязательно одинаковое), и при этом не должно остаться
лишних конфет. Награду до сих пор никто не получил. Объясните
почему.
1
Примечание для русского издания. В английском языке термин «последователь­
ные» обозначается словом «consecutive», которое гораздо менее интуитивно для де­
тей, чем его русский аналог.
Занятие 8. Чётные и нечётные числа II 79

РАЗБОР ПРИМЕРА 2. Сумма трёх нечётных чисел всегда нечётная.


Поэтому если каждый ребёнок получает нечётное число леденцов,
то общее количество леденцов тоже нечётное. А100 — чётное число.
Пример 3. На острове рыцарей и лжецов вы встречаете местно­
го жителя. Он говорит: «Вчера у меня были гости, и я испёк 39 плю­
шек. Каждый взрослый съел 2 плюшки, а каждый ребёнок—4 плюш­
ки. Плюшки всем очень понравились — ни одной не осталось!» Кого
вы встретили — рыцаря или лжеца?
РАЗБОР ПРИМЕРА 3. Каждый съел чётное число плюшек (2 или 4).
Сумма любого количества чётных чисел чётная. Но 39 — нечётное
число. Значит, вам встретился лжец.
Пример 4. Баба Яга пригласила на день рож­
дения одинаковое количество леших и водяных.
Она поймала 115 живых лягушек для раздачи го­
стям. Баба Яга хочет разделить лягушек между
гостями поровну. Но мудрая сова говорит, что
так не получится. Откуда сова это знает?
РАЗБОР ПРИМЕРА 4. Поскольку Баба Яга при­
гласила одинаковое количество леших и водяных,
общее число гостей чётное. Если каждый гость по­
лучит чётное число лягушек, то общее количество лягушек должно
быть чётным (сумма любого количества чётных чисел —чётное чис­
ло). Если каждому достанется нечётное число лягушек, то общее
количество лягушек всё равно чётное (сумма чётного количества
чётных чисел — нечётное число). Но 115 — нечётное число.

Чётность произведения
Для преподавателей. Если дети ещё не устали от избытка информации,
в заключение можно обсудить свойства умножения чётных и нечётных
чисел. Если же сил на восприятие этого материала уже не осталось, лучше
отложить его до другого раза. Можно, например, включить доказатель­
ство свойств умножения в какую-нибудь подборку задач или выделить
время для этой темы на следующем занятии, посвященном чётности.
Чётность произведения определяется по таким правилам:
чётное • чётное = чётное
нечётное • чётное = чётное
нечётное • нечётное = нечётное
80 Раздел 1. Планы занятий

Чтобы объяснить правила чётности произведения, проще всего


свести умножение к сложению. Это позволяет доказать правила чёт­
ности произведения, опираясь на уже известные свойства чётности
суммы.
Следующая задача иллюстрирует применение правил чётности
произведения.
Пример 5. На острове рыцарей и лжецов Макс из математиче­
ского кружка встречает местного жителя.
— Сколько тебе лет? — спрашивает Макс.
— Если умножить мой возраст на 18, получится 1421.
Макс сразу догадался, что перед ним лжец. Почему?
РАЗБОР ПРИМЕРА 5. Возраст жителя острова умножается на чётное
число 18. Значит, результат умножения должен быть чётным. Но
1421 —нечётное число. Следовательно, островитянин—лжец.

Подборка задач
Задача 1. Надо разрезать ножом бухан­
ку хлеба. Какое наибольшее количество кус­
ков можно получить, если разрешается сделать
только три прямых разреза? Каждый разрез
должен проходить через всю буханку. Перекла­
дывать куски и класть их друг на друга нельзя.
Задача 2. Петя говорит: «Позавчера мне было 10 лет, а в буду­
щем году будет 13». Как такое может быть, если известно, что Петя
говорит правду?
Задача 3. Можно ли замостить шахмат­
ную доску размерами 5 x 5 костяшками до­
мино? (Замостить — значит покрыть все по­
ля так, чтобы костяшки не перекрывались.)
Задача 4. Капитан Кук ведёт дневник
своих приключений. Во время путешествия
к островам Кука он сделал такую запись: «В нашей флотилии пять
кораблей. На каждом корабле нечётное число матросов. Всего в на­
шей экспедиции 500 матросов». Профессор истории Гоббс утвер­
ждает, что капитан ошибся. Как он это узнал?
Задача 5. Утром на космодроме планеты Пандора было 5 кос­
мических кораблей. В течение дня припандорилось ещё несколько
Занятие 8. Чётные и нечётные числа II 81

кораблей, а некоторые корабли уле­


тели. Космические корабли прибыва­
ли только парами или четвёрками,
а отправлялись только парами. Вече­
ром диспетчер насчитал на космодро­
ме 60 кораблей. Докажите, что дис­
петчер ошибся.
Задача 6. Волшебник Шмерлин получил в наследство старинную
книгу заклинаний. Переплёт этой книги сделан из мягкой кожи, а но­
мера страниц оттиснуты золотом. К несчастью, когда Шмерлин от­
крыл книгу, из неё выпало несколько листов. Собрав все 25 выпав­
ших листов, Шмерлин решил потренироваться в математическом
волшебстве. Он придумал заклинание, с помощью которого мож­
но мгновенно сложить номера страниц на всех выпавших листах.
Произнеся это заклинание, волшебник получил ответ: 2000. Старая
мудрая сова Шмерлина тут же заявила, что заклинание не работает.
Как она это поняла?
Задача 7. Во время летних каникул Белла и Таня часто приходи­
ли другу в гости поиграть. Приходя к Тане, Белла каждый раз дари­
ла ей стеклянный шарик. Таня, приходя к Белле, тоже каждый раз
дарила ей стеклянный шарик. В начале лета у Беллы было 50 стек­
лянных шариков. Всего девочки играли вместе 35 раз. Могло ли слу­
читься так, что в конце лета у Беллы снова оказалось 50 стеклянных
шариков? (Никто, кроме Тани, не дарил Белле шариков, и ни один
шарик не потерялся.)

Д О П О Л Н И Т Е Л Ь Н Ы Е ЗАДАЧИ

Задача 8. В копилке у Макса чётное число двухрублёвых и нечёт­


ное число пятирублёвых монет. Когда Макс решил подсчитать своё
богатство, у него получилось 60 рублей. Мама Макса уверена, что
он ошибся в расчётах. Почему?
Задача 9*. На доске в ряд написаны целые числа от 1 до 18.
Можно ли расставить между ними знаки «плюс» и «минус» так, что­
бы полученное выражение было равно нулю?
Занятие 9

Математический хоккей I

Мы уже два занятия подряд изучаем чётные и нечётные чис­


ла, и планируется ещё одно занятие по этой теме. Для того чтобы
немного переключиться, сегодня для участников кружка проводит­
ся новая игра —математический хоккей.

Математическая разминка
Задача для разминки 1. Серёжа выходит из библиотеки и едет
домой на велосипеде со скоростью 10 километров в час. Одновре­
менно его младший брат Дима выходит из дома и идёт в библиоте­
ку пешком со скоростью 4,5 километра в час. Кто из братьев будет
ближе в дому в тот момент, когда они встретятся?
Задача для разминки 2. Четырёхугольник —
это многоугольник с четырьмя сторонами. На
рисунке четырёхугольник (в данном случае это
квадрат) разделён прямой линией на два тре­
угольника. Существует ли четырёхугольник, ко­
торый можно разделить прямой линией на три
треугольника?

Событие дня:
игра «Математический хоккей»
В этой игре две команды соревнуются в решении задач на ско­
рость. В каждом раунде предлагается одна задача, которую решают
представители обеих команд. Команда, представитель которой ре­
шил задачу правильно, получает очко; в случае неправильного отве-
Занятие 9. Математический хоккей I 83

та очко присуждается другой команде. Счёт отмечается положением


шайбы на хоккейном катке, нарисованном на доске. Когда команда
зарабатывает очко, шайба перемещается в следующую зону —бли­
же к воротам команды-соперника.
Подробное изложение правил игры начинается на с. 238. Обра­
тите особое внимание на раздел «Советы преподавателю».
Преобладающая тематика задач — ведьмы, волшебники и вы­
прашивание сладостей — связана с тем, что это занятие проводится
в октябре, когда дети ждут праздника Хэллоуин.

Подготовка к игре
Готовясь к проведению математического хоккея, важно проду­
мать организационные вопросы: как показывать участникам усло­
вия задач и как выделить место для их решения. Чтобы игра бы­
ла честной, надо сделать так, чтобы представители обеих команд
могли прочитать задачу одновременно. Условия задач должны быть
видны и остальным членам команд. Советы по организации игры
приведены на с. 239.
У нас небольшое помещение и всего одна доска. Поэтому я при­
ношу заранее распечатанные крупным шрифтом задачи — по одной
на листе. Листы с задачами будут прикрепляться на доске так, чтобы
их могли видеть все участники.
Перед началом соревнования я делю доску пополам, выделяя ра­
бочее место для каждой команды. У верхнего края доски я рисую
посередине небольшой каток. Листы с условиями задач по очереди
прикрепляются под рисунком — поверх предыдущих листов.

Планирование времени
Сегодня на математическом хоккее будет предложено 16 задач.
(Ответы к задачам приведены на с. 288.)
Если (оптимистично) отвести на каждую задачу по три минуты,
игра должна занять примерно 45 минут. В этом случае до начала
игры будет достаточно времени, чтобы разобрать задачи из преды­
дущей подборки.
Для преподавателей. Чтобы придать соревнованию ещё больше азарта,
можно заменить вымышленные имена детей в условиях задач именами
реальных участников кружка.
84 Раздел 1. Планы занятий

Задачи для математического хоккея I


Задача 1. Три ведьмы — Гоусти, Вики и Слайми — родные сест­
ры. Их возраст: Гоусти —97 лет, Вики —105 лет, Слайми —115 лет.
Ведьма Стинки — их двоюродная сестра. Стинки и одна из трёх се­
стёр родились с разницей в 10 лет. Стинки и ещё одна сестра ро­
дились с разницей в 8 лет. Стинки и оставшаяся сестра родились
с разницей в 2 года. Сколько лет Стинки?
Задача 2. Белла, Таня и Лилит в день Хэллоуина надели костю­
мы ведьмы, чёрта и ангела. Лилит не была ни ведьмой, ни чёртом.
Белла не была ведьмой. Кем была каждая девочка?
Задача 3. У Тима есть раздвижная удочка, состоящая из пяти
секций. Длина первой секции — 50 сантиметров, длина остальных—
40 сантиметров. Когда удочка полностью вытянута, её соседние сек­
ции перекрываются на 5 сантиметров. Какой длины полностью вы­
тянутая удочка?
Задача 4. Четыре одинаковых бриллианта стоят столько же,
сколько два из них и ещё 600 тысяч рублей. Сколько стоит один
бриллиант?

Задача 5. В день Хэллоуина Александр, Катя, Мария, Бен и Пау­


ла отравились выпрашивать конфеты. Александр раздобыл больше
конфет, чем Катя, а Мария — меньше, чем Бен. Александру доста­
лось меньше конфет, чем Марии, а Бену—больше, чем Пауле. Кто
собрал больше всего конфет?
Задача 6. Три маленькие ведьмы и два двухкилограммовых па­
кета с конфетами весят столько же, сколько четыре маленькие ведь­
мы и один килограммовый пакет с конфетами. Сколько весит ма­
ленькая ведьма?
Задача 7. Ведьма приготовила зелье за 18 лет,
18 месяцев, 18 недель, 18 дней, 18 часов и 18 ми­
нут. Сколько полных лет ведьма готовила зелье?
Задача 8. Вики, Гоусти, Слайми и Стинки ле­
тят на метлах, выстроившись в ряд. Слайми летит
Занятие 9. Математический хоккей I 85

рядом с Вики, но не рядом со Стинки. Гоусти летит не рядом со


Стинки. Кто летит рядом с Гоусти?
Задача 9. На доске написано: 2 2 2 2 2 2 = 100. Расставьте ариф­
метические знаки (+, —, •, :) и скобки так, чтобы получилось вер­
ное равенство. Использовать все четыре знака не обязательно. Каж­
дый знак можно вставить несколько раз. Можно объединять двойки
в многозначные числа, например 22 или 2222.
Задача 10. Половина головки сыра стоит на 500 рублей больше,
чем четверть этой же головки сыра. Сколько стоит головка сыра?
Задача 11. Сеня, Веня и Женя живут на острове рыцарей и лжецов.
Сеня и Веня говорят, что все трое из них — лжецы. Женя утверждает,
что ровно двое из них — лжецы. Кто из них рыцарь и кто лжец?
Задача 12. В день Хэллоуина Дэвид, Алек­
сандр и Игорь нарядились привидениями. Один
из них напугал пять девочек, другой - трёх маль­
чиков, а третий— одного папу, который при­
сматривал за детьми. Дэвид не напугал девочек,
Игорь не напугал мальчиков, а Александр не на­
пугал папу. Известно также, что Александр и тот,
кто напугал мальчиков, выпрашивали конфеты
на одной улице. Кто напугал папу?
Задача 13. Сколько квадратов на рисунке?
Задача 14. Во сколько раз быстрее часовой
стрелки движется минутная стрелка?
Задача 15. Какое наименьшее возможное
количество детей в семье, если у каждого ре­
бёнка есть хотя бы одна сестра и хотя бы один
брат?
Задача 16. Возраст нескольких друзей в сумме составляет 29 лет.
Через три года он будет в сумме равен 41 году. Сколько этих друзей?

Подборка задач
Задача 1. Три одинаковых рубина дороже, чем четыре одинако­
вых сапфира. Что дороже: четыре рубина или пять сапфиров?
Задача 2. Пеликан-папа снимается с гнезда и летит к озеру ло­
вить рыбу. Одновременно пеликан-мама заканчивает ловлю рыбы
и летит к гнезду. Мамин клюв полон добычи, поэтому она летит
86 Раздел 1. Планы занятий

в два раза медленнее папы. Через четыре


минуты после вылета пеликаны встречаются
в воздухе. За сколько времени мама долетает
от озера до гнезда?
Задача 3. Можно ли разрезать квадрат
на 6 квадратов меньшего (не обязательно
одинакового) размера? Если да, то как? Если нет, то почему? Квад­
раты, состоящие из других квадратов, не считаются.
Можно ли разрезать квадрат на 7 квадратов? А на 8?
Задача 4. Смышлёная юная волшебница Нелли ищет Камень
счастья. Перед ней две двери: на одной написан номер 20, а на
другой —номер 21. Каждая дверь ведёт в заколдованную комнату.
Колдовство действует так: как только в комнату входит волшебник
или волшебница, на стене появляются случайно выбранные целые
числа (в первой комнате двадцать чисел, а во второй—двадцать
одно число). Нелли должна зачеркнуть одно число на стене. Если
сумма оставшихся чисел окажется чётной, откроется Камень сча­
стья. Если сумма окажется нечётной, Нелли погибнет. Какую дверь
должна выбрать Нелли, чтобы наверняка остаться в живых?
Задача 5. В Стране сладостей денеж­
ной единицей служит сугрик. Монеты там
бывают трёх видов: один сугрик, три суг-
рика и пять сугриков. Пара сапог-скорохо­
дов в Стране сладостей стоит 50 сугриков.
Кот в сапогах уверяет, что свои сапоги-
скороходы он купил в Стране сладостей,
отдав за них ровно 15 монет. Докажите, что кот говорит неправду.
Задача 6. В день Хэллоуина шесть детей пришли к вам выпраши­
вать конфеты. У одного из детей корзина с одной конфетой, а у пя­
ти остальных пустые корзины. У вас есть горшок с карамельками.
Запуская руку в горшок, вы каждый раз достаёте две карамельки
и даёте их двум детям — каждому по одной. Можно ли сделать так,
чтобы все дети ушли с одинаковым количеством конфет? Если да,
то как? Если нет, то почему?

ДОПОЛНИТЕЛЬНЫЕ ЗАДАЧИ
Задача 7*. Ровно в полночь шесть часовых занимают свои по­
сты на шести угловых башнях шестиугольного замка (по одному ча-
Занятие 9. Математический хоккей I 87

совому на каждой башне). Через каждые 15 минут каким-то двум


часовым становится скучно. Как только часовому становится скуч­
но, он переходит на одну из двух соседних башен (по своему выбо­
ру). Докажите, что, как бы ни двигались часовые, они никогда не
окажутся все вместе на одной башне.
Занятие 10

Чётные и нечётные числа III. Чередование

Это третье занятие, посвященное чётности. Мы уже обсудили опре­


деления и свойства чётных и нечётных чисел, освоили полезные спо­
собы решения задач и неоднократно применяли их на практике.
Сегодня мы познакомимся ещё с одним типом задач на чётность.
В этих задачах встречается чередование предметов, свойств или со­
бытий.
Что принести на урок
Маркеры разных цветов могут пригодиться на любом занятии
математического кружка, но при обсуждении сегодняшней темы
они будут особенно полезны.

Математическая разминка
Задача для разминки 1. Когда петух стоит на одной ноге, он
весит 4 килограмма. Сколько весит петух, когда он стоит на двух
ногах?
Задача для разминки 2. Как разбить 188 на две части по 100?

Тема занятия:
«Чередование»
Мы начинаем тему чередования с решения задач.
Пример 1. Изобретатель Макс расположил 11 одинаковых ше­
стерёнок в ряд. Каждая шестерёнка, кроме первой и последней,
сцеплена с обеими соседними шестерёнками. Макс поворачивает
крайнюю левую шестерёнку по часовой стрелке. Как будет вращать­
ся крайняя правая шестерёнка: по часовой стрелке или против?
Для преподавателей. Эта задача требует некоторого представления о ме­
ханике. Не все дети знают, что такое шестерёнки и как они применя­
ются для передачи движения. Поэтому можно сначала нарисовать пару
сцепленных шестерёнок и поговорить о том, каким образом вращение
одной шестерёнки влияет на вращение другой.
Занятие 10. Чётные и нечётные числа III. Чередование 89

Вырисовывать все одиннадцать шестерёнок на доске не обязатель­


но: для экономии времени можно нарисовать несколько штук, а потом
договориться с участниками, что остальные вы изобразите как обычные
окружности. Полезно также распечатать рисунок, где показано несколь­
ко сцепленных шестеренок.
РАЗБОР ПРИМЕРА 1. Главное в этой задаче — заметить, что направ­
ление вращения шестерёнок чередуется: любые две сцепленные ше­
стерёнки крутятся в противоположных направлениях. Вращаясь по
часовой стрелке, первая шестерёнка заставляет вторую поворачи­
ваться против часовой стрелки. Вторая шестерёнка передаёт вра­
щение третьей, которая поворачивается по часовой стрелке, и так
далее. Другими словами, все шестерёнки через одну, начиная с пер­
вой, вращаются по часовой стрелке. Проследив это чередование до
последней, одиннадцатой шестерёнки, мы увидим, что она враща­
ется по часовой стрелке.
Затем я спрашиваю, что будет, если шестерёнок 12, 13, 25, 100
и так далее. Устав от этих вопросов, участники сами формулируют
общее правило: каждая нечётная шестерёнка вращается в том же
направлении, что и первая, а каждая чётная — в противоположном
направлении.
Пример 2. Изобретательница Белла расположила 11 одинако­
вых шестерёнок по кругу. Каждая шестерёнка сцеплена с двумя со­
седними. Что произойдёт, если Белла попробует вращать первую
шестерёнку по часовой стрелке?
РАЗБОР ПРИМЕРА 2. Такая система шестерёнок не будет работать.
Поскольку шестерёнки вращаются в чередующихся направлени­
ях, одиннадцатая шестерёнка должна вращаться в том же направле­
нии, что и первая. Но одиннадцатая шестерёнка сцеплена с первой,
поэтому они не могут вращаться в одном направлении. У Беллы не
получится повернуть шестерёнку.
Как и в предыдущей задаче, мы выясняем, что будет происхо­
дить, если в такой конструкции 12 шестерёнок, 13, 14 и так да­
лее. Система, состоящая из нечётного числа шестерёнок, останется
неподвижной; если же шестерёнок чётное число, они будут вра­
щаться.
Пример 3. Можно ли поставить 9 мальчиков и девочек в круг
так, чтобы каждый мальчик стоял между двумя девочками, а каждая
девочка — между двумя мальчиками?
90 Раздел 1. Планы занятий

РАЗБОР ПРИМЕРА 3.Это невозможно. Присвоим каждому ребёнку


номер (от 1 до 9, по часовой стрелке). Предположим, что под номе­
ром 1 стоит девочка. Поскольку она стоит между двумя мальчиками,
под номером 2 должен стоять мальчик. По этой же причине под но­
мером 3 должна стоять девочка. Рассуждая точно так же и двигаясь
по кругу, выясняем, что под номером 9 должна стоять девочка. Но
она стоит рядом с девочкой под номером 1. Такое же противоречие
получается и в том случае, если под номером 1 стоит мальчик. По­
этому такая расстановка невозможна.
Обобщая все предыдущие задачи, мы формулируем два важных
вывода.
Вывод 1. Предположим, что у нас есть предметы двух видов,
например лягушки и жабы. Расположим их в ряд, чередуя лягушек
с жабами. Если начать считать этих животных слева направо, каж­
дое нечётное животное будет того же вида, что и первое, а каждое
чётное животное—другого вида.
Вывод 2. Предположим, что лягушки и жабы расположены по кру­
гу, причём любые два соседних животных относятся к разным ви­
дам (другими словами, лягушки чередуются с жабами). Это возмож­
но только в том случае, если общее количество животных чётное.
Первое замечание очевидно, но второе требует объяснения. До­
кажем, что чередование лягушек и жаб в круге возможно только
в том случае, если их общее количество чётное.
Выберем животное, с которого мы начнём обходить круг. По пу­
ти будем присваивать всем животным номера: 1, 2,3,... Поскольку
лягушки и жабы чередуются, все животные с нечётными номерами
относятся к одному виду.
Предположим, что животных в круге нечётное число. Тогда по­
следнее животное будет таким же, как и первое. Но эти два живот­
ных находятся рядом, поэтому они должны быть разного вида! Зна­
чит, наше предположение неверно: количество животных в круге не
может быть нечётным. Оно должно быть чётным.
Чтобы закрепить идею чередования, мы решаем ещё несколько
задач.
Пример 4. Кузнечик упражняется в прыжках на прямой доро­
ге, размеченной чёрными и красными камнями. Чёрные и красные
камни чередуются, а расстояние между соседними камнями — ров­
но один метр. В первый раз кузнечик прыгает с красного камня.
Занятие 10. Чётные и нечётные числа III. Чередование 91

Каждый раз он прыгает на один метр влево или вправо. В какую


сторону он прыгнет в следующий раз — неизвестно.
а) На каком камне окажется кузнечик после пяти прыжков — на
красном или на чёрном?
б) Тот же вопрос для двадцати прыжков.
в) Кузнечик закончил тренировку на том же камне, с которого
начал. Маша, которая всё это время наблюдала за кузнечиком, на­
считала 125 прыжков. Докажите, что рассеянная Маша ошиблась.
РАЗБОР ПРИМЕРА 4. Начнём с обсуждения пунктов а) и б), которые
позволят нам сформулировать общее правило: сделав чётное число
прыжков, кузнечик обязательно окажется на красном камне, а сде­
лав нечётное число прыжков — на чёрном.
Кузнечик начинает с красного камня. Где он приземлится по­
сле первого прыжка? Длина прыжка — один метр, значит, кузнечик
приземлится на соседнем камне. Оба камня, соседних с красным,
чёрные. Значит, в какую бы сторону кузнечик ни прыгнул, первый
прыжок закончится на чёрном камне.
Теперь рассмотрим второй прыжок. Он начинается на чёрном
камне. В какую бы сторону кузнечик ни прыгнул, он приземлится
на соседнем камне — красном.
Делаем вывод: при каждом приземлении цвет камня меняется.
С чёрного камня кузнечик прыгает на красный, а с красного — на
чёрный. Поэтому после любого чётного числа прыжков кузнечик
приземлится на камне того же цвета, что и камень, с которого он
начал тренировку. Это красный цвет. После любого нечётного числа
прыжков кузнечик приземлится на камне другого цвета —чёрного.
в) Предположим, что Маша права и кузнечик действительно прыг­
нул 125 раз. На каком камне он приземлился в последний раз? Ко­
личество прыжков нечётное, а начал кузнечик с красного камня.
Значит, он приземлился на чёрном камне. Но кузнечик начал тре­
нировку и закончил её на одном и том же камне. Следовательно,
тренировка закончилась на красном камне. Мы получили два утвер­
ждения, противоречащих друг другу. Поэтому можно с уверенно­
стью сказать, что Маша ошиблась.
Для преподавателей. При разборе пункта в) мы снова встречаемся с од­
ним важным аспектом математических рассуждений, который следует
ещё раз подчеркнуть.
Поскольку кузнечик выбирает направление прыжков случайным об­
разом, его точный маршрут неизвестен. Красота нашего решения за-
92 Раздел 1. Планы занятий

ключается в том, что оно не привязано ни к какой конкретной после­


довательности прыжков. Напротив, оно показывает, что, какие бы на­
правления ни выбирал кузнечик, результат будет одним и тем же: после
125 прыжков кузнечик окажется на чёрном камне.
Очень часто дети предлагают какой-нибудь один конкретный вари­
ант прыжков кузнечика, проверяют, что после 125-го прыжка кузнечик
будет на чёрном камне, и делают вывод, что Маша ошиблась. Но, может
быть, прыгая каким-то другим способом, кузнечик всё-таки окажется на
красном камне? Из частных случаев нельзя делать общие выводы!
Другой способ решить эту задачу — отдельно рассмотреть число прыж­
ков вправо и число прыжков влево. Поскольку суммарное количество
прыжков нечётно, эти два числа не могут быть равными. Это решение
тоже верное, но не такое наглядное.

Пример 5. На левом нижнем поле шахмат­


ной доски размерами 5 x 5 (на рисунке это
поле помечено звёздочкой) стоит ленивая ла­
дья. Она может ходить вверх, вниз, вправо
или влево, но только на одно поле.
Ленивая ладья должна обойти всю доску
(ей ещё повезло, что это не очень большая
доска) и при этом побывать на каждом поле ровно один раз.
а) Может ли ладья завершить обход в поле, помеченном цвет­
ком? (В каждом пункте этой задачи надо либо нарисовать маршрут
ладьи, либо объяснить, почему это невозможно.)
б) Может ли ладья завершить обход в поле, помеченном рожицей?
в) Может ли ладья, завершив обход, вернуться в поле, помечен­
ное звёздочкой?
РАЗБОР ПРИМЕРА 5. а) Для первого пункта задачи нетрудно на­
рисовать маршрут. Существует множество вариантов, и участники
наверняка предложат сразу несколько.
б) Придумать маршрут для этого пункта тоже несложно.
в) После нескольких неудачных попыток дети догадываются, что
такого маршрута не существует. Доказать это им помогут подсказки.
1. Обратите внимание на то, что чёрные поля чередуются с белыми.
2. Замечайте цвет полей, на которые встаёт ладья.
Ладья всегда переходит с чёрного поля на белое, а с белого — на
чёрное. Поэтому на каждом ходе цвет поля меняется.
Поле со звёздочкой в левом нижнем углу чёрное. Поэтому через
нечётное число ходов ладья будет на белом поле, а через чётное — на
Занятие 10. Чётные и нечётные числа III. Чередование 93

чёрном. Для того чтобы обойти всю доску и вернуться на исходное по­
ле, понадобится 25 ходов (на один ход больше, чем в пунктах а) и б)).
25 — чётное число, поэтому через 25 ходов ладья должна оказать­
ся на белом поле. Значит, такого маршрута обхода доски, при кото­
ром ладья побывала бы на каждом поле ровно один раз и вернулась
на исходное поле, не существует.
Для преподавателей. При решении пункта в) применяется тот же прин­
цип, что и в задаче про кузнечика: мы не пытаемся проверить все воз­
можные маршруты (это нереально), а доказываем некоторое свойство,
которое остаётся верным при любом маршруте.

Подборка задач
Задача 1. Разрежьте каждую фигуру, показанную на рисунке, на
две части, одинаковые по размеру и форме. Резать можно по лини­
ям сетки и по диагоналям квадратиков. Две части считаются оди­
наковыми, если после разрезания можно наложить их друг на друга
так, чтобы они точно совпали. При этом их можно поворачивать
и переворачивать.

Задача 2. Застенчивый хамелеон Лео


очень волнуется, когда встречает другого
хамелеона. Когда Лео волнуется, он изме­
няет свой цвет. Если Лео был зелёным, он
становится коричневым, а если был ко­
ричневым — становится зелёным. Сегодня
утром Лео проснулся коричневым. До обе­
да он встретил 17 хамелеонов. Какого цве­
та был Лео в начале обеда?
Задача 3. Неизвестный хронист оставил нам описание пира во
дворце короля Артура. Если верить этому описанию, на празднество
было приглашено 25 гостей — храбрых рыцарей и прекрасных дам.
Все гости уселись за круглым столом. Рядом с каждой дамой сидели
94 Раздел 1. Планы занятий

два рыцаря, а рядом с каждым рыцарем—две дамы... Докажите, что


в описании пира есть ошибка.
Задача 4. Может ли сумма четырёх последовательных целых чи­
сел быть нечётным числом?
Задача 5. Пассажирский теплоход шёл из Санкт-Петербурга в Ка­
лининград. На середине пути объявили штормовое предупрежде­
ние. Тогда теплоход увеличил скорость в два раза и прибыл в Кали­
нинград на три часа раньше срока. Сколько времени шёл теплоход
от Санкт-Петербурга до Калининграда?
Задача 6. Бусы состоят из 45 красных и синих бусинок.
а) Докажите, что найдутся две соседние бусинки одного цвета.
б*) Докажите, что найдутся две бусинки одного цвета, между ко­
торыми ровно две бусинки.
Задача 7. В этой задаче все события
происходят на длинной прямой дороге.
а) Кузнечик прыгает по дороге. При
каждом прыжке он перемещается на
один метр вправо или влево. Может ли
он после 21 прыжка оказаться точно
в том месте, откуда он сделал первый
прыжок? Если да, то как? Если нет, то
почему?
б) Кузнечик прыгает по дороге. Каждый раз он прыгает вправо
или влево на один метр. Может ли он после 33 прыжков оказаться
на расстоянии 10 метров от исходной точки? Если да, то как? Если
нет, то почему?
в) Два кузнечика — коричневый и зелёный — сидят на дороге.
Расстояние между ними —15 метров. Кузнечики одновременно на­
чинают прыгать. Когда один из них прыгает, другой тоже прыгает.
Длина каждого прыжка — 3 метра. Кузнечики могут прыгать впра­
во или влево. Могут ли они одновременно приземлиться в одном
месте? Если да, то как? Если нет, то почему?

ДОПОЛНИТЕЛЬНЫЕ ЗАДАЧИ

Задача 8. а) Можно ли нарисовать четырёхугольник и прямую


линию так, чтобы прямая пересекала каждую сторону четырёхуголь­
ника? (Во всех пунктах этой задачи прямая не может проходить че­
рез вершины многоугольника.)
Занятие 10. Чётные и нечётные числа III. Чередование 95

б) Можно ли нарисовать десятиугольник (многоугольник, име­


ющий 10 сторон) и прямую линию так, чтобы прямая пересекала
каждую сторону десятиугольника?
в) Можно ли нарисовать многоугольник, имеющий 11 сторон,
и прямую линию так, чтобы прямая пересекала каждую сторону
этого многоугольника?
Задача 9*. На столе шесть столбиков монет. В первом столбике
одна монета, во втором—две и так далее (в последнем столбике
шесть монет). За один ход можно выбрать любые два столбика и до­
бавить к ним по одной монете. Количество ходов не ограничено.
Можно ли сделать все столбики одинаковыми?
Занятие 11

Взвешивания и фальшивые монеты

Сегодняшнее занятие одновременно и полезное, и увлекатель­


ное. Мы будем распознавать фальшивые монеты, придумывать хит­
роумные способы взвешивания предметов и решать другие интерес­
ные головоломки.
В отличие от задач на чётность, которые мы решали на преды­
дущих занятиях, в задачах на взвешивание не нужно ничего дока­
зывать. Это конструктивные задачи: они заключаются в том, что­
бы придумать способ достижения определённой цели. Такие задачи
понятны и естественны для детей младшего возраста. Их успешно
решают не только пятиклассники и шестиклассники, но и школь­
ники младших классов.

Математическая разминка
На этот раз разминкой послужит математический фокус. Снача­
ла я расскажу, как проходит этот фокус на занятии, а потом раскрою
его секрет.
Я предлагаю участникам задумать целое число от 1 до 10 и хра­
нить его в тайне. Я отворачиваюсь, и дети договариваются, какое чис­
ло загадать. Я даю указания: прибавить к секретному числу 10, вы­
честь из полученного результата 8, затем прибавить 20, вычесть сек­
ретное число, умножить результат на 2, вычесть 4 и разделить на 4.
Теперь участникам пришло время удивиться. Я заявляю, что, хо­
тя не знаю секретного числа, могу угадать результат вычислений:
это 10. К общему изумлению, результат оказывается верным.
Затем я предлагаю детям найти объяснение моим телепатиче­
ским способностям. Поскольку они в полной растерянности, при­
ходится повторить фокус. Я прошу выбрать другое число и выпол­
нить следующие действия: вычесть из секретного числа 5, приба­
вить к результату 13, потом прибавить 12, вычесть секретное число,
умножить на 3, вычесть 10 и разделить результат на 2. Затем я объ­
являю результат: 25.
Занятие 11. Взвешивания и фальшивые монеты 97

На этот раз участники внимательнее следят за последовательно­


стью действий, и кто-то догадывается, в чём дело.
Действительно, нетрудно заметить, что в обоих случаях вычис­
ления проводятся по одной схеме.
1. Первые нескольких действий просто увеличивают или умень­
шают секретное число на известную величину.
2. Когда участникам даётся указание вычесть секретное число,
оно фактически удаляется из результата. Например, в первом раун­
де игры вычисления начинались так:

х + 1 0 - 8 + 20,

где х — секретное число. После вычитания х получается

х + 1 0 - 8 + 2 0 - * = 1 0 - 8 + 20 = 22.

3. Начиная с этого места результат не зависит от секретного числа.


Поэтому преподавателю нетрудно угадать конечный результат вы­
числений. Достаточно выбрать одновременно с участниками своё соб­
ственное секретное число (проще всего взять 0) и затем, давая де­
тям указания, самому выполнять те же действия со своим числом.
Получится тот же результат.
Для преподавателей.
• Секретное число, конечно, может быть любым, но во избежание
сложных вычислений лучше предложить выбрать число от 1 до 10.
• Последовательность действий можно сделать проще или сложнее
с учётом уровня участников.
• Если действие вычитания неизвестного числа перенести ближе
к началу, детям будет проще разгадать секрет фокуса.
• Разгадать фокус будет сложнее, если перед вычитанием неизвест­
ного числа добавить действие умножения. В этом случае важно не оши­
биться: чтобы скомпенсировать умножение, надо вычесть секретное
число соответствующее количество раз. Например, для схемы
(х + 5 ) - 2 - х + 1 0 - х
инструкции будут такими: прибавить к секретному числу 5, умножить
результат на 2, вычесть секретное число, прибавить 10, вычесть секрет­
ное число.
• Такая разминка проходит более оживлённо, если в счёт вовлечены
все участники кружка. Один раз я назначила только двух «счетоводов».
Делать этого не стоило: остальные участники скучали, а один из счето­
водов ошибся в вычислениях.
98 Раздел 1. Планы занятий

Тема занятия:
«Весы и фальшивые монеты»
В большинстве задач этого занятия
фигурируют весы с двумя чашами. Если
кто-то никогда не видел таких весов, на­
до объяснить, что это такое.
Чашечные весы служат для сравнения предметов по весу. Пред­
меты кладутся на противоположные чаши весов. Когда грузы на ча­
шах весят одинаково, весы находятся в равновесии. Если одна из
чаш ниже другой, значит, груз на ней тяжелее. На наших чашечных
весах нет ни стрелки, ни делений. Поэтому мы можем увидеть, что
один предмет тяжелее другого, но не можем определить, насколько
именно.
Привыкнуть к чашечным весам помогут несколько простых уп­
ражнений. Затем мы перейдём к более сложным задачам.
Упражнение 1. Три шалтая и 4 болтая вместе весят столько же,
сколько 5 шалтаев и 3 болтая. Кто тяжелее — шалтай или болтай?
Разбор упражнения 1. Это упражнение служит напоминани­
ем о том, что решать задачи такого типа значительно проще с по­
мощью рисунков. Другая важная идея состоит в следующем: если
убрать с обеих чаш груз одинакового веса, состояние весов не из­
менится.
Рисунок помогает понять, что если убрать с каждой чаши трёх
шалтаев, то весы останутся в равновесии. Теперь снимем с каждой
чаши трёх болтаев: равновесие снова сохраняется.

= шалтай
= болтай

Не первой чаше остался один болтай, а на второй—два шалтая.


Поскольку один болтай весит столько же, сколько два шалтая, бол­
тай тяжелее шалтая.
Упражнение 2. Два шалтая весят больше, чем 6 болтаев. Кто
тяжелее: один шалтай или 3 болтая?
Разбор упражнения 2. Это упражнение помогает усвоить новую
полезную вдею: если убрать с каждой чаши половину её содержимого,
то состояние весов не изменится. Более тяжёлая чаша по-прежнему
Занятие 11. Взвешивания и фальшивые монеты 99

будет тяжелее, а более лёгкая — по-прежнему легче. Поэтому можно


убрать с весов одного шалтая и трёх болтаев (см. рисунок). Весы
остаются в том же положении: один шалтай тяжелее трёх болтаев.

= шалтай
= болтай

Для преподавателей. Идею этого упражнения можно обобщить: если уб­


рать с каждой чаши (или добавить к каждой чаше) одну и ту же долю
лежащего на ней веса — например, убрать с каждой чаши треть веса или
удвоить вес на каждой чаше, — то состояние весов не изменится.
Упражнение 3. Три шалтая весят больше, чем 10 болтаев. Кто
тяжелее: один шалтай или 3 болтая?
Разбор упражнения 3. Это упражнение иллюстрирует ещё одну
простую мысль. Предположим, что одна чаша тяжелее другой. Что
произойдёт, если добавить к более тяжёлой чаше что-нибудь ещё?
Может ли она стать легче другой чаши? Конечно, нет! Она станет
ещё тяжелее.
Точно так же состояние весов не изменится, если убрать что-то
с более лёгкой чаши.
Исходя из этой идеи уберём с весов одного болтая. Состояние ве­
сов не изменится: ведь чаша с болтаями и так была легче. Осталось
3 шалтая на одной чаше и 9 болтаев на другой. Теперь уберём с каж­
дой чаши две трети веса (двух шалтаев с одной чаши и шесть болтаев
с другой). От этого состояние весов тоже не изменится. Теперь на ве­
сах один шалтай и три болтая, причём чаша с шалтаем тяжелее.
Упражнение 4. У золотоискателя Джека три килограмма золо­
того песка. Сможет ли он отмерить половину своего золота на ча­
шечных весах без гирь?
Разбор упражнения 4. Надо по очереди высыпать золотой песок
на обе чаши весов небольшими порциями, каждый раз восстанавли­
вая равновесие.
На практике этот способ не позволяет разделить золотой песок
пополам абсолютно точно. Рано или поздно мы будем высыпать по­
следнюю порцию, и её придётся разделить «на глазок». Но эту по­
следнюю порцию можно сделать настолько маленькой, что она су­
щественно не повлияет на результат. Поэтому в последующих за-
100 Раздел 1. Планы занятий

дачах этого типа мы будем исходить из предположения, что таким


способом можно добиться точного равновесия.
Теперь перейдём к более сложным задачам на взвешивание. На
этот раз мы не разбираем задачи у доски: участники работают в груп­
пах по два-три человека. Я перехожу от одной группы к другой, про­
веряя решения и при необходимости направляя ход рассуждений.
Такая форма работы выбрана не случайно. Если обсуждать зада­
чи вместе, самые сообразительные дети быстро предлагают реше­
ние, лишая остальных возможности прийти к нему самостоятельно.
Конструктивные задачи хороши тем, что каждый участник в состо­
янии решить их, если дать ему достаточно времени.
Пример 1. У золотоискателя Джека 24 килограмма золотого песка
и чашечные весы без гирь. Как ему отмерить 9 килограммов золота?
РАЗБОР ПРИМЕРА 1. Сначала надо разделить весь золотой песок на
две одинаковые кучки по 12 килограммов. Затем разделить пополам
одну из этих кучек: получатся кучки по 6 килограммов. И наконец,
разделить одну из них на кучки по 3 килограмма. Теперь Джек мо­
жет объединить две кучки — шестикилограммовую и трёхкилограм­
мовую. Получится ровно 9 килограммов золота.
Пример 2. Предположим, что у вас есть три монеты и вы точно
знаете, что одна из них фальшивая: она легче настоящих. Как опре­
делить фальшивую монету за одно взвешивание на чашечных весах
без гирь?
РАЗБОР ПРИМЕРА 2. Взвесим любые две монеты. Если одна из них
легче, она фальшивая. Если они весят одинаково, оставшаяся моне­
та фальшивая.
Пример 3. Предположим, что у вас есть три монеты и вы точ­
но знаете, что одна из них фальшивая, но неизвестно, легче она
настоящих или тяжелее. Как определить фальшивую монету за два
взвешивания на чашечных весах без гирь?
РАЗБОР ПРИМЕРА 3. При первом взвешивании выберем любые две
монеты и сравним их вес. Возможны только два результата.
• Если эти две монеты весят одинаково, значит, они обе настоя­
щие. В этом случае оставшаяся монета фальшивая.
• ЕСЛИ ЭТИ две монеты имеют разный вес, то одна из них фаль­
шивая. Мы не знаем, какая именно, но зато теперь знаем, что остав­
шаяся монета настоящая. Сравним, например, более лёгкую монету
с настоящей. Если окажется, что они весят одинаково, более лёгкая
Занятие 11. Взвешивания и фальшивые монеты 101
монета настоящая, а более тяжёлая фальшивая. Если нет, более лёг­
кая монета фальшивая.

Подборка задач
Задача 1. На пляже острова рыцарей и лжецов отдыхает компа­
ния местных жителей. За время их беседы каждый хотя бы один раз
сказал остальным: «Вы все лжецы!» Сколько рыцарей в этой ком­
пании?
Задача 2. Разрежьте фигуру, показанную на рисун­
ке, на четыре части одинаковой формы и одинакового
размера.
Задача 3. Известно, что 3 шалтая,
3 болтая и 3 тянитолкая весят столько
же, сколько 2 шалтая, 5 болтаев и 2 тяни­
толкая. Кроме того, тянитолкай тяжелее
шалтая. Кто тяжелее: болтай или тяни­
толкай?
Задача 4. а) Известно, что из трёх
монет одна фальшивая. Фальшивая мо­
нета легче настоящей. Определите фаль­
шивую монету за одно взвешивание на
чашечных весах.
б) Та же задача для четырёх монет и двух взвешиваний.
в) Та же задача для пяти монет и двух взвешиваний.
г) Та же задача для семи монет и двух взвешиваний.
д) Та же задача для девяти монет и двух взвешиваний.
Задача 5. У золотоискателя Джека есть 9 ки­
лограммов золотого песка и чашечные весы.
а) Как Джеку за одно взвешивание отмерить
5 килограммов золота, если у него есть одна ки­
лограммовая гиря?
б) Как Джеку за два взвешивания отмерить
3 килограмма золота, если у него есть одна кило­
граммовая гиря?
в) Как Джеку за три взвешивания отмерить
2 килограмма золота, если у него есть две гири:
200-граммовая и 50-граммовая? (Напомним, что
в одном килограмме 1000 граммов.)
102 Раздел 1. Планы занятий

Задача 6. Из четырёх монет одна фальшивая. Фальшивая моне­


та отличается по весу от настоящей, но неизвестно, легче она или
тяжелее. Как определить фальшивую монету за два взвешивания на
чашечных весах?
Задача 7. В здании музея математики 36 залов, расположенных
в виде квадрата 6 х 6. В каждом зале 4 двери (по одной в каждой
стене), которые ведут в другой зал или на улицу.
а) Тим собирается начать осмотр музея в юго-западном угловом
зале и закончить его в юго-восточном угловом зале. Он планирует
побывать в каждом зале ровно один раз и ни разу не выйти на ули­
цу. Сможет ли он это сделать? Нарисуйте маршрут или объясните,
почему это невозможно.
б) Катя собирается начать осмотр музея в юго-западном угловом
зале и закончить его в северо-восточном угловом зале. Она плани­
рует побывать в каждом зале ровно один раз и ни разу не выйти на
улицу. Сможет ли она это сделать? Нарисуйте маршрут или объяс­
ните, почему это невозможно.

ДОПОЛНИТЕЛЬНЫЕ ЗАДАЧИ

Задача 8. а) Из 75 золотых монет одна фальшивая. Фальшивая


монета отличается по весу от настоящей, но неизвестно, легче она
или тяжелее. За два взвешивания на чашечных весах определите,
какая монета легче — фальшивая или настоящая.
б) Та же задача для 76 монет.
Задача 9*. В награду за верную службу король раздал трём ры­
царям шесть монет: три золотые и три серебряные. Каждый рыцарь
получил две монеты. Первый рыцарь знает, какие монеты раздал
король и какие получил он сам, но не знает, какие монеты у каж­
дого из остальных. Узнайте, какие монеты у первого рыцаря, задав
ему только один вопрос. Вопрос должен быть таким, чтобы на него
можно было ответить только «Да», «Нет» или «Не знаю».
Занятие 12

Математическая олимпиада I

Сегодня последнее занятие перед зимними каникулами. Дети усерд­


но поработали и многому научились. Олимпиада будет достойным
завершением полугодия, тем более что это ещё и повод раздать призы.
В главе «Математическая олимпиада» (с. 241) даны подробные
рекомендации по организации и проведению таких соревнований.
Олимпиаду можно проводить в разных формах. Для нашего кружка
лучше подойдёт устная олимпиада: она проходит оживлённо и для
детей этого возраста проще письменной. Необходимость записы­
вать решения задач вряд ли вызвала бы энтузиазм у моих учеников.
Кроме того, на устной олимпиаде участники сразу узнают, пра­
вильно ли они решили задачу, и могут исправить или улучшить своё
решение прямо в ходе соревнования. А для преподавателя это отлич­
ная возможность индивидуальной беседы с большинством участни­
ков кружка. Благодаря этому устная олимпиада становится продол­
жением процесса обучения.
Для преподавателей. Если по каким-то причинам неудобно проводить олим­
пиаду на последнем занятии перед каникулами, можно перенести её.

Что принести на урок


• Призы и награды (желательно для всех участников, а не только
для победителей). Благодаря им олимпиада станет памятным собы­
тием.
• Счётные палочки или спички для задачи про вишню.

Математическая разминка
В день соревнования не хватит времени, чтобы разобрать все
задачи из предыдущей подборки. Но к следующему занятию дети
104 Раздел 1. Планы занятий

их забудут: перед ним будет длительный перерыв. Поэтому вместо


разминки я кратко разбираю те задачи, которые касаются послед­
ней темы — взвешивания. Обсудить их решения перед олимпиадой
будет полезно.

Событие дня:
математическая олимпиада
Олимпиада состоит из десяти задач. Они приведены (пример­
но) в порядке возрастания сложности и напечатаны на двух ли­
стах: шесть задач на первом и четыре — на втором. Некоторые за­
дачи на втором листе не очень простые, а последняя задача —по-
настоящему трудная: она не даст заскучать даже самым способным
и сообразительным участникам.
В начале олимпиады участникам выдаётся первый лист. Второй
лист получают те, кто решил хотя бы четыре задачи из первого.
Для преподавателей. В этой главе приведён также список дополнитель­
ных задач. Он позволяет скорректировать сложность олимпиады с учё­
том уровня подготовки участников, заменив часть задач более просты­
ми или, наоборот, более трудными. Но обязательно оставьте несколько
простых задач! Лучше всего подобрать задачи так, чтобы почти каждый
участник был в состоянии решить хотя бы половину.
Для преподавателей. Задачи из этой главы можно использовать и для
письменной олимпиады. Но на письменной олимпиаде задач должно
быть меньше, и лучше не давать таких задач, для которых первое при­
шедшее в голову решение, скорее всего, будет неверным. Такова, на­
пример, задача про электрика Бориса.
Перед тем как раздать задачи, я напоминаю важные правила.
• Олимпиада — это индивидуальное соревнование. Любые разговоры
между участниками запрещены.
• Решать задачи и рассказывать их решения можно в любом поряд­
ке. На каждую задачу даётся три попытки.
• Если вы готовы рассказать решение, но все преподаватели сейчас
заняты, надо записаться в очередь на доске.
Раздав первый лист с задачами, я зачитываю их вслух и предлагаю
задать вопросы.
В нашем кружке соревнование продолжалось 1 час 30 минут. Участво­
вало девять детей. У меня был один помощник, занимающийся в кружке
для старшеклассников. Сначала мы едва справлялись с потоком реше­
ний, но3 когда дети перешли к более трудным задачам, смогли вздохнуть
спокойнее.
Занятие 12. Математическая олимпиада I 105

В целом олимпиада прошла успешно: почти всё время участники бы­


ли поглощены решением задач. В заключение мы объявили результаты
и раздали призы.
Мы приготовили по два приза на каждого участника и выдали их
по системе, изложенной в главе «Математическая олимпиада» на с. 244.
Это отлично сработало: все дети вышли из класса весёлые и довольные.
Для преподавателей. В дни проведения олимпиад не даётся домашнее за­
дание, поэтому в этой главе нет обычной подборки задач. Если детям
очень хочется порешать задачи дома, они могут заняться нерешёнными
задачами олимпиады.

Первый лист задач


Задача 1. Мальчик и девочка сидят рядом. «Я мальчик!» —го­
ворит темноволосый ребёнок. «Я девочка!» — говорит рыжеволосый
ребёнок. Известно, что хотя бы кто-то из них говорит неправду. Ка­
кого цвета волосы у мальчика и какого — у девочки?
Задача 2. Разрежьте равносторонний треугольник (треуголь­
ник, у которого все три стороны равны) на три одинаковых тре­
угольника.
Задача 3. На рисунке показана составленная
из спичек рюмка, внутри которой лежит вишня.
Выньте вишню из рюмки, переместив только одну
спичку. Форма рюмки должна остаться той же, но
вишня должна находиться вне её.
Задача 4. Четыре монеты расположены в ряд.
Известно, что среди них есть хотя бы одна настоящая и хотя бы одна
фальшивая. Фальшивая монета легче настоящей. Известно также,
что все фальшивые монеты лежат слева от всех настоящих. Как
определить все фальшивые монеты за одно взвешивание на чашеч­
ных весах?
Задача 5. Отец оставил в наследство трём сыновьям 7 сунду­
ков, полных золота, 7 сундуков, наполовину заполненных золотом,
и 7 пустых сундуков. Как братьям поделить наследство, чтобы у каж­
дого было одинаковое количество золота и одинаковое количество
сундуков? (Перекладывать золото из одного сундука в другой не раз­
решается.)
Задача 6. В день Хэллоуина две девочки и два мальчика от­
правились выпрашивать конфеты. Девочек звали Дженни и Синди,
106 Раздел 1. Планы занятий

а мальчиков — Майкл и Питер. Когда они сравнили свою добычу,


оказалось, что Дженни получила больше всех, Синди — не меньше
всех, а Питер — больше, чем Майкл. Получили ли девочки больше
конфет, чем мальчики?

Второй лист задач


Задача 7. На небольшом острове живут кролики трёх видов: бе­
лые, серые и пятнистые. У каждой серой или пятнистой крольчихи
всегда рождается два крольчонка: белый и серый. У каждой белой
крольчихи всегда рождается три крольчонка: белый, серый и пят­
нистый. Когда-то давным-давно на этом острове не было ни одного
кролика, но потом на него попала одна крольчиха, от которой про­
изошли все остальные кролики. Они называют её Великой бабуш­
кой. Сейчас на острове живут 40 серых, 40 белых и 30 пятнистых
кроликов. Ни один кролик не умер и не покинул остров. Какого цве­
та Великая бабушка?
Задача 8. Царь Дадон обещал награду в 1000 золотых монет
тому, кто решит трудную шахматную задачу: «Конь стоит в левом
нижнем поле шахматной доски 8 x 8 . Как сделать 11 ходов этим
конём, чтобы после них он оказался на том поле, с которого начал?»
(За один ход конь перемещается на два поля по вертикали или гори­
зонтали и затем на одно поле в перпендикулярном направлении.)
Объясните, почему никто не получил эту награду.
Задача 9. Электрик Борис смонтировал электропроводку в но­
вом школьном здании. В длинном коридоре он установил 10 лам­
почек и 9 выключателей. От каждого выключателя провода идут
к двум лампочкам, как показано на рисунке.

При нажатии на каждый выключатель состояние этих двух лам­


почек изменяется: если лампочка горит, она гаснет, а если не го­
рит — загорается.
Занятие 12. Математическая олимпиада I 107

Когда Борис закончил работу, горела только одна лампочка —


крайняя справа. В конце рабочего дня директор школы хочет вы­
ключить все лампочки. Сможет ли он это сделать? Если да, то как?
Если нет, то почему?
Задача 10*. Есть десять мешков с монетами. В одном мешке все
монеты фальшивые, а в остальных — все настоящие. Каждая фаль­
шивая монета легче настоящей на один грамм. Имеются чашечные
весы со стрелкой, показывающие разницу между двумя грузами
в граммах. Как найти фальшивую монету за одно взвешивание? Из
каждого мешка можно взять сколько угодно монет.

Д О П О Л Н И Т Е Л Ь Н Ы Е ЗАДАЧИ

Задача 11. Саша положил на одну чашу весов головку своего лю-
3
бимого сыра. Маша положила на другую чашу -г головки такого же
сыра и килограммовую гирю. После этого весы оказались в равно­
весии. Сколько весит головка сыра? (Мы считаем, что все головки
сыра весят одинаково.)
(По уровню трудности соответствует задачам 1—3.)
Задача 12. Как достать из колодца ровно 6 литров воды с помо­
щью только двух вёдер — 4-литрового и 9-литрового?
(По уровню трудности соответствует задачам 1—3.)
Задача 13. Несколько целых чисел расположены по кругу. Из­
вестно, что сумма любых двух соседних чисел нечётная. Докажите,
что количество чисел чётное.
(По уровню трудности соответствует задачам 4—6.)
Задача 14. В детском велопробеге участвуют мамы с маленьки­
ми детьми —всего 36 человек. Мамы едут на двухколёсных вело­
сипедах, а дети — на четырёхколёсных (с двумя вспомогательными
колесиками). Всего у велосипедов 110 колёс. Сколько участвует мам
и сколько детей?
(По уровню трудности соответствует задачам 4—6.)
Задача 15. а) Космический аппарат «Марсоход» опустился на
Марс и приступил к его исследованию. Каждое утро Марсоход от­
правляется в путь и за день проходит один километр в одном из двух
направлений: на восток или на запад. Из-за ошибки в программе
диспетчер с Земли не может передать Марсоходу команду, в каком
108 Раздел 1. Планы занятий

направлении ему двигаться. Может ли Марсоход через 24 дня ока­


заться в трёх километрах к востоку от места посадки?
б) Из-за аварии произошёл сбой программы, управляющей дви­
жением Марсохода: теперь за день он может перемещаться только
на 7 километров к востоку или на 5 километров к западу. Может ли
Марсоход через 30 дней после аварии оказаться в трёх километрах
к востоку от места, где она случилась?
(По уровню трудности соответствует задаче 8.)
Задача 16*. Бусы состоят из 34 бусинок. Известно, что любые
две бусинки, между которыми находятся ровно две бусинки, одного
цвета. Докажите, что все бусинки в этих бусах одного цвета.
(Эта задача труднее, чем задача 9.)
Занятие 13

Знакомство с кубом

Первое занятие по стереометрии


Это и следующее занятие посвящены стереометрии (простран­
ственной, или трёхмерной геометрии). Может показаться стран­
ным, что мы начинаем заниматься стереометрией раньше, чем пла­
ниметрией (геометрией на плоскости). Но мы живём в трёхмерном
мире и естественным образом приобретаем способность ориенти­
роваться в нём. Важно научиться анализировать этот мир и пред­
ставлять его в виде изображений на двумерных поверхностях (на
бумаге или на экране компьютера).
Поэтому детям полезно развивать навыки пространственного
мышления — способности представлять себе предметы в простран­
стве, совершать с ними воображаемые действия и соотносить их
с двумерными изображениями.
Стереометрия —настоящий кладезь задач, достаточно трудных,
чтобы вызвать серьёзный интерес, и в то же время не требующих
специальных знаний. Решение этих задач развивает пространствен­
ную интуицию, навыки логических рассуждений применительно
к трёхмерным телам и способность к абстрактному моделированию
в пространстве.
Наконец, учителя и родители, как правило, незаслуженно обхо­
дят стереометрию своим вниманием. Большинство школьников по­
лучают на удивление мало возможностей развить пространствен­
ное воображение. Научившись отлично решать сложные уравнения,
они затрудняются представить себе шестиугольное сечение куба.
Итак, на следующих занятиях мы приступаем к изучению бо­
гатейшего мира стереометрии. Мы познакомимся с несколькими
трёхмерными телами, научимся работать с сечениями и начнём
осваивать пространственное мышление.
Тема сегодняшнего исследования — трёхмерный куб. Кубическая
форма — одна из самых простых и наглядных, к тому же мы посто­
янно встречаемся с её воплощениями в повседневной жизни.
110 Раздел 1. Планы занятий

Что принести на урок


• Большой куб для преподавателя.
• Кубик для каждого участника. (Подойдут, например, детские ку­
бики или игральные кости. Я пользуюсь кубиками из азбуки дочки.)
• Две большие развёртки куба, вырезанные из бумаги (см. с. 113).
• Несколько пар ножниц (для тех участников, которым при реше­
нии задач на развёртку куба проще вырезать настоящие развёртки).
• Упражнения, распечатанные в достаточном количестве экзем­
пляров. (Помимо традиционной подборки задач, в этот раз понадо­
бятся распечатки для самостоятельной практической работы с ку­
бами и их развёртками на занятии.)
Для преподавателей* Это первое занятие после математической олимпиа­
ды, поэтому у нас осталось много неразобранных задач. В день олимпи­
ады не было времени обсудить задачи из двух предыдущих подборок,
и, кроме того, не обсуждались задачи самой олимпиады. Что лучше:
разобрать эти задачи сейчас или забыть про них?
Как показывает мой опыт, дети не горят желанием возвращаться
к прошлому. Если начать обсуждать все неразобранные задачи, занятие
получится скучным. Разумный компромисс заключается в том, чтобы
разобрать только самые интересные задачи олимпиады. Те задачи из
подборок, которые важны для дальнейшего, можно задать заново, из­
менив их формулировку.

Математическая разминка
Задача для разминки 1. Паломник идёт из Авиньона в Вати­
кан. Навстречу ему попадаются семеро крестьян. Каждый крестья­
нин несёт корзину с гусем и мешок с тремя кошками. Сколько всего
людей и животных направляются в Ватикан?
Задача для разминки 2. Почему лондонский парикмахер счи­
тает, что лучше подстричь двух французов, чем одного немца?

Тема занятия:
«Кубы»
Сначала я предлагаю дать определение куба. Как объяснить ино­
планетянину, что такое куб? (Мы считаем, что инопланетянин уже
познакомился с основными понятиями геометрии на плоскости.
Поэтому общими усилиями мы формулируем такое определение:
куб — это фигура с шестью равными квадратными гранями.)
Занятие 13. Знакомство с кубом 111
Это подходящий момент, чтобы раздать участникам кубики и на­
чать разговор об основных свойствах куба. Сколько у куба граней?
Сколько вершин? Сколько рёбер?
Как мы видим куб? Сколько граней видны одновременно? Мож­
но ли повернуть куб так, чтобы видеть ровно три грани? Да, это
нетрудно. Можно ли видеть одновременно только две грани? Или
одну? Да, всё это возможно. А можно ли увидеть сразу четыре грани
или больше? Нет, не получается.
А теперь следующий вопрос: как изобразить куб на плоской дву­
мерной поверхности—листе бумаги? Имеет ли смысл рисовать куб
в положении, в котором видна только одна грань? Тогда на рисунке
будет просто квадрат. А в положении, в котором видны только две
грани? Получится два четырёхугольника, имеющие общую сторону.
Что, если нарисовать куб в положении, в котором видны три грани?
Похоже, что такой рисунок будет наиболее содержательным...
Обычно куб изображают так, чтобы были видны сразу три грани.
Принято рисовать куб так, как на первой картинке слева:

А на картинке справа показана так называемая каркасная модель


куба: то, что останется от куба, если убрать все грани и всё, что
внутри, оставив только рёбра. (Можно также представить себе куб,
сделанный из стекла.)
В чём различие между этими двумя рисунками? Какой из них
нам больше подходит?
Для урока рисования, конечно, лучше левый рисунок: именно
так выглядит настоящий куб. Но у него есть один недостаток: на
нём не видны скрытые рёбра и задние грани. Поэтому для занятий
по математике каркасная модель удобнее.
Но и каркасная модель не лишена недостатков. Глядя на нашу кар­
тинку каркасной модели куба, можете ли вы сказать, какая грань пе­
редняя? Оказывается, на этот вопрос есть два различных ответа:
112 Раздел 1. Планы занятий

Настроив по-разному фокусировку глаз, можно увидеть два раз­


ных куба: на одном передняя грань находится слева внизу, а на дру­
гом — справа вверху.
Для преподавателей. Чтобы лучше продемонстрировать разницу между
этими двумя вариантами, нарисуйте на доске две одинаковые каркас­
ные модели куба и закрасьте передние грани этих кубов разными цве­
тами.
Эта неоднозначность лежит в основе многочисленных оптических
иллюзий, т. е. рисунков, на которых можно разглядеть две разные ком­
позиции. Но математика — строгая наука, в ней важны точность и опре­
делённость, и каждый рисунок должен иметь однозначное толкование.
Поэтому надо исправить каркасную модель так, чтобы было понятно,
какие грани на самом деле видны, а какие скрыты.
Самое удобное — изображать скрытые рёбра не сплошными, а штри­
ховыми линиями. Тогда вместо неоднозначной каркасной модели (распо­
ложенной в центре предыдущего рисунка) получаются два таких изобра­
жения:

Здесь сплошными линиями показаны видимые рёбра куба, а штри­


ховыми линиями — скрытые. Так мы будем теперь изображать кубы
и другие трёхмерные тела.
Упражнение 1. Для этого упражнения у каждого участника дол­
жен быть куб.
а) Возьмите куб. Поворачивая руку, попробуйте расположить
куб так, чтобы увидеть его, как показано на левом рисунке:

б) Это же упражнение для правого рисунка.


Упражнение 2. Сделайте то же самое для этих рисунков:
Занятие 13. Знакомство с кубом 113

Для преподавателей. Эти безобидные на вид упражнения могут потребо­


вать значительного времени. Возможно, придётся объяснить каждому
в отдельности (особенно детям помладше), как понимать рисунки и как
держать куб. Но результат стоит затраченного времени: такая трениров­
ка пространственного воображения очень важна для участников.

Развёртки куба
Мы переходим к обсуждению двумерных фигур, из которых мож­
но сложить трёхмерные объекты. Нам понадобятся две бумажные
вырезки, форма которых показана на рисунке. Каждая вырезка со­
стоит из шести одинаковых квадратов и может быть сделана из од­
ного листа формата А4:

Покажите участникам первую вырезку. Можно ли сложить из неё


куб так, чтобы каждый квадрат стал его гранью? Да, можно (жела­
тельно, чтобы этот ответ дали сами участники). Вот как это дела­
ется. (Сгибы лучше наметить заранее.) Сначала сложим длинную
полоску, состоящую из четырёх квадратов, в вертикальную четы­
рёхугольную «трубу» с одним выступающим квадратом сверху и од­
ним снизу. Теперь закроем эти выступающие квадраты как крышки:
получатся верхняя и нижняя грани куба.
Мы только что сложили из двумерной фигуры трёхмерный куб.
Вопрос о том, из каких двумерных фигур можно сложить трёхмер­
ные тела, лежит в основе многих интересных задач. Перед тем как
перейти к этим задачам, введём несколько определений.
Определение 1. Развёртка — это двумерная фигура, из которой
можно сложить трёхмерное тело.
Определение 2. Развёртка куба — это двумерная фигура, из ко­
торой можно сложить куб.
Для преподавателей. На этом этапе мы считаем, что все развёртки куба
состоят из шести одинаковых квадратов. Такие развёртки получаются
разрезанием куба по нескольким рёбрам и последующим развёртыва­
нием.
114 Раздел 1. Планы занятий

Разрезая куб через грани, можно получить более сложные развёрт­


ки. Но мы отложим работу с ними на будущее.
Если фигура состоит из шести одинаковых квадратов, примыкаю­
щих сторонами друг к другу, — значит ли это, что из неё всегда можно
сложить куб? Оказывается, это не так. Примером может служить вторая
вырезка на предыдущем рисунке — полоска из шести квадратов, распо­
ложенных в ряд. Почему из неё нельзя сложить куб? Повернём полос­
ку так, чтобы её длинные стороны были расположены горизонтально,
а плоскость бумаги — вертикально. Теперь попробуем сложить из неё
куб. Как бы мы ни сгибали полоску, все линии сгиба будут оставаться
вертикальными. Значит, и грани будут только вертикальными: ни один
из квадратов не станет верхней или нижней гранью куба.

Для преподавателей. Для упрощения доказательства мы выбрали кон­


кретную исходную ориентацию, но в действительности она не имеет
значения: любую процедуру получения куба из развёртки можно видо­
изменить так, чтобы она начиналась с этой ориентации.
Теперь можно перейти к самостоятельной работе. Чем больше прак­
тического опыта обращения с трёхмерными телами, тем лучше. На этом
занятии, помимо домашнего задания в виде подборки задач, участни­
кам раздаются материалы для работы на кружке. Это упражнения на
двумерное представление куба и на его развёртку.

Упражнения для самостоятельной работы


«Знакомство с кубом»
Упражнение 1. На рисунке показаны два одинаковых эскиза куба.
Завершите каждый из них так, чтобы получились разные виды куба:

Для первого эскиза расположите модель куба так, чтобы были


видны передняя, правая и верхняя грани. (Можно положить кубик
на стол слева от себя.) Сделайте из эскиза рисунок куба так, как вы
его видите.
Для второго эскиза расположите модель куба так, чтобы были
видны передняя, левая и нижняя грани. (Можно держать кубик в ру­
ке перед собой, немного справа от себя и выше уровня глаз.) Сде­
лайте из эскиза рисунок куба так, как вы его видите.
Занятие 13. Знакомство с кубом 115

Упражнение 2. Какие из показанных на рисунке двумерных фи­


гур — развёртки куба? Другими словами, какие из них можно сло­
жить так, чтобы получился куб?

Упражнение 3. Марианна сделала несколько моделей куба из


бумажных вырезок. На каждом сложенном кубе она пометила гра­
ни буквами: верхнюю грань —буквой В, нижнюю — буквой Н, все
боковые — буквой Б. Затем Марианна снова развернула свои модели
и сохранила развёртки.
Но Белла — младшая сестра Марианны — стёрла некоторые бук­
вы. Теперь развёртки выглядят так:

Восстановите недостающие буквы.

Подборка задач
Задача 1. а) Из 10 монет одна фальшивая: она легче настоящих.
Как найти фальшивую монету за три взвешивания на чашечных ве­
сах без гирь?
б) Из 16 монет одна фальшивая: она легче настоящих. Как най­
ти фальшивую монету за три взвешивания на чашечных весах без
гирь?
в) Из 27 монет одна фальшивая: она легче настоящих. Как най­
ти фальшивую монету за три взвешивания на чашечных весах без
гирь?
Задача 2. У булочника Василия есть большой мешок гречневой
муки, весы с двумя чашами и одна килограммовая гиря. Как ему
отмерить 7 килограммов муки за три взвешивания?
116 Раздел 1. Планы занятий

Задача 3. Какие из этих фигур — развёртки куба (из каких мож­


но сложить куб)?

Задача 4. Какие кубы, показанные на рисунке, можно сложить


из развёртки, изображённой слева?

Задача 5. а) Можно ли разрезать квадрат на 9 квадратов мень­


шего (не обязательно одинакового) размера? Если да, то как? Если
нет, то почему? (Квадраты, состоящие из других квадратов, не счи­
таются.)
б) Тот же вопрос для 10 квадратов.
в) Тот же вопрос для 11 квадратов.
Задача 6. Белоснежка решила сшить лоскутное одеяло. Она взя­
ла большой квадратный кусок ткани и разрезала его на четыре оди­
наковых квадрата. Оставив их на столе, Белоснежка ушла на кухню.
Первый гном, проходивший мимо стола, взял один из квадратов. Он
разрезал его на четыре одинаковых квадрата и положил их обратно
на стол. Все остальные гномы (второй, третий, четвёртый, пятый,
шестой и седьмой) сделали то же самое: каждый из них выбрал ку­
сок ткани и разрезал его на четыре меньших квадрата. Сколько кус­
ков ткани было на столе, когда Белоснежка вернулась в комнату?
Задача 7. Хромой, но весёлый кузнечик прыгает по прямой ли­
нии. При каждом прыжке он перемещается на 3 сантиметра вправо
или на 5 сантиметров влево.
Занятие 13. Знакомство с кубом 117

а) Может ли кузнечик прыгать так, чтобы оказаться на один сан­


тиметр правее исходной точки? А на один сантиметр левее? Если
это возможно, укажите последовательность прыжков. Если нет, объ­
ясните почему.
б) Кузнечик прыгает 20 раз. Может ли он оказаться на один сан­
тиметр правее исходной точки?
в) Кузнечик прыгает 23 раза. Может ли он оказаться на 10 сан­
тиметров левее исходной точки?
Занятие 14

Сечения

Второе занятие по стереометрии


Мы продолжаем изучение стереометрии. Сегодня участники по­
знакомятся с понятием сечения и узнают, как выглядят сечения ку­
ба и некоторых других трёхмерных тел.

Что принести на урок


Некоторые из этих предметов знакомы детям по прошлому заня­
тию.
• Большой куб для преподавателя.
• Кубик для каждого участника. (Подойдут, например, кубики из
детских наборов или деревянных конструкторов.)
• Нож и тарелка или доска для резки.
• Соевый сыр (тофу) в полукилограммовой упаковке.
• Яблоко.
• Несколько пар ножниц (для тех участников, которым при реше­
нии задач на развёртку куба проще вырезать настоящие развёртки).

Математическая разминка
В прошлый раз мы говорили о трудностях, которые возникают
при изображении трёхмерных тел на двумерных рисунках. Мы так­
же отметили, что восприятие и интерпретация этих рисунков че­
ловеком порождают множество разнообразных оптических иллю­
зий. Лучшая разминка для сегодняшнего занятия — показать детям
несколько таких иллюзий.
Оптическая иллюзия 1. На этом рисун­
ке можно разглядеть либо шесть, либо десять
кубов. Если рассматривать белые ромбы как
верхние грани кубов, то их будет шесть. Ес­
ли представить себе, что белые ромбы —это
нижние грани кубов, то их окажется десять.
Занятие 14. Сечения 119

Оптическая иллюзия 2. На этом ри­


сунке можно увидеть куб, лежащий в уг­
лу комнаты. Посмотрев по-другому, вы уви­
дите висящий в воздухе куб, из которого
в нижнем переднем углу вырезан куб мень­
шего размера. (Нижняя поверхность это­
го неполного куба —тёмный многоуголь­
ник в виде буквы V.) Есть и другие вариан­
ты интерпретации этого рисунка.
Оптическая иллюзия 3. На рисунке по­
казана двумерная проекция несуществую­
щего трёхмерного куба. Почему такого ку­
ба не существует? Если закрыть правую
часть рисунка, то видна левая половина ку­
ба. Если закрыть левую часть, то видна пра­
вая половина куба. Обе половины нарисо­
ваны совершенно правильно. Но это половины разных кубов! (По­
пробуйте также отдельно посмотреть на верхнюю и нижнюю поло­
вины.)
Для преподавателей. В этой книге приведены рисунки небольшого разме­
ра. На занятиях лучше показывать большие рисунки, например, с веб­
сайта этой книги. Множество таких рисунков можно найти в Интернете
или в любой книге по оптическим иллюзиям.
Если искать в Интернете изображения по запросу «оптические ил­
люзии», получатся миллионы результатов. Все они по-своему интересны,
но большинство не имеет отношения к нашей теме — стереометрии. Для
этой разминки лучше всего перейти в расширенный поиск и искать чёрно-
белые изображения по ключевым словам «оптические иллюзии, куб».
Для преподавателей. Ещё один вариант для этой или следующей размин­
ки—показать участникам несколько графических работ М. К.Эшера.
Он изобразил множество невозможных конструкций. Двумерные фигу­
ры на его рисунках интерпретируются нашим мозгом как трёхмерные
тела, которых в действительности не существует. Можно принести аль­
бом или найти изображения в Интернете.

Тема занятия:
«Сечения»
Обсуждение сегодняшней темы лучше всего начать с яблока. Со­
общите участникам, что яблоко служит моделью трёхмерного те-
120 Раздел 1. Планы занятий

ла — шара. Отрежьте кусок яблока прямым разрезом. (Чтобы разрез


не прошёл через сердцевину, нож нужно приставить к яблоку в двух-
трёх сантиметрах от черенка.) Объясните детям, что вы только что
получили сечение шара.
Что именно называется сечением? Отрезанный кусок? Нет. Се­
чение — это новая плоская поверхность, появившаяся в результате
разреза.
Какое сечение может получиться при разрезании яблока? Если
не резать слишком близко к сердцевине, сечение всегда будет иметь
форму круга.
Новое понятие — сечение — относится не только к яблокам и да­
же не только к шарам. Если взять любое трёхмерное тело и разре­
зать его одним прямым движением ножа, то полученная двумерная
поверхность тоже будет называться сечением.
Во избежание путаницы надо ещё раз подчеркнуть некоторые
моменты.
• Чтобы получить сечение, надо сделать один-единственный пря­
мой разрез через всё тело. Нельзя останавливаться посередине, из­
менять наклон ножа во время разреза и вырезать замысловатые фи­
гурки несколькими разрезами.
• Сечение — это не отрезанный кусок, а плоская двумерная по­
верхность, появившаяся в результате разреза.
Пример 1. Какие виды сечений шара можно получить с помо­
щью ножа, сделав один прямой разрез?
РАЗБОР ПРИМЕРА 1. Одним разрезом шара в сечении можно полу­
чить только круг. Другими словами, все сечения шара —это круги
(разных размеров).
Продемонстрировать это можно с помощью другого яблока.
А лучше понять этот факт поможет рассуждение. Интуитивно яс­
но, что если разрезать шар по вертикали (или по горизонтали),
то в сечении обязательно получится круг. Это менее очевидно для
разрезов, сделанных под углом. Но разрезать шар под углом —это
то же самое, что повернуть его и затем сделать вертикальный
(или горизонтальный) разрез. Поэтому любое сечение шара будет
кругом.
Следующая серия задач служит двум целям. Участники попро­
буют представить себе сечения более сложного трёхмерного тела
и поупражняются в рисовании таких сечений.
Занятие 14. Сечения 121

Пример 2. Представим себе один прямой разрез куба ножом.


Очевидно, что в сечении получится какой-то многоугольник. Попро­
буем ответить на несколько вопросов об этом сечении.
• Можно ли разрезать куб так, чтобы получилось квадратное се­
чение?
• Может ли получиться прямоугольное, но не квадратное сече­
ние?
• Может ли получиться треугольное сечение?
• А сечение в форме трапеции?
• А сечение в форме многоугольника, у которого больше четырёх
сторон?
Нарисуйте все сечения, которые вы сможете придумать.
РАЗБОР ПРИМЕРА 2. Над квадратным сечением участники долго не
думают. Если бы куб был сделан из сыра, можно было бы отрезать
тонкий ломтик — и в сечении получился бы квадрат.
Следующий шаг — нарисовать это сечение. Как это сделать? Мож­
но просто нарисовать на доске квадрат. Но такой рисунок ничего не
говорит о том, как именно мы разрезали куб, чтобы получить такое
сечение. Поэтому обычно рисуют трёхмерное тело вместе с сечени­
ем, как на правом рисунке.

Перейдём к вопросу о сечении в форме прямоугольника, кото­


рый не является квадратом. Первая реакция участников: получить
такое сечение невозможно. Но у кого-то появляется идея: сделаем
вертикальный разрез, проходящий через две противоположные вер­
шины верхней грани.
Поскольку некоторым участникам трудно представить себе та­
кой разрез, я достаю упаковку соевого сыра. Твёрдый соевый сыр —
отличный материал для изучения сечений. Из него удобно вырезать
простые трёхмерные тела, которые легко разрезаются. В стандарт­
ной полукилограммовой упаковке — четыре кубика. Этого как раз
хватит для сегодняшнего занятия.
Я делаю разрез по диагонали, как на следующем рисунке слева,
и торжественно демонстрирую гладкий прямоугольный срез. Это
производит ожидаемое впечатление. Участники сразу же отмечают,
122 Раздел 1. Планы занятий

что при других вертикальных разрезах тоже получатся прямоуголь­


ники, как на рисунке справа. Мы уделяем некоторое время попыт­
кам правильно изобразить эти сечения на доске.

Следующий вопрос —о треугольном сечении. В первый момент


дети снова уверены, что такого сечения быть не может. Но Эмма,
немного поразмыслив, предлагает отрезать угол куба (рисунок сле­
ва). Чуть позже Дэвид находит треугольник большего размера: его
вершины совпадают с вершинами куба (рисунок справа). Мы про­
веряем обе идеи на кубике соевого сыра и рисуем сечения на доске.

Вопрос о сечении в форме трапеции ставит в тупик всех. (Неко­


торым детям нужно напомнить, что такое трапеция.) Приходится
дать подсказку: как и в случае треугольного сечения, разрез должен
быть наклонным. После этого участники предлагают несколько ва­
риантов, которые сразу же проверяются на новом кубике из тофу:

Следующий вопрос —о сечении, имеющем больше четырёх сто­


рон — воспринимается как очевидная нелепость. Куб имеет настоль­
ко «прямоугольную» форму, что такое сечение трудно себе предста­
вить. Но Эмма довольно быстро придумывает шестиугольное сече­
ние. Затем появляется и пятиугольный вариант:
Занятие 14. Сечения 123

В заключение я ставлю вопрос о сечениях, в которых больше ше­


сти сторон. Несколько неудачных попыток придумать такое сече­
ние убеждают участников в том, что его не существует. Как обычно,
я предлагаю объяснить это. После недолгого обсуждения кому-то
приходит в голову верная идея: каждая сторона сечения получается
в результате пересечения секущей плоскости с одной из граней ку­
ба. Но у куба только шесть граней, поэтому сечение не может иметь
больше шести сторон.
На этом разговор о сечениях заканчивается. Подводя итоги, я ещё
раз говорю о том, в чём заключается основная трудность этой те­
мы: думать приходится в пространстве, а рисовать — на плоскости.
Затем раздаётся очередная подборка задач.
Для преподавателей. Одна из важных целей этого занятия— практика
в рисовании трёхмерных тел сложной формы. Поэтому пользы будет го­
раздо больше, если сечения будут рисовать на доске сами участники.
Рисунки детей далеки от совершенства, но обнаружение и исправление
ошибок — само по себе полезное упражнение для остальных участников.

Подборка задач
Задача 1. На рисунке показаны два игральных кубика и их раз­
вёртки. Сколько точек на следующих гранях:
• на нижней грани каждого кубика;
• на задней (скрытой) грани каждого кубика;
• на левой (скрытой) грани каждого кубика?

Задача 2. На столе находится куб. Он касается поверхности стола


одной вершиной, а противоположная вершина направлена вертикаль­
но вверх. Прямо над кубом висит лампа. Как выглядит тень от куба?
Задача 3. Вершины треугольника на рисунке совпа­
дают с вершинами куба. Найдите углы этого треуголь­
ника. (Полезно будет напомнить, что сумма углов тре­
угольника равна 180°.)
124 Раздел 1. Планы занятий

Задача 4. Изобретатель Макс развернул у себя дома компьютер­


ную сеть, соединив все компьютеры в квартире новыми сверхско­
ростными кабелями. (В сети нет маршрутизаторов; каждый кабель
соединяет пару компьютеров; два компьютера могут быть соедине­
ны несколькими кабелями.) От компьютера на кухне отходит пять
кабелей, от каждого из двух компьютеров в комнате Макса — по три
кабеля, от каждого из пяти папиных компьютеров — по два кабе­
ля, а от маминого ноутбука — один кабель. Сколько всего кабелей
в этой сети?
Задача 5. Представьте себе, что сплошной верти­
кальный цилиндр разрезали плоскостью. Нарисуйте
сечения всех видов, которые могут получиться.
Задача 6. Король древней страны Фигурии прика­
зал своему придворному математику Прямоугольни-
усу придумать способ разрезать квадрат на любое число меньших
квадратов начиная с шести. (Квадраты, состоящие из других квад­
ратов, не считаются).
Математик работал день и ночь. В конце концов он решил задачу
короля. Но позднее рукопись с его решением пострадала во время
пожара. От неё осталось всего несколько строк. Вот они:
«Сначала покажем, как разрезать квадрат на 7, 8 и 9 меньших
квадратов: ...Чтобы разрезать квадрат на большее число квадратов,
надо...»
Восстановите решение Прямоугольниуса.
Задача 7*. Есть три бутылки: у одной круглое горлышко, у дру­
гой — квадратное, а у третьей — в форме равнобедренного треуголь­
ника. На рисунке показана форма каждого горлышка, если смотреть
сверху. Обратите внимание на то, что сторона квадрата, диаметр
круга, высота и ширина треугольника — равные величины.

а) Придумайте затычку, которая подходила бы ко всем трём бу­


тылкам.
б) Придумайте затычку, которая подходила бы ко всем трём бу­
тылкам и которую можно было бы протолкнуть внутрь каждой бу­
тылки.
Занятие 14. Сечения 125

Для преподавателей. Это трудная задача. В моем кружке я поступаю так:


на одном занятии предлагаю задачу, на следующем даю подсказку, без
которой обычно не обойтись даже самым способным участникам, и на
третьем окончательно объясняю решение, демонстрируя реальную мо­
дель (см. с. 307).
Задача 8. Тетраэдр — это пирамида с треугольным основанием
и треугольными боковыми гранями. Есть ли у тетраэдра прямо­
угольное сечение?
Занятие 15

Математический аукцион II

Сегодня мы проводим второй математический аукцион. Правила


аукциона приведены на с. 232. Более подробно об этой игре говорится
в главе, посвященной первому математическому аукциону (с. 46).

Что принести на урок


• Клетчатая бумага для рассеянных участников, забывших её дома.
• Трёхмерные модели для разъяснения задач на сечения из преды­
дущей подборки (с. 124). Об этих моделях подробно рассказывается
в главе с решениями начиная со с. 306.
• Подарки и призы для участников соревнования. Это, конечно,
не обязательно. Но дети будут рады скромным наградам—леден­
цам или шоколадкам.

Математическая разминка
Задача для разминки 1. Представьте себе, что вы водитель
школьного автобуса. На первой остановке вы запускаете в авто­
бус трёх мальчиков и двух девочек, на второй — четырёх мальчиков
и одну девочку. На третьей остановке в автобус садятся три девочки.
Сколько лет водителю?
Задача для разминки 2. Два разбойника хотят поделить на­
грабленное. Каждый из них считает, что он в состоянии разделить
добычу на две равноценные части. Но ни один из них не верит, что
его коллега разделит добычу по-честному. Как им поделить добычу,
чтобы каждый был уверен, что получил причитающуюся ему поло­
вину?
Занятие 15. Математический аукцион II 127

Событие дня:
игра «Математический аукцион»
Поскольку сегодняшний аукцион — только второе соревнование
такого рода, стоит кратко напомнить участникам правила игры.
Важно отметить следующие аспекты.
• Одна и та же задача выставляется на торги несколько раз, пока
есть команда, которая готова предложить лучшее решение. (Часто
дети ошибочно думают, что задачу можно выставить повторно толь­
ко один раз.) В частности, каждая команда имеет право представ­
лять разные решения одной и той же задачи несколько раз подряд.
• Стоимость каждой задачи достаётся одной команде — той, ко­
торая предложила наилучшее решение. Все команды (включая по­
бедившую в этом раунде) теряют свои ставки.
• Каждый член команды имеет право рассказать решение только
одной задачи. Это правило не действует, если задач больше, чем
членов команды. В любом случае разные решения одной задачи (ес­
ли она повторно выставляется на торги) может рассказывать один
и тот же участник.
А ведущему полезно помнить вот о чём.
• Обязательно прочитайте все задачи вслух, позаботившись о том,
чтобы дети действительно слушали (а не бросились тут же решать
первую задачу). Подчеркните, что на этапе решения задач можно
задавать вопросы. Неверно понятое условие может стать причиной
горького разочарования.
• Величина любой ставки должна делиться на пять. (Иначе тор­
говля затянется до бесконечности.)
• Пользуйтесь любой возможностью сэкономить время. Напри­
мер, заранее сделайте на доске все рисунки, которые могут приго­
диться. Дети, как известно, рисуют не слишком быстро. Например,
для задачи 1 можно нарисовать несколько квадратов 4 х 4, а для за­
дачи 2 — несколько планок с делениями на равном расстоянии (рас­
сказывая решение, участник обводит нужные деления жирной ли­
нией).
• При планировании времени разумно отвести десять минут на
подготовку (формирование команд, напоминание правил), 30 ми­
нут—на решение задач и 30 минут —на сам аукцион.
• Напомню, что в этой книге не приведены решения задач аук­
циона.
128 Раздел 1. Планы занятий

Задачи для математического аукциона


Каждая задача стоит 100 шмолларов (см. с. 232).
Задача 1. Придумайте как можно больше способов разделить
квадрат 4 х 4 на две равные части (одинаковые по форме и по раз­
меру). Резать разрешается только по линиям сетки (а не по диа­
гоналям клеток). Две части считаются равными, если их можно
наложить друг на друга так, чтобы они совпали. При этом их можно
поворачивать и переворачивать. Разрезать квадрат больше чем на
две части не разрешается.
Решение считается лучше предыдущего, если команда придумы­
вает фигуру, не продемонстрированную другими командами. (Пер­
вая команда показывает свои варианты. Остальные команды могут
демонстрировать те варианты, которых ещё не было.)
Задача 2. Имеется деревянная планка длиной 20 сантиметров.
Надо отметить на планке несколько делений так, чтобы она могла
служить линейкой. Это значит, что с её помощью можно отмерить
любое целое расстояние в сантиметрах от 1 до 20.
Отметьте на планке как можно меньше делений так, чтобы для
любого числа от 1 до 20 нашлась пара делений, между которыми
ровно столько сантиметров.
Решение считается лучше предыдущего, если предложен вариант
с меньшим числом делений.
Задача 3. Новая шахматная фигура «кентавр» похожа на ко­
ня: она ходит буквой Г и может перепрыгивать через другие фи­
гуры. Единственное отличие: размер буквы Г —не 2 х 3, как у коня,
а 2x4:

конь кентавр

Расположите на шахматной доске как можно больше кентавров


так, чтобы они не били друг друга.
Решение считается лучше предыдущего, если оно позволяет рас­
положить на доске больше кентавров.
Задача 4. Получите число 100 только из восьмёрок. Восьмёрок
должно быть как можно меньше.
Занятие 15. Математический аукцион II 129

Можно пользоваться арифметическими действиями «+», «—»,


«.», «ч-» и возведением в степень. Можно расставлять скобки. Каж­
дое действие может встречаться несколько раз. Не обязательно ис­
пользовать все действия. Не разрешается составлять из восьмёрок
многозначные числа, такие как 88 и 888.
(Пример. Число 16 можно получить из четырёх восьмёрок: 16 =
О _|_ о
= 8- —g—, а можно —всего из двух: 16 = 8 + 8.)
Решение считается лучше предыдущего, если оно позволяет по­
лучить число 100 из меньшего количества восьмёрок.

Подборка задач
Задача 1. В здании музея странностей 18 этажей. В лифте этого
здания всего две кнопки. При нажатии первой кнопки лифт подни­
мается на 9 этажей вверх, а при нажатии второй — опускается на
7 этажей вниз. (Если лифт должен оказаться выше 18-го или ниже
первого этажа, он остаётся на месте.)
а) Администратор здания находится на втором этаже. Сможет ли
он с помощью лифта попасть на первый этаж? Если да, то как? Если
нет, то почему?
б) Сможет ли он попасть с первого этажа на второй?
в) Новая уборщица Мария Петровна собирается помыть полы на
всех этажах, куда она сможет добраться на лифте. Сможет ли она
помыть полы на всех этажах здания, если начнёт с первого?
Задача 2. Вася с родителями едет из Москвы в Бабушкинск, где
живёт его бабушка. В субботу семья села на поезд, а через день —
в понедельник —приехала в Бабушкинск. По дороге Вася сделал
несколько наблюдений. В понедельник он заметил, что число меся­
ца совпадает с номером вагона. Ещё он обратил внимание на то, что
номер его места меньше номера вагона. Когда Васина семья сади­
лась в поезд, число месяца было больше номера вагона. Определите
номер вагона и номер места Васи.
Задача 3. В зоопарке есть пруд, где растут кувшинки. Первого
июня в пруду появилась первая кувшинка. С тех пор количество кув­
шинок каждый день увеличивалось в два раза. 15 июня кувшинками
покрылась вся поверхность пруда. Если бы первого июня вместо од­
ной кувшинки появилось две, какого числа пруд был бы полностью
покрыт кувшинками?
130 Раздел 1. Планы занятий

Задача 4. В стране драконов живут


трёхглавые и пятиглавые драконы. Одна­
жды в магазин головных уборов зашли
20 драконов. Они заказали себе огнен­
ные шляпы —по одной на каждую голо­
ву. Хозяин магазина считает, что ему на­
до изготовить 11 огненных шляп. Дока­
жите, что он ошибся в вычислениях.
Задача 5. Пятиклассник Денис при­
шёл в гости к своей подруге Тане, кото­
рая живёт на самом верхнем этаже 20-этажного дома. Он вызвал
Таню на соревнование: кто быстрее спустится с верхнего этажа на
первый этаж и вернётся обратно, если Таня будет ехать на лифте,
а Денис — бежать по лестнице?
Лифт движется вверх и вниз с одной и той же постоянной скоро­
стью. Денис бежит вниз в два раза быстрее лифта, а вверх —в два
раза медленнее лифта. Кто выиграет соревнование?

Д О П О Л Н И Т Е Л Ь Н Ы Е ЗАДАЧИ

Задача 6. На столе лежат четыре карточки. На верхней стороне


каждой карточки написана буква или цифра: на первой —цифра 4,
на второй — цифра 5, на третьей — буква А, на четвёртой — буква Б.
Неизвестно, написано ли что-нибудь на нижних сторонах карточек
и что именно. Макс утверждает: «Если на одной стороне карточки
написано чётное число, то на другой её стороне написана гласная
буква». Какое наименьшее количество карточек нужно перевернуть,
чтобы проверить, прав ли Макс? Какие карточки надо перевернуть?
Занятие 16

Комбинаторика I

Комбинаторика — искусство подсчёта — увлекательная и полез­


ная тема, к тому же широко представленная в математических кон­
курсах для средних классов1. Однако именно её популярность со­
здаёт определённую трудность для преподавателя. Детям, активно
участвующим в математических соревнованиях, сегодняшние зада­
чи могут показаться слишком простыми. Но остальным, не знако­
мым с понятиями комбинаторики, необходима пара вводных заня­
тий, чтобы освоиться с этой темой.
Чтобы смягчить эту ситуацию, можно чередовать комбинаторику
с другими темами: тогда её знатокам не придётся скучать несколько
занятий подряд. Другой вариант — подобрать для них более слож­
ные задачи. В этой главе приведены ссылки на обширные источни­
ки комбинаторных задач.

Что принести на урок


• Трёхмерные модели для разбора задач на сечения из позапро­
шлой подборки (если эти задачи не разобраны на прошлом заня­
тии, когда проводился математический аукцион). Об этих моделях
подробно говорится в главе с решениями на с. 306.

Математическая разминка
Задача для разминки 1. Что находится в середине пустоты?
Задача для разминки 2. Получите верное равенство, переме­
стив только одну цифру:

101-102 = 1.
1
В США комбинаторные задачи широко представлены в «массовых» школьных олим­
пиадах, так как в таких олимпиадах используются задачи с числовыми ответами.
132 Раздел 1. Планы занятий

Тема занятия:
«Введение в комбинаторику»
Сегодняшнее занятие посвящено основным правилам подсчёта.
Для их усвоения мы решим серию задач. Если группе нужна допол­
нительная практика, всегда можно придумать аналогичные задачи.
Лучше всего начать с задач про города и дороги: они решаются
с помощью рисунков, которые помогают наглядно представить ос­
новные понятия комбинаторики.
Пример 1. В Стране чудес три города, отмеченные на схеме бук­
вами А, В и С. Города А и В соединены тремя дорогами, а города В
и С—двумя. Сколько существует маршрутов для поездки из А в С?
Двигаться можно только в направлении стрелок —слева направо.

РАЗБОР ПРИМЕРА 1. Несколько участников дают правильный от­


вет: 6. Я прошу обосновать этот ответ. Рассуждения кружковцев
сводятся к двум идеям: «перечислить и подсчитать все возможные
маршруты» и «умножить 3 на 2».
Первую идею легко проверить: мы составляем на доске список
всех возможных маршрутов. Их действительно шесть.
Почему умножение 3 на 2 тоже приводит к верному решению?
Какую бы дорогу из А в В мы ни выбрали, у нас есть два способа
добраться из В в С. Другими словами, каждая дорога из А в В слу­
жит начальным этапом двух маршрутов из А в С. Значит, количество
маршрутов из А в С в два раза больше количества дорог из А в В.
Пример 2. В Стране чудес построили новый город D и проло­
жили четыре дороги из С в D. Сколько существует маршрутов для
поездки из А в D? Двигаться можно только слева направо.

РАЗБОР ПРИМЕРА 2. Все дают один и тот же ответ (3 • 2 • 4) и пред­


лагают одинаковое объяснение: маршрутов из А в D больше, чем из
Занятие 16. Комбинаторика I 133

А в С, в четыре раза. Моё предложение решить задачу путём пере­


числения всех возможных маршрутов решительно отвергается: это
займёт слишком много времени.
Для преподавателей. При разборе этой задачи стоит поговорить о том,
какой из способов решения удобнее: перечисление всех возможных ва­
риантов или сокращённый подсчёт, например, с помощью умножения.
Для простых задач вполне подходит перебор вариантов. Если задача бо­
лее сложная, иногда тоже полезно начать перебирать варианты: это мо­
жет навести на мысль о закономерности, позволяющей быстро полу­
чить ответ. Однако довести перебор до конца становится затруднитель­
ным. Представьте себе, что в задаче 2 больше четырёх городов. Тогда пе­
речислить все маршруты практически невозможно. Поэтому, когда вы
решаете комбинаторную задачу, всегда нужно пытаться найти удобный
и эффективный способ сокращённого подсчёта.
Ещё один полезный комментарий: решить сложную задачу по ком­
бинаторике часто помогает упрощённый вариант этой же задачи. На­
пример, если уже известно количество маршрутов из А в С, то нетрудно
подсчитать количество маршрутов из А в D.
Пример 3. В Стране чудес построили ещё один новый город (Z)
и несколько новых дорог. Карта дорог показана на следующем ри­
сунке. Сколько теперь существует маршрутов для поездки из А в D?
(Правило остаётся прежним: двигаться можно только в направле­
нии стрелок.)

РАЗБОР ПРИМЕРА 3. На этот раз дети дают множество разных от­


ветов (первым появляется ответ 48). Я записываю ответы на доске
и предлагаю их авторам обосновать свои точки зрения. Точку в дис­
куссии ставит участник, убедительно объяснивший ответ 26: есть
24 маршрута из А в D через В и С и ещё 2 маршрута через Z.
Для преподавателей. Этот участник фактически разбил задачу на две бо­
лее простые задачи: о количестве маршрутов А—В—С—D и о количе­
стве маршрутов A—Z—D. Чтобы получить ответ, надо сложить ответы
134 Раздел 1. Планы занятий

к этим двум задачам. Разбиение задачи на несколько независимых за­


дач — один из полезных методов комбинаторики.
Для преподавателей. Вопрос «складывать или умножать?» поначалу мо­
жет сбивать детей с толку. Решив несколько похожих задач, где требу­
ется и умножение, и сложение, участники научатся делать правильный
выбор.
Задачи о городах и дорогах естественным образом связаны с рисун­
ками, помогающими увидеть путь к решению. Для остальных задач это­
го занятия нарисовать подходящую схему не так просто, но это прине­
сёт неоценимую пользу в развитии наглядного мышления. Поэтому при
разборе следующих задач мы будем уделять время различным способам
их графического представления.
Пример 4. В Стране чудес открылся магазин чая. Там продают­
ся чайные чашки трёх видов и чайные ложки двух видов. Сколькими
способами можно составить набор из одной чашки и одной ложки?
РАЗБОР ПРИМЕРА 4. Чашку можно выбрать тремя способами. Для
каждой чашки можно выбрать ложку двумя способами. Это простое
рассуждение приводит к ответу: 3-2 = 6.
Посмотрим, как можно представить эту задачу наглядно.
Первый способ — свести её к уже известной задаче о городах и до­
рогах. Для этого возьмём схему из задачи 1 и нарисуем на трёх доро­
гах из А в В разные чашки, а на двух дорогах из В в С — разные ложки:

Представим себе, что мы едем на машине из А в С. По пути из А


в В мы подбираем чашку, а по пути из В в С—ложку. Таким образом,
разные маршруты из А в С соответствуют разным сочетаниям чашки
и ложки. Другими словами, количество всех возможных сочетаний
чашки и ложки равно количеству всех возможных маршрутов из А
в С. Поэтому к ответу приводит то же действие, что и в задаче 1:
умножение 3 на 2.
Занятие 16. Комбинаторика I 135

Второй способ — нарисовать схему в виде дерева возможностей:

корень

чашка 1 чашка 2 чашка 3

ложка 1 ложка 2 ложка 1 ложка 2 ложка 1 ложка 2

Подобные схемы, которые так и называются — деревьями, широ­


ко применяются в математике, информатике и других науках. (Как
вы легко можете заметить, математические деревья слегка отлича­
ются от настоящих: они растут не вверх, а вниз. Как вы думаете,
почему?)
Пользуясь случаем, введём некоторые термины из области мате­
матических деревьев.
Дерево состоит из вершин, соединённых ветвями (на рисунке
ветви показаны стрелками). Оно начинается с одной вершины, ко­
торая называется корнем. Вершины, непосредственно соединённые
с корнем, называются вершинами первого уровня. (У нашего дерева
есть три вершины первого уровня — чашки 1, 2 и 3.) Вершины, нахо­
дящиеся на уровень ниже, называются вершинами второго уровня,
и так далее. (Все ложки — вершины второго урвоня.) Все вершины,
в которые можно попасть из данной вершины (двигаясь в направ­
лении стрелок), называются его потомками. Вершины из которых
не выходят ветви, называются листьями.
Покончив с терминологией, обсудим, каким образом дерево мо­
жет помочь в решении задачи о магазине чая. Нарисованная схема
соответствует такому порядку выбора: сначала чашка, а затем лож­
ка. Корень дерева отмечает начало процедуры выбора. Три ветви,
ведущие от корня к вершинам-чашкам, означают три возможных
способа выбрать чашку. Ветви, начинающиеся в чашке, соответству­
ют всем возможным способам выбрать ложку, если чашка уже вы­
брана.
Таким образом, каждому сочетанию ложки и чашки можно сопо­
ставить некоторый путь от корня к вершине-ложке (вершине вто­
рого уровня). Поэтому способов выбрать набор из ложки и чашки
столько же, сколько путей от корня к вершинам второго уровня.
136 Раздел 1. Планы занятий

Число таких путей в два раза больше, чем число путей к вершинам
первого уровня (а таких путей ровно три). Как и следовало ожидать,
мы получили ту же формулу и тот же ответ: 3-2 = 6.
Для преподавателей. Мы только что разобрали две схемы, позволяющие
графически представить задачи по комбинаторике этого типа. Каждая
из них имеет свои преимущества, и выбор между ними — вопрос лич­
ного предпочтения. Детям обычно понятнее схема «городов и дорог».
К тому же нарисовать её проще, чем дерево.
Так как эти две модели демонстрируют два различных способа на­
глядного представления задач, при разборе каждой задачи сегодняш­
него занятия рекомендуется рисовать обе схемы. В науке часто бывает
полезно посмотреть на ситуацию с нескольких точек зрения. Умение
интерпретировать одну и ту же задачу разными способами — важное ка­
чество будущего исследователя.
Пример 5. В магазине чая в Стране чудес продаются чайные
чашки трёх видов, чайные ложки двух видов и чай четырёх сортов.
Сколькими способами можно составить подарок из одной чашки,
одной ложки и одной пачки чая?
РАЗБОР ПРИМЕРА 5. Эта задача решается аналогично предыдущей.
Задачу 5 можно представить графически с помощью схемы городов
и дорог (воспользовавшись рисунком к задаче 2) или в виде дерева
(рисовать придётся долго). ОТВЕТ: 3 • 2 • 4 = 24.
Пример 6. У Наташи есть две большие вазы — одна с белыми
шарами, другая с чёрными. Сколькими способами она может выло­
жить четыре шара в ряд? (Каждый способ —это определённая по­
следовательность цветов. Если обозначить белый цвет буквой «Б»,
а чёрный — буквой «Ч», то, например, ЧЧЧЧ, БЧБЧ и ЧББЧ — разные
способы.)
РАЗБОР ПРИМЕРА 6. Эта задача того же типа, что и все предыду­
щие. Однако большинство участников не замечают сходства, пото­
му что на этот раз мы имеем дело с группами одинаковых предме­
тов. Дети предлагают разные ответы, в том числе и правильный.
Эту задачу можно упростить, превратив её в серию задач воз­
растающей сложности. Сначала мы перечисляем способы выложить
один шар, затем два шара в ряд, затем три. Каждый раз мы запи­
сываем все возможные сочетания: для одного шара —Б и Ч, для
д в у х - Б Б , БЧ, ЧБ и ЧЧ, для т р ё х - Б Б Б , ББЧ, БЧБ, БЧЧ, ЧББ, ЧБЧ,
ЧЧБ и ЧЧЧ. Когда мы переходим к четырём шарам, большинство
Занятие 16. Комбинаторика I 137

участников уже замечают закономерность: при добавлении каждо­


го следующего шара количество способов увеличивается в два раза.
Для преподавателей. Выписывая все возможные последовательности цве­
тов, мы убеждаемся в том, как важно перечислять их в определённом
порядке. Чем длиннее ряды шаров, тем легче запутаться в многочислен­
ных вариантах и пропустить один из них. Поэтому в ходе разбора этой
задачи я предлагаю детям порядок записи вариантов, позволяющий из­
бежать таких ошибок (см. следующий пункт).
Продолжим разбор задачи: исследуем два способа графического
представления задачи 6.
• Нарисовать схему «города и дороги» нетрудно. Она показана
на рисунке: это ряд из пяти городов, в котором каждые два сосед­
них города соединены двумя дорогами. На каждой такой паре дорог
помещены два шара разных цветов.

Каждый маршрут из А в Е соответствует определённому способу


выложить четыре шара в ряд. Например, маршрут, показанный чёр­
ными стрелками, — это последовательность цветов ББЧБ. Поэтому
количество таких последовательностей равно количеству маршру­
тов и з А в Е : 2 - 2 - 2 - 2 = 16.
• Схему «дерево» легко представить себе, но трудно нарисовать.
В таком дереве, помимо корня, есть четыре уровня вершин. Из каж­
дой вершины (кроме листьев) ветви ведут к двум другим вершинам.

Упорядоченное перечисление возможных вариантов


Искусство комбинаторики состоит в том, чтобы придумать упро­
щённый способ подсчёта. Однако в некоторых случаях —например,
в предыдущей задаче — требуется записать длинный список всех
возможных вариантов. Если записывать их в том порядке, как они
приходят в голову, очень легко пропустить один или два варианта.
Поэтому важно продемонстрировать детям упорядоченный способ
записи.
138 Раздел 1. Планы занятий

Покажем его на примере задачи про шары.


Предположим, что мы уже выписали все последовательности
цветов определённой длины. Как разумным способом получить спи­
сок последовательностей, длина которых на единицу больше?
Заметим, что из каждой имеющейся последовательности можно
получить ровно две новые последовательности: справа от уже выло­
женного ряда шаров мы добавляем либо белый, либо чёрный шар.
Таким образом, имея полный список вариантов для определён­
ного числа шаров, нетрудно получить полный список для следую­
щего числа. Для этого нужно переписать каждый вариант из гото­
вого списка два раза, в первый раз добавляя в конце букву Б, а во
второй — букву Ч. Теперь у нас есть правило упорядоченного пере­
числения вариантов.
• Сначала выписываем все «последовательности длины 1».
• Поочерёдно составляем списки всех последовательностей дли­
ны 2, 3, 4, ..., пока не дойдём до нужной длины. Каждый раз по­
следний составленный список служит основой для нового списка,
который должен получиться при добавлении одного шара.
Чтобы проиллюстрировать это правило, перечислим все возмож­
ные способы выложить в ряд четыре шара, сначала составив списки
для одного, двух и трёх шаров.
• Один шар: Б и Ч.
• Два шара: ББ, БЧ, ЧБ, ЧЧ.
• Три шара: БББ, ББЧ, БЧБ, БЧЧ, ЧББ, ЧБЧ, ЧЧБ, ЧЧЧ.
• Четыре шара: ББББ, БББЧ, ББЧБ, ББЧЧ, БЧББ, БЧБЧ, БЧЧБ,
БЧЧЧ, ЧБББ, ЧББЧ, ЧБЧБ, ЧБЧЧ, ЧЧББ, ЧЧБЧ, ЧЧЧБ, ЧЧЧЧ.
Между прочим, варианты в полученном списке расположены
в алфавитном порядке. Можете ли вы объяснить, почему так по­
лучилось?
Для преподавателей, 1. В комбинаторике понимание приходит с практикой.
Давайте детям новые задачи, пока они не усвоят идею по-настоящему. Ни­
же приведено ещё несколько задач; при необходимости их можно приду­
мать сколько угодно. Книги [1, 2, 3, 22] и интернет-проект [25] служат
отличными источниками комбинаторных задач любого уровня сложности.
Для преподавателей. Перед тем как перейти к следующим задачам, умест­
но напомнить о том, что к каждой из них полезно нарисовать схему.
Рисунки делают задачу наглядной и помогают найти правильный ответ.
Когда мы научимся уверенно решать такие задачи, мы сможем обхо­
диться без рисунков.
Занятие 16. Комбинаторика I 139

Пример 7. Миша взял с собой в летний лагерь две пары боти­


нок, две пары шорт и шесть футболок. Сколькими способами Миша
сможет одеться в лагере? (Комплект одежды состоит из пары шорт,
футболки и пары одинаковых ботинок.)
Пример 8. Алиса взяла с собой в летний лагерь четыре пары
туфель, две юбки, пять блузок и три платья. Сколькими способами
Алиса может одеться в лагере? (Комплект одежды состоит либо из
юбки, блузки и пары одинаковых туфель, либо из платья и пары
одинаковых туфель.)
Пример 9. Белла пришла с мамой в магазин игрушек, чтобы
выбрать Максу подарок на день рожденья. В магазине есть фигурки
рыцарей трёх видов, коней трёх видов и пиратов двух видов, а ещё
есть игрушечный корабль (одного вида). Мама разрешает купить
либо рыцаря и коня, либо пирата и корабль. Сколькими способами
Белла может выбрать подарок для Макса?
Пример 10. Маша зашла в кафе-мороженое. В кафе продаются
молочные коктейли четырёх видов, мороженое десяти видов и ва­
фельные рожки трёх видов. У Маши хватит денег либо на коктейль,
либо на мороженое в вафельном рожке. Сколькими способами она
может выбрать себе угощение?
Пример 11. У Марка есть мешок с зелёными шарами, мешок
с красными шарами и мешок с синими шарами. Сколькими спосо­
бами он может выложить в ряд два шара? Три шара? Четыре шара?

Подборка задач
Задача 1. Медвежонок полез на дерево за мёдом и упал на зем­
лю. Его друг Ёжик — начинающий кинорежиссёр — снял падение
Медвежонка на камеру. Теперь Ёжик собирается озвучить свой
фильм. В каждый момент, когда Медвежонок задевает ветку де­
рева, Ёжик планирует добавить звук: «Ой-ой-ой!», «Ай-ай-ай!» или
«Бамс!»
а) Сколько разных звуковых дорожек может сделать Ёжик, если
во время падения Медвежонок задел две ветки?
б) А если Медвежонок задел три ветки? Четыре ветки? Пять
веток?
Задача 2. Четыре города на острове Буяне соединены дорогами,
как показано на рисунке. Сколько существует различных маршру-
140 Раздел 1. Планы занятий

тов из А в С? (Можно двигаться только в направлении стрелок.)

Задача 3. Алфавит планеты Бавгабав состоит всего из четырёх


букв: А, Б, В и Г. На этой планете две страны: Страна Говорунов
и Страна Болтунов.
а) Имена жителей Страны Болтунов —любые трёхбуквенные сло­
ва, в которых нет буквы Г. (Словом считается любое сочетание букв.
Например, АБВ и ББА—имена болтунов.) Сколько разных имён мо­
жет быть в этой стране?
б) Фамилии жителей Страны Болтунов—любые четырёхбуквен­
ные слова, оканчивающиеся буквой Г. (Например, БААГ и ВВВГ —
фамилии болтунов.) Сколько разных фамилий может быть в этой
стране?
в) Имена жителей Страны Говорунов —любые слова длиной не
более трёх букв, в которых нет буквы Г. (Например, А и ВАВ — имена
говорунов.) Сколько разных имён может быть в этой стране?
г) Фамилии жителей Страны Говорунов —любые четырёхбук­
венные слова ровно с одной буквой Г, которая находится либо в на­
чале, либо в конце слова. (Например, ГААА и БВБГ —фамилии го­
ворунов.) Сколько разных фамилий может быть в этой стране?
Задача 4. Иван-Царевич хочет освободить Ва­
силису Прекрасную, которая томится в замке. На
воротах замка есть десять кнопок с цифрами от 0
до 9. Чтобы открыть ворота, надо набрать сек­
ретный код —четыре цифры в определённом по­
рядке. Ворота охраняет голодный дракон Паш­
ка, который любит сосиски. Когда Пашка ест со­
сиску, он не замечает ничего вокруг себя. Иван-
Царевич пробует одно сочетание цифр за одну
секунду, а Пашка съедает одну сосиску за 20 секунд. Когда Иван
откроет ворота, ему понадобится ещё одна минута, чтобы подняться
в башню к Василисе и улететь с ней на ковре-самолёте.
Занятие 16. Комбинаторика I 141

а) Сколько сосисок надо взять с собой Ивану, чтобы улететь


целым и невредимым вместе с Василисой? (Запас сосисок должен
быть рассчитан на худший случай, когда Ивану придётся перебрать
все возможные сочетания и только последнее из них окажется пра­
вильным.)
б) Сколько сосисок понадобится Ивану, если он заранее знает,
что код состоит только из нечётных цифр?
в) Сколько сосисок понадобится Ивану, если он заранее знает,
что код состоит только из нечётных цифр и в нём есть ровно одна
цифра 5?
Задача 5. Давным-давно на острове рыцарей и лжецов жили три
друга: Рональд, Дональд и Арчибальд. Все они были храбрыми во­
инами, но только один из них был рыцарем. Одним из их много­
численных подвигов была битва с драконом, наводившим ужас на
всё королевство. Об этой битве доподлинно известно только одно:
дракона убил рыцарь. Историки нашли в старинном архиве письмо,
в котором Рональд утверждает, что дракона убил Арчибальд. Кто на
самом деле убил дракона?
Задача 6. У Аладдина есть два кувшина — трёхлитровый и пя­
тилитровый. Он хочет занять у своего соседа Хасана четыре литра
оливкового масла. У Хасана есть восьмилитровый кувшин, полный
масла, но ни у одного из соседей не нашлось мерной кружки. Помо­
гите Аладдину отмерить нужное количество масла, пользуясь толь­
ко двумя пустыми кувшинами и полным кувшином Хасана.
Задача 7. Разрежьте каждую из фигур, показанных на рисунке,
на две части, одинаковые по форме и размеру. Можно резать только
по линиям клетчатой бумаги и по диагоналям клеток.

Д О П О Л Н И Т Е Л Ь Н Ы Е ЗАДАЧИ
Задача 8*. Двадцать чисел расположены по кругу так, что каж­
дое из них равно среднему арифметическому двух соседних чисел.
Докажите, что все эти числа равны.
142 Раздел 1. Планы занятий

Задача 9*. Робот нарисовал на клетчатой бумаге белый квадрат


4 х 4 и закрасил левую верхнюю клетку чёрным цветом. Вы можете
выбрать в этом квадрате любой вертикальный или горизонтальный
ряд и дать роботу команду перекрасить все клетки в этом ряду в про­
тивоположный цвет.
Можно ли с помощью нескольких таких команд добиться того,
чтобы весь квадрат стал чёрным? Приведите последовательность
команд или докажите, что её не существует.
Занятие 17

Комбинаторика II
На этом занятии мы закрепим и расширим знания по комбина­
торике, полученные в прошлый раз. Мы познакомимся с двумя важ­
ными идеями: правилом произведения и понятием размещений без
повторений. Специально подобранная серия комбинаторных задач
поможет усвоить материал. Если приведённых здесь задач недоста­
точно, нетрудно придумать аналогичные.
Математическая разминка
Задача для разминки 1. Продавая на рынке попугая, старый
моряк расхваливал птицу: «Он повторяет каждое слово, которое
слышит!» Один молодой человек поверил моряку и купил попугая.
Он разговаривал с попутаем несколько дней, но тот не проронил ни
слова. Однако моряк сказал правду. Как такое может быть?
Задача для разминки 2. Стае и Тарас хотят купить по одина­
ковому мороженому. Но Стасу не хватает трёх рублей, а Тарасу—
35 рублей. Тогда они решают скинуться и купить одно мороженое
на двоих. Но им всё равно не хватает трёх рублей! Сколько стоит
мороженое?
Тема занятия:
«Правило заполнения пропусков
и размещения без повторений»
В комбинаторике трудно обойтись без рисунков. В прошлый раз
они помогли нам понять одну из разновидностей комбинаторных
задач и освоить быстрый способ подсчёта. Но рисовать карты «горо­
да и дороги» и схемы-деревья — долгое занятие. Можно ли выразить
суть задачи более простым способом?
Такой способ есть: он называется правилом произведения (или
правилом заполнения пропусков). Мы не будем вводить строгого
определения, а проиллюстрируем это правило на примерах.
Пример 1. Чтобы поцеловать Спящую красавицу, принц должен
пересечь тёмный лес, преодолеть ров, окружающий замок, и под-
144 Раздел 1. Планы занятий

няться на башню, где лежит Спящая красавица. Принц может про­


браться через лес на коне, на велосипеде или пешком. Он может
переправиться через ров на лодке, на спине гигантской черепахи
или на водном мотоцикле. Наконец, он может подняться к Спящей
красавице по канату, закреплённому на её окне, или по винтовой
лестнице. Сколькими способами принц может добраться до Спящей
красавицы?
РАЗБОР ПРИМЕРА 1. Сначала мы решаем эту задачу методом «го­
рода и дороги». Для этого рисуем такую схему:

комната
опушка леса край рва стена замка принцессы

пешком на лодке по канату


на вело­ на чере­
сипеде пахе
на мото­ по лест­
на коне цикле нице

Сформулируем вопрос так: «Сколько различных маршрутов со­


единяют опушку леса с комнатой Спящей красавицы?» Чтобы прий­
ти к ответу, решим серию более простых задач.
• Сколькими способами можно добраться от опушки леса до
края рва? (Тремя.)
• Сколькими способами можно добраться от опушки леса до сте­
ны замка? (Три умножаем на три: каким бы способом ни добрался
принц до края рва, он может выбрать один из трёх способов преодо­
леть ров.)
• Сколькими способами можно добраться до комнаты Спящей
красавицы? (Умножаем предыдущий результат на два: для каждого
способа достичь стены замка существует два варианта подняться
в комнату.)
Получаем ответ: 3 - 3 - 2 = 18.
Чтобы понять правило заполнения пропусков, представим себе
репортёра, который наблюдает за принцем и записывает все его
действия. Репортёр принёс с собой такую заготовку:
«Принц пробрался через лес , переправился через
ров и поднялся на башню ».
Занятие 17. Комбинаторика II 145

Пропущенные слова в этом мини-репортаже — это способы пе­


ресечь лес, преодолеть ров и забраться на башню. Если заполнить
все пропущенные места, получится полное описание маршрута от
опушки леса до комнаты Спящей красавицы.
Каждый пропуск можно заполнить несколькими способами. Для
первого места есть три варианта заполнения (пешком, на коне и на
велосипеде), для второго — тоже три варианта (на лодке, на чере­
пахе и на водном мотоцикле), для последнего — два варианта (по
канату и по лестнице). Отметим, что пропуски заполняются неза­
висимо друг от друга: вариант, выбранный для одного пропуска, не
влияет на возможности заполнения остальных. Каким бы способом
принц ни пробрался через лес, он может выбрать любой из трёх
способов переправы через ров.
Теперь мы знаем, для чего нужны пропуски, и можем изобразить
ещё одну схему этой задачи, на которой показаны пропуски и вари­
анты. Сначала рассмотрим полную схему:
пешком лодка
велосипед черепаха лестница
конь мотоцикл канат

ЛЕС РОВ БАШНЯ


Прежде всего заметим, что схема пропусков и вариантов по су­
ти не отличается от схемы городов и дорог (см. рисунок на с. 144).
Посмотрим, например, на пропуск «лес». Три варианта заполнения
этого пропуска (конь, велосипед и пешком) — это три «дороги», по
которым можно пересечь лес. Это же относится и к двум другим
этапам пути: рву и башне.
Возникает вопрос: чем же схема «пропуски и варианты» лучше? За­
чем она нам? Дело в том, что составлять полный список вариантов во­
все не обязательно: имеет значение только количество вариантов. По­
этому схема «пропуски и варианты» может выглядеть гораздо проще:
3 варианта 3 варианта 2 варианта

ЛЕС РОВ БАШНЯ


Мы уже знаем ответ к задаче про принца: 3 - 3 - 2 . Каждый из этих
трёх множителей равен количеству вариантов заполнения одного
из пропусков. Совпадение? Конечно, нет.
146 Раздел 1. Планы занятий

В этой задаче не имеет значения, какие именно варианты есть


у принца: важно только их количество. Если вместо трёх способов
пробраться через лес (пешком, на коне и на велосипеде) у принца
будут другие три способа (на самокате, на роликах и на машине),
ответ к задаче не изменится. Поэтому в модели пропусков можно
убрать ненужные подробности и оставить только то, что влияет на
ответ: количество вариантов заполнения каждого пропуска.
Эта упрощённая модель хороша тем, что она подходит для са­
мых разнообразных задач. Нам уже встретилось несколько задач,
похожих на задачу про принца: про магазин чая, про ряд из чёрных
и белых шаров и другие (с. 132 и дальше). Каждую из них можно
переформулировать, получив схему городов и дорог такого типа:

В такой формулировке цель задачи всегда одна: подсчитать ко­


личество маршрутов, соединяющих начальную точку с конечной.
И когда мы решаем задачу такого типа, мы всегда рассуждаем оди­
наково и приходим к одной и той же конечной формуле: сосчитать,
сколько дорог соединяет каждую пару соседних городов, и перемно­
жить эти числа.
Поэтому для обширного класса комбинаторных задач не нужны
сложные рисунки: вся необходимая информация содержится в про­
стой схеме «пропуски и варианты». Более того, не нужно заново
обосновывать способ подсчёта для каждой задачи. Достаточно опре­
делить пропуски, подсчитать количество вариантов их заполнения
и перемножить эти числа. Этот сокращённый способ подсчёта на­
зывается правилом произведения.
Для преподавателей. Строго говоря, рассуждения в этом пункте не явля­
ются доказательствами. Их можно перевести на точный математиче­
ский язык, но тогда они будут слишком сложными для детей. У это­
го занятия другая цель — развитие комбинаторной интуиции. Участники
должны понимать, почему правило произведения даёт верный ответ, и на­
учиться распознавать задачи, которые можно решить по этому правилу.
Следующие несколько задач решаются с помощью правила про­
изведения.
Пример 2. В ресторане «Детский рай» огромный выбор блюд.
На закуску предлагаются салат из леденцов и листья шпината с шо-
Занятие 17. Комбинаторика II 147

коладными жучками. На второе можно заказать фасоль из джема,


шоколадных лягушек, копчёные лапки пряничных мишек, яичницу
из пасхальных яиц, жареную пастилу, вафельную капусту и мака­
роны в соусе из мороженого. На десерт подаются печёные яблоки
с мармеладными червячками, леденцы в рисовой бумаге и скорпи­
оны на палочках. Сколькими способами можно выбрать в «Детском
раю» обед из трёх блюд — закуски, второго и десерта?
РАЗБОР ПРИМЕРА 2. Сначала определим пропуски. Их три: закус­
ка, второе и десерт. Для первого пропуска есть два варианта запол­
нения, для второго — семь, для третьего — три. ОТВЕТ: 2 • 7 • 3 = 56.
2 варианта 7 вариантов 3 варианта

ЗАКУСКА ВТОРОЕ ДЕСЕРТ

Пример 3. Каждое утро принцесса Белла должна выполнить од­


ну из государственных обязанностей: пойти на заседание парламен­
та или провести пресс-конференцию. Сегодня принцесса заполня­
ет свой рабочий календарь на ближайшую неделю. Сколькими спо­
собами она может запланировать свои утренние государственные
обязанности?
РАЗБОР ПРИМЕРА 3. Каждый день недели —это пропуск в кален­
даре. Для каждого пропуска есть два варианта заполнения — пар­
ламент и пресс-конференция, причём пропуски заполняются неза­
висимо друг от друга. Правило заполнения пропусков даёт ответ:
2 - 2 - 2 - 2 - 2 - 2 2 = 2 7 = 128.
парламент парл. парл. парл. парл. парл. парл.
конференция конф. конф. конф. конф. конф. конф.

ДЕНЬ! ДЕНЬ 2 ДЕНЬ 3 ДЕНЬ 4 ДЕНЬ 5 ДЕНЬ 6 ДЕНЬ 7

Пример 4. Лена — подруга Светы — только что вернулась из по­


ездки по Европе с родителями. Зная, что Света любит сладости, Ле­
на привезла ей в подарок три пакетика: один с французскими шоко­
ладками, один с немецкими карамельками и один с итальянскими
мармеладками.
Мама Светы убрала подарки и разрешила Свете есть только три
конфеты в день: одну на завтрак, одну на обед и одну после ужина.
148 Раздел 1. Планы занятий

Каждое утро Света записывает, какие конфеты она собирается се­


годня съесть и в каком порядке. Сколькими способами она может
составить этот список?
РАЗБОР ПРИМЕРА 4. В этой задаче три пропуска: завтрак, обед
и ужин. Варианты заполнения каждого пропуска одни и те же: фран­
цузская конфета, немецкая конфета и итальянская конфета. Для
каждого пропуска есть 3 варианта заполнения. ОТВЕТ: 3-3-3 = 27.
Во всех задачах, которые мы решали до сих пор, каждый пропуск
заполняется независимо от остальных. Теперь перейдём к задачам
другого типа, в которых каждое принятое решение влияет на воз­
можности последующего выбора.
Пример 5. Однажды утром мама сообщила Свете, что её кон­
феты почти закончились: остались только одна французская шоко­
ладка, одна немецкая карамелька и одна итальянская мармеладка.
Сколькими способами Света может в последний раз составить спи­
сок конфет?
РАЗБОР ПРИМЕРА 5. Света должна сделать выбор три раза: на зав­
трак, на обед и на ужин. В отличие от предыдущих задач, каждое
принятое ей решение влияет на последующий выбор: Света не мо­
жет выбрать одну и ту же конфету два раза. Поэтому вопрос стоит
так: сколькими способами можно составить список из трёх видов
конфет без повторений?
Поскольку эта задача относится к новому типу, нам придётся
придумать новый способ её анализа. Сначала нарисуем дерево:

Начало дня

Завтрак: франц. нем. итал.

Обед: нем. итал. франц. итал. франц. нем.

Ужин: итал. нем. итал. франц. нем. франц.

На этой схеме три вершины первого уровня, так как на зав­


трак Света может выбрать один из трёх видов конфет. Но на обед
у неё остаётся меньше вариантов — только два. Поэтому от каждой
вершины-завтрака идут всего две ветви к вершинам-обедам, ко-
Занятие 17. Комбинаторика II 149

торые соответствуют двум оставшимся конфетам. Например, если


Света выбрала на завтрак французскую шоколадку, на обед она
может выбрать немецкую или итальянскую конфету. Наконец, на
ужин у Светы остаётся только один вариант. Поэтому каждый обед
соединён только с одним ужином.
Каждый маршрут от корня до ужина соответствует одному из
способов расположить конфеты трёх видов в определённом поряд­
ке. Поэтому количество таких маршрутов и будет ответом к задаче.
Осталось подсчитать эти маршруты.
• Число способов перейти на первый уровень равно 3.
• Число способов перейти на второй уровень в 2 раза больше: 3 • 2.
• Число способов перейти на третий уровень такое же, что и на
второй: 3-2-1 = 6.
Эти шесть маршрутов можно сокращённо обозначить так: ФНИ,
ФИН, НФИ, НИФ, ИФН и ИНФ. Каждый маршрут —это способ за­
писать буквы Ф, Н и И в определённом порядке, используя каждую
букву только один раз.
Благодаря Свете мы только что познакомились с комбинаторны­
ми задачами нового типа. В таких задачах у нас есть определённое
количество разных предметов, например три конфеты или три бук­
вы алфавита. Требуется определить, сколькими способами можно
упорядочить эти предметы, т. е. расставить их в ряд.
В математике каждый способ упорядочить набор предметов на­
зывается размещением1. В этой задаче мы подсчитываем число раз­
мещений без повторений. Слово «размещение» в данном контек­
сте означает, что порядок расположения предметов имеет значение.
Слова «без повторений», как нетрудно угадать, указывают на то, что
каждый предмет можно взять только один раз.
Нам уже встречались задачи, где нужно подсчитать число спосо­
бов расположить несколько предметов в ряд, но в этом ряду может
быть несколько экземпляров каждого предмета. Это задачи на раз­
мещение с повторениями. Примерами могут служить первая задача
про Свету из сегодняшнего занятия и задача о чёрных и белых ша­
рах из прошлого занятия.
Стоит отметить, что не все комбинаторные задачи относятся
к этим двум типам. Можно вспомнить, например, первую задачу
1
В случае, когда все предметы должны быть использованы, размещения также
называют перестановками.
150 Раздел 1. Планы занятий

из прошлого занятия про города и дороги, задачи про магазин чая,


принца, «Детский рай» и другие.
Теперь вернёмся к нашей задаче на размещение без повторе­
ний — о последних трёх конфетах Светы. Мы решили её с помощью
дерева. Это помогло нам понять суть задачи и найти ответ. Однако,
как мы могли заметить, рисование деревьев занимает много време­
ни. Мы только что познакомились с упрощённым способом решения
комбинаторных задач — правилом заполнения пропусков. Что, если
применить это правило к задачам на размещение без повторений?
Получим ли мы правильный ответ?
Посмотрев на задачу более внимательно, мы увидим три пропус­
ка: завтрак, обед и ужин. Мы знаем, что первый пропуск (завтрак)
можно заполнить любым видом конфет. Мы не можем составить
список для заполнения остальных пропусков (обед и ужин): набор
вариантов зависит от того, что именно Света выбрала раньше. Но
количество вариантов для каждого пропуска известно: 2 для обеда
и 1 для ужина.
3 варианта 2 варианта 1 вариант

ЗАВТРАК ОБЕД УЖИН

Поэтому к этой задаче применимо правило произведения. Оно


даёт ту же формулу, что и схема: 3 - 2 - 1 .
Рассуждая точно так же, можно применить правило заполнения
пропусков к любой задаче на размещения без повторения. Это упро­
щает нашу жизнь. Пока мы только учимся решать такие задачи, нам
будут помогать деревья. Но уже скоро мы сможем быстро находить
ответ, пользуясь только правилом произведения.
Следующие задачи предлагаются для самостоятельного реше­
ния. Участники могут работать по отдельности или в небольших
группах. Если детям требуется дополнительная практика, нетрудно
придумать похожие задачи.
Пример 6. В солнечный воскресный день Робин Гуд собирается
сделать три вещи: сходить в гости к Мэриан, поупражняться в фех­
товании с Малюткой Джоном и поохотиться на оленя в Шервудском
лесу. Сколькими способами Робин может спланировать этот день?
Пример 7. Король Артур попросил королеву сшить шёлковое зна­
мя для рыцарского турнира. Он хочет, чтобы знамя состояло из четы-
Занятие 17. Комбинаторика II 151

рёх вертикальных полосок разных цветов: зелёного, чёрного, белого


и жёлтого. Сколькими способами королева может сшить знамя?
Пример 8. У Наташи есть пять кубиков разных цветов: крас­
ный, синий, жёлтый, розовый и зелёный. Сколькими способами она
может построить из этих кубиков вертикальную башню?
Для преподавателей. Ответ к комбинаторной задаче — это количество спо­
собов сделать что-то. Ответ в виде числа ничего не говорит о способе
подсчёта. Например, ответ к задаче про кубики — просто число 120. От­
вет будет более содержательным и лучше запомнится, если записать его
так: 5 • 4 • 3 • 2 • 1. Поэтому я всегда прошу участников писать в ответе не
числа, а формулы для подсчёта.
Для преподавателей. Если позволяет уровень подготовленности участни­
ков, можно ввести факториалы. Факториал — это сокращённое обозна­
чение (знаком «!») произведения нескольких последовательных целых
чисел начиная с 1. Например, 3! = 3-2-1; 5! = 5 • 4• 3• 2• 1. В комбина­
торных задачах факториалы часто упрощают запись ответов.

Подборка задач
Задача 1. Во владения великана вторглись четыре рыцаря в же­
лезных латах: сэр Аллистер, сэр Баллистер, сэр Валлистер и сэр Гал-
листер. Великан собирается сбросить их с коней одного за другим,
но ещё не решил, в каком порядке он это сделает. Сколькими спосо­
бами великан может сбросить всех рыцарей с коней?
Задача 2. В марсианском алфавите только шесть букв. Все мар­
сианские слова состоят из четырёх букв.
а) Предположим, что любая последовательность из четырёх
букв — осмысленное марсианское слово. Сколько слов в марсиан­
ском языке?
б) Предположим, что любая последовательность из четырёх
букв — осмысленное марсианское слово при условии, что буквы не
повторяются. Сколько слов в марсианском языке?
в) Предположим, что любая последовательность из четырёх
букв — осмысленное марсианское слово при условии, что хотя бы
одна буква повторяется. Сколько слов в марсианском языке?
(Подсказка. Пункт в) становится значительно проще, если уже
известны ответы к пунктам а) и б).)
Задача 3. Тимофею, Саше, Антону и Марине поручили органи­
зовать благотворительную лотерею. Они собираются напечатать
152 Раздел 1. Планы занятий

1000 лотерейных билетов, которые будут продаваться по 100 руб­


лей. На билетах будут указаны номера от 1 до 1000. Сейчас они
обсуждают, какие билеты будут считаться выигрышными.
• Тимофей предлагает выплачивать приз в 700 рублей по каж­
дому билету, номер которого состоит только из нечётных цифр (на­
пример, 11, 3, 199, 375, 111).
• Саша придумал другой вариант: выплачивать 800 рублей по
каждому билету, номер которого оканчивается цифрой 1 (напри­
мер, 31, 1, 191, 371, 111).
• Антон считает, что лучше всего объявить выигрышными биле­
ты, номера которых содержат ровно одну цифру 3 (например, 3, 31,
345, 131, 293), и назначить по ним приз в 300 рублей.
Мальчики начинают спорить: каждый из них считает, что его
план принесёт самую большую прибыль (если будут проданы все
билеты). Марина, которая занимается в математическом кружке,
делает какие-то подсчёты и объясняет, сколько денег можно будет
получить в каждом случае. Какой план Марина считает лучшим?
Задача 4. Медвежонок съедает бочонок мёда за 15 минут. Папа-
медведь съедает такой же бочонок в два раза быстрее. За сколько
времени они прикончат бочонок мёда вдвоём?
Задача 5. Таблица 3 x 3 заполнена нулями. За один
ход разрешается выбрать в ней любой квадрат 2 x 2
и увеличить каждое число в этом квадрате на 1. Можно
ли за несколько таких ходов получить таблицу, показан­
ную на рисунке? Если да, то как? Если нет, то почему?
Задача 6. Команда Солнечного города участвует в футбольном
турнире. По правилам турнира за каждую победу команде присуж­
дается одно очко, за каждое поражение снимается одно очко, а в слу­
чае ничьей количество очков не изменяется. За первые 20 матчей
команда Солнечного города набрала 10 очков. Затем она сыграла
ещё 25 матчей, и у неё стало 20 очков. Докажите, что хотя бы один
из последних 25 матчей закончился вничью.

Д О П О Л Н И Т Е Л Ь Н Ы Е ЗАДАЧИ
Задача 7. а) Придумайте четыре целых положительных числа,
сумма которых равна их произведению.
б) Придумайте 1000 целых положительных чисел, сумма кото­
рых равна их произведению.
Занятие 18

Математический хоккей II

Сегодня очередное игровое занятие: мы снова играем в матема­


тический хоккей. Поскольку участники уже знакомы с правилами
этой динамичной игры, она не должна занять больше половины все­
го занятия.

Математическая разминка
Задача для разминки 1. Британский офицер, служивший в Ин­
дии, вышел в отставку и вернулся домой. Однажды он заснул во
время церковной службы и увидел страшный сон. Ему приснился
палач, который занёс острый меч, чтобы отрубить ему голову. В этот
момент жена офицера решила разбудить мужа и коснулась его шеи
веером. Офицер пришёл в ужас и мгновенно умер от сердечного
приступа.
Эту печальную историю рассказала вдова офицера. Заслуживает
ли её рассказ доверия?
Задача для разминки 2. В одном племени была древняя тради­
ция: убивать всех путешественников, попавших на территорию это­
го племени. Сначала каждого пойманного путешественника спра­
шивали о цели его поездки. Если вождь племени считал, что путе­
шественник сказал правду, его сбрасывали со скалы. Если путеше­
ственник, по мнению вождя, солгал, его скармливали крокодилам.
Только один путешественник сумел придумать ответ, который спас
ему жизнь. Что он ответил?
Уточним, что путешественник не промолчал, а действительно
ответил на поставленный вопрос.
154 Раздел 1. Планы занятий

Событие дня:
игра «Математический хоккей»
Правила игры подробно изложены на с. 238—239. Здесь я напом­
ню самое главное.
• Разделите детей на две команды, приблизительно равные по
силе (но не обязательно по числу участников).
• Напомните участникам о необходимости ротации: каждый дол­
жен хотя бы один раз выступить у доски.
• Заранее обдумайте, как представить детям условия задач и ор­
ганизовать место для выступлений.
Сегодня на математическом хоккее предлагается восемнадцать
задач. Ответы приведены на с. 320.

Задачи для математического хоккея II


Задача 1. Света, Ира и Наташа съели конфету, зефир и пряник.
Света не ела конфету и зефир. Ира не ела зефир. Что съела каждая
девочка?
Задача 2. У учителя Математикова спросили, сколько ему лет.
«Я прожил 24 года, 24 месяца, 24 недели, 24 дня и 24 часа» — отве­
тил учитель. Сколько же ему лет? (Имеется в виду количество пол­
ных лет.)
Задача 3. На доске записаны числа 1, 2, 3, ..., 99, 100. Сколько
на доске цифр 5?
Задача 4. На доске написано выражение: 6 5 4 3 2 1 = 30. Рас­
ставьте в этом выражении знаки арифметических действий +,—,-,
: и скобки так, чтобы получилось верное равенство. Использовать
все четыре знака не обязательно. Один и тот же знак можно вста­
вить несколько раз. Не обязательно вставлять знаки между всеми
цифрами.
Задача 5. Один арбуз весит на три кило­
грамма меньше, чем три арбуза. Сколько весит
арбуз? (Все арбузы весят одинаково.)
Задача 6. У Макса три кармана и две оди­
наковые монеты. Сколькими способами он мо­
жет положить свои монеты в карманы?
Занятие 18. Математический хоккей II 155

Задача 7. Марина сидит на скамейке с мамой, бабушкой и кук­


лой. Бабушка сидит рядом с Мариной, но не рядом с куклой. Мама
сидит не рядом с куклой. Кто сидит рядом с мамой?
Задача 8. В этом сезоне наша футбольная команда сыграла 31
матч. Семь из них закончились вничью. За каждый выигрыш при­
суждалось 3 очка, за ничью — 1 очко, за проигрыш — 0 очков. Всего
команда набрала 64 очка. Сколько матчей она проиграла?
Задача 9. Сколько треугольников на рисунке?
Задача 10. Маша, Даша и Наташа пришли в гости
к Антону. Маша пришла на 5 минут раньше Даши, но
на 3 минуты позже Наташи. Наташе надо было уйти
первой. Она ушла за 3 минуты до Даши и за 5 минут до
Маши. На сколько минут Маша была у Антона дольше, чем Даша?
Задача 11. Сколько существует трёхзначных чисел, в которых
нет чётных цифр и все цифры разные? (Примеры таких чисел: 135,
351 и 975.)
Задача 12. У Макса в коробке лежат шары трёх цветов: красные,
белые и чёрные. Всего шаров 15. Белых шаров в 7 раз больше, чем
красных. Сколько чёрных шаров в коробке?
Задача 13. Кирилл учится плавать. В по­
недельник он смог проплыть 6 метров.
В каждый последующий день он удваивал
свой вчерашний результат. В день, когда он
проплыл 50 метров, мама купила ему моро­
женое. В какой день недели Кирилл получил
мороженое?
Задача 14. На острове рыцарей и лжецов живут два брата-близ­
неца—Тим и Том. Один из них лжец, а другой рыцарь. В день рож­
дения братьев Тим сказал гостям: «Теперь мне больше десяти лет!»
Том в тот же день заявил, что ему больше девяти лет. Сколько лет
близнецам?
Задача 15. Расшифруйте пример:
ААА + ВВ = СКВС.
(Разные буквы обозначают разные цифры, а одинаковые буквы —
одинаковые цифры.)
Задача 16. Четыре президента разных стран должны встать в оче­
редь, чтобы пожать руку победителям международной математиче-
156 Раздел 1. Планы занятий

ской олимпиады. Сколькими способами можно сформировать оче­


редь из президентов?
Задача 17. Какая из дробей on* oZ> 72 ближе в с е г о к
1?
Задача 18. У Сони неторопливый будильник: за каждый час
он отстаёт на три минуты. Сейчас на этом будильнике 11:41. Через
сколько минут на нём будет 12:00?

Подборка задач
Задача 1. Переложите две спички так, что­
бы дом был повёрнут в другую сторону (см.
рисунок).
Задача 2. У Васи четыре книги: учебник по
математике, учебник по русскому языку, книга
о Гарри Поттере и сборник кулинарных рецеп­
тов. Сколькими способами он может расставить эти книги в ряд на
полке?
Задача 3. Учитель написал на доске:
87654321=3
Затем он предложил вставить между некоторыми цифрами ми­
нус, чтобы получилось верное равенство. Как это сделать?
Задача 4. У золотоискателя Джека четыре золотых слитка. На
каждом слитке оттиснут его вес: 100, 200, 300 и 500 граммов. Джек
знает, что один слиток сделан из фальшивого золота, но не знает,
легче оно настоящего или тяжелее. Как Джеку определить фальши­
вый слиток за два взвешивания на чашечных весах? Никаких гирь,
кроме самих слитков, у него нет.
Задача 5. В древнем египетском городе Алексан­
дрии стоял храм Пи — бога математики и других точ­
ных наук. Святилище этого храма находилось в цен­
тре круга из тринадцати волшебных лампад. Когда
жрец входил в круг между двумя лампадами, состо­
яние этих двух лампад волшебным образом изменя­
лось: если лампада горела, то она гасла, а если не
горела, то загоралась. Никаким другим способом по­
гасить или зажечь лампады было невозможно.
Занятие 18. Математический хоккей II 157

Утром в день ежегодного праздника математики из тринадцати


лампад горят только две соседние лампады. Юный жрец пытается
зажечь все лампады. Сможет ли он это сделать? Если да, то как?
Если нет, то почему?
Занятие 19

Числовые ребусы I. Потерянные цифры

Потерянные цифры
Это занятие посвящено интересной и полезной теме — числовым
ребусам. Дети с удовольствием решают такие головоломки, а препо­
даватели видят в них возможность развить критическое мышление
и способность рассуждать логически. Кроме того, эти ребусы помо­
гают глубже понять свойства чисел и арифметические действия.
Большинство приведённых в этой книге ребусов может решить
любой участник кружка — конечно, при некоторой настойчивости.
Для каждой задачи важно найти правильное сочетание двух мето­
дов: перебора и логического анализа.

Математическая разминка
Задача для разминки 1. Этот фокус с тремя игральными куби­
ками взят из книги Мартина Гарднера «Mental Magic» [20].
Я выдаю участникам три игральных кубика — белый, фиолето­
вый и красный. (Важно, чтобы кубики были разными, иначе их лег­
ко перепутать.)
• Сначала я прошу участников бросить все три кубика на стол
и записать сумму очков, выпавших на белом и фиолетовом кубиках.
• Затем я предлагаю перевернуть фиолетовый и красный кубики
и записать сумму очков на этих двух кубиках.
• После этого я прошу перевернуть красный и белый кубики и за­
писать сумму очков на этих кубиках.
• Наконец я прошу детей сложить все три числа.
• В этот момент я разворачиваю листок бумаги, на котором за­
писано число. Ко всеобщему изумлению, это число равно сумме,
которую только что сосчитали участники. Я прошу детей объяснить,
как я угадала число.
Для преподавателей. В большой группе лучше принести несколько набо­
ров кубиков и разделить участников на подгруппы.
Если дети не разгадают секрет с первого раза, повторите фокус.
Занятие 19. Числовые ребусы I. Потерянные цифры 159

Задача для разминки 2. Купец купил товар за 7 рублей, продал


его за 8 рублей, затем снова купил его же за 9 рублей и продал за
10 рублей. Какую прибыль он получил?

Тема занятия:
«Потерянные цифры»
Пример 1. Гензель написал на доске равенство. Гретель стёрла
три цифры и заменила их звёздочками. Получилась такая запись:
* + * = *8
Восстановите исходное равенство.
Для преподавателей. Иногда детям приходится напомнить, что такое «циф­
ра»: это знак числа от 0 до 9. Числа составляются из цифр, как слова из
букв.
РАЗБОР ПРИМЕРА 1. Двузначное число в правой части равно сум­
ме двух однозначных чисел. Что мы можем узнать об этой сумме?
Каждое из слагаемых меньше 10. Значит, их сумма меньше 20. По­
скольку эта сумма оканчивается цифрой 8, она равна 18.
Теперь, когда мы знаем сумму, попробуем определить слагае­
мые. Если хотя бы одно из этих слагаемых меньше 9, то их сумма
меньше 18. Значит, каждое слагаемое равно 9.
Для преподавателей. Дети могут возразить: в этой задаче проще угадать
ответ, чем вывести его. Однако логическое рассуждение имеет важное
преимущество перед угадыванием: оно позволяет не только найти от­
вет, но и показать, что других решений не существует.
Приведённое решение иллюстрирует понятие оценки неизвест­
ного числа на основе имеющихся данных. Эта техника помогает ре­
шать не только числовые ребусы, но и многие другие задачи. Оцен­
ка, или граница, —это известное число, гарантированно большее
(или, наоборот, меньшее) числа, которое требуется найти. В данном
случае мы начали с оценки: число в правой части меньше 20.
Пример 2. Гретель написала на доске равенство. Гензель стёр
пять цифр и заменил их звёздочками. Получилась такая запись:
**5 — ** = 8.
Восстановите исходное равенство.
РАЗБОР ПРИМЕРА 2. Попробуем оценить величину двух неизвест­
ных чисел. Из трёхзначного числа вычитается двузначное, и ре­
зультат получается меньше 10. Насколько большим может быть это
160 Раздел 1. Планы занятий

трёхзначное число? Начнём с грубой оценки. Например, может ли


это быть 200 или больше? Конечно, нет. Иначе число справа от
знака равенства не могло бы быть таким маленьким. Значит, трёх­
значное число начинается с цифры 1 и выглядит так: 1*5.
Чтобы определить второе число, нам понадобится более точная
оценка. Сначала запишем неравенство по-другому: 1*5 = ** +8.
Справа от знака равенства стоит сумма двузначного числа и чис­
ла 8. Эта сумма не может быть больше 107: даже если взять наи­
большее двузначное число 99 и прибавить к нему 8, то получится
только 107. Значит, средняя цифра в числе 1*5 равна 0. Поэтому
задача имеет только одно решение: 105 — 97 = 8.
Для преподавателей. Отметим, что для решения задачи достаточно вто­
рой (более точной) оценки. Первую (грубую) оценку мы проводим для
того, чтобы продемонстрировать простую и удобную технику.

Пример 3. Восстановите недостающие цифры:

РАЗБОР ПРИМЕРА 3. Для решения этой задачи достаточно при­


менить правила сложения в столбик. Из таблицы сложения ясно,
что цифрой единиц в верхнем числе может быть только 6. При сло­
жении в разряде единиц в разряд десятков переносится 1, поэтому
неизвестная цифра десятков должна быть равна 4.
Цифру сотен в верхнем числе можно восстановить двумя спосо­
бами.
Во-первых, можно снова рассмотреть сложение в столбик. К этой
цифре прибавляется только единица (перенесённая из разряда де­
сятков). В результате возникает перенос в следующий разряд (ты­
сяч). Следовательно, перенесённая единица прибавлена к цифре 9
(для любой другой цифры сумма получилась бы меньше 10, и ново­
го переноса бы не было).
Второй способ — определить эту цифру с помощью оценки. Если
сумма трёхзначного и двузначного чисел больше 1000, то трёхзнач­
ное число должно быть больше 900. Поэтому в разряде сотен верх­
него числа стоит цифра 9.
Когда все цифры обоих слагаемых уже известны, восстановить
сумму не составляет труда. ОТВЕТ: 996 + 47 = 1043.

Пример 4. Восстановите недостающие цифры:


Занятие 19. Числовые ребусы I. Потерянные цифры 161

РАЗБОР ПРИМЕРА 4. Снова пользуемся оценкой. Из числа, больше­


го 1000, вычитается число, меньшее 1000, и результат получается
меньше 100. Насколько большим может быть первое число? Оно не
может быть больше 1098: даже если к самому большому трёхзнач­
ному числу (999) прибавить самое большое двузначное число (99),
то получится только 1098. Значит, верхнее число равно 1071. Теперь
легко восстановить остальные числа.
Для преподавателей. Большинство ребусов, предлагаемых на этом заня­
тии, допускают два способа решения. Первый состоит в том, чтобы
применить правила сложения в столбик. Второй основан на оценках
величин и понимании количественных соотношений между числами.
Обычно дети хорошо натренированы складывать в столбик, но не за­
думываются о логике, лежащей в основе этого алгоритма. Поэтому они
склонны решать ребусы первым способом. Задача преподавателя — про­
демонстрировать возможности метода оценок. Это побуждает детей
думать о смысле арифметических действий.

Пример 5. Восстановите недостающие цифры:

РАЗБОР ПРИМЕРА 5. Эта задача не так проста, как предыдущие.


Единственное, что можно заметить сразу: цифрой единиц в нижнем
числе (* 8 *) может быть только 4.
Теперь придётся подключить логический анализ. Рассмотрим
неизвестное двухзначное число в четвёртой строке: **. Это резуль­
тат умножения неизвестного двузначного числа в первой строке
на 5. В каком случае такое произведение может быть меньше 100?
Только если двузначный множитель меньше 20.
Это означает, что в разряде десятков первого множителя стоит
цифра 1. Цифру в разряде единиц найдём методом перебора, но сна­
чала сведём к минимуму набор возможных вариантов. Неизвестное
число, умноженное на 2, оканчивается цифрой 4. Значит, неизвест­
ное число оканчивается цифрой 7 или 2. Теперь остаётся подста­
вить 7 и 2 в разряде единиц. При умножении должно получиться
трёхзначное число с цифрой 8 в середине. Проверяем:
12 • 52 = 624 — не подходит,
17 • 52 = 884 — подходит.
Задача 6 остаётся для самостоятельного решения.
Раздел 1. Планы занятий
Пример 6. Восстановите недостающие цифры:

Подборка задач
Задача 1. Волшебник Шмерлин пригласил в гости несколько
ведьм. Он идёт в волшебный магазин за угощением — сушёными
лягушками. Все приглашённые должны получить одинаковое ко­
личество лягушек, и Шмерлин не хочет, чтобы после ухода гостей
лягушки оставались у него в доме. Какое наименьшее количество
лягушек надо купить Шмерлину в следующих случаях?
а) Придут либо 3 ведьмы, либо 5 ведьм.
б) Придут либо 4 ведьмы, либо 6 ведьм.
Задача 2. а) У золотоискателя Джека есть семь
золотых самородков. Первый самородок стоит один
доллар, второй—два доллара, третий — три долла­
ра и так далее. Собираясь в деревенскую лавку за
продуктами, Джек положил в карман только три
самородка. Он знает, что сможет расплатиться ими
за любую покупку, которая стоит целое число дол­
ларов от 1 до 7. Какие три самородка в кармане
у Джека?
б) Теперь у Джека есть пятнадцать самородков, которые стоят
1,2,3,..., 15 долларов. Какие четыре самородка ему надо взять с со­
бой, чтобы он мог заплатить любое целое число долларов от 1 до 15?
Задача 3. Замените все звёздочки цифрами от 0 до 9 так, чтобы
получились верные равенства:
Занятие 19. Числовые ребусы I. Потерянные цифры 163

Задача 4. а) Три мешочка наполнены золотым песком. Как рас­


положить их в порядке возрастания веса (от самого лёгкого до са­
мого тяжёлого) за три взвешивания на чашечных весах без гирь?
б) Четыре мешочка наполнены золотым песком. Как располо­
жить их в порядке уменьшения веса за пять взвешиваний на ча­
шечных весах без гирь?

ДОПОЛНИТЕЛЬНЫЕ ЗАДАЧИ

Задача 5. На столе лежат две золотые, две серебряные и две


бронзовые монеты. В каждой паре одна из монет фальшивая: она
легче настоящей. Все настоящие монеты весят одинаково, и все
фальшивые тоже весят одинаково. Как определить все три фаль­
шивые монеты за два взвешивания на чашечных весах без гирь?
Занятие 20

Числовые ребусы II.


Зашифрованные примеры

Зашифрованные примеры
В прошлый раз мы занимались восстановлением цифр в числовых
примерах. Сегодня мы познакомимся с другой разновидностью таких
задач. Это зашифрованные примеры, в которых каждая цифра замене­
на определённой буквой. Наша цель — расшифровать эти примеры.
Как правило, такие задачи труднее задач с потерянными цифра­
ми. Работа над ними похожа на расследование запутанной детек­
тивной истории. «Универсальные» методы (такие как оценка) рабо­
тают и здесь, но наряду с ними необходимы более точные приёмы.
Участникам придётся внимательно анализировать применение пра­
вил арифметических действий к конкретным числам.
Это занятие, как и предыдущее, одновременно увлекательно и по­
лезно.
Математическая разминка
Задача для разминки 1. Шесть стаканов поставлены в ряд. Пер­
вые три заполнены водой, а последние три пустые. Надо расположить
стаканы так, чтобы полные и пустые чередовались. При этом разре­
шается перемещать только один стакан из шести. Как это сделать?
Задача для разминки 2. Можно ли бросить мяч так, чтобы че­
рез некоторое время он остановился и начал двигаться в противо­
положном направлении?

Тема занятия:
«Зашифрованные примеры»
Пример 1. Предположим, что кто-то показал вам пример на сло­
жение:
Занятие 20. Числовые ребусы II. Зашифрованные примеры 165

Эта запись выглядит странно: она похожа на числовой пример,


но в ней нет ни одной цифры. Цифры зашифрованы по неизвест­
ному коду: каждая цифра заменена определённой буквой. Одина­
ковые буквы обозначают одинаковые цифры, а разные буквы — раз­
ные цифры. Наша цель — восстановить исходные цифры.
С чего начать? Когда криптоаналитик (специалист по расшиф­
ровке секретных сообщений) начинает работу с зашифрованным
текстом, он ищет «слабое звено»: знак, который проще всего уга­
дать. Мы будем поступать так же.
Заметим, что ни в одном из слагаемых нет цифры в разряде со­
тен. Значит, А равно числу, которое переносится из разряда десят­
ков в разряд сотен. Поскольку слагаемых всего два, переносится са­
мое большее 1. Следовательно, А равно 1:

Это рассуждение можно перевести на язык оценок: ASA —это


сумма двузначного и однозначного чисел. Значит, число ASA не
может быть больше 199, и его первая цифра А равна 1.
Чтобы расшифровать ещё одну букву, рассмотрим сложение в раз­
ряде десятков. В первом слагаемом на этом месте стоит цифра М,
а во втором — ничего. Однако в разряде десятков суммы другая циф­
ра: S. Это различие может объясняться только переносом из разряда
единиц. Перенесённое число (1) прибавляется к М, в результате чего
появляется цифра в разряде сотен. Если бы М было меньше 9, то
сложение М 4-1 не привело бы к переносу. Следовательно, М = 9
и S = 0. Теперь определить цифру Е нетрудно, поскольку все осталь­
ные цифры уже известны:

Тот же результат можно получить другим путём. Сумма двузнач­


ного и однозначного чисел не может быть больше 994-9 = 108, по­
этому цифра S должна быть равна 0. Цифра М должна быть равна 9:
иначе сумма ME 4-М не могла бы быть больше 100.
166 Раздел 1. Планы занятий

Для преподавателей. 1. У кружковцев такие задачи пользуются огромным


успехом. Всю работу по расшифровке выполняют сами участники. Все
тянут руки, желая внести свой вклад. Чтобы дать каждому возможность
высказаться, вводится правило: за один раз можно высказать только од­
но наблюдение или одну идею в отношении одной цифры. Если группа
заходит в тупик, я подсказываю букву, с которой проще всего начать на
данном этапе решения.
2. Это не очень трудная задача. Возможно, вы не успеете дописать на
доске пример, как кто-то уже выкрикнет правильный ответ. Но угаданный
ответ — ещё не решение. Расшифровать пример — значит не только найти
вариант расшифровки, но и объяснить, почему нет других вариантов.
Пример 2. Расшифруйте следующий пример. Одинаковые буквы
обозначают одинаковые цифры, а разные буквы — разные цифры.

РАЗБОР ПРИМЕРА 2. Эта задача значительно труднее предыдущей.


Сложение в разряде сотен показывает, что S не больше 4. К со­
жалению, это мало что даёт.
Сложение в разряде десятков выглядит более обнадёживающим.
При сложении цифр U и Р с возможным переносом из разряда еди­
ниц получается число, оканчивающееся цифрой Р. Значит, сумма U
и перенесённого числа равна 0 или 10. Это возможно только в двух
случаях: либо U = 0 и переноса нет, либо U = 9 и перенос (число 1)
есть. Вариант U = 0 сразу отпадает, если посмотреть на разряд сотен:
ясно, что цифра U должна быть больше нуля. Следовательно, U = 9:

Расшифровав U, из сложения в разряде сотен нетрудно сделать


вывод, что S = 4. Рассмотрев разряд единиц, находим значение Р:
сумма Р и 9 оканчивается цифрой 4. Значит, Р = 5:

Ко всеобщему удивлению, мы расшифровали пример, ничего не


зная об исходных цифрах!
Занятие 20. Числовые ребусы И. Зашифрованные примеры 167

Для преподавателей. Эти две задачи можно представить ещё одним спосо­
бом — расшифровкой в обратном порядке. Сначала предлагается исход­
ный числовой пример, в котором все вхождения какой-то одной цифры
заменены буквой. Затем задача усложняется — буквой заменяется ещё
одна цифра, и так далее. Этот способ подходит для детей более млад­
шего возраста. Он проще ДДЯ понимания, поскольку разные техники
расшифровки разбираются по отдельности.
Важно заранее продумать порядок замены цифр, позволяющий пе­
реходить от простых идей к более сложным. Например, в первой за­
даче лучше сначала дать пример М2 + М = 101, а затем М2 4- М = АОА
и М2 4- М = ASA. Во второй задаче — сначала S95 4- S59 = 95S, затем
S9P4-SP9 = 9PS и, наконец, SUP + SPU = UPS.
Пример 3. Расшифруйте пример:
АТ + АТ + АТ = ВАТ.
РАЗБОР ПРИМЕРА 3. Получив этот пример, дети обычно записыва­
ют его в столбик и пытаются рассуждать так же, как в предыдущей
задаче. Скорее всего, из этого ничего не выйдет. (Попробуйте сами
расшифровать пример таким способом: это довольно трудно.)
К этой задаче нужно подойти по-другому. Немного подумав, мож­
но заметить, что число в правой части равно В • 100 + AT, т. е. сумме
числа В, умноженного на 100, и числа AT. Теперь пример становится
проще: АТ + АТ = В-100. Поскольку AT—двузначное число, сумма
в левой части меньше 200. Следовательно, В = 1. Тогда А = 5 и Т = 0.
Следующие задачи дети решают самостоятельно. В каждом при­
мере одинаковые буквы обозначают одинаковые цифры, а разные
буквы — разные цифры.
Пример 4. Расшифруйте примеры:

Пример 5. Объясните, почему следующие ребусы не имеют ре­


шения:
168 Раздел 1. Планы занятий

Подборка задач
Задача 1. Замените все звёздочки цифрами от 0 до 9 так, чтобы
получилось верное равенство: а) 1*«*1 = 1*1; б) ***7 — *** = 8.
Задача 2. Мама-медведица поймала 18 карасей для дня рожде­
ния медвежонка. Сколько зверят было на дне рождения, если все,
включая медвежонка, съели одинаковое целое количество карасей?
Перечислите все возможные варианты.
В остальных задачах, как обычно, одинаковые буквы обозначают
одинаковые цифры, а разные буквы — разные цифры.
Задача 3. Белла придумала два зашифрованных примера, ко­
торые показаны ниже. Но как ни старалась, не смогла их решить.
Почему?

Задача 4. Расшифруйте примеры:

Задача 5. Замените все звёздочки цифрами от 0 до 9 так, чтобы


получилось верное равенство:

Задача 6. Посмотрите на зашифрованный пример:


ДУБ+ДУБ + ...+ДУБ = РОЩА.
Какое наибольшее количество «дубов» может быть в «роще»? По­
кажите, как может получиться такое количество, и объясните, поче­
му «дубов» не может быть больше.

ДОПОЛНИТЕЛЬНЫЕ ЗАДАЧИ

Задача 7*. Расшифруйте пример:


Занятие 21

Математическая олимпиада II

Сегодня в нашем кружке вторая математическая олимпиада. Она


снова проводится устно. Такая форма побуждает участников оттачи­
вать навыки математических рассуждений, а преподавателю помога­
ет лучше узнать, чему научился каждый участник и над чем ему ещё
нужно поработать. (Однако те же задачи, слегка уменьшив их коли­
чество, вполне можно предложить и на письменной олимпиаде.)
Об организации и проведении олимпиады подробно написано
на с. 241 и далее; см. также комментарии по случаю первой олим­
пиады (с. 103).

Что принести на урок


• Призы и награды (желательно для всех участников, а не только
для победителей). Благодаря им олимпиада станет весёлым и па­
мятным событием.

Событие дня:
математическая олимпиада
Эта олимпиада проводится так же, как и первая. Задачи напеча­
таны на двух листах приблизительно в порядке возрастания сложно­
сти. В начале олимпиады участникам выдаётся первый лист. Второй
лист получают те, кто решил хотя бы четыре задачи из первого.
Для группы из десяти человек мне понадобился помощник: им
стал папа одного из участников. Вдвоём мы вполне справлялись
с потоком желающих рассказать решение. Соревнование продол­
жалось 1 час 30 минут.
170 Раздел 1. Планы занятий

Олимпиада прошла успешно: задачи (особенно текстовые) ока­


зались интересными и достаточно трудными. Всем особенно понра­
вились задача про зверят и история про Змея Горыныча с нормаль­
ными и сумасшедшими головами.
Для преподавателей. В этой главе приведено несколько дополнительных за­
дач. Ими можно заменять основные задачи, если нужно отрегулировать
сложность олимпиады. Можно также составить из дополнительных задач
третий лист на случай, если кто-то быстро решит все основные задачи.

Первый лист задач


Задача 1. Разрежьте треугольник на че­
тыре треугольника меньшего размера так,
чтобы любые два из них имели общий уча­
сток границы. (Точка не считается общим
участком границы: общий участок должен
быть отрезком. На рисунке показан пример такого разбиения не на
четыре, а на три треугольника: любые два из этих трёх треугольни­
ков имеют общий участок границы.)
Задача 2. У Макса 20 шаров трёх цветов: красные, белые и чёр­
ные. Белых шаров у него в шесть раз больше, чем красных, а чёрных
меньше, чем белых. Сколько чёрных шаров у Макса?
Задача 3. Установите весы в положение равновесия, переме­
стив ровно пять спичек.

Задача 4. Тима едет на автобусе в библиотеку. Из окна автобуса


он замечает своего брата Сашу, который пешком идёт из библиоте­
ки. Тима решает догнать Сашу. Через одну минуту он выходит из
автобуса и начинает бежать за Сашей. Ещё через четыре минуты
Занятие 21. Математическая олимпиада II 171

Тима оказывается в том месте, где он увидел Сашу из автобуса. Тима


бежит в два раза быстрее, чем идёт Саша. Сколько времени придёт­
ся бежать Тиме, чтобы догнать Сашу?
Задача 5. У слонёнка, бегемотика и жирафика спро­
сили, сколько они весят. Вот что они ответили:
• Жирафик: «Я вешу 100 килограммов».
• Бегемотик: «Я вешу столько же, сколько
жирафик и полслонёнка».
• Слонёнок: «Я вешу столько же, сколь­
ко жирафик и бегемотик».
Сколько весит каждый из зверят?
Задача 6. а) Сколькими способами можно поставить на шах­
матной доске 8 x 8 белую и чёрную ладьи так, чтобы они не били
друг друга? (Ладья ходит по горизонтали и по вертикали на любое
расстояние.)
б) Решите ту же задачу для случая, когда обе ладьи чёрные. (Бу­
дем считать, что ладьи одного цвета могут бить друг друга.)

Второй лист задач


Задача 7. В каждой клетке таблицы была написана цифра 1, 2,
3, 4 или 5. В каждой строке, в каждом столбце и на каждой диа­
гонали каждая цифра встречалась только один раз. Пришла Белла
и стёрла почти все цифры. Какая цифра была в центральной клетке?

Задача 8. Флаг пиратского корабля имеет форму прямоугольни­


ка и состоит из чередующихся чёрных и белых вертикальных полос
одинаковой ширины. Количество полос на флаге равно количеству
пленных, которые в данный момент находятся на корабле. На про­
шлой неделе на корабле было двенадцать пленных, а на флаге —
двенадцать полос. Но за выходные двум пленным удалось бежать.
Корабельному юнге поручили переделать флаг. Новый флаг должен
172 Раздел 1. Планы занятий

быть прямоугольным, иметь ту же площадь и состоять из десяти


полос той же ширины, что и у исходного флага. Помогите юнге раз­
резать флаг на две части, из которых можно сшить новый флаг.
Задача 9. У Змея Горыныча 15 нормальных и 20 сумасшедших
голов. Волшебный меч Ивана-Царевича одним ударом отрубает од­
ну или две головы. Вместо отрубленных голов сразу вырастают но­
вые по таким правилам.
• Если отрублены две одинаковые головы, то вместо них вырас­
тает одна сумасшедшая голова.
• Если отрублены две разные головы, то вместо них вырастает
одна нормальная голова.
• Если отрублена одна голова, то вместо неё вырастает точно
такая же голова.
Когда у Змея Горыныча остаётся одна голова, он падает духом
и сдаётся.
а) В каком порядке нужно отрубать головы, чтобы у Змея Горы­
ныча осталась одна голова? Приведите пример.
б) Будет ли последняя голова нормальной или сумасшедшей?
Объясните, почему ваш ответ верен при любом порядке отрубания
голов.
Задача 10. Три брата — Ваня, Даня и Коля—любят блины, кото­
рые печёт бабушка Фёкла. Каждый раз, когда кто-то из братьев при­
ходит в гости к бабушке, он съедает столько блинов, сколько может
съесть (всегда одно и то же количество). В понедельник Ваня и Даня
пришли к бабушке и съели 25 блинов. Во вторник Даня и Коля съели
37 блинов. В среду Коля и Ваня съели 30 блинов. В воскресенье все
три брата собираются в гости к бабушке. Она хочет накормить их
досыта, но так, чтобы ни одного блина не осталось (она сама не
любит блины). Сколько блинов ей нужно испечь?

Д О П О Л Н И Т Е Л Ь Н Ы Е ЗАДАЧИ

Задача 11. Алина и её мама делают шоколадных Дедов Моро­


зов. Они расплавляют плитки шоколада и заливают его в специаль­
ные формы. Из каждой плитки получается один Дед Мороз, причём
немного шоколада остаётся. От трёх плиток остаётся столько же шо­
колада, сколько содержится в одной плитке. Сколько Дедов Морозов
можно сделать из 9 плиток? А из 14 плиток?
(Этой задачей можно заменять задачу 7 и последующие задачи.)
Занятие 21. Математическая олимпиада II 173

Задача 12. На гребне крыши в ряд сидят 19 кошек и 10 котов.


Каждая кошка сидит рядом с котом. Докажите, что каждый кот си­
дит рядом с кошкой.
(Этой задачей можно заменять задачу 7 и последующие задачи.)
Задача 13. У мышки праздник: она нашла кусок сыра в форме
куба 3 x 3 x 3 см. Этот куб состоит из 27 кубиков 1 x 1 x 1 см. Покон­
чив с одним кубиком, мышка переходит к другому кубику, который
имеет общую грань с только что съеденным кубиком. Мышка знает,
что в центральном кубике спрятан крючок, захлопывающий мыше­
ловку. Может ли она съесть все кубики, в которых нет крючка?
(Эта задача труднее всех задач из первых двух листов.)
Занятие 22

Делимость I. Определение и свойства

Определение и свойства
Мы переходим к теме делимости. Это необъятная тема, кото­
рая открывает путь ко многим разделам теоретической и приклад­
ной математики (и является важной составной частью этих раз­
делов). Делимость служит незаменимым инструментом в теории
чисел, теории групп, криптографии и множестве других наук. Вме­
сте с тем это благодатная тема для математического кружка. Её
различные аспекты — от самых простых к более сложным — можно
изучать в одном и том же кружке на протяжении нескольких лет.
Тема делимости очень полезна, но не так занимательна для млад­
ших детей, как многие другие разделы внешкольной математики.
Поэтому важно дозировать объём материала, изучаемого на каж­
дом занятии и на протяжении учебного года.
На предыдущих занятиях уже предлагалось несколько задач, име­
ющих отношение к делимости. Благодаря этому такие задачи не ста­
нут для участников полной неожиданностью.

Что принести на урок


Круглую кастрюлю с крышкой и квадратный пластиковый кон­
тейнер с крышкой. (Они пригодятся при разборе второй разминоч-
ной задачи.)

Математическая разминка
Задача для разминки 1. Почему в поездах ручка стоп-крана
красного цвета, а в самолётах — синего?
Задача для разминки 2. Вероятно, вы не раз видели на улице ка­
нализационные люки. Вопрос: почему крышка люка всегда круглая?
Для преподавателей. Будьте готовы ответить на множество вопросов по
этой задаче. Прежде всего нужно объяснить разницу между люком и во­
досточным отверстием. Люк —это проход, соединяющий подземную си-
Занятие 22. Делимость I. Определение и свойства 175

стему канализации с поверхностью. Он должен быть достаточно широ­


ким, чтобы в него мог пролезть человек. Водосточное отверстие служит
для стока воды. Его крышка вполне может быть прямоугольной, а вот
крышки люков делают круглыми.
Если окажется что дети «застрянут» на этой задаче, то наглядная
демонстрация может помочь им найти решение. (Описание приведено
на с. 335.)

Тема занятия:
«Делимость»
Мы начнём с того, что обсудим понятие делимости, используя
в качестве примеров пары конкретных чисел. Затем мы обобщим
понятие делимости и введём для него математическое обозначение.

Делимость
Рассмотрим равенство 30:5 = 6. Как известно, в этом равенстве
30 называется делимым, 5 —делителем, а 6 —частным.
Заметим, что в нашем равенстве все три числа —делимое, дели­
тель и частное — целые. Давайте введём несколько новых терминов,
которые позволят нам говорить не просто о делении, а именно о та­
ком делении, где все числа целые. В таком случае будем говорить,
что 30 делится на 5, или, что то же самое, 5 —делитель числа 30.
Давайте снова подчеркнём, что эти новые термины применимы
только тогда, когда и делитель, и делимое, и результат — целые чис­
ла. Когда говорят: «число а—делитель числа Ь», имеют в виду, что
частное от деления Ь на а — целое число.
Снова поясним это на примерах. Верно ли что число 40 делится
на 5 и 5 — делитель числа 40? Да, это так. А верно ли, что число
41 делится на 5 и 5 является делителем числа 41? Нет, это неверно,
так как результат такого деления не целый.
Продолжим усвоение новых терминов. Поскольку 30 = 5-6, чис­
ло 30 делится на 6 и 6 —делитель числа 30. Есть ли у числа 30 ещё
какие-нибудь делители? Есть: 5, 2, 15, 3, 10, 1, 30. (В моем кружке
возникли разногласия насчёт того, являются ли 1 и 30 делителя­
ми числа 30. Сомнения легко разрешаются с помощью равенства
30 = 1-30.)
Для преподавателей. Понятие делимости применимо как к положитель­
ным, так и к отрицательным целым числам. Однако на этом и после-
176 Раздел 1. Планы занятий

дующих занятиях мы ограничимся положительными числами. Работа


с отрицательными числами требует дополнительных усилий, но не даёт
ничего принципиально нового.
Настало время сформулировать общее определение делимости,
не связанное с конкретными числами. Как всегда, желательно вы­
работать определение вместе с детьми.
Для общих определений удобно заменять числа обозначениями.
Я пишу на доске две буквы: А и В. Эти буквы обозначают два неиз­
вестных целых числа. Затем я задаю участникам ряд вопросов.
• Как объяснить смысл утверждения «Л делится на В»?
Обычно дети отвечают примерно следующее: «Л делится на В,
если при делении не получается остатка».
• Как выразить это в записи!
Почти наверняка кто-нибудь выдаст формулу: А = В-К.
Теперь у нас есть формула. Осталось превратить её в математи­
чески точное определение делимости. Я формулирую такое опреде­
ление.
Определение 1. Пусть А и В — целые числа. Говорят, что А де­
лится на В, если можно найти такое целое число К, что А = В-К.
Это определение надо обязательно обсудить: ведь в нём впервые
используются буквенные обозначения. Сначала проверим, как ра­
ботает это общее определение для конкретной пары чисел.
Предположим, например, что А = 40 и В = 8. Делится ли 40 на 8,
если исходить из нашего определения? Можно ли найти целое чис­
ло К, которое при умножении на 8 даст 40? Конечно, можно: это
число 5. Значит, 40 делится на 8, и 8 — делитель числа 40.
Теперь предположим, что А = 7 и В = 2. Что говорит нам опреде­
ление насчёт делимости 7 на 2? Как известно, 7 = 2- 3,5. На первый
взгляд, мы действительно нашли такое число К, что 7 = 2 • К. Значит
ли это, что 7 делится на 2? Нет! Число 3,5 не подходит под наше
определение, так как это не целое число. Этот пример показывает:
перед словом число в определении не случайно стоит слово целое.
Если забыть об этом коротеньком слове, то определение не будет
работать.

Сколько делителей?
Мы узнали, что у числа могут быть делители. Теперь займёмся
изучением их свойств. Этот материал лучше всего усваивается в хо-
Занятие 22. Делимость I. Определение и свойства 177

де активного обсуждения, поэтому здесь он представлен в форме


вопросов и ответов.
Сколько делителей может быть у числа? Мы уже знаем, что чис­
ло 30 имеет 8 делителей: 1, 2, 3, 5, 6, 10, 15, 30. Рассмотрим другие
примеры. У числа 10 четыре делителя: 1, 2, 5, 10. Может ли у числа
быть три делителя? Два делителя? Один делитель? (Ответ на все
вопросы — положительный. Дети с удовольствием ищут и приводят
примеры таких чисел.)
Как вы думаете, есть ли числа, у которых больше 50 делителей?
А больше 100 делителей?
Оказывается, есть и такие числа. Давайте составим число, у ко­
торого точно не меньше 50 делителей.
Рассмотрим число, которое получится, если перемножить 1 • 2 •...
... • 50. Легко видеть, что оно делится на все целые числа от 1 до 50.
Значит, у него самое меньшее 50 делителей. (Участники могут при­
думать совсем другие числа, имеющие не меньше 50 делителей.
Может быть, кто-нибудь приведёт пример числа, у которого ровно
50 делителей.)
Подобным образом можно убедиться и в существовании числа,
имеющего не меньше 100 делителей.

Другие определения делимости


Определение делимости, сформулированное в начале занятия, —
алгебраическое: в нём используются обозначения. Попробуем при­
думать другие определения, показывающие суть делимости более
наглядно. (Вспомните разные определения чётных и нечётных чи­
сел, приведённые на с. 72—76.)
Вместе с участниками мы вырабатываем два наглядных опреде­
ления.
Определение 1. Целое число А делится на целое число В, если А
предметов можно разбить на несколько групп, в каждой из которых
ровно В предметов (см. рисунок).

В предметов

Число Л
178 Раздел 1. Планы занятий

Определение 2. Целое число А делится на целое число В, если


А предметов можно разбить на В групп, содержащих одинаковое
количество предметов (см. рисунок).

Число А
В предметов
Эти два определения тесно связаны между собой: количество
предметов в группе из первого определения равно количеству групп
во втором определении.
Разговор об определениях делимости завершается традицион­
ным вопросом: какое определение лучше? Каким из них мы будем
пользоваться? ОТВЕТ: надо пользоваться тем определением, кото­
рое лучше подходит для конкретной задачи.

Обозначение делимости
Математики —ленивый народ. Они всегда стараются придумать
самую короткую запись и самый простой способ решения. Если бы
не их лень, нам приходилось бы записывать пример на сложение
так: «5 плюс 7 равно 12». Но математики изобрели для часто исполь­
зуемых терминов сокращённые обозначения, и теперь мы пишем:
«5 + 7 = 12».
То же самое произошло с понятием «делимость»: для него при­
думали специальное обозначение. Например, вместо «10 делится
на 2» или «Л делится на В» пишется1 10:2 или А: В. Не следует
путать обозначение делимости с обычным знаком деления. Знак
деления заменяет команду «выполнить деление» (10:2 или 10/2
означают «разделить 10 на 2»). Он может также означать результат
деления (10 : 2 = 5). А обозначение «10 : 2» указывает на возмож­
ность деления (10 можно разделить на 2). (В этой книге обозна-
1
Существует еще один символ, которым можно обозначать делимость. «В | Л»
означает «число В — делитель числа Л». Например, 216. В некоторых странах матема­
тики предпочитают использовать этот символ. Англоязычные школьники, например,
всегда пишут «|».
Занятие 22. Делимость I. Определение и свойства 179

чение делимости не употребляется. Но на занятиях оно помогает


сэкономить время, а также место на доске.)
Теперь мы изучим несколько простых и полезных свойств дели­
мости.

Свойства делимости
В этом разделе рассматриваются и доказываются несколько про­
стых, но полезных свойств делимости. Для каждого свойства при­
ведено два объяснения: наглядное и алгебраическое. Скорее всего,
сами свойства и их наглядные доказательства покажутся детям ин­
туитивно очевидными и не вызовут затруднений. А вот понять ал­
гебраические доказательства тех же свойств им будет значительно
труднее. Тем не менее, следует пользоваться любой возможностью
обсуждать с детьми такие доказательства: это приучает их к работе
с буквенными обозначениями.
Каждое свойство делимости в этом разделе сначала разбирается
на примере конкретных чисел. Затем рассуждения приводятся к об­
щему виду, в котором все числа заменены символами.
Я задаю детям вопрос: если каждое из двух чисел делится на 7,
делится ли их сумма на 7? Сначала вопрос ставится на голосование.
Явно преобладает ответ «Да». Затем мы проверяем это для какой-
нибудь пары чисел, например 21 и 14. Я отмечаю, что, хотя это
свойство кажется очевидным, его необходимо доказать. Для боль­
шей убедительности можно привести похожее утверждение: «Если
каждое из двух чисел не делится на 7, то и их сумма не делится
на 7». Это утверждение тоже может показаться очевидным, но оно
неверно.
Алгебраическое доказательство. Поскольку речь идёт о произ­
вольной паре чисел, обозначим их буквами: Л и В. Оба эти числа
делятся на 7. Значит, каждое из них можно представить в виде про­
изведения числа 7 и неизвестного целого числа. Эти неизвестные
числа придётся обозначить новыми буквами: X и Y. Получаются
такие формулы:
А = 7-Х и B = 7-Y.

Чтобы доказать, что сумма чисел А и В делится на 7, займёмся


алгеброй:
A + B = 7-X + 7-Y = 7-(X + Y).
180 Раздел 1. Планы занятий

Итак, сумма чисел АиВ равна произведению числа 7 на какое-


то целое число. Значит, она делится на 7.
Наглядное доказательство. Поскольку оба числа делятся на 7,
каждое из них можно представить в виде прямоугольника с высо­
той 7 и с целочисленной длиной. Расположим эти два прямоуголь­
ника рядом. Вместе они образуют длинный прямоугольник с той же
высотой 7, соответствующий сумме чисел. Значит, эта сумма делит­
ся на 7.
Если нужно, мы можем ещё раз повторить аналогичное доказа­
тельство на примере другого конкретного числа.
Теперь дети готовы перейти от конкретных чисел к обобщённой
формулировке.

Делимость суммы
Если каждое из двух целых чисел АиВ делится на целое число С,
то и сумма чисел АиВ делится на С.
Алгебраическое доказательство. Поскольку А делится на С,
для некоторого целого числа X верно равенство А = С • X. Анало­
гично для некоторого целого числа Y верно равенство А = С -Y.
Следовательно,
A + B = C-X + C-Y = С-(Х + У).
Число X + Y целое. Поэтому равенство A + B = C-(X + Y) означа­
ет, что А + В делится на С.
Наглядное доказательство. Поскольку оба числа делятся на С,
каждое из них можно представить в виде прямоугольника с целой
высотой С. Расположим эти два прямоугольника рядом. Вместе они
образуют длинный прямоугольник с той же высотой С, соответству­
ющий сумме чисел. Значит, эта сумма делится на С.
Теперь на очереди делимость произведения. Мы обсуждаем это
свойство так же, как и предыдущее: сначала на примере конкретно­
го числа, затем в общем виде. Здесь я для краткости опускаю обсуж­
дение и привожу только формулировку и доказательство.

Делимость произведения
Если целое число А делится на целое число С, а В —любое целое
число, то А • В делится на С.
Занятие 22. Делимость I. Определение и свойства 181

Алгебраическое доказательство. Поскольку А делится на С,


для некоторого целого числа X верно равенство А = С - X. Далее,
A-B = X-C-B = X-B-C=(X-B)-C = Y-C,
где Y = X-B. Следовательно, А • В делится на С.
Наглядное доказательство. Предложите участникам придумать
его самим.
На этом заканчивается первое из занятий, посвященных делимо­
сти. Подборка задач к этому занятию содержит много упражнений —
простых и не очень—для закрепления пройденного материала.
Для преподавателей. Если остаётся время, полезно отметить ещё несколь­
ко свойств делимости.
Делимость разности. Если каждое из целых чисел АиВ делится на
целое число С, то и разность чисел АиВ делится на С.
Делимость суммы нескольких чисел. Если каждое из нескольких це­
лых чисел делится на целое число С, то и сумма этих чисел делится на С.

Подборка задач
Задача 1. Разрежьте каждую фигуру на рисунке на четыре оди­
наковые фигуры. Резать можно только по линиям сетки и по диаго­
налям клеток.

Задача 2. Галеон «Фортуна» потерпел крушение и затонул на


обратном пути из Пуреи — колонии Инпасии. Корабль отправился
в своё последнее плавание с тремя сундуками на борту: в одном
были только золотые монеты, в другом — только серебряные, в тре­
тьем—и золотые, и серебряные. Недавно остатки галеона были об­
наружены, и в его трюме действительно нашлись три сундука с со­
кровищами. На крышке каждого сундука была надпись. Эти надпи­
си гласили: «Золото», «Серебро», «Золото и серебро».
Сундуки передали в музей, где вы работаете, и назначили дату
публичной церемонии их открытия. Вы знаток истории, и вам из-
182 Раздел 1. Планы занятий

вестно, что в те времена в Пурее была традиция: надписи на сун­


дуках никогда не соответствовали их содержимому. (Считалось, что
это помогает запутать тёмные силы и отвести беду.) Чтобы на цере­
монии не произошёл конфуз, необходимо заранее знать, что нахо­
дится в каждом сундуке. Вы пообещали директору музея установить
это, взглянув только на одну монету из одного сундука по вашему
выбору. Как вы поступите?
Задача 3. Перечислите все делители числа 36. Сколько их?
Задача 4. Приведите пример числа, у которого ровно 10 дели­
телей.
Задача 5. а) Можно ли найти четыре последовательных нату­
ральных числа, ни одно из которых не делится на 4? Приведите при­
мер таких чисел или докажите, что их не существует.
б) Можно ли найти пять последовательных натуральных чисел,
ни одно из которых не делится на 5?
Задача 6. На острове рыцарей и лжецов живут мистер X, мистер
Y и леди Z.
а) Мистер X говорит, что если два целых числа делятся на 7, то
и их разность обязательно делится на 7. Кто мистер X —рыцарь или
лжец? Обоснуйте ответ.
б) Мистер Y говорит, что если разность двух целых чисел делится
на 7, то и каждое из этих чисел обязательно делится на 7. Кто мистер
Y—рыцарь или лжец? Обоснуйте ответ.
в) Леди Z утверждает, что если два целых числа делятся на третье
целое число (С), то и их разность обязательно делится на С. Кто
леди Z — рыцарь или лжец? Обоснуйте ответ.
Задача 7*. Начнём с наблюдения. Разделим все целые числа на
две группы: точные квадраты (1,4, 9,16, 25,...) и все остальные це­
лые числа (2, 3,5, 6, 7,...). Оказывается, у каждого числа из первой
группы нечётное количество делителей. Например, у числа 1 только
один делитель, у числа 4 — три, а у числа 16 — пять. Если же подсчи­
тать делители любого числа из второй группы, то обязательно полу­
чится чётное число. Например, у числа 7 два делителя, у числа 6 —
четыре, а у числа 12 — шесть.
Теперь можно сформулировать задачу. Докажите, что у любого
точного квадрата нечётное количество делителей, а у любого дру­
гого целого числа чётное количество делителей.
Занятие 22. Делимость I. Определение и свойства 183

ДОПОЛНИТЕЛЬНЫЕ ЗАДАЧИ

Задача 8. Гномик родился в 1122 году. На каждый день рождения


он получает в подарок от бабушки драгоценный камень: алмаз или
рубин. Если возраст гномика - делитель номера нынешнего года, то
бабушка дарит ему алмаз. Если это не так, то она дарит гномику ру­
бин. Гномик получил алмаз, когда ему исполнился один год (посколь­
ку 1123 делится на 1), затем на своё двухлетие (1124 делится на 2)
и трёхлетие (1125 делится на 3). Но, например, в пять и в семь лет он
получил рубин (1127 не делится на 5, а 1129 не делится на 7). В каком
возрасте гномик получит на день рождения свой последний алмаз?
Гномики и их бабушки живут много тысяч лет.
Задача 9*. В Стране сладостей есть монеты трёх видов: сугри-
ки, тугрики и шмоллары. Сугрик—самая мелкая монета: каждая
из остальных монет (тугрик и шмоллар) равна целому числу сугри-
ков. На воскресной ярмарке в Стране сладостей волшебный леде­
нец на палочке стоит 11 сугриков. У Стаса было 4 сугрика, 1 тугрик
и 9 шмолларов. Он купил несколько волшебных леденцов, и у него
не осталось денег.
У Тараса есть 15 сугриков, 12 тугриков и 9 шмолларов. Дока­
жите, что он тоже может потратить все свои деньги на волшебные
леденцы.
Задача 10*. Две горные деревни, Белвью и Хилвью, соединены
дорогой. На этой дороге нет горизонтальных участков: она везде
либо поднимается, либо опускается. За 6 часов автобус доезжает от
одной деревни до другой и возвращается обратно. На подъёме он
едет со скоростью 50 километров в час, а на спуске — со скоростью
100 километров в час. Найдите расстояние между деревнями.
Занятие 23

Делимость И. Простые числа


и разложение на простые множители

На прошлом занятии мы начали работу с делимостью. Мы по­


смотрели на делимость по-новому: дали её формальное определе­
ние, доказали несколько свойств и познакомились с разными спо­
собами решения задач на делимость.
Сегодня мы продолжим изучение этой темы: поговорим о про­
стых и составных числах и обсудим одно очень важное понятие —-
разложение на простые множители. Мы узнаем, как такое разложе­
ние помогает получать полезную информацию о делимости чисел.

Математическая разминка
Задача для разминки 1. Все числа от 0 до 9 разбили на две
группы.
• Первая группа: 1,4, 7.
• Вторая группа: 0, 2,3, 5, 6, 8, 9.
По какому признаку выделены эти группы?
Задача для разминки 2. Каким будет следующий рисунок в этом
ряду?

Тема занятия:
«Простые числа и делимость»
Начнём с разбора задач из домашнего задания, большинство из
которых связаны с темой делимости. Обсуждение этих задач на за­
нятии преследует две цели. Тем участникам, которые пропустили
прошлое занятие, оно поможет разобраться в теме, а для остальных
послужит напоминанием.
Занятие 23. Делимость II 185

Простые и составные числа


Я пишу на доске две группы чисел: 2, 3, 5, 7,11,13 и 4, 6,8, 9,10,
12,14,15.
По какому признаку числа разделены на две группы?
Несколько человек отвечают: первая группа — это простые чис­
ла, а вторая — составные. Итак, мы знаем, как называется каждая
группа; теперь дадим точное определение этих терминов.
Определение. Простое число — это положительное целое чис­
ло, у которого ровно два делителя: само это число и единица. Со­
ставное число — это положительное целое число, у которого больше
двух делителей.
Например, число 7 простое, так как у него ровно два делителя: 1
и 7. Число 12 составное, у него много делителей: 1,2, 3,4, 6,12.
А число 1 простое или составное? Ни то, ни другое! Это особое
число: у него только один делитель. Поэтому оно не относится ни
к одной из двух групп.
В заключение разговора о простых и составных числах я задаю
детям несколько вопросов.
• Сколько всего составных чисел? Можно ли найти наибольшее
составное число?
• Можно ли составить список всех простых чисел? Есть ли наи­
большее простое число?
Легко показать, что составных чисел бесконечно много. Напри­
мер, все чётные числа (кроме числа 2) составные, а чётных чисел
бесконечно много.
Простых чисел тоже бесконечно много. Первым объяснил это
древнегреческий математик Евклид. Он доказал, что, какое бы чис­
ло мы ни взяли, обязательно найдётся простое число, которое боль­
ше его. Это доказательство не очень простое, и здесь оно не приво­
дится.

Разложение на простые множители


Объяснить разложение на простые множители лучше всего на
конкретном примере.
Начнём с числа 360. У этого числа довольно много делителей.
Выберем какую-нибудь пару делителей, произведение которых рав­
но 360: например, 360 = 10 • 36.
186 Раздел 1. Планы занятий

Каждое из чисел в правой части этого равенства можно в свою


очередь записать как произведение двух делителей. Мы можем про­
должать этот процесс до тех пор, пока все делители в правой части
не окажутся простыми. В зависимости от того, какие пары дели­
телей мы выбираем, дойти до простых делителей можно разными
способами. Вот один из них:
360 = 10-36 =
= 5-2-36 =
= 5-2-6-6 =
= 5-2-2-3-2-3.
Итак, мы представили число 360 в виде произведения несколь­
ких чисел, каждое из которых простое.
Можно ещё сказать так: мы сопоставили числу 360 набор его
простых делителей. Этот процесс называется разложением на про­
стые множители.
Сразу возникает вопрос: а что, если начать разложение с другой
пары делителей? Получится такой же или другой результат? Про­
ведём эксперимент: разложим то же самое число другим способом,
например, так: 3 6 0 = 2-180 =

= 2-6-30 =
= 2-2-3-30 =
= 2-2-3-5-6 =
= 2-2-3-5-2-3.
Получился тот же набор простых множителей, только записанных
в другом порядке. Нам просто повезло или так будет всегда? Может
быть, можно разложить какое-нибудь число на простые множители
двумя способами и получить два разных набора множителей?
Оказывается, нет. Если взять любое число и разложить его на
простые множители всеми возможными способами, набор простых
множителей будет одним и тем же. Это свойство чисел — единствен­
ность разложения на простые множители — настолько важно, что
его называют основной теоремой арифметики.
Под «теоремой» в математике понимается утверждение, которое
необходимо доказать. Единственность разложения на простые мно­
жители можно доказать, но это доказательство довольно сложное,
и мы его опустим.
Занятие 23. Делимость II 187

Запись разложения на простые множители


Принято записывать простые множители в порядке возрастания.
Если в разложении есть одинаковые множители, то для экономии
времени и места их не выписывают несколько раз, а пользуются воз­
ведением в степень. Разложение числа 360 на простые множители,
записанное по этим правилам, выглядит так:
360 = 2-2-2-3-3-5 = 2 3 -3 2 -5.

Дерево разложения на простые множители


Чтобы не потерять простые множители числа, иногда при его
разложении рисуют схему в виде дерева:

Листья этого дерева — простые множители числа (на рисунке


они выделены тёмной границей).
Схема дерева упрощает процесс разложения на простые множи­
тели и делает его наглядным. Но в результате мы обязательно полу­
чим тот же набор простых множителей, что и без этой схемы.

Разложение на простые множители и делимость


Что нам даёт набор простых множителей числа? Он оказывается
очень полезным при решении задач. Разложив число на простые
множители, мы можем многое узнать о нём —в том числе о его
свойствах, связанных с делимостью.
Начнём с конкретного числа: 990. Запишем его разложение на
простые множители:
990 = 9-10 = 9-11 -2-5 = 3-3 -11 -2-5 = 2 -3-3 -5 -11.
Сейчас мы узнаем, как с помощью этого набора простых множи­
телей получить важную информацию о делимости числа 990. Точ-
188 Раздел 1. Планы занятий

нее, мы научимся строить из этих простых множителей другие чис­


ла, на которые делится 990. Кроме того, зная разложение какого-
нибудь числа на простые множители, мы сможем быстро опреде­
лить, делится ли 990 на это число.
Сначала рассмотрим делимость 990 на простые числа.
Делится ли 990 на 3? Запишем разложение 990 на простые мно­
жители так: 990 = 3 • (2 • 3 • 5 • 11). Значит, 990 делится на 3, а частное
от деления равно 2 • 3 • 5 • 11.
Делится ли 990 на 7? Если бы 990 делилось на 7, оно было бы
равно произведению 7 и какого-то целого числа X: 990 = 7 • X. Тогда
число 7 входило бы в набор простых множителей числа 990. Но мы
уже знаем разложение 990 на простые множители: 2 • 3 • 3 • 5 • 11.
В этом разложении нет множителя 7. Может быть, число 7 входит
в какое-нибудь другое разложение числа 990? Нет: согласно основ­
ной теореме арифметики существует только одно разложение числа
на простые множители. Значит, 990 не делится на 7.
Подведём итоги: число делится на простое число только в том
случае, если это простое число входит в набор простых множителей
исходного числа.
Теперь перейдём к делимости 990 на составные числа.
Делится ли 990 на 9? Да. Действительно, число 9 раскладывается
как 3 • 3, а простые множители числа 990 можно записать в таком
порядке: (3 • 3) • (2 • 5 • 11). Таким образом, мы не только проверили
делимость 990 на 9, но и нашли частное от деления: 990 —это про­
изведение чисел (3 • 3) и (2 • 5 • 11).
Делится ли 990 на 18? Да. Число 18 раскладывается как 2-3-3,
а разложение 990 на простые множители можно записать так:
(2-3-3)-(5-11).
Значит, 990 —это произведение чисел 18 и 5 • 11.
Делится ли 990 на 4? Нет. Число 4 раскладывается как 2 • 2. Зна­
чит, если бы число 990 делилось на 4, то в наборе его простых мно­
жителей было бы не меньше двух чисел 2. Но на самом деле в этом
наборе только одно число 2.
Делится ли 990 на 14? Нет. Число 14 раскладывается как 2 • 7.
Значит, если бы число 990 делилось на 14, то было бы верно ра­
венство 990 = 14-К = 2-7-К. Тогда в наборе простых множителей
числа 990 было бы число 7. Но мы знаем, что числа 7 в этом наборе
нет.
Занятие 23. Делимость II 189

Итак, разложение на простые множители раскрывает все тай­


ны делимости числа. С его помощью мы не только проверяем де­
лимость, но заодно и находим частное.
Чтобы наглядно представить взаимосвязь между разложением на
простые множители и делимостью, очень полезно нарисовать схему:

На схеме наверху написано исходное число 990, а под ним пока­


зан полный набор его простых множителей.
Простые множители можно представить себе в виде кирпичиков
для построения всех других делителей числа 990. Выбрав несколь­
ко простых множителей любого числа и перемножив их, мы обяза­
тельно получим делитель исходного числа. Таким образом, простые
множители служат кирпичиками, из которых можно собирать дели­
тели числа. На следующем рисунке показаны некоторые делители
числа 990 и кирпичики, из которых они построены:

Такие схемы позволяют не только быстро проверять делимость:


в качестве бесплатного приложения мы определяем частное от де­
ления. Из каких бы кирпичиков — простых множителей — мы ни по­
строили делитель, частное всегда равно произведению оставшихся
простых множителей (неиспользованных кирпичиков). Например,
результат деления 990 на 18 равен произведению чисел 5 и 11 —
двух кирпичиков, оставшихся неиспользованными после построе­
ния числа 18 (это видно на предыдущем рисунке посередине).
Мы увидели, как схемы помогают решать задачи на делимость.
Но для повседневной работы такие схемы не очень удобны: их ри­
сование занимает много времени. На следующем рисунке показан
более быстрый и экономичный способ графически изобразить раз­
ложение числа на простые множители. Эти три схемы, состоящие
190 Раздел 1. Планы занятий

из числа 840 и его простых множителей, подтверждают делимость


840 на 12, 21 и 56. В дальнейшем при изучении делимости мы будем
рисовать именно такие схемы: это сэкономит нам время.

Разложение на простые множители позволяет не только прове­


рить делимость, но и убедиться в её отсутствии.
Пусть X и У — два целых числа. Число X не делится на У, если из
простых множителей числа X невозможно составить У. Например,
840 не делится на 9. В разложении числа 9 на простые множители
два множителя 3, а в разложении числа 840 —только один.
Эта идея наглядно показана на следующем рисунке. Из него вид­
но, что 840 не делится на 9, на 16 и на 22.

Разложение произведения на простые множители


Предположим, что у нас есть два числа и мы знаем разложение
каждого из них на простые множители. Как разложить на простые
множители произведение этих чисел?
Это просто. Чтобы получить набор простых множителей произ­
ведения двух чисел, надо объединить наборы простых множителей
обоих чисел.
Проиллюстрируем это утверждение на нескольких примерах.
Набор простых множителей числа 12 —это 2, 2 и 3. Набор про­
стых множителей числа 21 —это 3 и 7. Зная это, определим набор
простых множителей для произведения 12-21: это 2, 2, 3, 3, 7.
Произведение чисел 990 и 840 равно 831600. Значит, чтобы по­
лучить полный набор простых множителей числа 831600, надо объ­
единить наборы простых множителей чисел 990 и 840. Это соответ­
ственно 2, 3, 3, 5,11 и 2, 2, 2, 3, 5, 7. В объединённом наборе мно-
Занятие 23. Делимость II 191

житель 2 встречается четыре раза, множитель 3 —три раза, мно­


житель 5—два раза, а множители 7 и 11 —по одному разу. Итак,
запишем
831600 = 2 4 -3 3 -5 2 -7-11.
Как и обычное разложение на простые множители, разложение
произведения удобно представить в виде схемы. Например:

В заключение сделаем несколько упражнений.


Упражнение 1. Назовите ещё несколько делителей числа 990.
Упражнение 2. Известно разложение некоторого числа на про­
стые множители: 2 • 3 • 5 • 17. Делится ли это число на 6? На 15? На 4?
Упражнение 3. Известно разложение некоторого числа на про­
стые множители: 2 • 2 • 7 • 11 • 13. Делится ли это число на 2? На 4?
На 8? На 26? На 24?
Упражнение 4. Известно разложение некоторых двух чисел на
простые множители: 2 • 3 • 5 • 17 и 2 • 2 • 7 • 11 • 13. Как раскладывается
на простые множители произведение этих чисел? Делится ли оно
на 8? На 16? На 24? На 32? На 35?

Подборка задач
Задача 1. Число раскладывается на простые множители так:
2 • З2 • 73 • 13. Делится ли это число на 2? На 4? На 14? На 12?
Задача 2. Два числа раскладываются на простые множители
так: 2 2 -3-7 3 -13и2-3 2 -7 2 .
а) Делится ли первое число на второе?
б) Делится ли произведение этих чисел на 8? На 36? На 27?
На 16? На 56?
Задача 3. Пусть р — простое число, большее 2. Число р — 1 чёт­
ное или нечётное? А р +1?
Задача 4. Пусть р — простое число, большее 2.
а) Докажите, что хотя бы одно из чисел р — 1 и р + 1 делится на 4.
б) Верно ли, что хотя бы одно из них обязательно делится на 5?
192 Раздел 1. Планы занятий

Задача 5. Мистер X и мистер Y живут на острове рыцарей и лже­


цов. Определите, кто они, исходя из их утверждений.
Мистер X: «Если число делится на 3 и на 4, то оно обязательно
делится на 12».
Мистер Y: «Если число делится на 6 и на 4, то оно обязательно
делится на 24».
Задача 6. Каждый из двух сундуков заполнен либо золотыми, ли­
бо серебряными монетами. На первом сундуке написано: «В этом сун­
дуке золото, а в другом серебро». На втором написано: «В одном из
этих сундуков золото, а в другом серебро». Что лежит в каждом сун­
дуке, если известно, что одна из надписей неверна, а другая верна?
Задача 7. Разложите числа на простые множители: 111, 1111,
1001.
Задача 8*. Расшифруйте пример: ВАО • ВА • В = 2002. (Одинако­
вые буквы обозначают одинаковые цифры, а разные буквы —раз­
ные цифры.)
Подсказка. Поможет разложение на простые множители одного
числа из предыдущей задачи.

Д О П О Л Н И Т Е Л Ь Н Ы Е ЗАДАЧИ

Задача 9. У вас есть ножницы и обычный лист бумаги. Вырежь­


те в этом листе такое отверстие, чтобы в него мог пройти слон.
Задача 10*. На острове рыцарей и лжецов был проведён опрос,
в котором приняли участие все жители острова. В ходе опроса некото­
рые жители сказали, что на острове чётное число рыцарей. Остальные
сказали, что на острове нечётное число лжецов. Докажите, что число
всех жителей острова чётное.
Занятие 24

Математический аукцион III

Сегодня у нас третий математический аукцион. Правила аукци­


она приведены на с. 232. Более подробно об этой игре говорится
в главе, посвященной первому математическому аукциону (с. 46).
Сегодняшняя игра служит перерывом в серии из четырёх занятий,
на которых мы изучаем делимость. Это очень важная и трудная тема.
По ней не стоит проводить четыре занятия подряд. Во-первых, дети
устанут, а во-вторых, им требуется время для усвоения новых знаний.

Что принести на урок


• Клетчатая бумага для рассеянных участников, забывших её дома.
• По желанию: леденцы, шоколадки или другие призы для побе­
дившей команды (или для всех участников).

Математическая разминка
Задача для разминки 1. У вас есть 3 бумажных стаканчика и
10 пластмассовых шариков. Можно ли положить все шарики в стакан­
чики так, чтобы в каждом стаканчике было нечётное число шариков?
Задача для разминки 2. Во время осенних каникул Вася и Петя
жили на соседних дачах. Однажды ночью в первый раз выпал снег.
Утром мальчики вышли на улицу. «Так нечестно! — воскликнул Ва­
ся. —На твоём участке в два раза больше снега, чем на моём!» Как
могло такое случиться?

Событие дня:
игра «Математический аукцион»
Участники уже освоились с правилами математического аукциона.
Поэтому время разумно распределить так: 5 минут — на формирова-
194 Раздел 1. Планы занятий

ние команд, 30 минут — на решение задач и 30 минут — на сам аук­


цион. В оставшееся время можно обсудить решение домашних задач.

Задачи для математического аукциона


Каждая задача стоит 100 шмолларов (см. с. 232).
Задача 1. Разделите квадрат 7 • 7 на как можно большее количе­
ство разных прямоугольников. Резать можно только по линиям сет­
ки. Прямоугольники должны заполнять весь квадрат и не должны
перекрываться. (Если один прямоугольник можно повернуть так,
что получится другой, то они считаются одинаковыми. Например,
прямоугольники 4 • 1 и 1 • 4 одинаковые.)
Решение считается лучше предыдущего, если квадрат разбит на
большее число прямоугольников.
Задача 2. Какое наибольшее количество слонов можно поста­
вить на шахматной доске 8 • 8 так, чтобы никакие два из них не били
друг друга? (Слоны ходят по диагонали на любое расстояние.)
Решение считается лучше предыдущего, если расставлено боль­
шее количество слонов.
Задача 3. Получите из шести троек как можно больше идущих
подряд натуральных чисел начиная с 1. Можно пользоваться дей­
ствиями арифметики (+, —, •, :) и скобками. (Одно и то же дей­
ствие может встречаться несколько раз. Использовать все четыре
действия не обязательно. Не разрешается составлять из троек мно­
гозначные числа, такие как 33 и 333.)
Пример: 1 = (3 + 3 + 3): (3 + 3 + 3).
Решение считается лучше предыдущего, если команда может
продолжить список с того числа, на котором остановилась высту­
павшая перед этим команда.
Задача 4. Получите число 100 только из единиц с помощью че­
тырёх арифметических действий (+, - , •, :) и скобок. Единиц долж­
но быть как можно меньше. Не разрешается составлять из единиц
многозначные числа, такие как 11 и 111.
(Пример: 100 = 1 + 1 + 1 + . .. + 1. В этом решении сто единиц.)
Решение считается лучше предыдущего, если оно позволяет по­
лучить число 100 из меньшего количества единиц.
Занятие 24. Математический аукцион III 195

Подборка задач
Задача 1. Найдите три целых числа, сумма которых равна их
произведению.
Задача 2. Найдите семь целых чисел с таким свойством: ни од­
но из них не делится на 7, но их сумма делится на 7.
Задача 3. Разложите на простые множители следующие числа:
555, 2222, 2277, 7007.
Задача 4. Байт (единица информации, используемая в компью­
терах) состоит из восьми битов. Каждому биту может быть назначе­
но одно из двух значений: 0 или 1. Разные сочетания битов соответ­
ствуют разным значениям байта. Сколько разных значений может
иметь байт?
Задача 5. Волшебник Шмерлин хочет отмерить 60 граммов зе­
лья. У Шмерлина есть полный кувшин зелья, но он потерял все свои
мерные стаканчики. Из посуды у него осталось только два флако­
на: в один помещается 40 граммов зелья, а в другой 90 граммов.
Шмерлин может переливать зелье из кувшина во флакон, из фла­
кона в кувшин или из одного флакона в другой сколько угодно раз.
Помогите ему отмерить 60 граммов зелья.
Задача 6. Расшифруйте пример. (Одинаковые буквы обознача­
ют одинаковые цифры, а разные буквы —разные цифры.)

Задача 7*. Докажите, что число 111... 111, состоящее из 99 еди­


ниц, делится на 37.
Занятие 25

Делимость III. Признаки делимости

На предыдущем занятии по делимости мы говорили о простых


числах и о разложении на простые множители. Теперь мы знаем,
что если число разложено на простые множители, то легко опреде­
лить, на какие числа оно делится.
Однако сначала нужно разложить число. Для этого приходится
проверять его делимость на разные числа. Сегодня мы изучим при­
знаки делимости. Они позволяют быстро узнать, делится ли одно
число на другое, не выполняя деления.
В заключение мы немного поговорим о том, какую роль играет
разложение на простые множители в современной криптографии.

Математическая разминка
Задача для разминки 1. Что дороже: вагон, доверху загружен­
ный 50-рублевыми золотыми монетами, или вагон, наполовину за­
груженный 100-рублевыми золотыми монетами?
Задача для разминки 2. Корабль бросил якорь в бухте. С борта
корабля свисает верёвочный трап, у которого десять перекладин.
Расстояние между перекладинами — 30 сантиметров.
Сегодня разница уровней между приливом и отливом составляет
90 сантиметров. Утром во время отлива нижняя перекладина была
под водой, а вторая снизу— точно на уровне воды. Сколько перекла­
дин будут под водой во время прилива?

Тема занятия:
«Признаки делимости»
Как мы уже видели, разложение числа на простые множители
даёт нам массу полезных сведений об этом числе. В частности, если
число уже разложено, то мы можем предсказать, разделится ли оно
на какое-то другое число.
Занятие 25. Делимость III. Признаки делимости 197

Но сначала нужно разложить число на простые множители. Лег­


ко это или трудно?
Всё зависит от конкретного числа. Есть числа, которые можно
разложить без труда, но есть и такие, которые придётся расклады­
вать очень и очень долго.
Возьмём, например, число 72 000. Сразу видно, что оно делится
на 1000. Дальнейшее разложение не представляет трудностей:
72000 = 1000-72 =
= 10-10-10-8-9 =
= 2-5-2-5-2-5-2-2-2-3-3 =
= 26-32-53.

Однако если взять число 2 049 209, то задача значительно услож­


няется. Мало того, что это число большое: оно ещё и не даёт нам
ни малейшего намёка на его делители. Поэтому всё, что мы можем
сделать, — это по очереди делить его на простые числа до тех пор,
пока оно не разделится или пока мы не дойдём до простого чис­
ла, квадрат которого больше 2049209. (Хотя бы один из простых
множителей числа должен быть не больше квадратного корня из
этого числа. Поэтому, дойдя до квадратного корня из 2 049 209, мож­
но прекратить поиск.) На самом деле 2 049 209 — это произведение
двух простых множителей: 1019 и 2011. Если бы мы действительно
проверяли делимость на все простые числа начиная с 2, то мы бы
нескоро добрались до этих множителей.
К сожалению, простого алгоритма, который позволял бы раскла­
дывать на множители очень большие числа, не существует. Поэтому
такую задачу не сможет быстро решить даже мощный компьютер.
Как ни странно, математики научились извлекать пользу из этого,
казалось бы, неприятного обстоятельства. В конце занятия (с. 200)
мы поговорим о современной криптографии и о том, как в ней при­
меняется разложение на простые множители.
Однако во многих случаях можно обнаружить делитель числа
путём простой проверки, не выполняя самого деления. Это будет
темой сегодняшнего занятия: мы познакомимся с признаками де­
лимости. Эти признаки позволяют быстро проверить делимость на
некоторые числа.
Оставшаяся часть этой главы посвящена самым употребитель­
ным признакам делимости.
198 Раздел 1. Планы занятий

Признак делимости на 2
Число делится на 2, если оно оканчивается цифрой О,2,4, 6 или 8.
Этот признак нам хорошо известен, но всё же его надо доказать.
Любое число можно представить в виде суммы числа, оканчи­
вающегося на 0, и однозначного числа (например, 134 = 130-1-4).
Число, оканчивающееся на 0, всегда делится на 10; значит, оно де­
лится и на 2. Поэтому делимость этой суммы на 2 зависит только
от делимости второго слагаемого (однозначного числа) на 2. Одно­
значные числа, делящиеся на 2, —это 0, 2, 4, 6 и 8. Следовательно,
число делится на 2 в том случае, если оно оканчивается одной из
этих цифр.

Делимость на 4
В этом случае «правило последней цифры» не действует. Напри­
мер, последняя цифра числа 32 не делится на 4, а само число делит­
ся. С другой стороны, последняя цифра числа 34 делится на 4, а са­
мо число — нет. К счастью, есть другой простой признак делимости
на 4.
Число делится на 4, если две его последние цифры образуют чис­
ло, делящееся на 4.
Например, 32 делится на 4. Значит, любое число, оканчивающе­
еся на 32, — например, 132, 232, 1732 — тоже делится на 4.
Докажем этот признак. Сначала заметим, что все числа, окан­
чивающиеся двумя нулями, — например, 100, 200, 900, 1000, 1100,
29800 —делятся на 4. Почему? Все они делятся на число 100, кото­
рое в свою очередь делится на 4.
Ещё одно полезное наблюдение: любое число можно предста­
вить в виде суммы числа, оканчивающегося двумя нулями, и дву­
значного числа (например, 1732 = 1700 + 32). Поскольку первое
слагаемое всегда делится на 4, делимость суммы на 4 зависит только
от второго слагаемого. Это доказывает наш признак делимости.
В приведённом примере 32 делится на 4; значит, 1732 тоже де­
лится на 4. Однако 1782 не делится на 4, так как 82 не делится на 4.

Делимость на 5
Число делится на 5, если оно оканчивается цифрой 0 или 5.
Признак делимости на 5 можно объяснить так же, как признак
делимости на 2.
Занятие 25. Делимость III. Признаки делимости 199

Делимость на 3
Число делится на 3, если сумма его цифр делится на 3.
Объяснить этот признак немного сложнее.
Сначала изучим свойства чисел 1,10,100,1000,100000, ... Каж­
дое из этих чисел на 1 больше какого-то числа, делящегося на 3.
Например, 1 = 0 + 1, 10 = 9 + 1, 100 = 99 + 1, 1000 = 999 + 1 и так
далее. Это можно показать на схеме:

Теперь рассмотрим другую группу чисел: те, у которых все циф­


ры, кроме первой, — нули. Например, 200, 500, 3000, 200000,... Ка­
кие из этих чисел делятся на 3?
Возьмём, например, число 500. Как мы уже знаем, 100 — это сум­
ма числа, делящегося на 3 (числа 99), и числа 1. Значит, 500 —это
сумма пяти одинаковых чисел, делящихся на 3, и пяти единиц. По­
скольку сумма пяти единиц (число 5) не делится на 3, число 500
также не делится на 3. Другой пример: число 6000 состоит из шести
чисел 999 и шести единиц (сумма которых равна 6). Поэтому 6000
делится на 3.
Мы показали, что 500 не делится на 3, так как 5 не делится на 3.
Мы также объяснили, что 6000 делится на 3, исходя из того, что 6
делится на 3.
Наше объяснение подходит для любого числа этого типа: если
первая цифра такого числа делится на 3, то и само число делится
на 3. Если первая цифра не делится на 3, то и число не делится на 3.
Покажем это на схеме:

делится на 3
Теперь мы готовы применить такое же рассуждение к любому
целому числу. Возьмём, например, 435. Это число состоит из четы-
Раздел 1. Планы занятий
рёх чисел 99 (которые делятся на 3), трёх чисел 9 (которые тоже
делятся на 3) и всех «остаточных» единиц. Сумма этих остаточных
единиц равна 4 + 3 + 5 = 12. Поскольку 12 делится на 3, исходное
число также делится на 3:

делится на 3 делится на 3
А число 835, например, не делится на 3. Действительно, подсчи­
таем для него сумму остаточных единиц: 8 + 3 + 5 = 16. Эта сумма не
делится на 3. Такое рассуждение подходит для любого числа. Итак,
мы доказали признак делимости на 3.

Делимость на 9
Число делится на 9, если сумма его цифр делится на 9.
Признак делимости на 9 доказывается так же, как признак дели­
мости на 3. (Все числа 9, 99, 999, ... делятся на 9.)
Например, число 1234 не делится на 9, так как сумма его цифр
1 + 2 + 3 + 4 = 1 0 не делится на 9.
Число 123 456 789 делится на 9, так как сумма его цифр
1 + 2 + 3 + 4 + 5 + 6 + 7 + 8 + 9 = 45
делится на 9.
Мы обсудили много нового материала, и теперь пора остано­
виться. Чтобы немного отдохнуть, поговорим о том, как разложение
на простые множители применяется на практике.

Делимость и криптография
В наши дни наука о числах и их делимости играет важнейшую
роль. В частности, она лежит в основе современной криптографии.
Что такое криптография? Это наука о тайной переписке. Она
изучает различные шифры, позволяющие превратить обычное со­
общение в скрытое (шифрованное), а затем по шифрованному со-
Занятие 25. Делимость III. Признаки делимости 201

общению восстановить исходное. Эти процессы называются шиф­


рованием и расшифровкой.
Шифрование возникло в глубокой древности: им пользовались
цари, военачальники и шпионы для передачи важных секретных
сведений. Сейчас шифрование вошло в повседневную жизнь. Каж­
дый раз, когда через Интернет нужно передать информацию, не
предназначенную для посторонних (например, пароль), сообщение
сначала шифруется на компьютере. Когда шифрованное сообщение
приходит на компьютер адресата, оно расшифровывается.
Как шифруются секретные сообщения? С изобретением компью­
тера традиционные шифры, применявшиеся на протяжении мно­
гих столетий, стали бесполезными: теперь их легко взломать. По­
этому математики изобрели новые способы шифрования данных.
Взломать эти современные шифры почти невозможно. Они разрабо­
таны на основе изучения свойств чисел, и главную роль в них играет
разложение на простые множители.
Мы уже говорили о том, как трудно разложить на простые мно­
жители большое число. Благодаря этой трудности математики смог­
ли придумать метод шифрования, практически не поддающийся
взлому. Предположим, что мне и вам нужно передавать секретные
сообщения между нашими компьютерами: например, я продаю че­
рез Интернет какие-нибудь безделушки, а вы посылаете мне номер
кредитной карты. Этот номер должен быть зашифрован: иначе, ес­
ли сообщение будет перехвачено, злоумышленник сможет восполь­
зоваться вашей кредитной картой.
Сообщения шифруются так. Программа шифрования на моём
компьютере выбирает пару очень больших простых чисел, которые
будут служить ключом при расшифровке принятого от вас сообще­
ния. Эта пара чисел называется «закрытым ключом»: информация
о нём не выйдет за пределы моего компьютера. Затем я посылаю
вам произведение этих чисел, которое будет служить ключом при
шифровании сообщения. Это произведение называется «открытым
ключом»: его сможет получить каждый, кто хочет отправить мне
шифрованное сообщение. Я могу поместить открытый ключ на сво­
ём сайте, напечатать его в газете или отправлять по электронной
почте всем желающим.
По моему открытому ключу программа шифрования на вашем
компьютере шифрует номер кредитной карты (или другую важную
информацию, которую вы хотите мне отправить). Шифрованное со-
202 Раздел 1. Планы занятий

общение передаётся на мой компьютер, где оно расшифровывается


по закрытому ключу (исходной паре простых чисел).
Почему шифр такого типа очень трудно взломать? Чтобы рас­
шифровать сообщение, зашифрованное по открытому ключу, надо
знать закрытый ключ. Но мой закрытый ключ хранится в тайне.
Предположим, что злоумышленник сумел подключиться к линии
связи между нашими компьютерами. Он увидит шифрованное сооб­
щение, но, чтобы прочитать его, ему придётся по произведению (от­
крытому ключу) угадать составляющие его множители (закрытый
ключ). Оба простых числа огромны (сейчас применяются не менее
чем 100-значные числа), не говоря уже об их произведении. Даже
самый быстрый в мире компьютер будет искать эти множители
практически бесконечно долго. Поэтому расшифровать сообщение
смогу только я: ведь только я знаю множители.

Подборка задач
Задача 1. Уберите из показанной на рисунке
фигуры заданное число спичек так, чтобы оста­
лось ровно пять одинаковых квадратов:
а) 8 спичек; б) 4 спички; в) 6 спичек.
Задача 2. Докажите, что число Ю2016 + 2 де­
лится на 3.
Задача 3. Может ли число, состоящее только из четвёрок, де­
литься на число, состоящее только из троек? А наоборот? Для каж­
дого вопроса либо приведите пример пары чисел, либо докажите,
что такой пары не существует.
Задача 4. Агент секретной службы 00Х подкрался к сверхсек­
ретному сейфу с кодовым замком. Агент знает, что сейф открыва­
ется семизначным кодом, состоящим из только двоек и троек. Ему
также удалось узнать, что двоек больше, чем троек, и что код делит­
ся на 3 и на 4. У агента 00Х есть только одна попытка: если он набе­
рёт неверный код, то сработает сигнализация и включится сирена.
Помогите агенту открыть сейф без лишнего шума.
Задача 5. Макс придумал зашифрованный пример: АВ - ВА= 7.
Но как он ни старался, решить этот пример он не смог. Объяс­
ните почему. (Одинаковые буквы обозначают одинаковые цифры,
а разные буквы — разные цифры.)
Занятие 25. Делимость III. Признаки делимости 203

Задача 6. Сколько двоек в наборе простых множителей числа


16! = 16-15-...-2-1?
Задача 7. Злой великан поймал трёх мудрых гномов. Он собира­
ется съесть гномов, но сначала хочет поиграть с ними. Великан по­
казывает гномам коробку с двумя красными и тремя чёрными шля­
пами. Затем великан вручает каждому гному листок бумаги, гасит
свет и надевает три из пяти шляп на головы гномов. Когда великан
снова зажигает свет, каждый гном может видеть шляпы двух дру­
гих гномов, но не видит своей собственной. Великан объясняет, что
каждый гном получит шанс угадать цвет своей шляпы. Если гном
угадает цвет правильно, то он сможет уйти, но в случае ошибки
великан его съест.
Угадывание будет проводиться по таким правилам: великан три
раза взмахнёт рукой. При каждом взмахе каждый гном сможет либо
записать на листке цвет своей шляпы и отдать листок великану, ли­
бо ничего не делать. После каждого взмаха великан вслух зачитает
те листки, которые он получит. После трёх взмахов великан отпу­
стит тех гномов, которые угадают цвет правильно.
Гномам запрещается разговаривать друг с другом и обмениваться
какими-либо знаками. Но эти гномы действительно мудрые (в школь­
ные годы все они занимались в одном математическом кружке).
Каждый гном хочет спастись сам и спасти своих друзей. Покажите,
что все гномы могут правильно угадать цвета своих шляп.

ДОПОЛНИТЕЛЬНЫЕ ЗАДАЧИ

Задача 8*. Робот Робби перемножает все числа от 1 до 100 и под­


считывает сумму цифр этого произведения. Затем он берёт полу­
ченную сумму и подсчитывает сумму её цифр. Так он продолжает
делать до тех пор, пока не получится однозначное число. Что это за
число?
Задача 9*. Предположим, что вы записали целое число, а затем
другое число, состоящее из тех же цифр, но в обратном порядке.
Докажите, что разность этих двух чисел делится на 9.
Занятие 26

Делимость IV. Взаимно простые числа

Взаимно простые числа


Сегодня последнее в этом году занятие, посвященное делимости.
Мы введём понятие взаимно простых чисел и узнаем, как с их по­
мощью получить сразу целый набор новых признаков делимости.

Математическая разминка
Задача для разминки 1. Цифры выписали в ряд по некоторому
правилу: 8, 2, 9, 0, 1, 5, 7, ... Какая следующая цифра в этом ряду?
Задача для разминки 2. Семья из семи человек собралась в од­
ной комнате. Дедушка читает газету, бабушка вяжет, папа играет
в шахматы, мама разгадывает судоку, старший сын чинит велоси­
пед, средний ему помогает. Чем занимается младший сын?

Тема занятия:
«Взаимно простые числа»
Эту тему не стоит начинать со строгого определения взаимно
простых чисел: для многих детей этого возраста оно будет слишком
абстрактным. Вместо этого мы начнём с задач, которые естествен­
ным образом приведут к этому понятию. Формальное определение
будет дано после того, как дети на практике поймут пользу и важ­
ность понятия взаимной простоты.
Какие задачи могут послужить этой цели? Мы будем развивать
тему делимости на составные числа, которая естественным образом
продолжает тему прошлого занятия — признаки делимости.
Мы начинаем занятие с повторения: вспоминаем уже изученные
признаки делимости на 2, 3, 4, 5 и 9.
Затем немного говорим о делимости на 11. Изучение признака
делимости на 11 в полном объёме лучше отложить до следующе­
го года: и сам признак, и его доказательство довольно сложны. Но
Занятие 26. Делимость IV. Взаимно простые числа 205

в некоторых случаях делимость на 11 легко увидеть. Например, все


числа вида АА, ВВВВ, АВВА и ААВВ делятся на 11. Эти частные слу­
чаи помогают решить многие задачи, в том числе одну задачу из
сегодняшней подборки.
Наконец мы вплотную подходим к теме занятия — взаимно про­
стым числам.
• Я задаю вопрос: как узнать, делится ли целое число на 6? Дети
выдвигают гипотезу: число делится на 6, если оно делится и на 2,
и на 3. Поэтому, чтобы проверить делимость числа на 6, надо убе­
диться в том, что оно чётное и что сумма его цифр делится на 3.
Интуитивно ясно, что гипотеза верна, но всё же её надо дока­
зать. Мы формулируем доказательство. Из делимости числа на 3
следует, что в наборе его простых множителей есть 3. Из делимости
на 2 следует, что в этом же наборе есть 2. Итак, среди простых
множителей рассматриваемого числа есть 3 и 2; значит, оно делится
на произведение этих простых множителей.
• Затем я спрашиваю о делимости на 10.
Мы решаем, что для делимости на 10 необходима делимость на 2
и на 5. Объясняем это так же, как в случае делимости на 6.
• Следующий вопрос — о делимости на 12.
Здесь я намеренно ввожу участников в заблуждение.
— Похоже, — говорю я, — мы обнаружили новый признак дели­
мости: если число делится на каждый из двух множителей по от­
дельности, то оно делится и на их произведение. Этот признак под­
ходит для делимости на 6 (произведение 2 и 3) и на 10 (произведе­
ние 2 и 5). Давайте проверим его для делимости на 12. Это произ­
ведение чисел 2 и 6. Если число делится на 2 и на 6, обязательно ли
оно делится на 12?
Дети быстро находят опровергающий пример: 30 делится на 2
и на 6, но не делится на 12. Значит, из делимости на 2 и на 6 не
следует делимость на 12.
Вместе мы находим объяснение этому факту. Поскольку 12 =
= 2 - 2 - 3 , для делимости на 12 необходимы хотя бы две двойки
и одна тройка в наборе простых множителей. Обязательно ли вы­
полнится это условие, если число делится на 6 и на 2?
Делимость на 6 означает, что в наборе простых множителей есть 2
и 3, а делимость на 2 —что в этом наборе есть 2. Возьмём какое-
нибудь число, у которого в наборе простых множителей есть тройка
и ровно одна двойка. Такое число будет делиться на 6 и на 2, но не
206 Раздел 1. Планы занятий

будет делиться на 12. (Например, 30 = 2 • 3 • 5. Поэтому 30 делится


на 6 и на 2, но не делится на 12.)
Итак, наша гипотеза не подтвердилась. Всё же хотелось бы раз­
ложить 12 в произведение двух множителей так, чтобы из делимо­
сти на оба эти множителя следовала делимость на 12. Кто-то шутит:
это 12 и 1. Затем поступает более интересное предложение: что,
если взять 3 и 4?
Отличная идея! Действительно, если число делится на 4, то в на­
боре его простых множителей точно есть две двойки. Если оно де­
лится ещё и на 3, то в этом наборе обязательно есть тройка. Итак,
разложив такое число на простые множители, мы получим как ми­
нимум две двойки и одну тройку. Но это в точности означает, что
число делится на 12.
Мы продолжаем формулировать признаки делимости на разные
числа. Каждый раз мы стараемся представить число в виде произве­
дения двух подходящих множителей: делимость на эти множители
должна гарантировать делимость на само число.
• Что необходимо для делимости на 15? Делимость на 3 и на 5.
• Что необходимо для делимости на 18?
Достаточно ли проверить делимость на 6 и на 3? Если у числа есть
простой множитель 2 и ровно один простой множитель 3, то оно бу­
дет делиться на 3 и на 6, но не будет делиться на 18 (например, чис­
ло 24). Значит, для делимости на 18 нужна делимость на 9 и на 2.
Мы переходим к делимости на 20 и на 24. Теперь дети хором вы­
крикивают правильные ответы. Судя по всему, настало время вве­
сти понятие взаимно простых чисел.
Я задаю вопрос: «Чтобы проверить делимость на число, мы запи­
сываем его как произведение двух множителей специальным спосо­
бом: 12 = 3-4, 15 = 3-5, 18 = 2-9, 20 = 4-5, 24 = 8-3. Как сформули­
ровать общее правило? Если нам понадобится проверить делимость
на другое число, по какому критерию мы будем подбирать подходя­
щую пару множителей?»
Пара кружковцев выкрикивает ответ: «У них не должно быть
общих простых множителей!» Остальные соглашаются. Ура! Мы
естественным образом пришли к понятию взаимно простых чисел,
и всем ясно, для чего они нужны и как применять их на практике.
Преподавателю остаётся только одно: ввести соответствующий тер­
мин. Я объявляю: это свойство чисел — не иметь общих делителей —
настолько важно, что ему дали специальное название.
Занятие 26. Делимость IV. Взаимно простые числа 207

Определение. Два целых числа называются взаимно простыми,


если у них нет общих делителей, кроме числа 1.
Зная, как называются такие числа, мы можем сформулировать
признак делимости на их произведение.
Признак делимости. Пусть два множителя взаимно просты. То­
гда если число делится на каждый из этих множителей, то оно де­
лится и на их произведение.
В заключение мы делаем ещё несколько упражнений на взаимно
простые числа.
Числа 15 и 24 взаимно просты? А 8 и 27?
Как быстро проверить делимость на 45? А на 28?
Для преподавателей. Если остаётся время и дети не устали, можно обоб­
щить этот признак на произведение трёх и более множителей. Пусть
несколько множителей попарно взаимно просты. Тогда из делимости на
каждый из них следует делимость на их произведение.
Например, 60 = 3 • 4 • 5, и в каждой паре (3, 5), (3,4) и (4, 5) числа
взаимно просты. Поэтому если число делится на 3, на 4 и на 5, то оно
обязательно делится на 60.
Подборка задач
Задача 1. Поставьте 12 стульев в три ряда так, чтобы в одном
ряду было 6 стульев, а в двух других рядах по 4 стула.
Задача 2. В королевстве Годор живут хоббиты, эльфы и гномы.
Все жители королевства по традиции соблюдают древние правила.
• Если хоббиты и эльфы живут в одной деревне, то в ней не долж­
ны селиться гномы.
• Если где-то живут эльфы, но нет гномов, то рядом с эльфами
должны поселиться хоббиты.
• Если хоббиты живут в одном селении с гномами, то там долж­
ны также жить эльфы.
Хоббит Бильбо хочет основать возле знаменитых горячих источ­
ников Годора приют «Только для хоббитов». Будет ли это нарушени­
ем древних правил?
Задача 3. Ответьте на следующие вопросы. В случае ответа «да»
обоснуйте его. В случае ответа «нет» приведите пример.
а) Известно, что число делится на 9 и на 2. Обязательно ли оно
делится на 18?
б) Известно, что число делится на 6 и на 9. Обязательно ли оно
делится на 54?
208 Раздел 1. Планы занятий

в) Известно, что число делится на 3 и на 8. Обязательно ли оно


делится на 24?
г) Известно, что число делится на 21 и на 10. Обязательно ли оно
делится на 210?
Задача 4. Сколькими нулями оканчивается произведение
25! = 25-24-...-3-2-1?
Задача 5. а) Делится ли 10000100008 на 18?
б) Делится ли 37000032 на 12?
в) Делится ли 12 345 678 900 на 60?
Задача 6. Произведение двух целых чисел равно 100, но ни одно
из них не делится на 10. Найдите эти числа. Докажите, что вы нашли
все возможные решения.
Задача 7*. Решите ребус: 2222=А х ВВ х ВСВ. (Одинаковые бук­
вы обозначают одинаковые цифры, а разные буквы — разные цифры.)

Д О П О Л Н И Т Е Л Ь Н Ы Е ЗАДАЧИ

Задача 8. В Тридевятом царстве есть десять


колодцев с мёртвой водой. Колодцы обозначены
номерами от 1 до 10. Вода в этих колодцах по
виду и вкусу не отличается от обычной, но да­
же один её глоток будет смертельным, если не
принять противоядие. От мёртвой воды известно
только одно противоядие: вода из колодца, номер которого больше.
(Например, тот, кто глотнул воды из колодца с номером 6, спасётся,
если после этого выпьет воды из колодца с номером 7, 8, 9 или 10.)
Противоядия от воды из десятого колодца не существует. Из первых
девяти колодцев воду может достать любой желающий, а десятый
колодец находится во дворце царя Тридевятого царства Афрона.
Иван-Царевич вызвал царя Афрона на поединок. По правилам
поединка каждый участник должен выпить воду из чаши, поднесён­
ной соперником. Царь Афрон охотно согласился. Ведь в его распо­
ряжении десятый колодец: вода из него обезвредит любую воду, ко­
торую нальёт в чашу Иван, а самого царевича точно убьёт.
Поединок состоялся по правилам: каждый участник выпил воду,
поднесённую соперником. Но к удивлению и радости всех жителей
царства, Иван-Царевич остался в живых, а царь Афрон умер. Как это
случилось?
Занятие 27

Математические игры со стратегией I

Следующие два занятия посвящены играм и выигрышным стра­


тегиям. Зачем заниматься играми на математическом кружке? Дети
любят играть, а анализ многих игр содержит важные математиче­
ские идеи. Кроме того, эти занятия проводятся в конце учебного
года. В это время дети начинают больше думают о приближающих­
ся летних каникулах, чем об учёбе. Игры и выигрышные страте­
гии — это та увлекательная тема, которая сможет удержать их вни­
мание.
В начале занятия мы обсудим понятия игры и выигрышной стра­
тегии. Затем мы разберём несколько игр и познакомимся с путями
поиска выигрышной стратегии.

Что принести на урок


Играть в первые две игры сегодняшнего занятия интереснее
с реальными предметами: фишками, монетами, спичками. Но мож­
но играть в них и на бумаге.

Математическая разминка
Сегодня в качестве разминки предлагаются две задачи про парик­
махеров. Первая служит примером логического парадокса, а вторая —
просто шутка.
Задача для разминки 1. Командир роты драгунов приказал рот­
ному парикмахеру побрить каждого, кто не бреется сам. Должен ли
парикмахер побриться сам?
Задача для разминки 2. В небольшом городке работает только
один парикмахер. Парикмахер говорит: «Я брею всех мужчин в этом
городе, которые не бреются сами!» Известно, что это правда и что
парикмахер не носит бороду. Как такое может быть?
210 Раздел 1. Планы занятий

Тема занятия:
«Игры и выигрышные стратегии»
Введём некоторые термины. Во всех играх, которые мы будем
обсуждать на этих двух занятиях, игроки делают ходы по очереди.
Игрока, который делает первый ход, мы будем называть первым иг­
роком, а его соперника — вторым игроком.
В каждой игре мы будем искать выигрышную стратегию. Выиг­
рышная стратегия — это набор инструкций, соблюдение которых
гарантирует одному из игроков победу в игре.
Уделим некоторое время прояснению этого определения. Какого
рода инструкции нам нужны? Прежде всего подчеркнём, что они
должны быть точными и полными. Это значит, что, какая бы ситу­
ация ни сложилась в игре, игрок может сделать ход согласно этим
инструкциям. Во-вторых, выигрышная стратегия должна быть про­
веряемой. Это значит, что можно объяснить, почему эта стратегия
обязательно принесёт игроку победу.
В одних играх мы сможем найти выигрышную стратегию для
первого игрока, в других—для второго. Но ни в одной игре нам
не удастся придумать выигрышную стратегию для обоих. Как это
объяснить? Почему невозможна игра, в которой у каждого игрока
есть план действий, гарантирующий выигрыш? Ответ прост. В та­
кой игре каждый раз побеждали бы оба игрока, что невозможно.
В любой ли игре есть выигрышная стратегия? Конечно, нет. Её
может не быть по разным причинам.
• Некоторые игры могут заканчиваться вничью, например кре­
стики-нолики на поле 3 x 3 или шашки. Оказывается, в обеих этих
играх у каждого игрока есть стратегия, позволяющая избежать про­
игрыша: придерживаясь этой стратегии, игрок обязательно сведёт
партию вничью или выиграет. Поэтому ни для крестиков-ноликов,
ни для шашек не существует выигрышной стратегии для одного из
игроков — вместо этого есть беспроигрышная для обоих. Вы, навер­
ное, знаете, как играть в крестики-нолики, чтобы точно не проиграть.
А как насчёт того, чтобы не проиграть в шашки? Беспроигрышная
стратегия для игры в шашки тоже существует, но она гораздо сложнее.
Такая стратегия была найдена только недавно, и она настолько слож­
на, что ей можно обучить компьютер, но не человека1. (Из этого, кста-
1
Мы говорим о шашках на поле 8 x 8 . Игра в шашки на поле 10x10 еще не
решена до конца.
Занятие 27. Математические игры со стратегией I 211

ти, следует, что два компьютера, обученных правильной стратегии,


обязательно должны сыграть вничью. А когда в шашки играют два
человека, результат по-прежнему определяется мастерством игроков.)
• Бывает, что игра ещё не «решена», и поэтому неизвестно, есть
ли в ней выигрышная стратегия. Рассмотрим, например, шахматы.
Шахматы — очень сложная игра. Говоря математическим языком,
верно одно из трёх утверждений: либо выигрышная стратегия есть
у первого игрока (играющего белыми), либо у второго (играющего
чёрными), либо у обоих есть «беспроигрышные» стратегии. Тео­
ретики шахмат столетиями пытались найти такую стратегию или
хотя бы выяснить, какая из этих трёх возможностей имеет место
в действительности, но ответа до сих пор нет! (Большинство специ­
алистов склоняются к третьему варианту, поскольку партии между
гроссмейстерами чаще всего заканчиваются вничью.) В шахматах
и других играх, где выигрышная стратегия неизвестна, результат
зависит от мастерства игроков.
• Во многих играх важен элемент случайности: как выпадет ку­
бик или как перемешается колода карт. Для победы в таких играх
нужны и мастерство, и удача. К ним относятся многие популярные
игры, например нарды и монополия.
• В таких играх, как рулетка или «змеи и лестницы», мастерство
вообще не требуется: результат зависит только от везения.
Но мы сейчас не будем заниматься играми такого рода. Во всех
играх, которые мы будем изучать, играют двое и элемент случайно­
сти отсутствует. Кроме того, эти игры не могут закончиться вничью
или продолжаться бесконечно. В таких играх у одного из игроков
(либо у первого, либо у второго) всегда есть выигрышная стратегия
(возможно, очень сложная). Это значит, что одному из игроков га­
рантирована победа, если он знает, как правильно играть.
Настало время для первой игры.
Пример 1. На столе лежат две кучки конфет —по 15 конфет
в каждой. Тим и Алекс по очереди берут любое количество конфет
из одной из кучек. Проигрывает тот, кто не может сделать ход.
Тим ходит первым. У кого есть выигрышная стратегия и в чём она
заключается?
РАЗБОР ПРИМЕРА 1. Чтобы освоиться с правилами игры, я показы­
ваю на доске пару первых возможных ходов Тима и Алекса. Затем
дети разбиваются на пары и начинают играть. (Чтобы игра шла ве-
212 Раздел 1. Планы занятий

селее, я раздаю им фишки.) Цель состоит в том, чтобы найти выиг­


рышную стратегию. Если такая стратегия найдена, у соперника нет
шансов: каждый раз он будет проигрывать.
Дети быстро находят выигрышную стратегию: второй игрок
(Алекс) должен повторять все годы первого: брать то же количество
конфет, но из другой кучи. Тогда после каждого хода Алекса будет
восстанавливаться симметрия — количество конфет в кучках будет
одинаково. Поэтому если Тим сможет сделать ход, то Алекс всегда
сможет ответить. Значит, второй игрок обязательно выиграет.
Пример 2. Те же правила, что и в предыдущей задаче, но в од­
ной кучке —15 конфет, а в другой —16. У кого есть выигрышная
стратегия и в чём она заключается?
РАЗБОР ПРИМЕРА 2. Дети быстро понимают, что первый игрок вы­
игрывает. Первым ходом он берёт одну конфету из кучки с 16 кон­
фетами и уравнивает кучки. Дальше он должен использовать вы­
игрышную стратегию из предыдущей задачи — копировать каждый
ход второго игрока.
В обеих задачах, которые мы пока разобрали, ключом к победе
была симметрия: каждым ходом один из игроков возвращает пози­
ции некую симметрию. При этом симметрия должна быть такой,
чтобы для каждого хода противника имелся ответный ход, который
снова возвращает симметрию.
Выигрышные стратегии, которые основаны на симметрии, — это
основная тема сегодняшнего урока.
Пример 3. Два игрока по очереди кладут одинаковые монеты
на круглый стол. Монеты могут соприкасаться, но не должны пере­
крываться. Двигать уже положенные монеты не разрешается. Про­
игрывает тот, кто не может положить монету. У кого есть выигрыш­
ная стратегия и в чём она заключается?
РАЗБОР ПРИМЕРА 3. У первого игрока есть выигрышная страте­
гия. Первым ходом он кладёт монету точно в центр стола. Каж­
дым последующим ходом он симметрично отражает ходы второ­
го игрока относительно центра стола. (Симметрия относительно
точки называется центральной симметрией.) При такой стратегии
первый игрок каждым своим ходом восстанавливает на столе цен­
тральную симметрию. Поэтому, куда бы ни положил монету второй
игрок, первый всегда сможет положить монету в симметричное
положение.
Занятие 27. Математические игры со стратегией I 213

Для преподавателей. Следующая задача может оказаться намного слож­


нее предыдущих. Поэтому сейчас наступил подходящий момент для то­
го чтобы обобщить: напомнить детям, что во всех предыдущих зада­
чах ключом к решению была симметрия. В двух первых задачах мы ис­
пользовали осевую (зеркальную) симметрию, а в третьей — централь­
ную (относительно точки). И во всех последующих задачах симметрия
поможет нам найти правильное решение.
Пример 4. Тим и Алекс по очереди
ставят слонов на поля шахматной доски
8 x 8 так, чтобы они не били друг друга.
Проигрывает тот, кто не может поставить
слона. Тим ходит первым. У кого есть вы­
игрышная стратегия и в чём она заключа­
ется? (Шахматный слон ходит и бьёт по
диагонали на любое количество клеток.
Цвет фигур в этой игре не имеет значе­
ния.)
РАЗБОР ПРИМЕРА 4. В этой игре тоже основная идея — симметрия.
Второй игрок (Алекс) должен отражать ходы первого игрока (Тима)
относительно вертикальной линии, проходящей посередине доски.
Посмотрим, как работает эта стратегия. Во-первых, каждым сво­
им ходом Алекс восстанавливает симметрию относительно верти­
кальной оси. Далее, рассмотрим поле, на которое Тим собирается
поставить слона. Ни один из стоящих на доске слонов не бьёт это
поле; значит, ни один из них не бьёт и симметричное ему поле: ведь
слоны расположены на двух половинах доски симметрично. Новый
слон, которого ставит Тим, также не бьёт симметричное ему поле,
так как слоны бьют по диагонали.
Итак, после любого хода Тима Алекс сможет поставить слона на
симметричное поле. Поэтому Алекс выиграет.
Для преподавателей. Если вы хотите, чтобы разбор этой игры прошёл
быстрее, то раздайте участникам заранее напечатанные листки с шах­
матными полями. Помните, что в шуме интересного занятия детям
очень легко прослушать, что цвет фигур не имеет значения. Поэтому
повторите это условие несколько раз, и если вы раздаёте детям фишки,
то обязательно используйте одноцветные.
Для преподавателей. После того как мы обсудили идеи центральной и осе­
вой симметрии, можно задать детям пару полезных дополнительных во­
просов.
214 Раздел 1. Планы занятий

— Могли бы мы использовать идею


центральной симметрии в решении при­
мера 4 про шахматных слонов? Предполо­
жим, что второй игрок всегда ставит сло­
на на клетку, симметричную относитель­
но центра доски. (Пример показан на кар­
тинке.) Является ли такая стратегия вы­
игрышной? Подсказка. Допустим, первый
игрок поставит своего слона на главную
диагональ. Что тогда происходит с ответ­
ным ходом второго игрока?
— А почему мы не можем использовать осевую симметрию в игре
про монеты? Существует ли ответный ход на такой ход, в котором мо­
нета пересекает ось симметрии?
Если у вас осталось время, то можно закончить урок более сложной
задачей, которая тоже использует идею симметрии. (Другой вариант —
эту полезную стратегическую игру можно предложить в качестве раз­
минки на следующем занятии.)
Пример 5. Пятнадцать фишек выставлены в ряд. Тим и Алекс по
очереди убирают одну, две или три соседние фишки. («Соседними»
считаются фишки, которые расположены рядом. Если между фиш­
ками есть промежуток, то они не соседние.) Проигрывает тот, кто
не может сделать ход. Алекс ходит первым. У кого есть выигрышная
стратегия и в чём она заключается?

РАЗБОР ПРИМЕРА 5. Чтобы освоиться с правилами игры, мы при­


глашаем двух желающих сыграть партию на доске. Затем дети разби­
ваются на пары и начинают играть. После нескольких партий я спра­
шиваю, готов ли кто-нибудь предложить выигрышную стратегию.
Первой отвечает Эмма. Она предлагает выигрышную стратегию
для первого игрока (Алекса). Первым ходом Алекс должен убрать
какие-то три соседние фишки. После этого каждый ход Алекса опре­
деляется предыдущим ходом второго игрока (Тима): общее количе­
ство фишек, убранных за эти два хода, должно быть равно четырём.
Если Тим убирает одну фишку, то Алекс убирает три; если Тим уби­
рает две, то Алекс тоже убирает две; если Тим убирает три, то Алекс
убирает одну.
Я знаю, что эта стратегия неправильная. Но как объяснить это
кружковцам? Я предлагаю сыграть партию на доске: Эмма будет
Занятие 27. Математические игры со стратегией I 215

первым игроком, а я вторым. Моя задача — показать, что страте­


гия не работает: я буду играть так, чтобы Эмме пришлось нарушить
стратегию.
Моя план прост: я каждый раз я убираю одну фишку. Причём
я выбираю её так, чтобы разбивать длинные последовательности
соседних фишек. Эмма, согласно своей стратегии, каждым ходом
должна убирать три соседние фишки.
В ходе игры промежутков между фишками становится всё боль­
ше. Наступает момент, когда Эмма не может найти три соседние
фишки и вынуждена отказаться от своей стратегии. Мы продолжа­
ем играть, и я выигрываю. Кто-то говорит, что Эмме надо было сде­
лать другой предпоследний ход: тогда она бы выиграла. Я возра­
жаю: если бы Эмма и выиграла, то благодаря своей сообразитель­
ности, а не правильной стратегии. Стратегия Эммы привела её в ту­
пик: она оказалась в ситуации, когда её план ничего не говорит
о том, каким должен быть следующий ход.
После этого кто-то предлагает первым ходом убрать централь­
ную фишку. Я отмечаю, что это отличное начало. Сразу же возника­
ет идея симметрии. Наконец мы вырабатываем выигрышную стра­
тегию! Первый игрок должен сначала убрать центральную фишку,
а затем каждый раз зеркально отражать ход второго игрока (относи­
тельно промежутка, оставшегося посередине). Тогда после каждого
хода первого игрока будет восстанавливаться симметрия. Поэтому,
какие бы фишки ни убрал своим ходом второй игрок, первый всегда
сможет сделать ход. Значит, первый игрок обязательно выиграет.
Пример 6. Та же игра, что и в предыдущей задаче, но фишек
не 15, а 16.
РАЗБОР ПРИМЕРА 6. Дети сразу предлагают выигрышную страте­
гию для первого игрока: первым ходом он должен убрать две цен­
тральные фишки, а последующими ходами зеркально отражать хо­
ды соперника.
Затем мы обобщаем найденную стратегию на любое количество
фишек: если оно нечётное, то первый игрок сначала убирает одну
центральную фишку, если чётное —две центральные фишки. После
этого он симметрично отражает ходы второго игрока.
Мы разобрали несколько игр, в которых найти выигрышную
стратегию помогает симметрия. Эта идея получит дальнейшее раз­
витие в сегодняшней подборке задач.
216 Раздел 1. Планы занятий

Для преподавателей. Когда дети предлагают неверные стратегии, разубе­


дить их бывает непросто. Иногда это удаётся сделать только одним спо­
собом: сыграть с участником так, чтобы он либо проиграл, либо попал
в ситуацию, в которой следовать предложенной стратегии невозможно.
Но такая игра будет для вас трудной задачей: с одной стороны, вам на­
до победить, а с другой — не выдать выигрышную стратегию. Поэтому
участник может перехитрить вас и выиграть! В этом случае можно пред­
ложить вторую партию, в которой вы будете выбирать ходы за участника
в соответствии с его стратегией. Контролируя ходы обоих игроков, вы
сможете реализовать вариант игры, опровергающий ошибочную страте­
гию. (Напомним, что для того, чтобы опровергнуть стратегию, не обяза­
тельно победить противника. Достаточно поставить игрока в ситуацию,
когда он не может сделать ход согласно своей стратегии.)

Подборка задач
Задача 1. а) В трактире на двух столах стоит по 23 грязные та­
релки. Два поварёнка —Джимми и Билли —по очереди уносят от
одной до пяти тарелок с одного (любого) стола. Хозяин даст кон­
фету тому поварёнку, который унесёт последнюю тарелку. Джимми
уносит тарелки первым. У кого есть выигрышная стратегия и в чём
она заключается?
б) Та же задача, но про три стола: на двух столах стоит по 28 та­
релок, и на третьем столе — ещё 4 тарелки.
Задача 2. В кувшине 2 литра грога, а в бутылке — полтора литра
эля. Пираты Длинный Джон и Толстый Роджер по очереди отпивают
либо из кувшина, либо из бутылки. Пират, выпивший последнюю
каплю, выигрывает, а его напарник должен лезть в трюм за добав­
кой. Стоит ли Длинному Джону начинать первым или вторым, если
он во что бы то ни стало хочет выиграть?
Задача 3. На доске написаны числа от 1 до 10. Двое игроков
по очереди вычёркивают по числу, пока не останется всего 2 числа.
Может ли второй игрок играть так, чтобы разница двух оставшихся
чисел была не меньше 5?
Задача 4. а) Стае и Тарас играют в игру. На полоске размером
2 х 8 на двух узких концах стоят по две шашки: чёрные на стороне
Стаса, белые на стороне Тараса. Игроки ходят по очереди: двигают
одну из своих шашек на одну или две клетки вперёд. Тот, кто не
может сделать ход, проигрывает. Стае ходит первым. У кого есть
выигрышная стратегия и в чём она заключается?
Занятие 27. Математические игры со стратегией I 217

б) А какова выигрышная стратегия для полосы 2 х 80?

Задача 5. Тим и Алекс по очереди кладут костяшки домино на


клетчатую доску 10 х 10. Каждая костяшка должна закрывать ровно
две клетки. Костяшки не должны перекрываться. Проигрывает тот,
кто не может положить костяшку. Тим ходит первым. У кого есть
выигрышная стратегия и в чём она заключается?
Задача 6. Пусть р и q — простые числа.
• Сколько делителей у числа р 2 ?
• Сколько делителей у числа р 5 ?
• Сколько делителей у числа р*, где х — целое положительное
число?
• Сколько делителей у числа р • q?
• Сколько делителей у числа р 2 • q?
• (*) Сколько делителей у числа рх -q?
• (*) Сколько делителей у числа рх • qy?

ДОПОЛНИТЕЛЬНЫЕ ЗАДАЧИ

Задача 7*. Придумайте число с суммой цифр 17, которое окан­


чивается на 17 и делится на 17.
Занятие 28

Математические игры со стратегией II

Сегодня второе занятие по математическим играм со стратеги­


ей. На первом занятии мы определили понятия игры и выигрышной
стратегии. Мы также узнали, что основой для выигрышной страте­
гии может служить симметрия.
Сегодня мы увидим, что не всякую игру можно выиграть с по­
мощью симметрии. Сначала мы познакомимся ещё с одной стра­
тегической идеей —дополнением ходов соперника. Затем обсудим
математические игры особого типа — игры-шутки, или псевдоигры.

Что принести на урок


Для второй игры понадобятся фишки. (Принесите их из расчёта
примерно семь фишек на человека. Если у вас не найдётся столько фи­
шек, подойдут монеты, спички, леденцы или, например, фасолины.)

Математическая разминка
Задача для разминки 1. В доме двое часов. Одни отстают на
одну минуту, а другие и вовсе стоят. Какие часы чаще показывают
правильное время?
Задача для разминки 2. На баобабе сидело двадцать птиц. Охот­
ник подкрался к дереву и одним выстрелом убил трёх птиц. Сколько
птиц осталось на баобабе?

Тема занятия:
«Дополнение ходов»
Сейчас мы поиграем в игру, иллюстрирующую принцип допол­
нения ходов.
Пример 1. На столе лежит полоска бумаги, состоящая из 16 квад­
ратиков. Тим и Алекс по очереди берут 1, 2, 3 или 4 соседних квадра­
тика, отрезая их от противоположных концов полоски. Выигрывает
Занятие 28. Математические игры со стратегией II 219

тот, кто забирает последний квадратик. Тим ходит первым. У кого


есть выигрышная стратегия?

Разбор задачи 1. Это полезная задача в методическом отноше­


нии: при её разборе дети часто делают поучительные ошибки.
Сначала Эмма предлагает для второго игрока (Алекса) выиг­
рышную стратегию на основе симметрии. Я предлагаю ей испытать
эту стратегию в реальной игре. Игра начинается. Я создаю ситу­
ацию, в которой после хода Эммы остаётся четыре центральных
квадратика, затем беру их и выигрываю.
Брейдй предлагает для второго игрока другую стратегию. Вто­
рой игрок отвечает на ходы первого игрока так: 1—3,2—2,3—1,4—4.
(Первое число обозначает количество квадратиков, взятых Тимом,
второе — количество квадратиков, взятых Алексом.) Я предлагаю
Брейдй сыграть за второго игрока и выигрываю. Другие участники,
внимательно следившие за игрой, замечают: Брейдй мог бы выиг­
рать, если бы сделал другой предпоследний ход. Но предложенный
ход расходится со стратегией Брейдй. Поэтому такой выигрыш не
подтвердил бы правильность этой стратегии.
На этом идеи иссякают. Очевидно, задача не так проста, как мне
казалось. Поэтому я предлагаю серию промежуточных задач, кото­
рые должны подвести детей к выигрышной стратегии.
Сначала я спрашиваю: кто выиграет, если полоска состоит из пя­
ти квадратиков? Дети сразу отвечают: выиграет второй игрок. Ему
надо дополнить количество квадратиков, взятых первым игроком,
до пяти: 1—4, 2—3, 3—2, 4—1.
Я задаю тот же вопрос для полоски из 10 квадратиков. Участники
догадываются: Алекс может свести задачу к предыдущей, т. е. оста­
вить после своего хода полоску из пяти квадратиков. Для этого надо
дополнить количество квадратиков, взятых Тимом, до пяти: 1—4,
2—3, 3—2, 4—1. После этого Тиму придётся сделать ход в условиях
предыдущей игры, и Алекс сможет применить уже известную выиг­
рышную стратегию.
Затем я предлагаю разобрать случаи, когда квадратиков 15 или 20.
Подводя итоги, мы формулируем выигрышную стратегию для вто­
рого игрока. Эта стратегия работает всякий раз, когда исходное ко-
220 Раздел 1. Планы занятий

личество квадратиков делится на 5. Второй игрок отвечает на каж­


дый ход первого так, чтобы сумма квадратиков, взятых за эти два
хода, была равна 5. Тогда после каждого хода второго игрока коли­
чество оставшихся квадратиков делится на 5. Это количество с каж­
дым разом становится меньше и в конце концов станет равным 0.
Итак, у нас есть выигрышная стратегия для 15 квадратиков. Ка­
кой будет выигрышная стратегия для 16 квадратиков? На этот раз
дети обходятся без подсказки. Эндрю выдаёт идею: если первый иг­
рок первым ходом отрежет один квадратик, то он сведёт ситуацию
к игре с пятнадцатью квадратиками. Причём в этой игре он будет
вторым игроком (победителем). Значит, в игре с шестнадцатью
квадратиками выигрышная стратегия есть у первого игрока.
В заключение мы обсуждаем выигрышные стратегии для случа­
ев, когда квадратиков 17,18,19 или любое другое число, не деляще­
еся на 5. Во всех этих играх может выиграть первый игрок. Для это­
го он должен первым ходом взять столько квадратиков, чтобы число
оставшихся делилось на 5, а последующими ходами дополнять ходы
соперника до пяти.

Псевдоигры
В этом разделе мы рассмотрим игры другого рода — игры-шутки,
или псевдоигры. Результат псевдоигры определён заранее: что бы
ни делали игроки, либо всегда выигрывает первый игрок, либо все­
гда выигрывает второй.
Я предлагаю участникам разбиться на пары, выдаю каждой паре
по 13 фишек и объясняю правила игры.
Пример 2. Тим и Алекс играют в такую игру. Сначала у них есть
куча из 13 камней. Каждым ходом игрок разделяет одну кучу на две
кучи любого размера. Проигрывает тот, кто не может сделать ход.
Тим ходит первым (разделяет кучу из 13 камней на две кучи). У кого
есть выигрышная стратегия и в чём она заключается?
Разбор задачи 2. Дети играют несколько партий. Вскоре они
замечают, что второй игрок каждый раз выигрывает. Как это объ­
яснить? Эндрю замечает, что с каждый ходом куч становится на од­
ну больше. Я подогреваю дискуссию вопросом: какую кучу нельзя
разделить на две? Кучу из одного камня! Значит, игра остановится,
когда в каждой куче будет только один камень. (Это значит, что куч
будет ровно 13.)
Занятие 28. Математические игры со стратегией II 221

Подведём итоги.
• Игра начинается с одной кучи.
• С каждым ходом количество куч увеличивается на 1.
• Игра заканчивается, когда куч становится 13.
Теперь понятно, почему всегда выигрывает второй игрок. После
каждого хода первого игрока количество куч становится чётным,
а после каждого хода второго — нечётным. Поэтому, какие бы стра­
тегии ни избрали игроки, 13 куч получится после хода второго иг­
рока. Значит, второй игрок обязательно выиграет.
Пример 3. Тим и Алекс по очереди ставят ладьи на полях шах­
матной доски 8 x 8 так, чтобы они не били друг друга. (Цвет ладей
в этой игре не имеет значения.) Проигрывает тот, кто не может по­
ставить ладью. Алекс ходит первым. У кого есть выигрышная стра­
тегия и в чём она заключается?
Разбор задачи 3. Обсуждение этой задачи можно начать с то­
го, чтобы предложить детям поискать выигрышную стратегию, ис­
пользуя идею прошлого урока — симметрию. Несложно увидеть, что
второй игрок обязательно выиграет, если он будет копировать ходы
противника симметрично относительно центра доски. (Почему такая
стратегия будет работать? А будет ли работать осевая симметрия?)
Теперь докажем, что это псевдоигра: в ней всегда выигрывает
второй игрок, независимо от того, какие ходы он делает.
Две ладьи атакуют друг друга, если они стоят на одной горизон­
тали или на одной вертикали. Поэтому каждым ходом игрок должен
поставить ладью на пересечение свободной горизонтали и свобод­
ной вертикали. Кроме того, каждая новая ладья, поставленная на
доску, исключает из игры ровно одну горизонталь и ровно одну вер­
тикаль.
Поэтому после первого хода в любом случае остаётся семь сво­
бодных горизонталей и семь свободных вертикалей. Когда постав­
лена вторая ладья, остаётся шесть свободных горизонталей и вер­
тикалей, и т. д. Когда настанет время ставить восьмую ладью, сво­
бодными будут одна горизонталь и одна вертикаль. После того как
игрок поставит ладью на пересечение этих горизонтали и верти­
кали, все горизонтали и вертикали будут заняты. Поэтому всегда
выигрывает тот, кто ставит восьмую ладью, — второй игрок.
Итак, у нас есть два полезных инструмента для разработки выиг­
рышных стратегий: симметрия и идея дополнения ходов соперника.
222 Раздел 1. Планы занятий

Кроме того, теперь мы знаем, что результат некоторых игр не зави­


сит от стратегии игроков. Пришло время решать задачи.
Подборка задач к этому занятию содержит несколько игр. Среди
них есть и такие, для которых недостаточно применить изученные
методы: потребуется придумать что-то новое.

Подборка задач
Задача 1. Много лет назад, когда отцу было 27 лет, сыну было
три года. Сейчас отец втрое старше сына. Сколько лет сыну?
Задача 2. Обо всех зоопарках Зазеркалья извест­
но следующее.
1. Если в каком-нибудь зоопарке есть и гиппопо­
тамы, и носороги, то в этом зоопарке нет жирафов.
2. В каждом зоопарке есть гиппопотам или носо­
рог, а в некоторых зоопарках есть и тот и другой.
3. Если в каком-нибудь зоопарке есть и гиппопо­
там, и жираф, то в этом зоопарке обязательно есть
носорог.
В зоопарке столицы Зазеркалья есть жираф. Есть
ли там носорог? А гиппопотам?
Задача 3. В первом классе учительница разда­
ла ученикам по три шара. Некоторые шары были
жёлтыми, а остальные — красными. Сначала учительница сказала:
«Поднимите руки те, у кого не меньше двух красных шаров». Подня­
лось 13 рук. Затем она велела поднять руки тем, у кого не меньше
двух жёлтых шаров. Поднялось 15 рук. Наконец подняли руки те уче­
ники, у кого были шары разных цветов. Таких оказалось 17. Сколько
учеников получили все три шара одного цвета?
Задача 4. Илья и Наташа играют в такую игру. У них есть две
тарелки с крекерами: на одной тарелке 31 крекер, на другой 27.
Каждым ходом игрок может взять с одной тарелки не больше ше­
сти крекеров (взять хотя бы один крекер обязательно). Выигрывает
тот, кто берёт последний крекер. Илья ходит первым. У кого есть
выигрышная стратегия и в чём она заключается?
Задача 5. Катя и Лёша играют в такую игру. У них есть куча
из 56 шаров. Каждым ходом игрок может взять из кучи не больше
восьми шаров (взять хотя бы один шар обязательно). Проигрывает
Занятие 28. Математические игры со стратегией II 223

тот, кто не может сделать ход. Катя ходит первой. У кого есть выиг­
рышная стратегия и в чём она заключается?
Задача 6. Максим и Олег по очереди ломают прямоугольную
плитку шоколада на куски. Размеры плитки —10 квадратиков в дли­
ну и 6 в ширину. За один ход игрок разламывает один из имеющихся
кусков шоколада на два куска меньшего размера. Разламывать мож­
но только по линиям между квадратиками. Проигрывает тот, кто не
может сделать ход. Максим ходит первым. У кого есть выигрышная
стратегия и в чём она заключается?
Задача 7. Алекс и Юстас играют в такую игру. На доске в ряд
написаны числа от 1 до 10. Игроки по очереди вставляют между
этими числами плюсы или минусы. Когда вставлены все девять зна­
ков, вычисляется результат (выполняются все действия сложения
и вычитания). Если результат окажется чётным, выигрывает пер­
вый игрок; если нечётным — второй. Алекс ходит первым. У кого
есть выигрышная стратегия и в чём она заключается?
Задача 8. Двадцать солдат стоят в шеренге в случайном поряд­
ке. Все они разного роста. По вашей команде какие-нибудь два сол­
дата, стоящие через одного, меняются местами. Вы можете дать та­
кую команду сколько угодно раз. Всегда ли можно расставить солдат
по росту?

Д О П О Л Н И Т Е Л Ь Н Ы Е ЗАДАЧИ

Задача 9. Два рабочих на складе играют в следующую игру.


Они начинают заполнять пол в комнате 5 х 25 метров ящиками
2 x 2 метра, располагая ящики на целом расстоянии от стен. Ра­
бочие по очереди ставят по одному ящику, и тот, кто не может
вместить следующий ящик, проигрывает. У кого — начинающего
или его партнёра — есть выигрышная стратегия?
Занятие 29

Математическая олимпиада III

Сегодня последнее занятие и последняя олимпиада в этом году.


Об организации и проведении олимпиады подробно написано на
с. 241. Ещё несколько полезных советов можно найти в главе, посвя­
щенной первой олимпиаде (с. 103).
В наших кружках дети всегда с нетерпением ждут очередную
олимпиаду. Им нравятся трудные задачи, азарт соревнования и,
конечно, призы.
Для преподавателей. Это соревнование завершает учебный год. Как сде­
лать его запоминающимся? Один вариант—дипломы и призы. Дру­
гой — совместное чаепитие после олимпиады.
Для преподавателей. Сегодня последняя возможность узнать впечатления
участников и их родителей от кружка. Можно раздать два небольших
опросных листа — один для детей, другой для родителей.
Продуманные вопросы позволяют получить множество полезной
информации. Какая тема была самой интересной, а какая — наименее
интересной? Какая тема была самой трудной? Что больше всего по­
нравилось в кружке? Ответы участников помогут увидеть их глазами
ваш стиль преподавания, а также скорректируют ваши представления
о трудности и занимательности конкретных тем.
Родителям можно задать более отвлечённые вопросы. Чем полезен
кружок для образования вашего ребёнка? Что дали ребёнку занятия
в кружке? Что помогло бы сделать кружок лучше? Помимо прочего,
ответы на эти вопросы помогут распространить информацию о кружке.
Например, некоторые отзывы можно поместить на сайте кружка.

Математическая разминка
В день соревнования не хватит времени на подробный разбор
домашнего задания. Но это последнее занятие в году. Поэтому вме-
Занятие 29. Математическая олимпиада III 225

сто разминки можно обсудить те домашние задачи, которые вы счи­


таете наиболее важными. Желательно уделить внимание задачам,
иллюстрирующим новый материал прошлого занятия.

Событие дня:
математическая олимпиада
Эта олимпиада снова проводится устно. Задачи приблизитель­
но упорядочены по возрастанию трудности и напечатаны на двух
листах: семь на первом и три на втором. Задачи на втором ли­
сте довольно сложные, а последняя задача по-настоящему трудная.
Она поможет избежать ситуации, когда самый способный участник
быстро всё решил и не знает, чем заняться.
В начале олимпиады участникам выдаётся первый лист задач.
Те, кто решил хотя бы четыре задачи из первого листа, получают
лист с остальными задачами.
Для преподавателей, В этой главе приведено несколько дополнительных
задач. В зависимости от продолжительности соревнования и требуемо­
го уровня сложности можно добавить их к основным задачам или заме­
нить ими часть основных задач.

Первый лист задач


Задача 1. В зоопарке Макс и Белла кормили крекерами медведя,
носорога, слона и гиппопотама. Животные съели 2010 крекеров:
носорог на один крекер больше, чем медведь, а гиппопотам — на
один больше, чем носорог, и на один меньше, чем слон. Сколько
крекеров съел слон?
Задача 2. Давным-давно принц Баддам из королевства Тадам
вознамерился искать руки прекрасной принцессы Лилли из коро­
левства Хадам. Король Хадама решил подвергнуть принца испыта­
нию. Он привёл его в зал с двумя дверями. Надпись на каждой двери
гласила: «За обеими дверями никого нет». Король сказал принцу,
что каждая дверь ведёт в помещение, где либо никого нет, либо
сидит голодный тигр. Он рассказал ещё кое-что: если за первой две­
рью никого нет, то надпись на этой двери правдива, а если там тигр,
то ложна. Если за второй дверью никого нет, то надпись на этой
двери ложна, а если там тигр, то правдива.
Король предложил Баддаму открыть любую дверь по своему вы­
бору. Если за дверью окажется тигр, то Баддам погибнет. Если же
226 Раздел 1. Планы занятий

там никого не окажется, то он получит руку Лилли. Помогите Бадда-


му пройти испытание исходя из предположения, что король не лжёт.
Задача 3. Записав подряд все числа от 10 до 20, получили число
101112... 1920. Делится ли это число на 12?
Задача 4. Разрежьте показанную на рисунке фи­
гуру на две одинаковые части. (Разрешается резать
только по линиям сетки и по диагоналям клеток.)
Задача 5. Макс и Белла наблюдают за краси­
вой золотой рыбкой, плавающей в прямоуголь­
ном аквариуме. Белла стоит перед аквариумом, а Макс — справа от
него. С точки зрения Беллы, рыбка движется по линии, показанной
на рисунке а). На рисунке б) показано движение рыбки с точки зре­
ния Макса. Папа Беллы, благодаря своему высокому росту, смотрит
на аквариум сверху. Нарисуйте путь рыбки так, как его видит папа.

Задача 6. Как-то раз Винни-Пух за­


метил, что его часы с кукушкой остано­
вились. Он завёл часы, но не смог уста­
новить правильное время, потому что
в его доме не было других часов.
В тот же вечер Винни-Пух пошёл
в гости к Кролику. У Кролика есть от­
личные настенные часы, всегда пока­
зывающие точное время. Вернувшись
домой, Винни-Пух смог установить на своих часах правильное вре­
мя. Как ему это удалось? (Часы Винни-Пуха оставались у него дома.
Часы Кролика никто не снимал со стены. Хотя дорога между домами
Винни-Пуха и Кролика ровная и Винни-Пух всегда идёт по ней с од­
ной и той же скоростью, он не знает, за сколько времени он доходит
от своего дома до дома Кролика.)
Задача 7. У Тима и Алекса есть большой мешок с монетами и две
копилки. В одну копилку помещается 300 монет, а в другую 375.
Занятие 29. Математическая олимпиада III 227

Тим и Алекс по очереди бросают монеты в копилки. За один ход


можно бросить сколько угодно монет, но только в одну копилку.
Проигрывает тот, кто не может сделать ход. Тим ходит первым.
У кого есть выигрышная стратегия?

Второй лист задач


Задача 8. Компьютер сгенерировал 100-значное число, которое
делится на 9. Илья сложил все цифры распечатанного числа и запи­
сал результат на доске. Макс сложил все цифры числа, написанного
на доске, и записал результат в своей тетради. Наконец, Белла сло­
жила все цифры числа из тетради Макса и записала сумму в своём
тайном дневнике. Какое число записано в дневнике у Беллы?
Задача 9. На каждой половинке костяшки домино по шесть то­
чек. Сколькими способами можно положить эту костяшку на шах­
матную доску 8 x 8 ? (Размеры костяшки: 1 х 2. Её можно класть
по горизонтали или по вертикали, при этом она должна полностью
закрывать два поля.)
Задача 10*. На склонах Радужной Горы живёт 10 драконов. Они
бывают разных цветов: красные, синие и т.д. Известно, что из любых
четырёх драконов как минимум два будут одинакового цвета. Докажи­
те, что по крайней мере 4 из всех драконов — одинакового цвета.

ДОПОЛНИТЕЛЬНЫЕ ЗАДАЧИ

Задача 11. В нескольких 17-литровых и 10-литровых кувшинах


хранится 223 литра масла. Все кувшины заполнены доверху. Сколь­
ко кувшинов каждого вида?
(По уровню трудности соответствует задачам 4—7.)
Задача 12. Докажите, что произведение любых пяти идущих
подряд натуральных чисел делится на 120.
(По уровню трудности соответствует задачам 4—7.)
Задача 13. Более сложный вариант задачи про аквариум. Рисун­
ки к задаче 5 заменяются следующими:
228 Раздел 1. Планы занятий

Задача 14. На одной половинке костяшки домино три точки,


а на другой — шесть точек. Сколькими способами можно положить
эту костяшку на шахматную доску 8 x 8 ? (Размеры костяшки: 1 x 2 .
Её можно класть по горизонтали или по вертикали, при этом она
должна полностью закрывать два поля.)
(Немного труднее задачи 9.)
Задача 15. Двадцать точек расположены по кругу. Тим и Алекс
по очереди соединяют пары точек отрезками. Новый отрезок не мо­
жет пересекать уже нарисованные отрезки, но может иметь с ни­
ми общие концы. Выигрывает тот, кто нарисует последний отрезок.
У кого есть выигрышная стратегия и в чём она заключается?
(По уровню трудности соответствует задачам 8 и 9.)
Задача 16. Антон записал двузначное число. Саша приписал
к этому числу спереди цифру 6. Получившееся трёхзначное число
оказалось в девять раз больше числа, записанного Антоном. Какое
число записал Антон?
(По уровню трудности соответствует задачам 8 и 9.)
Задача 17*. В клетках таблицы 5 x 5 сидят 25 божьих коровок
и тараканов — по одному насекомому в каждой клетке. Докажите,
что в таблице можно выбрать два столбца и две строки так, чтобы
в четырёх клетках на пересечениях этих столбцов и строк сидели
либо четыре божьи коровки, либо четыре таракана.
(Трудная задача. Соответствует задаче 10.)
Задача 18*. Света заполнила таблицу 4 x 4 числами +1 и — 1,
записав по одному числу в каждой клетке. Подсчитав произведение
чисел в каждой строке, она получила четыре одинаковых результа­
та: +1. Тогда она подсчитала произведение чисел в каждом столбце
и снова каждый раз получила +1. Обрадованная этим открытием,
Света задалась вопросом: сколькими способами можно расставить
числа +1 и — 1 так, чтобы таблица обладала этим свойством? Помо­
гите Свете найти ответ.
(Трудная задача. Соответствует задаче 10.)
РАЗДЕЛ 2

Математические турниры
Математические соревнования
Математические конкурсы, соревнования и олимпиады поль­
зуются огромным успехом как у детей, так и у преподавателей.
В наших кружках школьники с удовольствием и азартом сражаются
в нескольких излюбленных видах командных соревнований. Мы
регулярно проводим математические аукционы, математические
хоккей и математическое домино. Все эти соревнования зародились
в российских математических кружках, но, как оказалось, школьни­
ки из других стран играют в них с таким же удовольствием.
В этой книге изложены правила двух соревнований: аукциона
и хоккея. Несколько глав в первой части книги посвящены прове­
дению математических аукционов и хоккейных матчей.
Соревнование «Математическое домино» —это недавнее очень
удачное дополнение к нашей коллекции игр, которое мы, к сожа­
лению, не успели добавить в эту книгу1.
Но русскоязычные читатели, в отличие от зарубежных, могут
прочитать правила игры в Интернете и даже могут загрузить себе
варианты заданий домино.

1
Наши правила математического домино взяты с веб-сайта Кировской летней
математической школы.
Математический аукцион
Аукцион — одно из самых увлекательных математических сорев­
нований. В него с удовольствием играют кружковцы всех возрастов.
Задачи математического аукциона, в отличие от традиционных
задач, допускают несколько верных ответов, причём одни ответы
ценятся выше других. Некоторые решения совсем просты: их может
найти почти каждый участник. Чтобы найти более сложные ответы,
надо как следует потрудиться. По мере того как команды улучшают
результаты и находят более сложные решения, задачи снова и снова
выставляются на аукцион, и игровые деньги переходят из рук в ру­
ки, добавляя азарта к игре.
Задачи аукциона конструктивны: ответы к ним легко проверя­
ются и не требуют доказательств. Поэтому аукцион можно прове­
сти даже в только что открывшемся кружке, участники которого не
имеют опыта математических рассуждений и слабо знакомы с по­
нятием доказательства.
Ещё одна замечательная особенность этой игры состоит в том,
что формат проведения аукциона стимулирует детей думать над за­
дачей даже тогда, когда другая команда уже рассказывает её реше­
ние. Поскольку задачи аукциона имеют несколько неравноценных
решений, объединение собственных идей с идеями соперников ча­
сто помогает улучшить результат.
Правила математического аукциона
Математический аукцион — это соревнование двух или несколь­
ких команд. В начале игры каждая команда получает один и тот же
список задач. На их решение отводится определённое время. Затем
команды собираются вместе и начинаются аукционные торги.
В начале торгов каждая команда располагает одинаковой суммой
в придуманной валюте. В нашем кружке имеют хождение шмолла-
ры. (Можно также использовать тугрики, пиастры и другие экзо­
тические денежные единицы.) Каждой задаче из списка назначена
стоимость в шмолларах. Одну за другой, преподаватель выставляет
задачи на аукцион. Команды делают ставки, соревнуясь за право
рассказать решение этой задачи.
Команда, сделавшая наибольшую ставку, публично представляет
решение задачи. Если решение оказалось верным, у этой команды
есть шанс получить сумму, равную стоимости задачи. В таком слу-
Математический аукцион 233

чае прибыль команды —это стоимость задачи за вычетом сделан­


ной ставки.
Однако события могут развиваться по-другому. В отличие от обыч­
ного аукциона, команда может потерять приобретённое, если другая
команда предложит лучшее решение. (Напомним, что задачи мате­
матического аукциона имеют несколько неравноценных решений.)
Итак, если команда А предложила правильное решение, но команда В
считает, что её решение лучше, задача снова выставляется на торги.
Если команда В делает в этом втором раунде наибольшую ставку,
она получает право представить своё решение. Если это решение
оказалось верным и оно действительно лучше решения команды А,
то право на награду переходит от команды А к команде В.
Одна и та же задача может выставляться на торги сколько угодно
раз: этот процесс прекращается только тогда, когда ни одна команда
не считает, что может предложить улучшенное решение. (Возможна
также ситуация, когда команда улучшает своё собственное решение!)
Стоимость задачи в конечном итоге достаётся той команде, которая
нашла наилучший ответ. Чистая прибыль этой команды меньше сто­
имости задачи на сумму всех сделанных ей ставок. Другие команды
теряют ставки, давшие им возможность рассказать более слабые ре­
шения. (Пример игры см. на с. 235 в конце этой главы.)
Торговля и перепродажа придают игре стратегический характер
и делают её ещё более увлекательной. Благодаря возможности переку­
пить задачу дети активно решают задачи на всём протяжении сорев­
нования. Участников вдохновляют идеи других команд и желание вы­
играть. На математических аукционах решения часто придумываются
«на лету». Бывает, что за считанные минуты команда покупает одну
и ту же задачу несколько раз подряд, каждый раз улучшая решение.
Когда все задачи окончательно проданы, преподаватель подсчи­
тывает деньги у каждой команды. Команда с наибольшей суммой
выигрывает.

Как сравнивать решения


Как определить, какое решение лучше? На этот важный вопрос нет
универсального ответа: на математическом аукционе к каждой зада­
че применяются свои критерии. В каждом условии должен быть чётко
сформулирован принцип, по которому будут сравниваться решения.
В конце этой главы приведено несколько задач для аукционов
с критериями сравнения решений.
234 Раздел 2. Математические турниры

Как продавать и перепродавать задачу


Преподаватель объявляет выставленную задачу и начинает при­
нимать ставки. Он громко повторяет каждую новую ставку. «Кто
готов заплатить больше?» (Рука преподавателя указывает на ту ко­
манду, которая заявила эту ставку.) Если после очень неторопливо­
го счёта до трёх не следует нового предложения, задача считается
проданной. Чтобы торг шёл быстрее, в нашем кружке не принима­
ются ставки ниже 5 шмолларов, и перебить чужую ставку меньше
чем на 5 шмолларов тоже нельзя. А для того чтобы на аукционе был
порядок, ставки могут объявляться только заранее выбранными ка­
питанами команд. Как и на настоящем аукционе, объявляя ставку,
капитан должен поднимать руку.
После того как решение рассказано и проверено, преподаватель
спрашивает, нет ли у какой-нибудь команды лучшего решения. Если
такое решение есть, начинается следующий раунд торгов по этой же
задаче. Минимальный размер ставки остаётся прежним. Команда,
сделавшая наибольшую ставку, покупает попытку рассказать своё
решение. Та команда, которая только что представила решение, мо­
жет участвовать в торгах наравне с другими.
Торги продолжаются, пока находятся команды, желающие про­
демонстрировать улучшенные решения.

Денежные вопросы
Все команды начинают игру с одинаковой суммой денег. Жела­
тельно, чтобы эта сумма была примерно равна половине общей сто­
имости всех задач. (Если у команд слишком много денег, то появля­
ется соблазн тратить их не задумываясь. Если денег слишком мало,
то они могут закончиться, и тогда команда больше не сможет участ­
вовать в игре.)
Естественным ограничением ставки служит стоимость задачи:
делать ставки большего размера в любом случае невыгодно.
В наших кружках мы назначаем задачам стоимость порядка ста
шмолларов, а минимальная ставка и минимальная надбавка состав­
ляют пять шмолларов.
Правила игры в команде
Математический аукцион можно проводить как командную игру
или как индивидуальное соревнование. В наших кружках обычно
соревнуются две команды.
Математический аукцион 235

Вот несколько важных правил командной игры.


• В каждой команде должен быть капитан. Обязанность капита­
на—делать ставки во время соревнования. (Стратегию игры, в том
числе размер ставок, обсуждает вся команда. Капитан объявляет
ставку и поручает одному из членов команды рассказать решение.)
• Ни один участник не может рассказывать решение второй раз,
пока все члены его команды не выступали хотя бы по одному разу.
(Но улучшенные решения одной и той же задачи может рассказы­
вать тот же самый кружковец.) Это правило можно отменить, если
команды очень маленькие.
• Количество команд зависит от численности кружка. Оптималь­
ный размер команды — от четырёх до восьми человек. В больших
кружках можно проводить аукцион с тремя и даже с четырьмя ко­
мандами, если делать их не очень большими.

Пример игры
Разберём подробно вымышленный пример математического аук­
циона, на котором соревнуются две команды: А и В. На аукционе
предлагается всего одна (очень упрощённая) задача.
Задача 1. Используя цифру 1 пять раз, получите как можно боль­
ше идущих подряд натуральных чисел начиная с 1. Можно пользо­
ваться действиями арифметики +, —, •, : и скобками. Одно и то же
действие может встречаться несколько раз. Использовать все четы­
ре действия не обязательно.
(Стоимость задачи: 100 шмолларов.) Решение считается лучше
предыдущего, если команда может продолжить список с того числа,
на котором остановилась выступавшая перед этим команда.
Допустим, что за время, отведённое для решения задачи, коман­
да А придумала, как представить числа 1, 2 и 4:
1 = 1 - 1 + 1 - 1 + 1, 2 = 1:1 + 1 : 1 1 , 4 = (1 + 1) 1 + 1 + 1.
Команда В за это же время сумела найти представления для чи­
сел 1, 2 и 3:
1 = (11 - 11) + 1 , 2 = (1 • 1 + 1 : 1) • 1, 3 = 1 + 1 + 1 + 1-1.
Первый раунд торгов выигрывает команда В. Сделав наиболь­
шую ставку в 45 шмолларов, она показывает свои решения — пред­
ставление чисел 1, 2 и 3. Но согласно критериям задачи у команды А
236 Раздел 2. Математические турниры

есть улучшенное решение: представление числа 4. Поэтому коман­


да А требует второго раунда торгов. Во втором раунде команда А
за 25 шмолларов приобретает право продемонстрировать решение.
(Цена оказалась такой высокой, потому что команда В блефовала.
Она тоже повышала ставки, делая вид, что ей есть что показать.)
Команда А показывает представление числа 4. Теперь команда А
уверена, что деньги достанутся ей. (В этом случае её прибыль соста­
вила бы 100 — 25 = 75 шмолларов.) Но команда В, увидев решение
команды А, тут же придумывает, как представить число 5:
5 = 1 + 1 + 1 + 1 + 1.
Команда В требует третьего раунда. Она платит 5 шмолларов и де­
монстрирует представление числа 5. Поскольку ни одной команде не
удалось представить число 6, торги по этой задаче завершаются.
Команда В получает номинальную стоимость задачи: 100 шмол­
ларов. Её прибыль составляет 100 — 45 - 5 = 50 шмолларов. Коман­
да А теряет 25 шмолларов.

Советы преподавателю
1. Многие задачи, например с шахматными досками или разре­
занием фигур, удобнее решать на бумаге в клетку: принесите её
с собой в достаточном количестве. Для некоторых задач могут пона­
добиться спички и другие предметы. Заранее продумайте, что при­
нести на занятие, чтобы аукцион прошёл без заминок.
2. Если на доске недостаточно места, подготовьте большой лист
бумаги и клейкую ленту. Закрепите лист на стене и отмечайте на
нём текущие результаты. Можно также пользоваться флипчартом.
3. Прочитайте все задачи вслух, убедившись, что участники вни­
мательно слушают. Подчеркните, что на этапе решения задач мож­
но задавать вопросы. Если условие задачи понято неверно, решение
не засчитывается.
4. Пока команды решают задачи, подготовьте доску к выступле­
ниям: это поможет сэкономить драгоценное время. Нарисуйте всё,
что точно понадобится: шахматные доски, разлиновку в клетку, гео­
метрические фигуры.
5. На большом листе бумаги (см. пункт 2) нарисуйте таблицу. Её
столбцы —это команды, а строки —задачи. Сделайте соответству­
ющие пометки. Теперь нетрудно отслеживать «финансовое состоя­
ние» команд. Ставки обозначаются отрицательными числами, а вы-
Математический аукцион 237

игрыши — положительными. В конце аукциона достаточно сложить


значения в столбцах.
6. На соревновании вы будете судьёй, и вам неминуемо придётся
принимать решения в неоднозначных ситуациях. Будьте благожела­
тельны, но последовательны. Типичная ситуация, не регламентиро­
ванная общими правилами: участник, рассказывающий решение,
пытается исправить или улучшить его прямо по ходу выступления.
Сколько времени можно дать на такую попытку? Вы — судья и ор­
ганизатор торгов, и ваша задача — пойти навстречу этому участни­
ку в такой степени, чтобы не нарушить интересы других команд.
Поэтому можно установить правило: на обдумывание решения во
время выступления отводится не больше двух минут.

Примеры задач для математического аукциона


Задача 1. Разрежьте круг на части семью прямыми линиями
так, чтобы получилось как можно больше треугольников. «Треуголь­
ники» с кривыми сторонами не считаются. Разрезанные треуголь­
ники (состоящие из нескольких треугольников или многоугольни­
ков) тоже не считаются.
Решение считается лучше предыдущего, если команда может по­
казать рисунок, на котором больше треугольников.
Задача 2. Найдите как можно больше решений числового ре­
буса BACK + BOA = SCAM. (Одинаковые буквы обозначают одинако­
вые цифры, а разные буквы —разные цифры.)
Решение считается лучше предыдущего, если команда может
представить хотя бы один ответ, не найденный ранее.
Задача 3. Используя цифры 1, 2, 3,4, 5, 6 в этом порядке, получи­
те как можно больше идущих подряд натуральных чисел начиная с 1.
Можно пользоваться действиями арифметики +, —, •, : и скобками.
Одно и то же действие может встречаться несколько раз. Использо­
вать все четыре действия не обязательно, но окончательное выраже­
ние должно содержать все шесть цифр в порядке возрастания.
Решение считается лучше предыдущего, если команда может
продолжить список с того числа, на котором остановилась высту­
павшая перед этим команда.
Математический хоккей
Математический хоккей —это динамичное соревнование двух
команд с простыми правилами. Возможность участия предоставля­
ется каждому члену команды. В наших кружках в него с удоволь­
ствием играют все — от третьеклассников и старше

Правила математического хоккея


Цель игры такая же, как и в обычном хоккее: забить как можно
больше голов в ворота соперника. Но в математическом хоккее ка­
ток, шайба и ворота не настоящие: они нарисованы на доске. Чтобы
забить гол, надо успешно решать задачи, которые вбрасывает в игру
судья.

Игра состоит из коротких раундов, которые проводятся так. Сна­


чала каждая команда выдвигает своего представителя (защитника),
который будет защищать ворота в этом раунде. Затем преподава­
тель — он же судья — зачитывает условие задачи. Оба кружковца на­
чинают работать над задачей, не прибегая к помощи своих команд.
Тот, кто решил задачу первым, громко объявляет ответ. Если от­
вет верный, команда этого игрока выигрывает раунд. В противном
случае выигрыш присуждается другой команде, и при этом она не
должна объявлять свой ответ. (Это правило очень важно — оно ра­
ботает против привычки некоторых детей выдавать ответы быстро
и неправильно.)
Может случиться, что за отведённое время (минута или две) ни
один защитник не решит задачу. Тогда судья обращается за помо­
щью к командам: он объявляет, что команды должны написать на
бумаге правильный ответ и показать его судье. Если обе команды
имеют ответ, то обе получают по очку и шайба остаётся на месте.
Если же одна из команд ошиблась, то шайба передвигается ближе
Математический хоккей 239

к её воротам. Возможность помощи защитнику оживляет игру и по­


буждает всю команду решать задачи.
Ещё одно важное правило заключается в том, что время у доски
должно быть равномерно распределено между всеми участниками
игры. Ни один участник не может выходить к доске второй раз, по­
ка все члены его команды не выступили по одному разу, не может
выходить третий раз, пока все остальные не выступили по два раза,
и так далее.
Команда, выигравшая раунд, перемещает шайбу в следующую
зону —ближе к воротам соперника. Когда шайба попадает в зону
ворот, засчитывается гол. Шайба возвращается в центр, и игра про­
должается.
Команды сражаются до тех пор, пока у судьи не заканчивают­
ся задачи или не истекает время. Выигрывает команда, забившая
больше голов.
Советы преподавателю
Преподавателю, который готовится к математическому хоккею,
надо учитывать некоторые особенности игры.
• В отличие от других соревнований, в математическом хоккее
не отводится время на подготовку: задачи решаются прямо по ходу
игры. Поэтому математический хоккей занимает меньше времени,
чем, например, математический аукцион. В день хоккейного матча
можно успеть разобрать задачи из предыдущей подборки или на­
чать решать новые задачи.
• Продумайте заранее, как вы будете выдавать условия задач.
Чтобы игра была честной, представители обеих команд должны
получить условия одновременно. При этом прочитать задачу вслух
недостаточно. Нужен письменный текст, который смогут прочитать
не только представители, но и все члены команд: тогда они будут
вовлечены в игру.
Предъявлять задачи можно разными способами.
— Показывать в виде слайдов (при наличии проектора).
— Напечатать крупными буквами на отдельных листах. На каждом
листе должна быть одна задача, набранная очень крупным шрифтом
в альбомной ориентации. Прикрепите задачу к стене или к доске так,
чтобы все её видели. Этот способ подходит для небольших помещений.
— Распечатать несколько экземпляров. В начале каждого раун­
да судья или его помощник одновременно выдаёт по экземпляру
240 Раздел 2. Математические турниры

каждому защитнику, а затем ещё по одному или нескольким экзем­


плярам каждой команде. Это удобно в больших помещениях, где
распечатанный текст на доске будет виден не всем.
• Не менее важно заранее решить, где и как защитники будут ре­
шать задачи. Игра будет живее и интереснее, если они работают не
на бумаге, а на доске: тогда остальные члены команд могут следить
за ходом событий. Но если вашим кружковцам спокойнее работать
так, чтобы за ними не наблюдали, то вы можете сажать защитников
за две парты перед доской.
• Участников надо разбить на две команды, приблизительно рав­
ные по силе, но не обязательно равные по численности. Поддержи­
вайте высокий темп игры: отводите на каждую задачу самое боль­
шее несколько минут. Если задача оказалась слишком трудной, про­
сите помощи у команд.
• Задачи для хоккея можно подбирать разными способами. Мож­
но составить подборку из разнообразных задач, а можно связать их
с недавно изученным материалом. Это особенно полезно, если игра
проводится после изучения новой темы, усвоение которой требует
интенсивной практики. Примеры подходящих тем — комбинатори­
ка, графы (особенно изоморфные графы и графы Эйлера), алгебра­
ические формулы.
• Задачи для игры должны быть сравнительно простыми, но, как
и положено в математическом кружке, интересными и увлекатель­
ными. В этой книге приведено несколько готовых подборок таких
задач. Составить собственную подборку непросто, но усилия оправ­
дывают себя. Много примеров задач для математического хоккея
можно найти в Интернете. Можно также брать задачи с других ма­
тематических соревнований, например из олимпиады «Кенгуру».
• Оптимальный размер команды — от трёх до восьми игроков.
Математические олимпиады
Олимпиада — это индивидуальное математическое соревнова­
ние. Устраивать олимпиады на каждом занятии не стоит, но про­
водить их время от времени очень полезно по ряду причин.
• Дети сосредоточенно решают задачи все отведённое время. Не
приходится удивляться, что уровень их вовлечённости гораздо вы­
ше, чем на обычном занятии.
• Дети любят соревноваться и получать призы. Поэтому олимпи­
ады — это и событие в жизни кружа, и развлечение, и дополнитель­
ный стимул к учёбе.
• Для преподавателя олимпиада — отличная возможность узнать
о сильных и слабых сторонах каждого участника и оценить успеш­
ность кружка в целом.
Виды олимпиад
Математическая олимпиада может быть устной или письменной.
Письменные олимпиады, в свою очередь, могут проводиться в двух
формах: участники должны либо выбрать правильный ответ из не­
скольких вариантов, либо написать подробное решение. Каждый из
трёх видов олимпиад имеет свои преимущества и недостатки.
• Письменная олимпиада с несколькими вариантами ответа су­
щественно ограничивает выбор задач. Не всегда просто подобрать
задачи, которые связаны с темами кружка и в то же время позволя­
ют оценить правильность рассуждений только на основании ответа.
Зато такую олимпиаду очень просто проводить и проверять. Резуль­
таты можно объявить сразу после соревнования.
• На олимпиадах с записью решения участникам даётся инструк­
ция писать подробные решения: обоснованность рассуждений важ­
нее, чем правильный ответ. Это позволяет подобрать более интерес­
ные задачи. Однако не все дети, особенно начинающие кружковцы,
способны ясно излагать свои мысли в письменном виде. К тому же
проверка и оценка решений занимает много времени.
• Устная олимпиада предполагает активное взаимодействие меж­
ду преподавателем и участниками. Для детей это отличная трени­
ровка способностей к логическим рассуждениям. Ещё одно преиму­
щество устной олимпиады — возможность объявить результаты сра­
зу после её окончания. Трудность состоит в том, что один препо­
даватель обычно не успевает выслушать все решения и приходится
242 Раздел 2. Математические турниры

приглашать помощников в жюри. Члены жюри должны уметь оце­


нивать правильность решений и указывать участникам на ошибки
в рассуждениях.
В наших кружках мы предпочитаем устные олимпиады. Обыч­
но мы просим помочь друзей, родителей и выпускников матема­
тических кружков, обладающих необходимыми умениями. Мы про­
водим такие соревнования раз в семестр, и у нас всегда получает­
ся найти достаточное количество добровольцев. Если мы не вполне
уверены в квалификации члена жюри, мы поручаем ему проверку
более простых задач из первого листа или просим его слушать зада­
чи совместно с кем-нибудь ещё.

Олимпиадные задачи
Успех олимпиады в первую очередь зависит от правильно подо­
бранных задач. Олимпиада должна быть трудной для всех кружков­
цев, но такой, чтобы каждый мог уйти с чувством удовлетворения
сделанной работой. Вот несколько советов по достижению этой цели.
• Подберите задачи разной степени сложности. Олимпиада долж­
на быть интересна всем — от новичков до самых сильных участни­
ков. Если всего задач десять, разумное соотношение будет пример­
но таким: четыре задачи, которые может решить практически лю­
бой участник вашего кружка, четыре — рассчитанные на более под­
готовленных участников и две по-настоящему трудные.
• Включите в подборку несколько задач по недавно изученным
темам. Это служит двум целям: закреплению пройденного матери­
ала и поощрению участников, которые активно занимались в те­
чение года. Но не забудьте добавить задачи, для решения которых
нужны только логика и сообразительность.
• Задачи должны иметь относительно короткие решения. Это
важно как для устной, так и для письменной олимпиады. Вам будет
нелегко, если придётся слушать или читать долгие рассуждения.
• Условия задач должны быть интересными и понятными. Ес­
ли задача скучная или слишком длинная, переформулируйте её или
выберите другую задачу.
• Расположите задачи в порядке возрастания трудности. Это по­
может участникам ориентироваться в подборке задач.
• Мы обычно разбиваем задачи на две группы, распечатывае­
мые на отдельных листах. На первом листе задачи в целом менее
Математические олимпиады 243

трудные, чем на втором. В начале олимпиады участникам раздаёт­


ся первый лист. Решив определённое количество задач из первого
листа, они могут получить второй. Такая система помогает участни­
кам сосредоточить усилия на относительно небольшом количестве
задач. К тому же она позволяет избежать соблазна сразу перейти
к трудным задачам, пропустив более простые.

Планирование устной олимпиады


Устная олимпиада требует тщательной подготовки. Вот некото­
рые важные рекомендации.
• Пригласите помощников в достаточном количестве, чтобы справ­
ляться с потоком решений. Угадать нужное количество членов жюри
бывает непросто, но в случае сомнений лучше перестраховаться. По
моему опыту, на олимпиады из этой книги в кружке из десяти детей
достаточно пригласить одного члена жюри (помимо преподавателя),
но в начале соревнования, когда дети решают простые задачи, прихо­
дится нелегко. (Олимпиада у нас длится полтора часа.)
• Разработайте удобную систему регистрации решённых задач.
Вполне подойдёт таблица, в которой строки помечены инициалами
участников, а столбцы — сокращёнными обозначениями задач. Мож­
но нарисовать такую таблицу на бумаге или, если вы предпочитаете
технологичный подход, завести электронную таблицу на компьютере.
• Обязательно заранее обсудите с членами жюри задачи и возмож­
ные решения. Это очень важно: если член жюри не знаком с програм­
мой кружка и с вашими требованиями, то его представление о пра­
вильном решении может не совпадать с вашим. Чтобы соревнование
было честным, критерии должны быть единообразными.
• Каждому члену жюри на всё время соревнования надо выде­
лить отдельное место, чтобы слушать решения задач и обсуждать
их с участниками. Это может быть стол в классе или пара стульев
в коридоре.
Правила олимпиады
В начале олимпиады напомните детям правила и временные
рамки. Сделайте это до выдачи листов с задачами, иначе многие
дети не обратят внимания на ваши слова.
• Участники работают самостоятельно, любые разговоры запре­
щены. (Обязательно напомните об этом: ведь обычно в кружках по­
ощряется совместная работа.)
244 Раздел 2. Математические турниры

• Задачи можно решать в любом порядке.


• Если в условиях задач что-то непонятно, можно и нужно зада­
вать вопросы.
• На письменной олимпиаде не забудьте напомнить детям о важ­
ности разборчивого почерка. (Если призвать детей писать чётко, то
проверять решения будет гораздо проще.) Решения должны быть
написаны подробно, с обоснованием каждого шага.
• На устной олимпиаде надо поддерживать очередь желающих
рассказать решения. Решив одну или несколько задач, участник до­
бавляет своё имя в список на доске. Когда очередь участника по­
дойдёт, его вызовет первый освободившийся член жюри. При этом
можно рассказать решения сразу нескольких задач.
• Участник может пытаться рассказывать решение каждой зада­
чи не больше трёх раз. Член жюри должен отмечать как успешные,
так и неуспешные попытки.
Посоветуйте детям не копить решённые задачи, особенно ближе
к концу олимпиады. Решив хотя бы одну из ещё не рассказанных
задач, лучше сразу записаться в список.
• Не забывайте напоминать кружковцам, сколько времени оста­
лось до конца соревнования. (Разумно делать это за полчаса, за пят­
надцать минут и за пять минут до конца.)

Олимпиады в этой книге


Все три олимпиады в этой книге —устные. Они рассчитаны на
полтора или два часа.
Те же задачи можно предложить и на письменной олимпиаде.
В этом случае нужно уменьшить количество задач.

Награды и призы
Если олимпиада проводится устно или с выбором правильного
варианта ответа, результаты объявляются сразу после её оконча­
ния. Результаты олимпиад с записью решения становятся известны
позднее.
Желательно завершить олимпиаду раздачей наград и призов.
Призами могут служить красиво распечатанные дипломы, конфеты
и шоколадные медали (простой способ обрадовать детей), сувени­
ры (ручки, карандаши, математические головоломки) или что-то
более существенное. Выбирая призы для конкретного кружка, надо
Математические олимпиады 245

учитывать его размер, возраст детей, степень активности родителей


и многое другое.
Как распределять призы? Это тоже зависит от размера кружка
и других обстоятельств.
В нашем маленьком домашнем кружке мы придерживались тра­
диции «двух призов». Мы подготавливали призы в количестве, вдвое
превышающем число участников, и раздавали их в два круга. Каж­
дый участник выбирал один приз в первом круге и один во втором.
Различие состоит в том, что в первом круге дети выбирали при­
зы в порядке уменьшения полученных баллов: первыми выбирали
победители. Во втором круге порядок изменялся на противополож­
ный. Таким образом, участник с наибольшим числом баллов выби­
рал один приз первым, а другой — последним. Участник с наимень­
шим числом баллов выбирал два приза в середине.
В больших кружках мы обычно подготавливаем один приз на
участника, и дети выбирают призы по очереди, начиная с победи­
теля. Призы для проигравших могут быть совсем маленькими, но
мы всегда стараемся наградить каждого.
Кто покупает призы? Это опять-таки зависит от размера кружка
и других факторов. Призы может приносить преподаватель или кто-
то из родителей. Если кружок небольшой, а у преподавателя хоро­
ший контакт с родителями, можно попросить каждого из родителей
подготовить по призу. Мы не перестаём удивляться разнообразию
призов, которые приносят родители. Вкусы и предпочтения самих
участников тоже совершенно разные: то, что совсем неинтересно
одному, может быть заветной мечтой другого.
Короткие увлекательные игры
Эта глава посвящена коротким математическим играм, в кото­
рые можно играть на занятиях. Такие игры развивают логическое
мышление и помогают занять детей, если в конце осталось немного
свободного времени.

«Джотто» и «Математическое джотто»1


«Джотто» — это простая логическая игра, сочетающая в себе
элементы математики и лингвистики.
Ведущий игры — преподаватель — загадывает слово. Это должно
быть существительное, в котором все буквы разные. Нельзя загады­
вать имена собственные. Цель игры — определить загаданное слово.
Ведущий сообщает игрокам длину задуманного слова. Игроки
начинают высказывать свои предположения: они называют слова
той же длины, а ведущий сообщает количество угаданных букв. Ме­
сто буквы в слове не имеет значения: например, если загадано сло­
во КЛЮВ, а игрок предлагает слово ВОЛК —значит, он угадал три
буквы.
По ходу игры преподаватель записывает на доске каждое назван­
ное слово, а рядом с ним — количество угаданных букв. Выигрывает
игрок (или команда), назвавший(-ая) задуманное слово.
В зависимости от длины задуманного слова игра будет более или
менее сложной и займёт больше или меньше времени. Например,
играя со школьниками третьего класса, загадывайте трёхбуквенные
слова. Для детей постарше можно загадать слово из четырёх букв.
Вариант для более старших детей и для взрослых — пятибуквенное
слово.
В математическом кружке «Джотто» можно проводить как инди­
видуальную или командную игру. В командном варианте участники
разбиваются на две команды (или на несколько команд). Команды
называют слова по очереди. Выигрывает команда, определившая за­
гаданное слово. В индивидуальном варианте каждый играет за себя.
1
Про игру «Джотто» мне рассказал на конференции по математическим круж­
кам Марк Саул — замечательный учитель и переводчик из Нью-Йорка. Марк перевел
на английский язык книгу «Ленинградские математические кружки» С. А. Генкина,
И. В. Итенберга и Д. В. Фомина. Этот перевод впервые подробно познакомил англо­
язычный мир с идеями математического кружковского движения.
Короткие увлекательные игры 247

«Математическое джотто» — такая же игра, но с числами вме­


сто слов. (При этом все цифры в числе должны быть разными.) Как
правило, обычное «Джотто» вызывает у детей больше интереса, чем
математическое.

«Ним»
«Ним» — это игра для двух игроков с очень простыми правилами.
Игровое поле представляет собой три кучи фишек. Игроки по
очереди берут фишки: за один ход можно взять любое количество
фишек из одной кучи (взять хотя бы одну фишку обязательно). Вы­
игрывает тот, кто берёт последнюю фишку. Несмотря на предель­
ную простоту правил, эта игра очень увлекательна.
Играть можно и на бумаге. В этом случае фишками служат круж­
ки или другие знаки, нарисованные в три ряда. Игроки по очереди
зачёркивают по несколько кружков в одном ряду.
Количество фишек в каждой куче выбирается случайным обра­
зом, но лучше, чтобы их было не больше десяти. Например, если
в одной куче 3 фишки, в другой 5, а в третьей 7, то игра уже будет
достаточно сложной и после нескольких партий у игроков не воз­
никнет ни малейшего представления о выигрышной стратегии.
«Ним» может быть как индивидуальной, так и командной игрой.
Например, можно устроить чемпионат кружка, а можно разбить
участников на две команды. Команды выдвигают делегатов для иг­
ры друг с другом, как в математическом хоккее.
В игре «Ним» у одного из игроков есть выигрышная стратегия.
Когда дети подрастут, будет очень полезно посвятить изучению этой
стратегии отдельное занятие. Но пока это просто занимательная игра.

«Чёрный ящик»
Играя в «Чёрный ящик», участники пытаются понять принцип дей­
ствия загадочной «математической машины» — чёрного ящика. Веду­
щий (преподаватель) задумывает правило, по которому будет рабо­
тать машина. Затем участники загружают входные данные, а чёр­
ный ящик (т. е. ведущий) обрабатывает их по задуманному правилу
и объявляет результаты.
Цель игры — определить правило чёрного ящика.
Игроки по очереди предлагают значения для ввода в чёрный
ящик. После ввода каждого значения ведущий сообщает результат.
248 Раздел 2. Математические турниры

Если кто-то из игроков считает, что угадал задуманное правило, он


запрашивает проверку. Проверка проводится так: ведущий пред­
лагает несколько входных значений, а игрок каждый раз называет
результат согласно предполагаемому правилу. Если гипотеза оказа­
лось верной, то ведущий объявляет игроку, что тот верно определил
правило, но вслух это правило не объявляется. Игра продолжается
до тех пор, пока несколько кружковцев не угадают правило. Когда
достаточно детей уже знают правило, преподаватель приглашает
одного из угадавших объявить его.
Входными данными для чёрного ящика могут служить числа или
слова. Правила обычно формулируются так, чтобы в результате по­
лучались числа. Ведущий сообщает участникам, какие данные мож­
но вводить в чёрный ящик: любые числа, только целые числа, слова
и т. п. Если игрок предлагает число, к которому невозможно при­
менить задуманное правило, то ведущий произносит заранее ого­
ворённую фразу. (Например, если х = О, то невозможно применить
правило - . Перед началом игры ведущий предупреждает, что в по­
добных случаях будет говорить: «Не понял».)
По ходу игры преподаватель записывает на доске входные и вы­
ходные данные.
В эту игру можно играть с детьми любого возраста, начиная
с первоклассников. Играют индивидуально или небольшими ко­
мандами. Чем старше дети, тем сложнее и необычнее должно быть
правило чёрного ящика.
Вот несколько возможных правил.
• Любое арифметическое действие: умножить на 2, прибавить 4,
возвести в квадрат.
• Арифметическое правило, по-разному действующее на различ­
ные группы чисел. Например, отнять 3 от чётного и делящегося
на 4, поделить на 2 чётное и не делящееся на 4, прибавить 5 к нечёт­
ному.
• Остаток от деления на некоторое целое число.
• Округление до ближайшего числа из набора. Например, округ­
ление к ближайшему числу, делящемуся на 6.
• Сумма цифр числа.
• Количество букв в названии числа.
• Количество общего числа кружочков в написании цифр числа.
(Например, 56 переводится в 1, а 58 —в 2.)
РАЗДЕЛ 3

Еще несколько советов


преподавателю
Как стать первоклассным преподавателем
математического кружка
В чём секрет хорошего математического кружка? На этот вопрос
трудно дать исчерпывающий ответ, но можно отметить ряд важных
деталей. Три необходимые составляющие успешного кружка — увле­
чённые дети, интересная программа и первоклассный преподаватель.
Вести математический кружок — непростая, но очень увлекатель­
ная работа, которая сочетает в себе элементы ремесла, творчества
и искусства. Очень часто люди, которые ведут кружок, сами прошли
через кружковскую систему и имели перед глазами пример своих
замечательных учителей. А если это ваш первый кружок? Как его
проводить, как выбирать задачи, как общаться с детьми?
Собранные в этой главе идеи и размышления помогут читателю
в этом увлекательном деле (Так как задолго до этой книги над теми
же вопросами размышляли многие талантливые педагоги, я буду при­
водить их высказывания наряду с моими собственными советами.)
• Прежде всего, помните о том, что на кружок приходят дети
разного уровня подготовки и разных способностей. Их объединя­
ет одно: все они хотят учиться. Однако их знания и возможности
могут существенно различаться. Вести кружок так, чтобы каждый
выходил из класса с ощущением сделанных открытий и одержанных
побед, — настоящее искусство. Как достичь этой цели?
• Одно из важнейших условий успешного обучения в кружке —
преподавание в форме свободного диалога. Прислушайтесь к сове­
ту Тома Дэвиса — координатора нескольких математических кружков
в районе Сан-Франциско: «Старайтесь не читать лекции. Хотя пере­
дача знаний и методов от учителя к ученику - неотъемлемая часть
процесса обучения, на занятиях должно быть как можно больше вза­
имодействия. Давайте сами себе баллы по такой системе: 1 балл — за
ту минуту, когда говорите вы, 5 баллов — за ту минуту, когда рассуж­
дают дети, 10 баллов —за минуту, когда кружковец спорит с вами,
и 50 баллов — за ту минуту, когда кружковцы спорят между собой».
• Приведу ещё одну цитату. По словам философа и педагога Ри-
ка Гарликова, вопросы учителя «должны возбуждать любопытство
и вместе с тем служить чётким пошаговым руководством, помога­
ющим детям разобраться в сложной теме на основе собственных
мыслей и догадок».
252 Раздел 3. Еще несколько советов преподавателю
• Помните, что вопросы и задачи, предлагаемые на математи­
ческом кружке, не всегда рассчитаны на быстрый ответ. «Если дети
не могут сразу ответить на вопрос, дайте им время подумать. Если
им нечего не приходит в голову, подскажите, но не давайте готового
ответа» (Том Дэвис).
• Не забывайте о конечной цели: обучить детей правильным ма­
тематическим рассуждениям. Том Дэвис: «Поощряйте детей давать
ответы, даже если эти ответы неверны. Находите что-то позитивное
в каждой попытке, но не удовлетворяйтесь нестрогим решением.
После разбора каждой задачи подведите итог, ещё раз обозначив
важнейшие этапы и идеи решения».
• Обязательно закрепляйте новые понятия и идеи решением за­
дач. Работа над задачами — это важнейшая составная часть процес­
са обучения. Поэтому на каждом занятии должна быть предложена
полезная и интересная подборка задач. Эту подборку надо хорошо
продумать с учётом нескольких критериев.
Трудность. Хотя участники кружка различаются по способно­
стям и уровню подготовки, надо сделать так, чтобы все они горди­
лись своими достижениями. Как этого добиться? Подберите инте­
ресные и творческие, но не слишком сложные задачи; таких задач
должна быть примерно половина. Одновременно включите в под­
борку пару задач, которые заставят серьёзно задуматься даже самых
сильных участников.
Выбор задач. Чтобы подборка не была скучной, задачи должны
быть разнообразными. Включите в подборку несколько задач по но­
вой теме и по материалам предыдущего занятия. Не забудьте до­
бавить задачи, которые решаются без каких-либо специальных зна­
ний — только с помощью сообразительности и здравого смысла.
Размер. Если дать слишком большое задание, дети могут утра­
тить к нему интерес. Поэтому не увлекайтесь количеством.
• Не забывайте, что у вас было гораздо больше практики в аб­
страктном мышлении и логических рассуждениях, чем у ваших уче­
ников. Некоторые понятия и принципы вам могут казаться очевид­
ными и само собой разумеющимися, а детям нужны работа и время,
чтобы их освоить.
— Перед тем как дать задачу, подсчитайте, сколько новых и труд­
ных идей потребуется для её решения. Если больше одной — это уже
много. Отложите задачу до следующего раза или дайте перед ней
пару более простых задач, иллюстрирующих отдельные шаги.
Как стать первоклассным преподавателем 253

— Точно так же проверяйте на сложность новый теоретический


материал или метод решения задач. Разбейте его на шаги, простые
для понимания; не пропускайте шагов только потому, что они оче­
видны для вас и для самых сильных кружковцев. И, как всегда, ва­
жен баланс. Основная часть материала должна быть по силам боль­
шинству участников. Но не забудьте сделать так, чтобы сильные де­
ти тоже не скучали: потратьте последние пять минут на более слож­
ные вещи, дайте задачу, которая по зубам только самым сильным
ученикам.
• Задачи— эффективное средство обучения. Если задача иллю­
стрирует важный теоретический принцип или метод решения, вклю­
чите её в несколько подборок, каждый раз изменяя её сюжет. Так дети
научатся распознавать базовые идеи, под каким бы обличьем они не
скрывались.
• Старайтесь давать детям возможность рассказывать решения
вслух —и перед всеми, и один на один с преподавателем.
• Не работайте над одной темой много занятий подряд. Лучше
разнообразить материал, а к поднадоевшей важной теме вернуться
позднее.
• Чаще устраивайте развлекательные мероприятия. Дети оста­
ются детьми — они любят играть. Если посвящать одно занятие в ме­
сяц математическим конкурсам и играм, мотивация участников будет
существенно выше.
• Всегда имейте план, как занять детей, если в конце занятия
останется десять свободных минут. Для того чтобы весело и с поль­
зой провести это время, отлично подходят короткие игры, имеющие
отношение к математике: «Джотто», «Ним», «Чёрный ящик», голо­
воломки «Кен-кен».
• Давайте персонажам задач имена участников кружка: детям
это нравится.
• Не забывайте о небольших наградах и призах. Когда вы ра­
ботаете с детьми, обыкновенная конфетка может сыграть немалую
роль.
Будни математического кружка
В этой главе я постараюсь ответить на некоторые вопросы, с ко­
торыми преподаватель математического кружка сталкивается в по­
вседневной работе. Как организовать и спланировать урок? Сколь­
ко времени должно продолжаться занятие? Сначала рассказываем
решения задач или обсуждаем новую тему?
Я начну с того, что расскажу, как проходят занятия в нашем
кружке. Затем поделюсь мыслями о том, как наши разработки мож­
но применить в вашем кружке с учётом его особенностей. Помните,
что преподавание —творческая работа. Хотя приведённые здесь ре­
комендации проверены на практике, не следуйте им безоговороч­
но. Выберите те из них, которые подойдут для вашего кружка.

Обычное занятие в нашем кружке


Традиционно большая часть наших занятий проходит по одному
и тому же плану.
• Обычно мы начинаем занятие с математической разминки —
пары несложных забавных задач, над которыми кружковцы с увле­
чением думают первые несколько минут занятия.
• После разминки начинается основная часть занятия. Это мо­
жет быть обсуждение нового теоретического материала или спосо­
ба решения задач, соревнование, олимпиада, математическая игра.
• Следующая часть посвящена решению задач. Мы раздаём но­
вую подборку задач, и участники начинают работать над ними. Од­
новременно кружковцы рассказывают решения задач, заданных на
предыдущем занятии.
• В заключение мы разбираем решения задач, заданных в про­
шлый раз.
• Нерешённые задачи из этого занятия становятся домашним
заданием. Таким образом, детям всегда есть над чем поработать до­
ма, а их родители тоже могут попробовать свои силы в решении
нестандартных задач.
Такой порядок соблюдается не всегда: он может изменяться в за­
висимости от продолжительности занятия и от характера основной
части. Например, если основная часть длится дольше обычного, то
можно уменьшить время, которое дети проводят за решением задач.
Давайте обсудим каждую часть подробнее.
Будни математического кружка 255

Разминка
Разминка занимает пять-десять минут минут в начале занятия.
Для разминки подходят несложные занимательные задачи. Это
могут быть головоломки, загадки или задачи-шутки, математиче­
ские фокусы или упрощённые варианты задач, которые разбира­
лись на прошлом занятии или будут даны на этом.
Задачи зачитываются вслух по одной. Обычно каждая задача по­
рождает множество идей и разнообразных ответов. Дайте возмож­
ность нескольким участникам рассказать их идеи. Среди них будут
и забавные, и совершенно неправдоподобные, и правильные. В лю­
бом случае разминка помогает детям включиться в работу: каждый
может выступить со своей идеей и принять участие в обсуждении.
Кроме того, разминочные задачи — отличный способ занять детей,
пока подходят опоздавшие.
Основная часть
Основная часть занятия — это, например, обсуждение нового тео­
ретического материала или способа решения задач, математическая
игра или олимпиада. Мы стараемся, чтобы занятия в нашем круж­
ке проходили интересно и разнообразно. Поэтому мы перемежаем
более серьёзные занятия (теория, решение задач) более весёлыми
(конкурсы, соревнования, игры).
Не стоит много раз подряд заниматься одной и той же темой. На­
пример, после пары трудных занятий по делимости сделайте пере­
рыв — проведите какой-нибудь конкурс. Нет ничего плохого и в том,
чтобы иногда отдохнуть — провести весёлое и нетрудное занятие:
устроить, например, чемпионат по решению спичечных головоло­
мок, или же поклеить ленты Мёбиуса, или поучиться решать голо­
воломки «Кен-кен».
Решение задач
Самостоятельное решение задач — одна из важнейших составля­
ющих обучения в математическом кружке.
Как правило, на каждом занятии нашим кружковцам выдаётся но­
вая подборка задач и выделяется время для работы над ними. Дети
могут решать эти задачи самостоятельно или небольшими группами.
Обычно в это время они могут подходить к преподавателю, чтобы об­
суждать задачи и рассказывать решения. Объяснять задачу вслух —
это лучший способ научиться ясно и чётко излагать свои мысли.
256 Раздел 3. Еще несколько советов преподавателю

Как организовать процесс рассказа решений? Мы разработали


стандартную процедуру, которая вполне оправдывает себя в нашем
кружке и, возможно, пригодится вам. (Подробное описание этой
процедуры приведено в конце главы.)

Разбор решений
Разбор решений очень важен по нескольким причинам. Дети не
только узнают, как решить задачи, которые у них не получились, но
и слушают строгие математические рассуждения.
Чаще всего мы разбираем задачи, выданные на предыдущем
занятии. Дети недавно работали над этими задачами и хорошо пом­
нят их. Каждое решение может быть рассказано преподавателем
или участником кружка. Дети любят рассказывать решения сами,
но их выступления могут занять много времени. (Поэтому если
время ограничено, то все задачи может рассказать преподаватель.)
В решениях детей нередко встречаются ошибки и недоработки.
Предложите другим участникам задать вопросы и заполнить про­
пуски в рассуждениях. В заключение разбора каждой задачи по­
вторите важнейшие этапы решения своими словами и подчеркните
главные идеи.
Развлекательное занятие в нашем кружке
Мы стараемся проводить развлекательные занятия примерно
раз в месяц. Такие занятия помогают кружковцам обучаться и об­
щаться. Дети не только с увлечением решают задачи, но и налажи­
вают отношения и заводят новых друзей.
В зависимости от того, как мы развлекаемся, план занятия мо­
жет существенно отличаться от обычного. Например, чтобы сыграть
в математический аукцион или провести математическую олимпиаду,
хорошо бы иметь полтора часа времени. А для математического хок­
кея или конкурса головоломок вполне хватит сорока минут. Откуда
взять время? Мы сокращаем другие части занятия. Можно пропу­
стить решение задач (новая подборка задач даётся в качестве до­
машнего задания или переносится на следующий раз). Можно так­
же сократить (или перенести) разбор решений.
В наших кружках мы проводим много разных развлекательных
игр и соревнований, командных и индивидуальных.
Наши наиболее популярные командные игры — математический
аукцион и математический хоккей — подробно описаны в соответ-
Будни математического кружка 257

ствующих главах (с. 232,238). Ещё мы очень любим математическое


домино. Эта игра, к сожалению, не успела попасть в это издание
книги, но на с. 231 рассказано о том, где можно найти правила.
Время от времени мы стараемся придумывать и пробовать но­
вые типы развлекательных занятий. Мы проводили криптографиче­
ские бои (это очень весело), устраивали чемпионаты головоломок,
складывали математическое оригами. В прошлом году мы в первый
раз устроили математические поисковые игры. Какие-то из наших
новых занятий оказываются более удачными, какие-то менее. Но
если не пробовать, то никогда не придумаешь ничего нового.
Не следует забывать и про индивидуальные математические олим­
пиады. На устной или письменной олимпиаде дети будут увлечён­
но решать задачи, а преподаватель может больше узнать о каждом
участнике кружка. Подробное описание и рекомендуемые правила
олимпиад приведены в главе «Математические олимпиады» (с. 241).
Очень полезно иметь в запасе несколько коротких увлекатель­
ных игр, которыми можно занять десять минут свободного вре­
мени. В главе «Короткие увлекательные игры» (с. 246) приведено
несколько игр такого типа.

Как спланировать занятие в вашем кружке


План и составные части обычного занятия, рассмотренные в пре­
дыдущем разделе, отлично зарекомендовали себя в нашем кружке.
Поскольку двух одинаковых кружков не бывает, вам предстоит
решить, как проводить занятия в вашем кружке и как пользоваться
этой книгой.
Наши материалы рассчитаны на занятия продолжительностью
полтора-два часа. Для таких занятий эта книга может служить гото­
вым учебным планом. Если занятие продолжается всего час, то по­
рядок работы, скорее всего, придётся изменить. Пытаться уместить
полуторачасовую программу в один час точно не стоит: дети обу­
чаются в естественном темпе, который нельзя ускорить. При этом
и решение задач, и разбор решений — слишком важные составляю­
щие, чтобы убрать их из программы. Простой способ обойти эту
трудность — проходить каждую главу этой книги за два занятия.
При этом возможны разные варианты.
• Можно чередовать занятия: на одном изучать новый материал,
а на другом решать задачи. В каждой главе этой книги достаточ-
258 Раздел 3. Еще несколько советов преподавателю
но материала для двух таких уроков. (Дети получат необходимую
порцию умственной работы на каждом занятии, поскольку теория
всегда подаётся через решение задач.)
• Другой вариант — прорабатывать одну тему и одну подборку
задач за два занятия. В этом случае придётся разбить подборку на
две половины и, возможно, увеличить каждую половину, добавив
дополнительные задачи. Где их взять? Можно творчески перерабо­
тать задачи из других глав или воспользоваться литературой по ма­
тематическим кружкам.
При планировании занятий придётся также учесть многое дру­
гое. Это и размер кружка, и количество помощников, и уровень под­
готовки участников, и ваши личные предпочтения.

Варианты организации занятия


Определив содержание занятия, нужно решить ещё несколько
вопросов. Подумайте о схеме занятия. С чего начать? Стоит ли сна­
чала разобрать домашнее задание? Или лучше сразу перейти к но­
вому материалу?
В наших кружках занятие обычно начинается с новой темы, по­
сле которой выдаётся очередная подборка задач. Разбор домашних
задач проводится в конце. Однако возможны и другие варианты.
Можно начать с разбора домашнего задания, затем обсудить
основную тему и ближе к концу выдать новую подборку задач. Это
вполне допустимо, но при одном условии. Если разбор решений
проводится вначале, расскажите их у доски, а не выслушивайте
каждого участника один на один. Иначе тем, кто уже побеседовал
с преподавателем, будет нечем заняться.
• Если основная часть занимает много времени — например, про­
водится математическая олимпиада, — то план придётся изменить.
В день соревнования на решение задач и на разбор домашнего за­
дания времени не останется или останется совсем мало. (Для уско­
рения процесса преподаватель может в такой день сам рассказать
решения всех домашних задач или обсудить только самые важные.)
Важно помнить: если отложить обсуждение домашнего задания,
то в следующий раз придётся разобрать две подборки задач. Это
займёт много времени, и дети, скорее всего, заскучают. Поэтому
в день соревнования домашнее задание можно сократить.
Будни математического кружка 259

• Если занятие полностью посвящено решению задач, ближе


к концу дети могут утратить интерес. Поэтому такое занятие лучше
завершить десятиминутной математической игрой.

Журнал кружка
Журнал кружка служит нескольким целям. В нём можно хранить
подборки задач, отмечать решение задач участниками и вести всю
бумажную работу, связанную с кружком.
Эти записи можно хранить и на компьютере, но бумажный фор­
мат имеет ряд преимуществ. Бумажный журнал можно носить по
классу, не опасаясь уронить его. Если нужно посмотреть задачи
из предыдущего занятия, достаточно перелистнуть страницу. На­
конец, если вы работаете с помощником, то может быть проще
передать ему бумажный журнал, чем предоставить доступ к элек­
тронному ресурсу.
Отмечать решение задач очень удобно: это позволяет оценить
работу каждого участника и группы в целом. (Даже неполная ста­
тистика даёт ценную информацию об успехах детей.) Эти сведения
пригодятся также при разговоре с родителями. Кроме того, раз в па­
ру месяцев можно награждать тех, кто решает задачи лучше всех.

Как организовать решение задач


В этом разделе подробно рассматривается организация решения
задач в наших математических кружках. Отработанная нами проце­
дура подтвердила свою эффективность и может оказаться полезной
в других кружках.

Раздача подборки задач


Не удивляйтесь — правильно раздать задачи тоже важно. Выдав
каждому участнику экземпляр задач, обязательно прочитайте каж­
дую задачу вслух и при необходимости сделайте рисунок на доске.
Спросите, есть ли вопросы. Зачем это нужно? Дети часто понимают
условия задач неправильно, причём так, как вы не сможете даже
предположить.
Если задача трудная (или связана с новым материалом), то мож­
но подсказать основные идеи решения. Небольшие подсказки зна­
чительно ускоряют процесс обучения: они задают верное направле-
260 Раздел 3. Еще несколько советов преподавателю

ние рассуждений и избавляют от необходимости выслушивать мно­


жество ошибочных решений.

Правила решения задач


Мы разрешаем участникам работать над задачами самостоятель­
но или небольшими группами. Негромкие разговоры вполне допу­
стимы, если они относятся к задачам.

Диалог ученика с преподавателем


Обсуждение решений с преподавателем крайне важно, и время,
отведённое для решения задач, лучше всего подходит для таких
бесед. Дети усердно работают над новой подборкой задач. Скоро
у большинства из них появятся решения, которые они будут готовы
рассказать. Дома они решали задачи, оставшиеся с прошлого раза.
Поэтому им всегда есть что обсудить с преподавателем.
Как организовать индивидуальные беседы? Это зависит от раз­
мера кружка. Если вам повезло и у вас есть помощник или если
кружок маленький, то на каждом занятии можно выслушать всех
участников. В большом кружке на всех не хватит времени. В этом
случае постарайтесь сделать так, чтобы за несколько занятий у каж­
дого была возможность поговорить с преподавателем.
Перед тем как пригласить участников рассказывать решения, мы
устанавливаем очередь. Если времени точно хватит на всех, то дети
сами решают, кто пойдёт первым, кто вторым и т. д.
В большом кружке очерёдность лучше установить преподавате­
лю: надо позаботиться о том, чтобы на протяжении нескольких за­
нятий был охвачен каждый участник. Например, в одном из круж­
ков я на каждом занятии записываю имена участников на доске,
чередуя порядок: один раз от А до Я, в следующий раз от Я до А.
Когда список составлен, участники рассказывают решения пре­
подавателю (или преподавателям) в установленном порядке. (Луч­
ше попросить детей зачёркивать свои имена, когда подходит их оче­
редь.) Если кто-то хочет подойти к преподавателю ещё раз, можно
снова записаться в конце списка.
Каждому помощнику нужно относительно спокойное место в клас­
се или в коридоре, где можно обсуждать решения. Однако руково­
дитель кружка должен оставаться в классе, чтобы присматривать за
остальными детьми.
Где найти ответы на вопросы
Вы прочитали эту книгу, но у вас всё равно осталось множе­
ство вопросов. Где искать ответы? Так как вы читаете эту книгу
по-русски, получить ответы на эти вопросы вам гораздо легче, чем
всем остальным читателям.
Наверное, нужно переехать в другую страну, чтобы понять, на­
сколько богата и уникальна традиция российских математических
кружков. Многие десятки лет талантливейшие математики и препо­
даватели вкладывали свои усилия в её развитие. В результате созда­
на культура математических кружков, школ, лагерей и олимпиад,
похожей на которую не найти. И, конечно же, количество литерату­
ры и других ресурсов на русском языке на порядок больше, чем на
всех остальных языках, вместе взятых.
Давайте начнём с книг. На русском языке напечатана масса за­
мечательной литературы по внешкольной математике. В том чис­
ле, очень много хороших книг выпущено издателем этой книги,
Московским центром непрерывного математического образования
(МЦНМО). В конце книги я привожу список литературы, которой я
регулярно пользуюсь, готовясь к занятиям кружка.
Кроме книг, существует масса интернет-ресурсов. Сайт problems.
ru является неоценимым источником прекрасно классифицирован­
ных задач. Малый мехмат МГУ поддерживает многолетние архивы
своих занятий (mmmf.msu.ru). На сайте zaba.ru вы можете найти
интересные подборки задач к разным математическим конкурсам,
включая математические аукционы и математические бои. А есть
ещё и материалы Кировской летней школы, и материалы самарско­
го кружка Мат Гуру (кружок Александра Савина), и заочные мате­
матические кружки 30-й школы Санкт-Петербурга, и ещё много по­
лезных ресурсов. И не следует забывать, что на сайте МЦНМО вы­
ложено огромное количество замечательных книг.
И не забудьте про личное общение. Если вы начинающий препо­
даватель математического кружка, внимательно оглядитесь вокруг.
Велики шансы, что вы можете найти более опытного коллегу, кото­
рый готов поделиться с вами своими знаниями. Если вы родитель и
хотите попробовать учить вашего ребёнка и нескольких его друзей,
попробуйте найти других родителей или учителей, разделяющих ва­
ши интересы.
262 Раздел 3. Еще несколько советов преподавателю

Математические кружки во всём мире


За последние годы, во многом благодаря людям, которые пере­
ехали в другие страны после распада Советского Союза, математи­
ческие кружки приобретают известность во всём мире, от Амери­
ки до Японии. Кружки организовывают совершенно разные люди:
профессора университетов, математики, занимающиеся приклад­
ной наукой, родители способных школьников. Нам кажется, что
это совершенно замечательная тенденция. В США, например, за
последнее время была организована ассоциация математических
кружков, проводятся регулярные конференции, печатаются кни­
ги. Американское математическое общество (AMS) выпускает се­
рию книг «Библиотека математического кружка». Много лет назад
в ней была переведена и напечатана книга «Ленинградские мате­
матические кружки» С. А. Генкина, Д. В. Фомина и И. В. Итенберга.
За последнее время в ней были изданы книги «Московский матема­
тический кружок» С. А. Дориченко, «Приглашение на математиче­
ский фестиваль» И. В. Ященко, замечательные книги, посвященные
кружкам для малышей, А. К. Звонкина и Н. Рожковской. Та книга,
которую вы сейчас читаете, тоже была написана для этой серии.
Я очень надеюсь, что она будет полезна читателям и внесёт свой
вклад в развитие культуры математических кружков.

Пожелания
Уважаемый читатель, в какой бы точке земного шара вы ни на­
ходились, я хочу закончить эту книгу пожеланием вам успеха. Ес­
ли вам интересно учить детей математике, не бойтесь пробовать
и не откладывайте. Если вы вложите достаточно труда и энтузиазма,
у вас обязательно получится!
Пусть ваши дети (или ученики) растут умными и увлечёнными
математикой. А когда они вырастут, пусть им тоже захочется поде­
литься своими знаниями и начать свой собственный математиче­
ский кружок!
РАЗДЕЛ 4

Решения
Занятие 1

Как решить задачу

Решение задачи для разминки 1. Всего шесть детей: пять бра­


тьев и одна сестра.
Решение задачи для разминки 2. Надо положить по монете
в каждый кошелёк, а потом один кошелёк в другой.
Решение примера 6 (предложенного на заня­
тии). См. рисунок.
Решение задачи 1. а) Каждый раз, когда бо­
бёр делает распил, количество кусков увеличивает­
ся на 1. Вначале был 1 кусок. Поэтому ответ —26.
б) Вначале было 2 куска (два длинных бревна).
При каждом распиле общее количество кусков увеличивается на 1.
Следовательно, после 40 распилов получается 42 куска.
Для преподавателей. Некоторые участники пытаются объяснить ответ
так: «Предположим, что бобёр распилил первое бревно 15 раз, а вто­
рое—25 раз. Тогда общее количество кусков будет 16 + 26 = 42». Это
неверное решение. Вот как можно объяснить ошибку: «Ты рассмотрел
только один способ сделать 40 распилов. Но есть много других способов:
например, можно распилить первое бревно 1 раз, а второе —39 раз.
Почему при другом способе не получится другое количество кусков?»
в) Вначале было 3 куска. Поэтому количество кусков всегда на 3
больше количества распилов. Значит, после 50 распилов получится
53 куска.
Решение задачи 2. Передвиньте три спички, показанные на ри­
сунке штриховой линией.

Решение задачи 3. а) Сначала докажем, что если взять мень­


ше чем 5 шаров, то трёх шаров одного цвета может не оказаться.
266 Раздел 4. Решения

Например, если мы возьмём 4 шара, из них может быть 2 синих


и 2 красных.
Теперь докажем, что если взять 5 шаров, то 3 из них обязательно
будут одного цвета. Предположим, что это не так. Тогда у нас не
больше двух красных шаров и не больше двух синих, значит, всего
не больше четырёх шаров. Но мы взяли 5 шаров, поэтому предполо­
жение неверно.
б) 17 шаров. Сначала докажем, что если взять меньше чем 17 ша­
ров, то синих шаров может оказаться меньше трёх. Например, если
взять 16 шаров, из них может оказаться 14 красных и 2 синих.
Теперь докажем, что если взять 17 шаров, то среди них обяза­
тельно будет три синих. Действительно, из 17 шаров красных не
может быть больше чем 14. Поэтому синих не меньше чем 3.
Решение задачи 4. В коробке ровно один красный носок. Если
бы красных носков было два (или больше), было бы неверно, что
среди любых двух носков есть один синий.
Решение задачи 5. Из п. 1 следует, что Маша живёт не в Челя­
бинске. Из п. 2 следует, что Маша не из Москвы и не из Самары. Зна­
чит, Маша живёт в Санкт-Петербурге. Из п. 3 следует, что Ксюша не
из Москвы и не из Челябинска. Она также не из Санкт-Петербурга,
потому что там живёт Маша. Значит, Ксюша из Самары. Остались
Даша и Наташа: они должны быть из Москвы и Челябинска. Из п. 4
мы знаем, что Наташа не из Москвы. Следовательно, Наташа живёт
в Челябинске, а Даша —в Москве.
Решение задачи 6. Самолёт приземлится далеко от места взлёта.
Замечание 1. Что означает направление на север или на запад?
Это движение по меридианам (линии долготы) и параллелям (окруж­
ности широты). Когда самолёт летит на север или на юг, он движется
по меридиану. Когда он летит на восток или на запад, он движется
по параллели.
Замечание 2. Выберем в Северном полушарии две точки на од­
ной и той же широте. Начнём перемещать эти точки к северу (по со­
ответствующим меридианам) таким образом, чтобы они всё время
оставались на одной широте. Тогда по мере приближения этой пары
точек к Северному полюсу расстояние между ними по параллели
будет становиться всё меньше.
Первый и третий участки пути самолёта направлены по мери­
дианам, а второй и четвёртый — по параллелям. Кроме того, место
вылета (Мурманск) находится в Северном полушарии, и полёт на-
Занятие 1. Как решить задачу 267

чинается с движения на север. Поэтому из замечания 2 следует, что


расстояние между местом вылета и местом последнего поворота (на
восток) больше, чем 300 километров. Значит, пролетев последний
участок пути (300 километров к востоку), самолёт не достигнет ме­
ста вылета. Он приземлится к западу от Мурманска.
Для преподавателей. Эту задачу гораздо проще объяснить на глобусе. Ес­
ли глобуса под рукой нет, подойдут воздушный шарик и маркер. Марке­
ром можно нарисовать решение на шарике.
Решение задачи 7. а) 51 перчатка. Сначала докажем, что если
перчаток меньше 51, то пары одного цвета может не найтись. Пред­
положим, что перчаток 50 и из них 30 оранжевых левых и 20 чёрных
правых. В таком наборе нет пары одного цвета. Теперь докажем, что
51 перчатки в любом случае достаточно. Действительно, оранжевых
перчаток без пары не может быть больше 30, а чёрных без пары не
может быть больше 20. Поэтому всего перчаток без пары не может
быть больше 50.
б) 81 перчатка. Сначала докажем, что если перчаток меньше 81,
то они могут не содержать чёрной пары. Предположим, что перча­
ток 80 и из них 60 оранжевых и 20 чёрных правых. В таком набо­
ре нет чёрной пары. Теперь докажем, что в наборе из 81 перчатки
обязательно есть чёрная пара. Действительно, оранжевых перчаток
не может быть больше 60. Значит, чёрных перчаток не меньше 21.
Чёрных перчаток без пары не может быть больше 20. Следователь­
но, найдётся чёрная пара.
Решение задачи 8. Два слона и девять верблюдов.
Всего есть 22 уха, а значит, 11 животных. Сколько из них могут
быть верблюдами? Поскольку горбов в девять раз больше, чем хобо­
тов, количество горбов должно делиться на 9. Горбов не может быть
больше 22 (2 • 11), значит, их может быть только 0, 9 или 18. Но если
горбов 0, то и хоботов 0:9 = 0, что невозможно. Поскольку горбов
чётное число, их не может быть 9. Итак, горбов 18. Следовательно,
верблюдов 9, а слонов 2.
Для преподавателей. Не стоит объяснять решение этой задачи с помощью
уравнений, так как многие участники их ещё не изучали.
Занятие 2

Рыцари и лжецы
Решение задачи для разминки 1. Десять ершей.
Нарисуйте две корзины — одну с пятью ершами и другую с поло­
виной улова. Это поможет детям понять, что пять ершей составляют
вторую половину улова.
Решение задачи для разминки 2. Невозможно определить, ка­
кое животное тяжелее. У нас недостаточно информации.
Решение задачи для разминки 3. Подсказка. Есть разные си­
стемы записи чисел.
Решение. Передвинув две спички, можно получить число VI, за­
писанное римскими цифрами.
Решение задачи 1. Эту задачу можно пояснить схемой, похожей
на ту, которая была нарисована на занятии.
существа, любящие плавать

рыбы маленькие
дети

Мама сказала Максу, что рыбы относятся к существам, кото­


рые любят плавать. Но есть и другие любители плавать (например,
дети). Поэтому Макс неправ: его утверждение подразумевает, что
плавать любят только рыбы.
Решение задачи 2. Есть несколько вариантов такого вопроса.
Например: «Ты лжец?», «Ты всегда говоришь правду?» и другие.
Решение задачи 3. 15 килограммов.
Рисунок объясняет решение лучше, чем много строк текста. Боль­
шой прямоугольник — это большой золотой слиток. Рисунок помо-
1
гает понять, что пятикилограммовыи слиток весит « всего слитка.
весь слиток

2 5 кг
- слитка
Занятие 2. Рыцари и лжецы 269

Для преподавателей. Решение этой задачи с помощью рисунка подго­


тавливает почву для темы одного из ближайших занятий — наглядного
представления уравнений.
Для преподавателей. Если вы считаете, что дети к этому готовы, сейчас
подходящий момент, чтобы ввести уравнения. Представим решение
2
с помощью уравнения. Пусть х — вес слитка; тогда - х + 5 = х.

Решение задачи 4. См. рисунок.

Решение задачи 5. 15 ночей и 16 дней. За сутки (день и ночь)


коала поднимается на один метр. Значит, к утру шестнадцатого дня
он будет на высоте 15 метров. А вечером того же дня коала достиг­
нет верхушки.
Решение задачи 6. Ни один житель острова не скажет: «Я лжец»
(к такому выводу мы пришли при разборе одной из задач на заня­
тии). Поэтому Боря точно лжёт, когда будто бы повторяет утвер­
ждение Миши. Боря —лжец. Значит, Слава говорит правду. Однако
определить, кем является Миша, невозможно.
Для преподавателей. Другой способ решения — отдельно разобрать два
случая: Миша —лжец и Миша —рыцарь. В обоих случаях получается,
что Боря лжёт.
Решение задачи 7. Разобраться в этой задаче поможет следую­
щая схема.

или или

Поскольку все совы, кроме двух, ушастые, общее количество по­


лярных и болотных сов равно 2. Аналогично ушастых и полярных
сов вместе тоже 2. Сколько может быть полярных сов? Исходя из
того, что нам уже известно, их может быть 0, 1 или 2.
• Если полярных сов нет, то сов каждого из двух других видов
(ушастых и болотных) по 2.
270 Раздел 4. Решения

• Если полярная сова одна, то сов каждого из остальных видов


тоже по одной.
• Наконец, если полярных сов две, то других сов нет.
Решение задачи 8. а) У каждого кубика в углу большого куба
окрашены три грани. Таких кубиков 8.
У каждого кубика в середине ребра большого куба окрашены две
грани. Таких кубиков 12.
У каждого кубика в середине грани большого куба окрашена од­
на грань. Таких кубиков 6.
Только у кубика в центре большого куба нет раскрашенных гра­
ней.
б) Получить тот же результат меньше чем за 6 разрезов невоз­
можно. Чтобы объяснить это, рассмотрим кубик в центре. Ни одна
из шести его граней не окрашена, поэтому каждая из них может
получиться только в результате распила. Поскольку все они принад­
лежат одному кубику, никакие две из них нельзя получить одним
распилом. Поэтому, как бы Илья ни ставил друг на друга части куба,
ему понадобится 6 распилов, чтобы вырезать центральный кубик.
Занятие 3

Как превратить ложь в правду

Решение задачи для разминки 1. Петухи не откладывают яйца.


Этим занимаются курицы.
Решение задачи для разминки 2. Это простое упражнение на
тему рыцарей и лжецов. Сеня говорит правду, а Веня лжёт.
Решение задачи 1. Утверждение Маши означает, что количе­
ство съеденных Сашей конфет может быть любым числом из набора
(12,13,14,...}. Из слов Саши следует, что количество съеденных
им конфет может быть любым числом от 0 до 13. Поскольку оба
говорят правду, Саша съел 12 или 13 конфет. Но Саша съел в два
раза больше конфет, чем его сестра; следовательно, он съел чётное
количество конфет. Значит, Саша съел 12 конфет.
Решение задачи 2. Тао сказал, что все трёхголовые животные ум­
ные. Но он не утверждает, что только трёхголовые животные на этой
планете умные. Например, на Крэбби может жить умное животное
с двумя головами. Следовательно, первое утверждение неверно.
Трёхголовое животное на Крэбби обязательно умное. Значит,
там нет ни одного трёхголового животного, которое не является
умным. Потому второе утверждение верно.
Третье утверждение неверно по той же причине, что и первое: Тао
не утверждает, что только трёхголовые животные на планете умные.
Решение задачи 3. Иван не может быть лжецом: иначе его утвер­
ждение о том, что хотя бы один из братьев—лжец, было бы правдой.
Значит, он рыцарь. Поскольку он говорит правду, один из братьев
действительно лжец. Следовательно, Пётр—лжец.
Решение задачи 4. Поскольку среди любых 12 детей есть хо­
тя бы один мальчик, девочек не может быть больше 11. Посколь­
ку среди любых 20 детей есть хотя бы одна девочка, мальчиков не
может быть больше 19. Теперь докажем, что девочек ровно 11. Ес­
ли бы девочек было меньше, всего детей было бы самое большее
10 4-19 = 29. Поскольку всего детей 30, это неверно. Значит, девочек
ровно 11. Из тех же соображений мальчиков ровно 19.
272 Раздел 4. Решения

Решение задачи 5. Сначала предположим, что в комнате дей­


ствительно есть лжецы. Тогда утверждение, что в комнате есть лже­
цы, — правда. Но это утверждение повторяет каждый, кто находится
в комнате, включая лжецов. Мы пришли к противоречию: находя­
щиеся в комнате лжецы говорят правду.
Значит, лжецов в комнате нет. Но тогда ни один рыцарь не может
сказать, что там есть лжецы. Получается, что рыцарей в комнате
тоже нет. Там одни туристы.
Решение задачи 6. См. рисунок.

Решение задачи 7. Надо задать вопрос: «Живут ли в этой де­


ревне люди твоего типа?» В деревне рыцарей и рыцарь, и лжец от­
ветят «Да». В деревне лжецов и рыцарь, и лжец ответят «Нет».
Решение задачи 8. Утверждения Маши и Даши можно перефор­
мулировать так: Маша заявляет, что 12 рубинов дороже, чем 18 сап­
фиров, а Даша —что 12 рубинов дороже, чем 16 сапфиров.
Но если 12 рубинов стоят дороже, чем 18 сапфиров, они тем бо­
лее стоят дороже, чем 16 сапфиров. Значит, если бы утверждение
Маши было верным, утверждение Даши тоже было бы верным. Дру­
гими словами, если Маша — рыцарь, то и Даша — рыцарь. Но рыца­
рем является только одна из девочек. Следовательно, Маша—лжец
(а рыцарь—Даша). Значит, 12 рубинов не дороже, чем 18 сапфиров.
Ответ к задаче: «Нет».
Решение задачи 9. Рассмотрим утверждение Вити: «Антон и Бо­
ря—два лжеца». Если бы Витя был рыцарем, Антон и Боря действи­
тельно были бы лжецами. В этом случае Антон не сказал бы, что
Боря—лжец (ведь он не может сказать правду). Следовательно, Ви­
тя—лжец. Поскольку лжец Витя говорит, что Гриша—лжец, на са­
мом деле Гриша —рыцарь.
Теперь мы знаем, что утверждение Вити «Антон и Боря—два
лжеца» неверно. Это значит, что либо они оба рыцари, либо один из
них — рыцарь. Если бы и Антон, и Боря были рыцарями, Антон не
сказал бы, что Боря—лжец. Значит, только один из них —рыцарь.
Рыцарь Гриша говорит, что Антон—лжец. Следовательно, Боря —
рыцарь.
Занятие 3. Как превратить ложь в правду 273

Решение задачи 10. Сначала выберем любых двух политиков из


разных областей. Если окажется, что они из разных партий, то за­
дача решена: мы нашли двух политиков, которые живут в разных
областях и состоят в разных партиях.
Если же эти два политика состоят в одной партии, рассмотрим
третьего политика — допустим, его зовут Потап, — принадлежащего
к другой партии. Такой политик обязательно найдётся, так как из­
вестно, что за ужином есть политики из разных партий.
Поскольку первые два политика живут в разных областях, хотя
бы один из них —назовём его Макар —не из той области, в кото­
рой живёт Потап. Вспомним, как мы выбрали Потапа: он не из той
партии, в которой состоит Макар. Значит, и в этом случае задача
решена.
Занятие 4

Математический аукцион I

Решение задачи для разминки 1. На первом.


Решение задачи для разминки 2. 12 секунд. Между первым
и девятым этажами лестничных пролётов в четыре раза больше, чем
между первым и третьим.
Решение задачи 1. У этой задачи много решений. Подойдёт лю­
бой вопрос (требующий ответа «да» или «нет»), ответ на который
заранее известен. Например: «Ты девочка?» или «Земля — планета?»
Решение задачи 2. Отметим, что информация о китах не влияет
на решение этой задачи. Теперь представим себе огромные весы, на
одной чаше которых лежат 10 дельфинов, а на другой— 6 акул. Ча­
ша с дельфинами легче чаши с акулами. Если убрать одного дельфина
с чаши, она станет ещё легче. Поэтому 9 дельфинов легче, чем 6 акул.
Если уменьшить груз на каждой часе в три раза, весы останутся в том
же состоянии. Поэтому 3 дельфина легче, чем 2 акулы.
Решение задачи 3. Да. Одно из решений показано на
рисунке.
Решение задачи 4. Если бы Полли была лжецом или
честным попугаем, она не могла бы сказать, что она лжец. Значит,
Полли — обманщица. Поэтому утверждение Долли, что она обманщи­
ца, ложное. Следовательно, Долли — лжец, а Арчи — честный попугай.
Решение задачи 5. ОТВЕТ: 39. Сначала докажем, что если в де­
легации меньше 39 человек, то в ней может не оказаться трёх на­
стоящих колдунов. Предположим, что мы пригласили 38 человек
и к нам приехали все ведьмы (21 человек) и все шарлатаны (15 че­
ловек). В этом случае 36 членов делегации — не настоящие колдуны.
Значит, настоящих колдунов в ней только два. А если в делегации
меньше 38 человек, то этот случай тем более не подходит.
Теперь докажем, что если пригласить 39 человек, то к нам обяза­
тельно приедут три настоящих колдуна. В делегацию могут войти са­
мое большее 21 ведьма и самое большее 15 шарлатанов. Значит, в де­
легации самое большее 36 человек, которые не являются настоящими
колдунами. Поэтому в ней самое меньшее три настоящих колдуна.
Занятие 5

Текстовые задачи и здравый смысл

Решение задачи для разминки 1. Одному из детей надо дать


яблоко вместе с корзиной.
Решение задачи для разминки 2. Совет был такой: «Сядь на
ишака твоего брата».
Решение задачи 1. На рисунках показаны возможные решения.
(В первом случае есть много разных способов.)

Решение задачи 2. Следующий рисунок поясняет решение. После


того как Макс подарил Ксюше две конфеты, у неё стало на четыре кон­
феты больше, чем у него. Поскольку теперь его конфеты составляют
половину Ксюшиных конфет, эти четыре конфеты составляют вторую
половину. Значит, у Ксюши 8 конфет (а у Макса — 4). Следовательно,
вначале у них было по 6 конфет.

половина конфет Ксюши

Макс:
Ксюша:

конфеты Ксюши

Решение задачи 3. Следующий рисунок поясняет решение. По


нему нетрудно определить удвоенное количество съеденных Сашей
конфет: 141 — 7 = 134. Значит, Саша съел 67 конфет, а Гриша 74. Им
276 Раздел 4. Решения

обоим грозит промывание.

Гриша: 7 конфет
Саша:
141 конфета 134 конфеты

Решение задачи 4.
• Алла — рыцарь.
• Белла — рыцарь. Например, 3 + 5 = 8.
• Вика — рыцарь. Сумма двух чётных чисел всегда чётная.
• Глаша—лжец, потому что Даша — лжец.
• Даша —лжец. Например, 3 + 5 = 8.
Для преподавателей. Не нужно требовать от детей строгого доказатель­
ства того, что сумма чётных чисел всегда чётная: немного позже чётным
и нечётным числам будут посвящено несколько занятий. Цель этой зада­
чи — подтолкнуть детей к размышлениям о чётных и нечётных числах.
Решение задачи 5. Следующий рисунок поясняет решение. По
нему видно, что количество съеденных Филимоном камней, умно­
женное на четыре, равно 2010 — 1 — 2 — 3 = 2004. Значит, Филимон
съел 501 камень, Парамон — 502, Капитон — 503, а Архип — 504.

Филимон:
Парамон:
Капитон:
Архип:

2010 2004

Решение задачи 6. Наименьшее общее кратное чисел 12 и 20 —


это 60. Если бы Дед Мороз работал 60 месяцев, то он приготовил
бы 5 наборов подарков. Если бы Снегурочка работала 60 месяцев,
то она приготовила бы 3 таких набора. Значит, вместе они за 60 ме­
сяцев приготовили бы 8 наборов. Для приготовления одного набора
им понадобится 6 0 : 8 = 7,5 месяца.
Решение задачи 7. На рисунке видно, что один пони весит столь­
ко же, сколько три гнома. Значит, пони в три раза тяжелее.

3 пони и 1 гном:
2 пони и 4 гнома:
Занятие 5. Текстовые задачи и здравый смысл 277
Решение задачи 8. Найдём наименьшее общее кратное чисел 2,
3 и 6. Это 6. За 6 дней лошадь съест 3 тюка сена, корова — 2 тюка,
а коза — 1 тюк. Вместе за 6 дней эти животные съедят 3 + 2 + 1 = 6 тю­
ков. Значит, один тюк они съедят за 6:6 = 1 день.
Решение задачи 9. Девочек самое большее 16: если бы их бы­
ло больше, можно было бы составить группу из 17 девочек. Но мы
знаем, что среди каждых 17 участников есть хотя бы один мальчик.
Точно так же можно доказать, что мальчиков самое большее 9.
Из этого следует, что девочек ровно 16: если бы их было 15 или
меньше, то (с учётом того, что мальчиков самое большее 9) общее
количество участников было бы меньше 25, что противоречит усло­
вию задачи.
Итак, из 25 участников девочек ровно 16; значит, мальчиков 9.
Решение задачи 10. Каждый раз, когда бобриха делает распил,
общее количество кусков увеличивается на 1. После 83 распилов по­
лучилось 109 кусков; значит, до начала работы их было 109 — 83 = 26.
Бобриха нашла 26 брёвен.
Занятие 6

Ещё текстовые задачи

Решение задачи для разминки 1. Ровно через 168 часов снова


будет полночь: 168 = 24 • 7.
Решение задачи для разминки 2. Номера квартир. Одна цифра
стоит 10 рублей.
Если детям ничего подобного не приходит в голову, можно при­
вести другие примеры, наводящие на мысль, что цена загадочного
товара зависит от количества цифр. Например, 17 стоит 20 рублей,
столько же стоит 93, а вот 400 стоит уже 30 рублей.
Решение задачи 1. За 9 шоколадок Настя заплатила бы на 60
рублей больше, чем за 5 шоколадок. Значит, 4 шоколадки стоят
60 рублей. Одна шоколадка стоит 15 рублей.
Решение задачи 2. Сначала подсчитаем те квадраты, стороны
которых параллельны линиям сетки. Можно нарисовать 9 квадра­
тов 1 х 1, 4 квадрата 2 х 2 и 1 квадрат 3 x 3 . Квадратов большего
размера нет.
Теперь считаем квадраты со сторонами, не параллельными ли­
ниям сетки. Рассмотрим несколько случаев.
— Квадраты, стороны которых —диагонали квадратов 1 x 1 . Два
таких квадрата показаны на первом рисунке. Все возможные поло­
жения левых вершин таких квадратов помечены звёздочками. Всего
8 возможностей.

— Квадраты, стороны которых — диагонали квадратов 2 х 2. На вто­


ром рисунке показан один из таких квадратов, а все возможные поло­
жения левых вершин помечены звёздочками. Всего 2 возможности.
Занятие 6. Ещё текстовые задачи 279

— Квадраты, стороны которых — диагонали прямоугольников


1 x 2 . Таких квадратов 3: все возможные положения левых вершин
помечены звёздочкой на третьем рисунке.
— Квадраты, стороны которых — диагонали прямоугольников
2 x 1 . Таких квадратов 3: см. последний рисунок.
ОТВЕТ: 14 + 8 + 2 + 3 + 3 = 30.
Решение задачи 3. Разделим всех учеников на четыре группы.

Мальчики Девочки
Сдавшие Сдавшие мальчики Сдавшие девочки
Не сдавшие Не сдавшие мальчики Не сдавшие девочки

Общее число девочек равно сумме числа девочек, сдавших экза­


мен, и числа девочек, не сдавших его. Это сумма значений в столбце
«Девочки».
Общее число учеников, сдавших экзамен, равно сумме числа де­
вочек, сдавших экзамен, и числа мальчиков, сдавших его. Это сумма
значений в строке «Сдавшие».
Поскольку число мальчиков, сдавших экзамен, равно числу де­
вочек, не сдавших его, эти две суммы равны.
Решение задачи 4. 20 — наименьшее общее кратное чисел 4 и 5.
Первая черепаха проползёт 20 метров за 9 • 5 = 45 часов. Вторая про­
ползёт то же расстояние за 11•4 = 44 часа. Значит, вторая черепаха
движется быстрее.
Решение задачи 5. Если временно предположить, что все драко­
ны двуглавые, получится 38 • 2 = 76 голов. Остальные 93 — 76=17 го­
лов — это третьи головы трёхглавых драконов. Значит, трёхглавых
драконов 17, а двуглавых 38 — 17 = 21.
Решение задачи 6. Каждый мальчик потерял на 4 предмета одеж­
ды больше, чем каждая девочка. Если бы каждый мальчик потерял
только три свитера, все 30 учеников потеряли бы 3 • 30 = 90 предме­
тов. На самом деле потеряно 134 предмета; значит, мальчики «до­
полнительно» потеряли 134 — 90 = 44 свитеров. Поскольку каждый
мальчик дополнительно потерял 4 свитера, количество мальчиков
равно 44 : 4 = 11. Количество девочек равно 30 - 11 = 19.
Решение задачи 7. Разница между покупками Кати и Маши —
это 4 попугая минус 4 хорька. Поскольку попутай в три раза до­
роже хорька, эта разница в денежном выражении равна стоимости
280 Раздел 4. Решения

восьми хорьков. Она составляет 4080 рублей. Значит, хорёк стоит


510 рублей. Попугай стоит 1530 рублей.
Решение задачи 8. Если в конце числа приписать 0, оно станет
в 10 раз больше. Поэтому сумма получилась больше ожидаемой на
второе число, умноженное на девять. Значит, второе число равно
(7641 - 2331): 9 = 590, а первое равно 2331 - 590 = 1272.
Решение задачи 9. Если бы каждый крупный мафиози выкурил
столько же сигар, сколько и каждый мелкий, общее количество сигар­
ных окурков делилось бы на 15 и было бы меньше 99. Кроме того,
разность между 99 и этим числом (т. е. количество дополнительных
ситар, выкуренных крупными мафиози) должна делиться на 4.
Чисел, которые делятся на 15 и меньше 99, не так много: 0,15, 30,
45, 60, 75 и 90. Числа 0, 30, 60 и 90 чётные, поэтому разность между
99 и такими числами не может делиться на 4. Остаются 15, 45 и 75.
Число 45 также не подходит: 99 — 45 = 54 не делится на 4. Остаётся
два возможных варианта: 15 и 75.
В первом случае каждый мелкий мафиози выкурил одну сига­
ру. Дополнительных сигар, выкуренных крупными мафиози, было
99 — 15 = 84, а их самих 84:4 = 21. Итак, этот вариант тоже не годит­
ся: крупных мафиози не может быть больше 15. Остаётся число 75.
Значит, каждый мелкий мафиози выкурил 75:15 = 5 сигар, крупных
мафиози было (99 — 75): 4 = 6, а мелких 1 5 - 6 = 9.
Решение задачи 10. Если бы каждый член шайки получил по
3 монеты, всем разбойникам вместе досталось бы 60 монет. Значит,
старшие разбойники присвоили 109 — 60 = 49 дополнительных монет.
Поскольку все старшие разбойники получили одно и то же коли­
чество дополнительных монет, возможны всего три варианта: 1 стар­
ший разбойник и 49 дополнительных монет; 7 старших разбойников
и 7 дополнительных монет; 49 старших разбойников и 1 дополнитель­
ная монета. Последний вариант сразу отпадает: старших разбойников
не может быть больше 20. Остаётся два варианта: 1 старший разбой­
ник, забравший себе 49 + 3 = 52 монеты, или 7 старших разбойников,
получивших по 7 + 3 = 10 монет.
Решение задачи 11. а) Выберем любую пару противоположных
стен (не имеющих общей башни). Докажем, что, как бы Макс ни
спрятал свои талисманы, непробиваемой станет ровно одна из этих
двух стен.
«Сила» первой стены равна суммарному количеству талисманов
в двух башнях, которые она соединяет. «Сила» противоположной
Занятие 6. Ещё текстовые задачи 281

стены равна суммарному количеству талисманов в двух других баш­


нях. Поэтому если сложить силы этих двух стен, то получится общее
количество талисманов во всех четырёх башнях, т. е. 21. Обозначим
силу первой стены буквой х, а силу второй стены буквой у. Получа­
ется равенство х + у = 21. Поскольку х и у — целые неотрицатель­
ные числа, это равенство возможно только тогда, когда одно из них
больше 10, а другое не больше 10. Поэтому одна из этих двух стен
непробиваемая, а другая пробиваемая.
Точно так же из второй пары противоположных стен одна непро­
биваемая, а другая пробиваемая. Поэтому, как бы Макс ни спрятал
талисманы, непробиваемыми станут ровно две стены замка.
б) Эта задача решается аналогично задаче а). Рассмотрим волшеб­
ную силу двух противоположных башен (например, первой и четвёр­
той). Волшебная сила первой башни —это общее количество вол­
шебных фонарей на ней и на двух соседних башнях (второй и ше­
стой). Волшебная сила противоположной башни равна общему ко­
личеству волшебных фонарей на остальных трёх башнях (третьей,
четвёртой и пятой). Поэтому суммарная волшебная сила этой (как
и любой другой) пары противоположных башен равна 31. Но если
сумма двух неотрицательных целых чисел равна 31, то одно из них
должно быть больше 15, а другое не больше 15. Поэтому из любых
двух противоположных башен волшебным светом светится ровно
одна. Значит, всего волшебным светом светятся три башни.
в) Эта задача решается аналогично задачам а) и б). Для любой
группы из пяти расположенных подряд башен остальные пять ба­
шен также расположены подряд. Из этих двух групп ровно одна
обеспечивает замок защитной силой. Всего есть 5 таких пар групп,
поэтому замок получает 5 защитных сил.
Занятие 7

Чётные и нечётные числа I.


Волшебные бумажные стаканчики

Решение задачи для разминки 1. Для случаев 3 и 4 стульев ре­


шение показано на рисунках. В случае 12 стульев расставим 4 стула
по углам и по 2 у каждой стены (не в углах). Тогда у каждой стены
окажется 4 стула.

Решение задачи для разминки 2. См. рисунок.


Решение задачи для разминки 3. Звонивший не
может жить в городе А: иначе он сказал бы, что живёт
в городе А. Он также не может жить в городе В: иначе
он не сказал бы, что живёт в городе В. Следователь­
но, звонил житель города С. Второе из его высказываний неверное,
значит, первое верное. Надо ехать в город С.
Решение задачи 1. На рисунке показано по одному варианту
разрезания пиццы на 4, 5, 6 и 7 частей.

Решение задачи 2. а) Если нажать первую кнопку (на 8 этажей


вверх) 12 раз, лифт приедет на 97-й этаж. Теперь нажмём вторую
кнопку (на 6 этажей вниз) три раза: лифт опустится на 18 этажей.
Наконец, нажмём первую кнопку два раза. Лифт поднимется на 16
этажей и окажется на 95-м этаже.
Занятие 7. Чётные и нечётные числа I 283

Для преподавателей. Важно следить за тем, чтобы лифт не поднимался


выше 100-го этажа. Пример неверного решения: «Надо нажимать кноп­
ки по очереди. Тогда после каждых двух нажатий лифт будет поднимать­
ся на два этажа (на 8 вверх, потом на 6 вниз) и в конце концов окажется
на 95-м этаже». Ошибка в том, что на 95-й этаж невозможно попасть
с помощью кнопки «на 6 этажей вниз»: для этого лифт должен был бы
находиться на 101-м этаже.
б) Попасть на 96-й этаж невозможно. Лифт всегда останавлива­
ется на нечётных этажах: он начинает движение с первого (нечёт­
ного) этажа и поднимается или опускается только на чётное число
этажей. Поэтому он не может оказаться на чётном этаже.
Решение задачи 3. Представим себе момент, когда Петя догнал
Юлю, и подумаем, сколько времени шёл каждый из них до этого мо­
мента. Петя идёт в полтора раза быстрее; значит, время Юли в пол­
тора раза больше, чем время Пети. Кроме того, мы знаем, что время
Юли на 5 минут больше: ведь Петя вышел на 5 минут позже неё.
Получается такой рисунок:

Время Пети времени Пети

Время Юли:
Время Юли:
Время Пети 5 минут

На рисунке видно, что 5 минут —это половина времени Пети.


Значит, Юля получит свой телефон через 10 минут после того, как
Петя вышел из школы.
Решение задачи 4. а) После одного заклинания количество не­
счастных принцесс может увеличиться на 2, уменьшиться на 2 или
остаться неизменным.
б) Количество несчастных принцесс остаётся нечётным, потому
что одно заклинание не изменяет это количество или изменяет его
на 2.
в) Поскольку количество несчастных принцесс остаётся нечёт­
ным после каждого заклинания, оно будет нечётным и после не­
скольких заклинаний.
г) Это невозможно, так как 0 —чётное число.
Решение задачи 5. Сначала плодов было 29 (нечётное число).
Их количество либо остаётся неизменным, либо увеличивается или
284 Раздел 4. Решения

уменьшается на 2. Значит, что бы колдунья ни делала, количество


плодов останется нечётным. Поэтому на дереве никогда не будет
О плодов.
Решение задачи 6. Всего Робин провёл в школе 798:2 = 399 пе­
ремен. Каждый день, когда он бывал в школе, количество уроков
было на 1 больше, чем количество перемен. Например, если бы он
провёл в школе всего 2 дня, то общее количество уроков было бы
на 2 больше, чем общее количество перемен. Если 3 дня —то на 3,
и так далее. Так как количество уроков на 100 больше, Робин провёл
в школе 100 дней.
Для преподавателей. Типичное неверное рассуждение: «Если бы было
99 учебных дней с пятью уроками каждый и 1 день с четырьмя уроками,
то количество перемен было бы 99 • 4 4-1 • 3 = 399». Ошибка этого рас­
суждения в том, что оно опирается на конкретный вариант расписания
уроков. А если бы расписание было другое? Может быть, получилось бы
другое количество перемен?
Решение задачи 7. Начнём с двух замечаний.
• В полном наборе домино каждое количество точек встречается
на восьми квадратиках.
• Костяшки всегда соприкасаются одинаковыми квадратиками.
Это значит, что одинаковые квадратики в ряду костяшек обяза­
тельно расположены парами, за исключением двух крайних — слева
и справа.
Теперь рассмотрим все возможные варианты для квадратика
в правом конце ряда. Может ли на нём быть 0 точек? Тогда осталь­
ные семь квадратиков, на которых тоже 0 точек, находились бы
в середине ряда. Но в середине ряда одинаковые квадратики распо­
лагаются парами, а семь квадратиков невозможно разбить на пары.
Значит, на квадратике в правом конце ряда не может быть 0 точек.
Рассуждая точно так же, приходим к выводу, что на нём не может
быть 1, 2, 3, 4 или 5 точек. Итак, на квадратике в правом конце ряда
6 точек.
Занятие 8

Чётные и нечётные числа II.


Определения и свойства

Решение задачи для разминки 1. а) Таких форм можно приду­


мать очень много. Примерами могут являться буквы S или Е. Каж­
дая из них режется вертикальным разрезом на 4 части.
б) Таких форм тоже много. Один пример —волна (синусоида)
с двумя низкими и двумя высокими точками.
Решение задачи для разминки 2. За завтраком были трое: де­
душка, отец и сын.
Обычно перед тем, как догадаться до правильного ответа, дети
высказывают разные забавные идеи. Например, Эндрю предполо­
жил, что один человек за завтраком проглотил другого, который пе­
ред этим успел съесть яйцо. Очевидно, он вспомнил разминочную
задачу про два кошелька и две монеты.
Решение задачи 1. Буханку можно разрезать на 4,5, 6, 7 и 8 ча­
стей. Эта задача дополняет задачу из предыдущей подборки — о том,
как разрезать пиццу тремя разрезами. Разрезать буханку на 4, 5, 6
и 7 частей можно точно так же, как и пиццу. Но между пиццей и бу­
ханкой есть существенное различие: буханка имеет толщину. По­
этому её можно разрезать по горизонтали (как для сэндвича). Если
сделать такой горизонтальный разрез и ещё два вертикальных раз­
реза, перпендикулярных друг другу, получится 8 кусков.
Решение задачи 2. Пете исполнилось одиннадцать лет 31 декаб­
ря, а сегодня 1 января.
Посмотрим, как можно прийти к этому ответу (и заодно дока­
жем, что других вариантов не существует).
Если бы сегодня Пете было 10 лет, ему не могло бы в будущем
году исполниться 13. Значит, вчера или сегодня ему исполнилось 11.
Поскольку в будущем году ему исполнится 13, до 31 декабря будуще­
го года у него будет ещё два дня рождения. Итак, между вчерашним
днём и 31 декабря будущего года у Пети три дня рождения.
286 Раздел 4. Решения

Период между сегодняшним днём и 31 декабря будущего года не


больше двух лет (если сегодня 1 января, это ровно два года). Зна­
чит, период между вчерашним днём и 31 декабря будущего года не
больше двух лет и одного дня. Но на этот период приходится три дня
рождения, поэтому он не может быть меньше двух лет и одного дня.
Следовательно, он равен двум годам и одному дню. Это позволяет
точно определить сегодняшнюю дату: 1 января (см. рисунок).

Этот год Следующий год

Решение задачи 3. Это невозможно. Каждая костяшка домино


покрывает два поля, следовательно, все костяшки покрывают чёт­
ное число полей. Но количество полей на доске нечётное (5 • 5 = 25).
Решение задачи 4. Если сложить пять нечётных чисел, получит­
ся нечётное число. А 500 —чётное число.
Решение задачи 5. Утром количество космических кораблей
было нечётным. При каждом прибытии группы кораблей их коли­
чество увеличивается на чётное число. При каждом отправлении
группы кораблей оно уменьшается на чётное число. Если прибав­
лять к нечётному числу чётное или вычитать из него чётное, всегда
будет получаться нечётное число. Поэтому количество космических
кораблей остаётся нечётным. Диспетчер насчитал чётное число ко­
раблей. Значит, он ошибся.
Решение задачи 6. Рассмотрим два номера страниц на лицевой
и обратной стороне одного и того же листа. Эти номера различают­
ся на единицу. Значит, один из них чётный, а другой — нечётный.
Поэтому их сумма нечётная. Получается, что сумма номеров стра­
ниц на 25 выпавших листах равна сумме 25 нечётных чисел. Эта
сумма нечётная и поэтому не может быть равна 2000.
Решение задачи 7. Рассмотрим число шариков, которые Белла
подарила за лето, и число шариков, которые она получила в пода­
рок. Сумма этих двух чисел равна 35. Следовательно, одно из них
чётное, а другое — нечётное, и они не могут быть равны между со­
бой. Обозначим первое из этих чисел х, а второе у.
Занятие 8. Чётные и нечётные числа II. Определения и свойства 287

Тогда количество шариков у Беллы в конце лета равно 50 — х + у.


Поскольку х и у — разные числа, это выражение не может быть рав­
но 50.
Решение задачи 8. Пятирублёвые монеты Макса вместе состав­
ляют нечётное число рублей (так как произведение двух нечётных
чисел нечётно). Двухрублёвые монеты вместе составляют чётное
число рублей (произведение двух чётных чисел четно). Сумма нечёт­
ного и чётного чисел нечётная. Следовательно, в копилке нечётное
число рублей. А 60 — чётное число.
Решение задачи 9. На доске написано 9 чётных и 9 нечётных
чисел. Значит, как бы мы ни расставили знаки «плюс» и «минус»,
в выражении будет 9 чётных и 9 нечётных членов. Поскольку ко­
личество нечётных членов нечётное, значение выражения будет
нечётным числом.
Занятие 9

Математический хоккей I

Решение задачи для разминки 1. Когда бра­


тья встретились, они были на одинаковом рассто­
янии от дома.
Решение задачи для разминки 2. См. рису­
нок.

Решение задач математического хоккея


Решение задачи 1. Ведьме Стинки 107 лет.
Решение задачи 2. Лилит была ангелом, Таня — ведьмой, Бал­
ла — чёртом.
Решение задачи 3. 190 сантиметров.
Решение задачи 4. Один бриллиант стоит 300 тысяч рублей.
Решение задачи 5. Больше всего конфет получил Бен.
Решение задачи 6. Одна маленькая ведьма весит 3 килограмма.
Решение задачи 7. 19 полных лет.
Решение задачи 8. Рядом с Гоусти летит Слайми.
Решение задачи 9. (222 - 22): 2 = 100.
Решение задачи 10. Головка сыра стоит 2000 рублей.
Решение задачи 11. Сеня и Веня—лжецы, Женя —рыцарь.
Решение задачи 12. Папу напугал Игорь.
Решение задачи 13. Квадраты по линиям сетки: 1 квадрат 3 x 3 ,
4 квадрата 2 х 2, 9 квадратов 1 x 1 . Квадраты, повёрнутые на 45°:
5 квадратов 2 х 2 и 12 квадратов 1 x 1 . Всего

(1 + 4 + 9) + (5 + 12) = 14 + 17 = 31 квадрат.

Решение задачи 14. В 12 раз быстрее.


Решение задачи 15. 4 ребёнка.
Решение задачи 16. 4 друга.
Занятие 9. Математический хоккей I 289

Решение задач из подборки


Решение задачи 1. Три рубина дороже четырёх сапфиров и тем
более — трёх сапфиров. Значит, один рубин дороже одного сапфира.
Теперь мы знаем две вещи: три рубина дороже четырёх сапфиров,
а один рубин дороже одного сапфира. Следовательно, три рубина
и один рубин вместе стоят дороже, чем четыре сапфира и один сап­
фир. Другими словами, четыре рубина дороже пяти сапфиров.
Решение задачи 2. Поскольку мама летит в два раза медленнее
папы, она долетит от места встречи до гнезда за 8 минут. Значит,
она прилетит домой через 4 + 8 = 12 минут после вылета.
Решение задачи 3. См. рисунки.

Решение задачи 4. Сначала покажем, что Нелли не должна от­


крывать дверь с номером 20. Если она откроет эту дверь, может
случиться так, что на стене будет 20 нечётных чисел. Когда Нелли
зачеркнёт одно число, останется 19 нечётных чисел. Нелли погиб­
нет, так как сумма 19 нечётных чисел нечётная.
Теперь покажем, что Нелли в любом случае сможет спастись, ес­
ли откроет дверь с номером 21. Ей надо убрать из суммы всех чисел
одно число так, чтобы оставшаяся сумма оказалась чётной.
• Предположим, что сумма всех чисел нечётная. Тогда среди этих
чисел есть хотя бы одно нечётное (так как сумма чётных чисел была
бы чётной). Зачеркнув нечётное число, Нелли получит чётную сумму.
• Теперь предположим, что сумма чётная. Тогда среди всех чисел
есть хотя бы одно чётное (так как сумма двадцати одного нечётного
числа была бы нечётной). Зачеркнув чётное число, Нелли и в этом
случае получит чётную сумму.
Решение задачи 5. Количество монет нечётное, и каждая моне­
та—это нечётное число сугриков. Следовательно, все монеты вме­
сте составляют нечётное число сугриков. Но 50 —чётное число.
Решение задачи 6. Первый способ. Рассмотрим общее количе­
ство конфет у всех детей. Когда они пришли, у них была одна кон­
фета. Каждый раз, когда вы даёте им две карамельки, общее коли­
чество конфет увеличивается на два. Значит, это количество всегда
290 Раздел 4. Решения

остаётся нечётным. Если бы все дети ушли с одинаковым числом


конфет, общее количество конфет делилось бы на 6 и поэтому бы­
ло бы чётным. Но число не может быть чётным и нечётным одно­
временно. Следовательно, уравнять количество конфет в корзинах
детей невозможно.
Второй способ (идея доказательства). Рассмотрим количество
детей, у которых в корзинах нечётное число конфет. Вначале это ко­
личество было нечётным (один ребёнок). Можно доказать, что, ко­
гда дети получают две конфеты, это количество либо остаётся неиз­
менным, либо увеличивается на два, либо уменьшается на два. Зна­
чит, оно всегда остаётся нечётным. Но если бы все дети ушли с оди­
наковым числом конфет, детей с нечётным числом конфет было бы
либо 0, либо 6 — чётное число.
Решение задачи 7. Покрасим башни через одну в белый и чёр­
ный цвета и рассмотрим общее количество часовых на белых баш­
нях. В полночь это количество нечётное, так как белых башен три
и на каждой из них стоит один часовой. Если какой-нибудь часо­
вой переходит на соседнюю башню, количество часовых на белых
башнях увеличивается или уменьшается на 1, и в любом случае его
чётность изменяется. Но в нашей задаче на соседние башни всегда
переходят два часовых одновременно, поэтому чётность каждый раз
изменяется дважды. Значит, после каждого перехода чётность оста­
ётся прежней — той, какая была в начале. Поэтому часовые никогда
не соберутся на одной башне: иначе количество часовых на белых
башнях стало бы равным 0 или 6, а оба эти числа чётные.
Занятие 10

Чётные и нечётные числа III. Чередование

Решение задачи для разминки 1. Те же 4 килограмма.


Решение задачи для разминки 2. См. рисунок.
Решение задачи 1. См. рисунки.

Решение задачи 2. После первой встречи Лео становится ко­


ричневым. После второй— зелёным. Поскольку цвет Лео меняется
после каждой встречи, он будет коричневым после встреч с нечётным
числом хамелеонов и зелёным после встреч с чётным числом хамелео­
нов. 17 —нечётное число, поэтому к обеду Лео будет коричневым.
Решение задачи 3. Пронумеруем все стулья числами от 1 до 25
по часовой стрелке. Если рядом с каждой дамой сидят два рыцаря,
а рядом с каждым рыцарем—две дамы, значит, рыцари чередуются
с дамами. Поэтому на всех стульях с нечётными номерами сидят
гости одного пола. Получается противоречие: на 1-м и 25-м стульях
должны сидеть гости разного пола.
Решение задачи 4. Среди четырёх последовательных целых чи­
сел всегда два нечётных и два чётных. Поэтому сумма таких чисел
чётная.
Решение задачи 5. На середине пути между двумя городами
скорость теплохода удваивается, поэтому вторую половину пути он
проходит за вдвое меньшее время, чем первую (или, что то же са­
мое, за вдвое меньшее время, чем прошёл бы вторую половину при
хорошей погоде). Сэкономленное время (3 часа)—это половина
времени, за которое теплоход проходит половину пути с обычной
292 Раздел 4. Решения

скоростью. Значит, с обычной скоростью он прошёл бы половину


пути за 6 часов, а весь путь —за 12 часов. Вычитая сэкономленные
3 часа, получаем время, за которое теплоход на самом деле прошёл
весь путь: 9 часов.
Решение задачи 6. а) Предположим, что в бусах нет соседних
бусинок одного цвета Тогда цвета бусинок чередуются. Значит, все
нечётные бусинки одного цвета. Но 1-я и 45-я бусинки соседние,
поэтому они не могут быть одного цвета. Это противоречие пока­
зывает, что исходное предположение неверно.
б) Заметим, что 45 делится на 3, причём при делении получается
15 —нечётное число. Рассмотрим «окружность» из бусинок, распо­
ложенных через две, например 1-я, 4-я, 7-я, ..., 43-я. В этой окруж­
ности 15 бусинок. Рассуждая так же, как в пункте а), приходим к вы­
воду, что в ней должны быть две соседние бусинки одного цвета.
Эти две бусинки — «соседние» в окружности из 15 бусинок: в самих
бусах они не расположены рядом — между ними ровно две бусинки.
Именно такие бусинки нам нужно было найти.
Решение задачи 7. а) Сначала заметим, что все возможные точ­
ки приземления кузнечика расположены через один метр. Покра­
сим эти точки поочерёдно в синий и красный цвета. Поскольку дли­
на каждого прыжка —один метр, прыжок всегда начинается и за­
канчивается в точках разных цветов. Другими словами, кузнечик
всё время приземляется в точках с чередующимися цветами.
Теперь предположим, что кузнечик начал прыгать из красной
точки. Тогда он приземлится в синей точке после любого нечётного
числа прыжков: 1,3, ...,21. Поскольку точка, в которой кузнечик
приземлится после 21 прыжка, синяя, она не может совпадать с на­
чальной точкой —красной. Такое же рассуждение подходит и для
случая, когда кузнечик сделал первый прыжок из синей точки.
б) Это невозможно по тем же соображениям, которыми мы поль­
зовались в пункте а). Любые две точки, отстоящие друг от друга
на 10 метров, окрашены в один цвет. Но после 33 прыжков кузнечик
не может приземлиться в точке того же цвета, что и исходная точка.
в) Любые две точки, отстоящие друг от друга на 3 метра, окраше­
ны в разные цвета. Поэтому каждый кузнечик приземляется в точ­
ках с чередующимися цветами. Поскольку между точками, из кото­
рых начинают прыгать кузнечики, нечётное число метров (15), эти
точки окрашены в разные цвета. Значит, после любого одинакового
количества прыжков кузнечики приземлятся в точках разных цветов.
Занятие 10. Чётные и нечётные числа III. Чередование 293

Решение задачи 8. а) и б) Да. Примеры нетрудно нарисовать.


в) Нет. Объясним это с помощью идеи чередования.
Предположим, что нам удалось нарисовать такой многоугольник
и прямую линию, пересекающую каждую сторону. Пронумеруем
вершины многоугольника числами от 1 до 11 по порядку. Это зна­
чит, что одна из сторон многоугольника соединяет вершины 1 и 2,
другая — вершины 2 и 3 и так далее; последняя сторона соединяет
вершины 11 и 1.
Прямая делит весь рисунок на две части. Представим себе, что
одна из этих частей окрашена в красный цвет, а другая — в синий.
Поскольку каждая сторона многоугольника пересекает эту прямую,
любые две вершины, соединённые одной стороной, окрашены в раз­
ные цвета. Другими словами, при движении по краю многоуголь­
ника цвета вершин чередуются. Например, если вершина 1 синяя,
такого же цвета будут вершины 3, 5,..., 11. Но вершины 1 и 11 не
могут быть одного цвета, так как они тоже соединены одной сто­
роной. Это противоречие показывает, что нарисовать такой много­
угольник невозможно.
Решение задачи 9. Вначале общее количество всех монет нечёт­
ное (1 + 2 + 3 + 4 + 5 + 6 = 21). Если выбрать два столбика и до­
бавить к каждому по монете, общее количество монет на столе
увеличится на 2 и, значит, останется нечётным. Если бы можно
было сделать все столбики одинаковыми, общее количество монет
делилось бы на 6 и поэтому было бы чётным. Поскольку число не
может быть чётным и нечётным одновременно, сделать все столби­
ки одинаковыми невозможно.
Занятие 11

Взвешивания и фальшивые монеты

Решение задачи 1. Если бы в этой компании не было рыцарей,


в ней все были бы лжецами. Тогда высказывание «Вы все лжецы»
было бы правдой. Но лжецы никогда не говорят правду. Получается
противоречие. Значит, в компании есть рыцари.
Если бы в компании было хотя бы два рыцаря, высказывание
одного из них «Вы все лжецы» было бы ложью. Значит, в компании
ровно один рыцарь.
Решение задачи 2. См. рисунок.
Решение задачи 3. Поставим на левую чашу весов
трёх шалтаев, трёх болтаев и трёх тянитолкаев, а на
правую—двух шалтаев, пять болтаев и двух тянитолка­
ев. Теперь начнём убирать с каждой чаши равное количество одина­
ковых существ. При этом каждый раз равновесие сохраняется.
Ш, Ш, Ш, Б, Б, Б, Т, Т, Т = Ш, Ш, Б, Б, Б, Б, Б, Т, Т
Ш, Б, Б, Б, Т, Т, Т = Б, Б, Б, Б, Б, Т, Т
Ш,Т,Т,Т = Б,Б,Т,Т
Ш, Т = Б, Б
Теперь мы знаем, что шалтай и тянитолкай весят столько же,
сколько два болтая. Если бы и шалтай, и тянитолкай в отдельности
были легче болтая, левая чаша весов оказалась бы легче правой. Ес­
ли бы каждый из них был тяжелее болтая, левая чаша бы перевеси­
ла. Следовательно, вес болтая находится где-то между весом шалтая
и весом тянитолкая.
Из условия известно, что тянитолкай тяжелее шалтая. Значит,
тянитолкай самый тяжёлый из всех троих, шалтай — самый лёгкий,
а болтай — средний.
Решение задачи 4. а) 3 монеты, одно взвешивание. Взвесим лю­
бые две монеты. Если одна из них легче, она фальшивая. Если они
весят одинаково, оставшаяся монета фальшивая.
б) 4 монеты, два взвешивания. Возможны разные способы; при­
ведём один из них.
Занятие 11. Взвешивания и фальшивые монеты 295

При первом взвешивании выберем любые две монеты и сравним


их. Если они имеют разный вес, более лёгкая из них фальшивая.
Если же они весят одинаково, фальшивая монета находится сре­
ди тех, которые мы ещё не взвешивали. При втором взвешивании
сравним эти две монеты. Более лёгкая из них фальшивая.
в) 5 монет, два взвешивания. Разделим монеты на три группы:
2 монеты, 2 монеты и одна монета. При первом взвешивании срав­
ним первые две группы. Если они весят одинаково, оставшаяся мо­
нета фальшивая. Если одна из групп легче, она содержит фальши­
вую монету. Тогда мы можем определить эту монету при втором
взвешивании.
г) 7 монет, два взвешивания. Разделим монеты на три группы:
2 монеты, 2 монеты и 3 монеты. При первом взвешивании сравним
первые две группы. Если они весят одинаково, фальшивая монета —
в третьей группе. Если одна из групп легче, фальшивая монета на­
ходится в ней.
В любом случае после первого взвешивания мы знаем, в какой
группе фальшивая монета. Теперь рассматриваем только эту груп­
пу. За второе взвешивание можно найти в ней фальшивую моне­
ту: если это группа из трёх монет, то применим тот же способ, что
и в пункте а).
д) 9 монет, два взвешивания. Разделим монеты на три группы
по 3 монеты. При первом взвешивании выберем любые две груп­
пы и сравним их. Если они весят одинаково, то фальшивая моне­
та—в третьей группе. Если одна из групп легче, то фальшивая мо­
нета находится в ней. В любом случае после первого взвешивания
мы знаем, в какой группе фальшивая монета. Продолжение — как
в пункте г).
Решение задачи 5. а) Джек ставит килограммовую гирю на ле­
вую чашу и высыпает всё золото на обе чаши так, чтобы весы оста­
вались в равновесии. На правой чаше будет 5 килограммов золота.
б) Первым взвешиванием Джек отмеряет 5 килограммов золота.
Для второго взвешивания он убирает с весов весь золотой песок, но
оставляет на левой чаше килограммовую гирю. Затем он высыпает
5 килограммов золота на обе чаши, поддерживая весы в равнове­
сии. На правой чаше будет 3 килограмма золота.
в) Первым взвешиванием Джек отмеряет 4,5 килограмма золо­
та (разделив 9 килограммов пополам). Вторым взвешиванием он
отмеряет 2,25 килограмма золота (разделив 4,5 килограмма попо-
296 Раздел 4. Решения

лам). После этого он оставляет 2,25 килограмма на одной чаше, кла­


дёт 200-граммовую и 50-граммовую гири на другую чашу и насы­
пает на эту чашу золото, до тех пор пока весы не окажутся в равно­
весии.
Решение задачи 6. Для первого взвешивания выберем любые
две монеты и положим их на разные чаши. Возможны два варианта.
• Если эти две монеты имеют разный вес, то одна из них фаль­
шивая. Теперь у нас есть две монеты (не взвешенные в первый раз),
о которых точно известно, что они настоящие. Сравним на весах бо­
лее лёгкую монету с настоящей. Если они весят одинаково, значит,
более лёгкая монета настоящая (а более тяжёлая —фальшивая). Ес­
ли нет, то более лёгкая монета фальшивая.
• Если при первом взвешивании монеты весят одинаково, зна­
чит, они обе настоящие. Сравним любую из них с одной из остав­
шихся монет. Если они весят одинаково, эта оставшаяся монета на­
стоящая, если нет —фальшивая.
Решение задачи 7. а) Нарисовать такой маршрут нетрудно.
б) Представим себе, что все залы в музее раскрашены в чёрный
и белый цвета в шахматном порядке. Тогда любые два зала, соеди­
нённые дверью, окрашены в разные цвета. Значит, цвета залов, че­
рез которые проходит Катя, чередуются. Предположим, что первый
зал на Катином маршруте (в юго-западном углу музея) окрашен
в чёрный цвет. Тогда в чёрный цвет окрашены также 3-й, 5-й и во­
обще все нечётные залы на этом маршруте. Последний, 64-й зал
окрашен в белый цвет. Но зал в северо-восточном углу чёрный (как
и все залы на диагонали, направленной с юго-запада на северо-во­
сток). Поэтому Катя не сможет закончить осмотр музея в этом зале.
Решение задачи 8. а) Разделим монеты на три кучки по 25. Сна­
чала сравним первые две кучки.
• Если они весят одинаково, то фальшивая монета находится
в третьей кучке. Сравнив третью кучку с любой из первых двух, мы
узнаём, легче ли фальшивая монета настоящей.
• Если первые две кучки имеют разный вес, то в одной из них
есть фальшивая монета, а в третьей кучке все монеты настоящие.
Сравним более лёгкую из первых двух кучек с третьей. Если более
лёгкая кучка весит столько же, сколько кучка настоящих монет, то
фальшивая монета находится в более тяжёлой кучке и, значит, тя­
желее настоящей. Если более лёгкая кучка весит меньше, чем кучка
настоящих монет, то фальшивая монета легче настоящей.
Занятие 11. Взвешивания и фальшивые монеты 297

б) Разделим монеты на три кучки: по 25 монет в первых двух


кучках и 26 в третьей. Сравним первые две кучки.
• Если они весят одинаково, то в них только настоящие монеты,
а фальшивая монета — в третьей кучке из 26 монет. Для второго взве­
шивания отберём 26 настоящих монет и сравним их с третьей кучкой.
Если 26 настоящих монет тяжелее, значит, фальшивая монета легче
настоящей. Если наоборот, то фальшивая монета тяжелее.
• Если первые две кучки имеют разный вес, то в третьей куч­
ке все 26 монет настоящие. Возьмём из неё 25 настоящих монет
и сравним их с более лёгкой кучкой.
Решение задачи 9. Таких вопросов можно придумать несколь­
ко. Вот два примера.
• У кого-нибудь, кроме тебя, есть две золотые монеты?
• Кому-нибудь заплачено больше, чем тебе?
Занятие 12

Математическая олимпиада I

Решение задачи 1. Предположим, что у мальчика тёмные воло­


сы, а у девочки — рыжие. Тогда и мальчик, и девочка говорят правду.
Но мы знаем, что кто-то из них лжёт. Значит, у мальчика рыжие
волосы, а у девочки — тёмные. Они оба лгут.

Решение задачи 2. См. рисунок.

Решение задачи 3. См. рисунок.

Решение задачи 4. Обозначим монеты буквами А, В, С, D (слева


направо). Известно, что среди этих монет есть хотя бы одна настоя­
щая и хотя бы одна фальшивая. Поскольку все настоящие монеты
лежат справа от всех фальшивых, монета D настоящая, а монета
А —фальшивая. Чтобы определить тип монет В и С, положим их на
правую чашу весов. На левую чашу положим монеты А и D. На левой
чаше — настоящая и фальшивая монета. Значит, если левая чаша тя­
желее, обе монеты В и С фальшивые. Если левая чаша легче, то обе
монеты В и С настоящие. Если весы в равновесии, то среди монет
В и С ровно одна фальшивая; это может быть только монета В, так
как фальшивые монеты находятся слева от настоящих.
Решение задачи 5. Это можно сделать двумя способами.
• Два брата получают по три полных сундука, по одному сундуку,
заполненному наполовину, и по три пустых сундука. Третий брат
получает один полный сундук, пять сундуков, заполненных наполо­
вину, и один пустой сундук.
• Один брат получает три полных сундука, один сундук, запол­
ненный наполовину, и три пустых сундука. Другие два брата полу-
Занятие 12. Математическая олимпиада I 299

чают по два полных сундука, по три сундука, заполненных наполо­


вину, и по два пустых сундука.
Чтобы прийти к решению, заметим, что общий размер наслед­
ства—10,5 сундука золота, а самих сундуков —21. Значит, каждый
должен получить 3,5 сундука золота и 7 сундуков. Если попробо­
вать, например, дать первому брату как можно больше полных сун­
дуков, то он получит три полных сундука, один заполненный на­
половину и три пустых. Тогда остаётся чётное количество сундуков
всех трёх видов, и их можно поровну поделить между другими дву­
мя братьями.
Решение задачи 6. Попробуем расставить детей по порядку в со­
ответствии с размером их добычи. Кто раздобыл меньше всех? Не
Дженни, не Синди и не Питер —он получил больше Майкла. Зна­
чит, меньше всех получил Майкл. У нас недостаточно информации,
чтобы определить, чья добыча больше — Синди или Питера. Но до­
быча Синди больше, чем добыча Майкла, а добыча Дженни больше,
чем добыча Питера. Значит, обе девочки вместе раздобыли конфет
больше, чем оба мальчика вместе.
Для преподавателей. Типичное неверное решение этой задачи заключа­
ется в попытке доказать общий случай, предложив какие-то конкрет­
ные числа. Например, кто-то может рассуждать так: «Предположим, что
у Дженни 20 конфет, у Питера 19, у Синди 18 и у Майкла 17. При таком
результате у девочек конфет больше, чем у мальчиков. Поскольку это
наихудший из всех возможных случаев, девочки точно получили больше
мальчиков».
Почему этот случай наихудший из всех возможных? Такое рас­
суждение не является доказательством.
Решение задачи 7. Заметим, что каждый раз, когда рождаются
крольчата, один из них белый, а другой — серый. Поэтому и количе­
ство белых кроликов, и количество серых кроликов одновременно
увеличиваются на 1.
Предположим, что Великая бабушка белая. Тогда сразу после её
прибытия на остров там был бы 1 белый кролик и 0 серых кроликов.
Поскольку количество белых и серых кроликов всегда увеличива­
ется на 1 одновременно, белых кроликов всё время оставалось бы
на 1 больше, чем серых. Но теперь на острове 40 белых и 40 серых
кроликов. Значит, Великая бабушка не может быть белой.
По той же причине она не может быть серой. Значит, она пятни­
стая.
300 Раздел 4. Решения

Решение задачи 8. Рассмотрев на доске все возможные ходы ко­


ня, проверим, что он всегда переходит на поле другого цвета. (Ещё
можно объяснить это так: ход коня состоит из трёх переходов на
соседнее поле, и каждый переход на соседнее поле меняет цвет.)
Значит, за нечётное число ходов конь перейдёт на поле другого
цвета. Поэтому он не может оказаться на исходном поле.
Решение задачи 9. Это невозможно.
Докажем, что чётность количества горящих лампочек не изме­
нится, что бы ни делал с выключателями директор. Если директор
нажимает на выключатель, соединённый с парой горящих лампо­
чек, то они гаснут. Горящих лампочек становится на две меньше.
Если выключатель соединён с двумя лампочками, которые не горят,
то горящих лампочек становится на две больше. Если горит ровно
одна из двух лампочек, соединённых с выключателем, то количе­
ство горящих лампочек не изменяется. Ни в одном из этих случаев
чётность количества горящих лампочек не изменяется. Поскольку
вначале это количество было равно 1, оно не может стать равным 0.
Для преподавателей. Пример неверного рассуждения: «Если нажимать на
выключатели слева направо, одна лампа всегда остаётся горящей». Воз­
разить можно так: «А вдруг существует какой-то хитроумный способ на­
жимать на выключатели в таком порядке, чтобы все лампы погасли?»
Решение задачи 10. Пронумеруем мешки с монетами: 1,2,3,...,
10. Возьмём одну монету из первого мешка, две из второго, три из
третьего и так далее. Всего мы взяли 1 + 2 + 3 + ... + 10 = 55 монет.
Теперь положим эти монеты на левую чашу весов со стрелкой, а на
правую положим 55 монет из первого мешка. Если монеты в первом
мешке настоящие, то разница в весе (в граммах) будет равна коли­
честву монет, взятых из мешка с фальшивыми монетами (а значит,
и номеру этого мешка). Если монеты в первом мешке фальшивые,
то разница в весе составит 2 + 3 + ... + 10 = 54 грамма. Поскольку
во всех случаях показания стрелки будут различными, мы можем
определить мешок с фальшивыми монетами.

Д О П О Л Н И Т Е Л Ь Н Ы Е ЗАДАЧИ

Решение задачи 11. Килограммовая гиря весит столько же, сколь­


ко четверть головки сыра. Значит, головка весит 4 килограмма.
Решение задачи 12. Сначала отмерим ровно один литр воды.
Для этого можно заполнить 9-литровое ведро и два раза вылить из
Занятие 12. Математическая олимпиада I 301

него 4 литра воды, заполняя 4-литровое ведро. Теперь в 9-литровом


ведре один литр воды.
Выльем этот литр воды из 9-литрового ведра в 4-литровое. Заме­
тим, что для заполнения этого ведра теперь нужно 3 литра воды.
Снова заполним 9-литровое ведро водой из колодца. Будем вы­
ливать воду из 9-литрового ведра в 4-литровое, до тех пор пока оно
не заполнится. Теперь в 9-литровом ведре 9 — 3 = 6 литров воды.
Решение задачи 13. Если сумма двух целых чисел нечётная, зна­
чит, одно из них чётное, а другое — нечётное. Поэтому из любой
пары соседних чисел одно должно быть чётным, а другое — нечёт­
ным. Значит, чётные и нечётные числа чередуются. Но мы знаем,
что нечётное количество чередующихся предметов невозможно рас­
положить в круг. Значит, количество целых чисел чётное.
Решение задачи 14. Поскольку у каждого из 36 велосипедов два
обычных колеса, всего у них 72 обычных колеса. Значит, вспомо­
гательных колёс 110 - 72 = 38. У каждого детского велосипеда два
вспомогательных колеса, поэтому в велопробеге участвует 38 : 2 =
= 19 детей. Мам участвует 36 — 19 = 17.
Решение задачи 15. а) Предположим, что на параллели, где ра­
ботает Марсоход, расставлены километровые столбы, один из кото­
рых находится в месте посадки. Представим себе, что километровые
столбы окрашены в чередующиеся цвета: красный и синий. Будем
считать, что столб в месте посадки красный. За каждый день Мар­
соход переходит к столбу другого цвета. Поэтому вечером каждого
нечётного дня Марсоход останавливается у синего столба, а вече­
ром каждого чётного дня —у красного. Значит, через 24 дня Марсо­
ход будет у красного столба. Но столб, отстоящий от места посадки
на три километра, синий. Следовательно, через 24 дня Марсоход не
может оказаться в трёх километрах от места посадки.
б) Идея решения такая же, как в предыдущем пункте. После ава­
рии цвета километровых столбов, у которых в конце дня останавли­
вается Марсоход, по-прежнему чередуются. Если считать, что ава­
рия произошла у красного столба, то через 30 дней Марсоход снова
окажется у красного столба. Но в трёх километрах от места аварии
находится синий столб. Значит, через 30 дней Марсоход не может
оказаться в трёх километрах к востоку от места аварии.
Решение задачи 16. Присвоим бусинкам номера 1, 2, 3, ..., 34
по часовой стрелке и будем двигаться по кругу, начиная с бусинки 1.
Предположим, что бусинка 1 красная. Мы знаем, что если между
302 Раздел 4. Решения

бусинками ровно две бусинки, то они одного цвета. Между бусинка­


ми 1 и 4 ровно две бусинки: 2 и 3. Значит, бусинка 4 того же цвета,
что и 1. Двигаясь дальше, замечаем, что каждая третья бусинка —
1, 4, 7, 10, ..., 34 —того же цвета. Значит, бусинка 34 красная. Но
между бусинками 34 и 3 ровно две бусинки. Поэтому бусинка 3 то­
же красная. Того же цвета и бусинки 6, 9, ..., 33. Поскольку бусин­
ка 33 красная, бусинка 2 тоже красная. Но бусинки 5, 8,..., 32 того
же цвета, что и бусинка 2. Мы пришли к выводу, что если бусинка 1
красная, то все остальные тоже красные. Если бусинка 1 какого-
нибудь другого цвета, такое же рассуждение показывает, что и все
остальные бусинки того же цвета.
Занятие 13

Знакомство с кубом

Решение задачи для разминки 1. В Ватикан идёт только один


человек — паломник.
Решение задачи для разминки 2. За две стрижки заплатят в два
раза больше, чем за одну.
Решение задачи 1. а) Эту задачу можно решить разными спосо­
бами. Например, разделим 10 монет на такие группы: 3, 3 и 4 мо­
неты. При первом взвешивании сравним первые две группы. Если
они весят одинаково, то фальшивая монета в третьей группе. Если
нет, она в более лёгкой из первых двух групп. Осталось два взвеши­
вания. Если фальшивая монета — в группе из трёх монет, её можно
определить за одно взвешивание (см. предыдущее занятие). Если
она в группе из четырёх монет, разделим эту группу на две пары мо­
нет. Сравнив эти пары, найдём ту из них, в которой есть фальшивая
монета. При третьем взвешивании определим, какая монета в этой
паре фальшивая.
б) Эту задачу можно решить разными способами. Как и в пунк­
те а), разделив все монеты на три группы, можно за одно взвешива­
ние узнать, в какой из них фальшивая монета. Разделим 16 монет,
например, на такие группы: 5, 5 и 6 монет. Определив при первом
взвешивании группу с фальшивой монетой, разделим её на такие
группы: 2, 2 и 1 или 2, 2 и 2. Тогда при втором взвешивании мы
определим либо фальшивую монету, либо пару монет, одна из ко­
торых фальшивая. В последнем случае найдём фальшивую монету
ещё за одно взвешивание.
в) Разделим монеты на три группы по девять монет. При первом
взвешивании определим группу, в которой есть фальшивая монета,
и разделим её на три группы по три монеты. При втором взвеши­
вании найдём группу из трёх монет, одна из которых фальшивая,
а при третьем — саму фальшивую монету.
Решение задачи 2. При первом взвешивании булочник отмеря­
ет один килограмм муки. При втором взвешивании он кладёт на
304 Раздел 4. Решения

первую чашу только что отмеренную муку вместе с килограммовой


гирей. Затем он высыпает муку на вторую чашу, до тех пор пока
весы не придут в равновесие. Так он отмеряет два килограмма муки.
Теперь у него отмерено три килограмма муки. При третьем взве­
шивании он кладёт на первую чашу три килограмма муки и ки­
лограммовую гирю, а затем уравновешивает весы четырьмя кило­
граммами муки. Объединив всю отмеренную муку — 3 килограмма
и 4 килограмма, — булочник получает 7 килограммов.
Решение задачи 3. а), б) и в), но не г).
Решение задачи 4. Первый куб сложить можно.
Сложить второй куб невозможно, так как чёрный и белый круж­
ки должны находиться на противоположных гранях.
Сложить третий куб невозможно по другой причине. На кубе,
сложенном из развёртки, оба треугольника должны указывать в сто­
рону от белого кружка. Но треугольник на передней грани третьего
куба указывает на грань с белым кружком.
Решение задачи 5. См. рисунок.

Решение задачи 6. Главное в этой задаче — заметить, что, ко­


гда гном разрезает квадрат на четыре меньших квадрата, общее ко­
личество квадратов увеличивается на три (вместо одного квадрата
получается четыре). Поэтому, какие бы куски ткани ни разрезали
гномы, после семи разрезаний получится 4 + 7 • 3 = 25 кусков.
Для преподавателей. Типичное неверное решение состоит в том, чтобы
предложить какой-то конкретный способ разрезания. Например, круж­
ковец приносит преподавателю рисунок, где изображён один вариант
разрезания ткани на квадраты. Получается 25 квадратов, значит, по его
словам, правильный ответ — 25. Возражение состоит в том, что гномы
могут выбирать квадраты для разрезания многими способами. Если мы
проверим какой-то один способ (или даже несколько способов) и по­
лучим 25, то отсюда не следует, что и все остальные способы приведут
к тому же результату.
Решение задачи 7. а) Да. Прыгнув два раза вправо и один раз
влево, кузнечик окажется на один сантиметр правее исходной точ­
ки. Прыгнув три раза вправо и два раза влево, он окажется на один
сантиметр левее исходной точки.
Занятие 13. Знакомство с кубом 305

б) Воспользуемся методом, изученным на занятии 10 (чередова­


ние).
Заметим, что все точки возможного приземления кузнечика от­
стоят друг от друга на один сантиметр. Раскрасим эти точки в синий
и красный цвета так, чтобы точки разных цветов чередовались. По­
скольку кузнечик прыгает на 3 или 5 сантиметров, после каждого
прыжка он попадает в точку другого цвета. Другими словами, цвета
точек приземления чередуются. Теперь предположим, что кузнечик
начал прыгать из красной точки. Тогда после второго прыжка и по­
сле любого чётного числа прыжков он приземлится в красной точке.
Но точка, отстоящая от исходной на 1 сантиметр, синяя. Поэтому
кузнечик не может приземлиться в этой точке после чётного числа
прыжков.
в) После 23 прыжков кузнечик приземлится в синей точке. Но
точка, отстоящая от исходной на 10 сантиметров, красная. Значит,
приземлиться в этой точке после 23 прыжков кузнечик не может.
Занятие 14

Сечения

Решение задачи 1. а) 4 на нижней грани, 3 на задней, 6 на левой,


б) 2 на нижней грани, 5 на задней, 4 на левой.
Решение задачи 2. Правильный шестиугольник.
Решение задачи 3. Особенность этого треугольника состоит
в том, что все его стороны—диагонали граней куба. Поскольку все
грани куба — квадраты одинакового размера, все их диагонали равны
по длине. Поэтому все стороны треугольника равны, а значит, и все
его углы равны. Следовательно, каждый угол равен 60 градусам.
Решение задачи 4. Попробуем придумать простой способ под­
счёта кабелей. У каждого кабеля два разъёма — по одному на каж­
дом конце. Что, если вместо кабелей подсчитать разъёмы? К ком­
пьютеру на кухне подсоединено 5 разъёмов, к компьютерам Мак­
са—6, к папиным компьютерам —10, а к маминому—1. Всего по­
лучается 22 разъёма. Зная, что у каждого кабеля два разъёма, опре­
деляем количество кабелей: 11.
Для преподавателей. Типичная ошибка при решении этой задачи состоит
в том, что количество кабелей подсчитываете^ для какой-то конкретной
схемы их подключения. Но откуда мы знаем, что в другой схеме не по­
лучится другое количество кабелей?
Решение задачи 5. На рисунке показано несколько возможных
сечений (в виде двумерных фигур):

Это круг, овал, прямоугольник, овал без одного конца, овал без
двух концов, кончик овала и очень узкий прямоугольник. Читателю
предлагается самостоятельно придумать, как разрезать цилиндр,
чтобы получить такие сечения.
Для преподавателей. Для демонстрации этих сечений могут быть исполь­
зованы огурец или морковь.
Занятие 14. Сечения 307

Решение задачи 6. Сначала вспомним, как разрезать квадрат


на 6, 7 и 8 квадратов меньшего размера:

Теперь предположим, что квадрат уже разрезан на несколько


меньших квадратов. Что будет, если выбрать один из них и разре­
зать его на четыре квадрата? Какой бы квадрат мы ни выбрали,
количество квадратов увеличится на три!
Выполним эту операцию для каждого из приведённых выше ри­
сунков. Получатся квадраты, разрезанные на 9, 10 и 11 квадратов:

Теперь повторим эту же операцию ещё раз: выберем на каждом из


предыдущих рисунков маленький квадрат и разрежем его на четыре
части. Получатся квадраты, разрезанные на 12, 13 и 14 квадратов.
Точно так же можно получить схемы разрезания на 15, 1 6 и 1 7
квадратов. Продолжая в таком же духе, мы можем разрезать квадрат
на любое количество квадратов (большее пяти).
Решение задачи 7. а) Скорее всего, дети придумают конструк­
ции для пункта а) без особых затруднений. Один из возможных ва­
риантов—затычка в форме буквы Y, состоящая из трёх стержней
круглой, квадратной и треугольной формы. Другой вариант — стер­
жень с тремя обычными затычками, расположенными в ряд: первая
в форме треугольной призмы, вторая в форме куба и третья в форме
цилиндра. (Чтобы такая затычка работала, куб должен находиться
в центре.)
б) Требование, чтобы приспособление можно было протолкнуть
внутрь каждого горлышка, существенно усложняет задачу. В этом
случае не подходит ни одна из затычек, предложенных в пункте а).
Это трудная головоломка, и я всегда растягиваю её решение на
два занятия. Разбирая остальные задачи этой подборки, к этой за­
даче я только даю подсказку, приведённую ниже. Окончательный
разбор откладывается до следующего раза.
Подсказка. Начнём процесс конструирования с цилиндрической
затычки. Для удобства обозначим её диаметр буквой d. Такая затыч-
308 Раздел 4. Решения

ка подойдёт к бутылке с цилиндрическим горлышком, и её можно


протолкнуть внутрь такой бутылки.
Теперь сделаем высоту этого цилиндра тоже равной d. Если поло­
жить такой цилиндр на бок и посмотреть на него сверху, мы увидим
квадрат со стороной d. Этот квадрат по форме и размеру соответ­
ствует квадратному горлышку. Значит, такой цилиндр, расположен­
ный горизонтально, может служить затычкой для бутылки с квад­
ратным горлышком. Кроме того, его можно протолкнуть в квадрат­
ное горлышко.
Итак, мы получили затычку, подходящую для бутылок с круглым
и квадратным горлышком. Как изменить её, чтобы она подошла
и к треугольному горлышку?
Решение. Расположим цилиндр вертикально. На верхнем осно­
вании цилиндра проведём диаметр, соединяющий северную и юж­
ную точки окружности. На нижнем основании пометим восточную
и западную точки окружности.
Теперь сделаем два прямых разреза. Первый
разрез должен начинаться на диаметре «север —
юг» верхнего основания и заканчиваться в запад­
ной точке нижнего основания. Второй диаметр
должен начинаться на том же диаметре «север —
юг» и заканчиваться в восточной точке нижнего
основания. Тело, полученное после отрезания двух кусков, обладает
такими свойствами: сверху оно выглядит как круг, с востока или
запада —как квадрат, а с севера или юга —как треугольник. Поэто­
му такая затычка подойдёт к любой из трёх бутылок, и её можно
протолкнуть в горлышко каждой из них.
Для преподавателей. Представить себе трёхмерное тело такой сложной
и необычной формы довольно трудно. Желательно не только объяснить
детям, как строится такая модель, но и продемонстрировать её. Отлич­
ной заготовкой послужит обрезок палки для обучения плаванию. (Такие
палки делаются из вспененного полиэтилена, который удобно резать.)
Лучше принести на занятие не только несколько экземпляров выре­
занной затычки, но и «треугольные» куски, отрезанные от цилиндра.
Для развития пространственного мышления полезно показать, как из
затычки и обрезков составляется цилиндр.
Решение задачи 8. Это сечение выглядит так:
Занятие 15

Математический аукцион II

Решение задачи для разминки 1. Это задача «на внимание».


Водитель автобуса — это вы. Ответ к задаче — ваш возраст.
Решение задачи для разминки 2. Один грабитель должен раз­
делить добычу, а другой — выбрать свою долю.
Решение задачи 1. а) Добраться со второго этажа на первый
можно так:
+ 9 (на 11-й этаж), — 7 (на 4-й этаж), + 9 (на 13-й этаж),
- 7 (на 6-й этаж), 4- 9 (на 15-й этаж), - 7 (на 8-й этаж),
- 7 (на 1-й этаж).
Обратите внимание на то, что на каждом этаже в этом списке мож­
но выбрать только одну кнопку: лифт либо слишком близко к 18-му
этажу, чтобы ехать вверх, либо слишком близко к 1-му, чтобы ехать
вниз.
б) Добраться с первого этажа на второй можно так:
+9 (на 10-й этаж), - 7 (на 3-й этаж), +9 (на 12-й этаж),
- 7 (на 5-й этаж), +9 (на 14-й этаж), - 7 (на 7-й этаж),
+9 (на 16-й этаж), —7 (на 9-й этаж), —7 (на 2-й этаж).
Обратите внимание на то, что почти на каждом этаже в этом списке
можно выбрать только одну кнопку. Обе кнопки могли бы сработать
только на 9-м этаже.
в) Поскольку Марья Петровна начинает с первого этажа, она мо­
жет выбрать маршрут из пункта б). Тогда она вымоет полы на всех
этажах из пункта б) и окажется на 2-м этаже. После этого, нажи­
мая кнопки, как в пункте а), она может побывать на всех этажах из
пункта а) и вернуться на 1-й этаж. Итак, мы получили «замкнутый
контур» из всех этажей, упомянутых в пунктах а) и б). На всех этих
этажах Марья Петровна сможет вымыть полы.
Чтобы найти другие этажи, на которые можно добраться на лиф­
те, рассмотрим «точки разветвления» на этом контуре. Это те этажи,
310 Раздел 4. Решения

на которых у Марьи Петровны есть выбор: подняться на 9 этажей


вверх или спуститься на 7 этажей вниз. Такой этаж только один: 9-й.
Все остальные этажи находятся слишком близко либо к верхнему,
либо к нижнему этажу.
Посмотрим, что получится, если с 9-го этажа поехать не вниз,
а вверх. Поднявшись на 9 этажей, Марья Петровна попадёт на 18-й
этаж. Там работает только кнопка, опускающая лифт на 7 этажей.
Лифт оказывается на 11-м этаже. Но этот этаж уже есть в нашем
списке. Значит, поиск других маршрутов закончен, и мы получили
полный список этажей, на которые может попасть Марья Петров­
на: 2, 11, 4,13, 6, 15, 8,1,10, 3,12, 5, 14, 7,16, 9 и 18. В этом списке
нет только одного этажа —семнадцатого.
Решение задачи 2. В субботу число месяца совпадает с номером
вагона, а в понедельник —меньше его. Значит, в субботу—день от­
правления—число месяца больше, чем в понедельник—день при­
бытия. Такое возможно только в том случае, если в воскресенье
или в понедельник начался новый месяц. Следовательно, понедель­
ник — первое или второе число месяца, и Вася едет в вагоне с номе­
ром 1 или 2. Учитывая, что номер места (положительное целое число)
меньше номера вагона, получаем ответ: вагон №2, место №1.
Решение задачи 3. Если бы 1 июня вместо одной кувшинки по­
явилось две, то в пруду каждый день было бы в два раза больше кув­
шинок, чем в действительности: 2 июня —четыре кувшинки вместо
двух, 3 июня — восемь кувшинок вместо четырёх и так далее. Зна­
чит, пруд полностью зарос бы кувшинками в тот день, когда он на
самом деле зарос наполовину.
Когда пруд зарос наполовину? Поскольку количество кувшинок
удваивается каждый день, это произошло за день до того, как пруд
зарос полностью. Итак, ответ —14 июня.
Решение задачи 4. В магазине было чётное число драконов,
и каждый дракон заказал нечётное количество шляп. Поэтому об­
щее количество заказанных шляп —это сумма чётного количества
нечётных чисел. Такая сумма чётная, а 77 — нечётное число. Значит,
хозяин магазина ошибся.
Решение задачи 5. Первое решение (которое обычно придумы­
вают дети). Денис бежит вниз в два раза быстрее, чем Таня едет
в лифте. Значит, когда Денис добежит до первого этажа, Таня будет
на середине пути (между 10-м и 11-м этажами).
Занятие 15. Математический аукцион II 311

Вверх Денис бежит в два раза медленнее, чем Таня едет на лифте.
Поэтому пока Таня проедет оставшуюся половину пути и окажется
на 1-м этаже, Денис пробежит четверть пути. Начиная с этого мо­
мента они оба движутся вверх. Таня едет в два раза быстрее, поэто­
му они достигнут середины пути одновременно. Теперь им осталось
преодолеть одинаковое расстояние, но Таня едет быстрее и вернётся
на 18-й этаж раньше.
Второе решение (более красивое). Мне кажется, проще такое ре­
шение: Денис бежит вверх в два раза медленнее, чем Таня едет
в лифте. Поэтому он пробежит всю лестницу вверх за такое же
время, за какое Таня проедет на лифте вниз и вверх. Но Денису
надо ещё пробежать лестницу вниз. Поэтому на весь маршрут ему
понадобится больше времени, чем Тане.
Решение задачи 6. Докажем, что надо перевернуть карточки
«4», «Б» и «5», а карточку «А» можно не переворачивать.
• Почему надо перевернуть карточку с цифрой 4? Предположим,
что на обратной стороне этой карточки ничего не написано (или на­
писано что-то отличное от гласной буквы). В этом случае утвержде­
ние Макса неверно. Поэтому необходимо проверить эту карточку.
• Почему надо перевернуть карточку с буквой Б? Предположим,
что на обратной стороне этой карточки написана цифра 2 (или дру­
гое чётное число). В этом случае утверждение Макса неверно. По­
этому необходимо проверить эту карточку.
• Почему надо перевернуть карточку с цифрой 5? Предположим,
что на обратной стороне этой карточки написана цифра 2 (или дру­
гое чётное число). В этом случае утверждение Макса неверно. По­
этому необходимо проверить эту карточку.
• Почему можно не переворачивать карточку с буквой А? Ес­
ли на обратной стороне этой карточки написано чётное число, это
соответствует утверждению Макса. Если же на обратной стороне
написано что-то другое (или ничего не написано), это тоже не про­
тиворечит утверждению Макса: поскольку на карточке нет чётного
числа, утверждение к ней не относится.
Занятие 16

Комбинаторика I

Решение задачи для разминки 1. Буква «т». Она находится в се­


редине слова «пустота».
Решение задачи для разминки 2. Это трудная задача. Вот не­
сколько подсказок.
• Цифры можно перемещать не только по горизонтали, но и по
вертикали.
• Попробуйте приподнять или опустить цифру.
• Обратите внимание на цифру 2.
2
ОТВЕТ: 101-10 = 1.
Решение задачи 1. Для каждой задетой ветки у Ёжика есть три
варианта озвучки. Поэтому для двух веток получается 3-3 = 9 вариан­
тов, для трёх веток — 3-3-3 = 27, для четырёх веток — 3-3-3-3 = 81,
для пяти — 3 • 3 • 3 • 3 • 3 = 243.
Решение задачи 2. В город С можно добраться либо через В,
либо через D. Количество различных маршрутов через В: 3-3. Ко­
личество различных маршрутов через D: 2-2. Следовательно, всего
маршрутов 9 + 4 = 1 3 .
Решение задачи 3. а) Общее количество возможных слов длины
три, составленных из трёх букв: 3 • 3 • 3 = 27. (Если бы мы хотели
проиллюстрировать решение схемой типа «города-дороги», мы бы
расположили в ряд четыре города и соединили каждую пару сосед­
них городов тремя дорогами. Эти три дороги соответствовали бы
буквам А, Б и В.)
б) Слов длины четыре, оканчивающихся буквой Г, ровно столько
же, сколько слов длины три. Почему? Если взять любое слово длины
три и приписать в конце букву Г, то получится одно из таких слов.
Поскольку в фамилиях допускаются все четыре буквы, количество
таких слов длины три равно 4 • 4 • 4 = 64. (Если бы мы хотели проил­
люстрировать решение схемой типа «города-дороги», мы бы распо­
ложили в ряд пять городов. Первые три пары соседних городов со­
единены четырьмя дорогами (буквы А, Б, В и Г), а последняя пара —
одной дорогой (буква Г).)
Занятие 16. Комбинаторика I 313

в) Количество однобуквенных имён: 3. Количество двухбуквен-


ных имён: 3-3 = 9. Количество трёхбуквенных имён: 3 • 3 • 3 = 27.
Значит, всего возможных имён 27 + 9 + 3 = 39.
г) Подсчитаем количество слов длины четыре, которые начина­
ются с буквы Г и не содержат других букв Г. Таких слов столько же,
сколько слов длины три, состоящих только из букв А, Б и В. Почему?
Если взять любое трёхбуквенное слово из букв А, Б, В и приписать
спереди букву Г, то получится одна из таких фамилий. Значит, этих
фамилий 3-3-3 = 27.
Такое же рассуждение показывает, что четырёхбуквенных фами­
лий с единственной буквой Г в конце столько же, сколько трёхбук­
венных слов, состоящих только из букв А, Б и В. Значит, таких фа­
милий тоже 27. Всего фамилий 27 + 27 = 54.
Решение задачи 4. После открытия ворот Ивану понадобятся
три сосиски, чтобы занять ими дракона на время подготовки к от­
лёту. Чтобы определить количество сосисок, гарантирующее Ивану
спокойную работу с воротами, рассмотрим наихудший из возмож­
ных случаев: Иван угадывает секретный код с последней попытки.
а) Количество всех возможных четырёхзначных кодов равно ко­
личеству целых чисел от 0 до 9999: таких чисел 10000. Поэтому
Ивану нужно усыпить бдительность Пашки на 10000 секунд. По­
скольку дракон съедает одну сосиску за 20 секунд, потребуется
10000 : 20 = 500 сосисок.
Следовательно, Ивану нужно взять с собой 500 + 3 = 503 сосиски.
б) Всего нечётных цифр пять: это 1, 3, 5, 7 и 9. Поэтому суще­
ствует ровно 5 • 5 • 5 • 5 • 5 = 625 четырёхзначных кодов, составлен­
ных только из этих цифр. (Объяснить это можно так же, как в зада­
че 3 (а).) Значит, в худшем случае Ивану придётся отвлекать внима­
ние Пашки от ворот 625 секунд. Царевичу надо взять с собой

625 : 20 + 3 = 3 1 ^ + 3 = 34^ сосиски.


Округляя до целого числа, получаем ответ: 35 сосисок.
в) В четырёхзначном коде цифра 5 может находиться на первом,
втором, третьем или четвёртом месте. Подсчитаем коды отдельно
для каждого из этих случаев.
Сначала определим количество четырёхзначных кодов из цифр
1, 3, 5, 7 и 9, в которых на первом месте стоит цифра 5, а других
314 Раздел 4. Решения

цифр 5 нет. Таких кодов столько же, сколько трёхзначных кодов из


цифр 1, 3, 7 и 9. Почему? Если взять любой трёхзначный код из
цифр 1, 3, 7, 9 и приписать спереди 5, получится как раз один из
кодов, которые мы подсчитываем. Итак, таких кодов 4-4-4 = 64.
Теперь подсчитаем четырёхзначные коды из цифр 1, 3, 5, 7 и 9,
в которых единственная цифра 5 стоит на втором месте. Их столько
же, сколько трёхзначных кодов из цифр 1, 3, 7 и 9. Чтобы убедиться
в этом, возьмём любой трёхзначный код из цифр 1, 3, 7, 9 и вставим
цифру 5 между первой и второй цифрами. Следовательно, кодов та­
кого вида тоже 4 • 4 • 4 = 64.
Рассуждая таким же образом, приходим к выводу, что существует
ровно 64 кода с единственной цифрой 5 на третьем месте и ровно 64
кода с единственной цифрой 5 на четвёртом месте. Количество всех
возможных кодов равно 64 + 64 + 64 + 64 = 256. Осталось рассчи­
тать количество сосисок: 256:20 + 3 = 12,8 + 3 = 15,8. Царевич дол­
жен взять с собой 16 сосисок.
Решение задачи 5. Предположим, что Рональд —рыцарь. Тогда
дракона убил Арчибальд. Известно, что дракона убил рыцарь. Зна­
чит, Арчибальд —рыцарь. Но мы знаем, что из трёх друзей только
один рыцарь. Это противоречие показывает, что наше предположе­
ние неверно: Рональд не рыцарь, он лжец.
Поскольку Рональд лжёт, дракона убил не Арчибальд. Значит,
Арчибальд тоже не рыцарь. Следовательно, рыцарь—Дональд. Он
и убил дракона.
Решение задачи 6. Заполним 5-литровый кувшин из 8-литрово­
го. Теперь в 8-литровом кувшине 3 литра масла, а в 5-литровом —
5 литров.
Заполним 3-литровый кувшин из 5-литрового. Теперь в 3-литро­
вом кувшине 3 литра масла, а в 5-литровом —2 литра. Выльем со­
держимое 3-литрового кувшина в 8-литровый, а содер­
жимое 5-литрового кувшина —в 3-литровый. Теперь
в 8-литровом кувшине 6 литров масла, 5-литровый
кувшин пуст, а в 3-литровом кувшине 2 литра.
Заполним 5-литровый кувшин из 8-литрового. Те­
перь в 5-литровом кувшине 5 литров масла, а в 3-лит­
ровом — 2 литра.
Заполним 3-литровый кувшин из 5-литрового. По­
сле этого в 5-литровом кувшине будет 4 литра масла.
Все переливания показаны на схеме справа.
Занятие 16. Комбинаторика I 315

Решение задачи 7. См. рисунок.

Решение задачи 8. Рассмотрим наибольшее число и два сосед­


них с ним числа. (Если таких чисел несколько, выберем любое из
них.) Поскольку это число наибольшее, оно не меньше каждого из
соседних чисел. С другой стороны, это число — среднее арифметиче­
ское соседних чисел и, значит, по величине находится между ними:
не больше наибольшего из них и не меньше наименьшего. Эти два
условия могут быть соблюдены только в одном случае: когда все три
числа равны. Значит, наибольшее число в этом круге равно каждому
из соседних чисел.
Но тогда каждое число, соседнее с наибольшим, само является
наибольшим. Значит, как мы уже выяснили, оно равно обоим чис­
лам, соседним с ним. Двигаясь по кругу и рассуждая точно так же,
убеждаемся в том, что все числа равны.
Решение задачи 9. Докажем, что количество чёрных клеток все­
гда остаётся нечётным.
• Заметим, что на исходном рисунке нечётное число чётных кле­
ток (оно равно 1).
• Покажем, что при перекрашивании горизонтального или вер­
тикального ряда чётность числа чёрных клеток не изменяется. Сна­
чала предположим, что ряд состоит из четырёх чёрных клеток. То­
гда при перекрашивании все они становятся белыми. Количество
чёрных клеток уменьшается на 4. Пусть теперь в ряду три чёрные
клетки и одна белая. При перекрашивании три клетки становятся
белыми, а одна —чёрной. Количество чёрных клеток уменьшается
на 2. Точно так же рассматриваем остальные случаи. Если в ряду
две чёрные и две белые клетки, то при перекрашивании количество
чёрных клеток не изменяется. Если в ряду только одна чёрная клет­
ка, то количество чёрных клеток увеличивается на 2. Наконец, если
весь ряд белый, то количество чёрных клеток увеличивается на 4.
316 Раздел 4. Решения

• Поскольку количество чёрных клеток может уменьшаться или


увеличиваться только на 2 или 4, его чётность не изменяется. Следо­
вательно, какие бы команды ни выполнял робот, количество чёрных
клеток останется нечётным. 16 — чётное число, поэтому сделать все
16 клеток чёрными невозможно.
Занятие 17

Комбинаторика II

Решение задачи для разминки 1. Попугай был глухим.


Решение задачи для разминки 2. Стасу не хватает трёх рублей
на одно мороженое. После того как он скинулся с Тарасом, ему по-
прежнему не хватает трёх рублей на одно мороженое. Значит, у Та­
раса нет денег. Стало быть, мороженое стоит 35 рублей.
Решение задачи 1. Это типичная задача на «размещения без по­
вторений». Решим её по правилу заполнения пропусков. У нас есть
четыре пропуска: они должны быть заполнены именами рыцарей,
сбитых с коня первым, вторым, третьим и четвёртым. Первый про­
пуск можно заполнить четырьмя способами; второй —тремя спо­
собами, так как один рыцарь сбит первым ударом; третий—двумя
способами, так как на конях остались два рыцаря; четвёртый —
только одним способом.
ОТВЕТ: 4-3-2-1 = 24.
Решение задачи 2. а) Это задача на «размещения с повторени­
ями». Каждой из четырёх букв слова соответствует один пропуск.
Каждый пропуск можно заполнить шестью способами.
ОТВЕТ: 6- 6- 6- 6 = 1296.
б) Это задача на «размещения без повторений». Первый пропуск
(первая буква в слове) можно заполнить шестью способами, вто­
рой — пятью способами и так далее. ОТВЕТ: 6 • 5 • 4 • 3 = 360.
в) Все сочетания четырёх марсианских букв можно разделить
на две группы: те, в которых хотя бы одна буква повторяется, и те,
в которых нет повторяющихся букв. Ответ к пункту а) даёт нам об­
щее количество четырёхбуквенных сочетаний, а ответ к пункту б) —
количество сочетаний без повторяющихся букв. Значит, количе­
ство сочетаний, в которых хотя бы одна буква повторяется, равно
1296-360 = 936.
Решение задачи 3. План Тимофея. Нечётных цифр всего пять: 1,
3, 5, 7, 9. Поэтому из нечётных цифр можно составить 5 однознач­
ных, 5 • 5 двузначных и 5 • 5 • 5 трёхзначных номеров. Если выбрать
318 Раздел 4. Решения

план Тимофея, выигрышных номеров будет 5 + 25 + 125 = 155. По­


скольку приз составляет 700 рублей, по выигрышным билетам при­
дётся выплатить 155 • 700 = 108500 рублей. Это даже больше, чем
выручка от продажи билетов (100000 рублей).
План Саши. Подсчитать количество выигрышных билетов (окан­
чивающихся на 1) можно разными способами.
• Долгий способ. Отдельно подсчитаем количество выигрышных
билетов с однозначными, двузначными и трёхзначными номерами.
Однозначный выигрышный билет только один: это билет с номе­
ром 1. Двузначных выигрышных билетов девять (так как в разряде
десятков выигрышного номера может стоять цифра 1,2,..., 9). Ко­
личество трёхзначных выигрышных билетов подсчитаем по прави­
лу заполнения пропусков. У нас есть 9 вариантов заполнения про­
пуска для разряда сотен (1,2, ...,9) и 10 вариантов —для разряда
десятков (0,1, 2,..., 9). Значит, трёхзначных выигрышных номеров
9 • 10 = 90, а всего выигрышных номеров 1 + 9 + 90 = 100. Если
каждый победитель получит 800 рублей, то по выигрышным биле­
там будет выплачено 100 • 800 = 80 000 рублей. Прибыль от лотереи
в этом случае составит 20 000 рублей.
• Быстрый способ. Заметим, что каждый номер билета до 999
(билет с номером 1000 точно не выигрышный) можно предста­
вить трёхзначным числом. Например, 1 — это 001, 5 — это 005, 31 —
это 031. Тогда выигрышный номер —это трёхзначное число, в кото­
ром цифра единиц равна 1, а цифру десятков и цифру сотен можно
выбрать десятью способами. Найдём количество выигрышных би­
летов по правилу заполнения пропусков: 10 • 10 = 100.
План Антона. Подсчитать количество выигрышных билетов мож­
но разными способами. Пожалуй, проще всего представить каждый
номер от 0 до 999 трёхзначным числом. Тогда мы можем отдельно
подсчитать количество билетов с единственной цифрой 3 в разряде
сотен, в разряде десятков и в разряде единиц. Во всех трёх случаях
каждую из двух неизвестных цифр можно выбрать девятью способа­
ми (0,1,2, 4, 5, 6, 7,8, 9). Значит, количество выигрышных билетов
равно 9-9 + 9-9 + 9-9 = 243. Если выбрать план Антона, прибыль от
лотереи составит 100 000 - 300 • 243 = 27100 рублей.
Итак, наибольшую прибыль принесёт план Антона.
Решение задачи 4. За 15 минут медвежонок с медведем съедят
три бочонка мёда: сын съест один бочонок, а папа—два. Значит,
один бочонок они съедят за 15:3 = 5 минут.
Занятие 17. Комбинаторика II 319

Решение задачи 5. Сначала заметим, что каждый квадрат 2 x 2


содержит центральную клетку квадрата 3 x 3 . Значит, количество
ходов должно быть равно числу в центральной клетке: 19.
Теперь обратим внимание на то, что каждый квадрат 2 x 2 содер­
жит ровно одну угловую клетку квадрата 3 x 3 . Поэтому количество
ходов должно быть равно сумме чисел в четырёх угловых клетках.
Но эта сумма равна 3 + 7 + 5 + 6 = 21.
Подсчитав количество ходов двумя способами, мы получили раз­
ные ответы. Следовательно, такой таблицы получиться не может.
Решение задачи 6. Предположим, что ни один из последних
25 матчей не закончился вничью. Тогда сумма числа побед и числа
поражений равна 25. Значит, одно из этих чисел чётное, а другое
нечётное. При этом разность числа побед и числа поражений равна
количеству очков, полученных за эти матчи. Следовательно, коман­
да получила нечётное число очков. Но по условию задачи команда
получила 20 — 10 = 10 очков — чётное число. Это показывает, что на­
ше предположение неверно и хотя бы один матч закончился вничью.
Решение задачи 7. а) Числа 4, 2, 1, 1. Пожалуй, проще всего
прийти к этому ответу методом проб и ошибок.
б) У этой задачи есть несколько ответов. Покажем, как найти
один из них.
Пункт а) позволяет заметить важное свойство: если к набору чи­
сел добавить число 1, то произведение этих чисел не изменяется,
а сумма увеличивается на 1. На этом основана идея решения: взять
два числа, которые больше единицы, и достаточное количество еди­
ниц, чтобы при их прибавлении к этим двум числам получилось их
произведение. Поскольку всего чисел должно быть 1000, надо взять
998 единиц. Значит, произведение первых двух чисел должно быть
на 998 больше их суммы. Чтобы упростить поиск таких чисел, вве­
дём дополнительное ограничение: предположим (основываясь на
ответе к пункту а)), что одно из этих чисел равно 2. Итак, надо най­
ти число, удвоенное значение которого на 998 больше, чем сумма
этого числа и числа 2. Это число 1000. Получаем один из возможных
ответов: 2, 1000 и девятьсот девяносто восемь единиц.
Занятие 18

Математический хоккей II

Решение задачи для разминки 1. Офицер умер мгновенно. По­


этому его жена не могла знать, что именно ему приснилось перед
смертью.
Решение задачи для разминки 2. Путешественник говорит, что
он будет скормлен крокодилам. Если вождь считает, что путеше­
ственник сказал правду, то его должны сбросить со скалы. Но тогда
утверждение путешественника неверно. Если вождь полагает, что
путешественник солгал, то его должны скормить крокодилам. Тогда
получается, что путешественник сказал правду. Поэтому вождь не
может решить, сказал путешественник правду или солгал.

Решение задач математического хоккея


Решение задачи 1. Ира съела конфету, Света — пряник, а Ната­
ша — зефир.
Решение задачи 2. 26 лет.
Решение задачи 3. 20.
Решение задачи 4. 6-5 + 4 - 2 - 3 + 1 = 30.
Решение задачи 5. 1,5 килограмма.
Решение задачи 6. 6 способов.
Решение задачи 7. Бабушка.
Решение задачи 8. 5 проигрышей.
Решение задачи 9. 20 треугольников.
Решение задачи 10. 7 минут.
Решение задачи 11. 5 • 4 • 3 = 60.
Решение задачи 12. 7 шаров.
Решение задачи 13. Пятница.
Решение задачи 14.10 лет.
Решение задачи 15. 999 + 22 = 1021.
Решение задачи 16. 4 • 3 • 2 • 1 = 24.
« -« 60
Решение задачи 17. -о.
Решение задачи 18. 20 минут.
Занятие 18. Математический хоккей II 321

Решение задач из подборки

Решение задачи 1. См. рисунок.

Решение задачи 2. Это задача на размещения без повторений.


Надо заполнить четыре пропуска: для первой книги (4 варианта),
для второй (3 варианта), для третьей (2 варианта) и для четвёртой
(1 вариант). ОТВЕТ: 4 • 3 • 2 • 1 = 24.
Решение задачи 3. Эта задача отлично иллюстрирует принцип:
прежде чем действовать методом подбора, лучше немного подумать.
Где может стоять первый минус? Если вставить его между 7 и 8,
то первым числом в выражении будет 8, и тогда значение всего вы­
ражения наверняка получится отрицательным. Если же поставить
первый минус правее цифры 6, то первое число будет не меньше
чем 876. При таком большом первом числе значение всего выраже­
ния тоже будет слишком большим. Значит, первый минус должен
стоять между 7 и 6.
Теперь попробуем определить место второго минуса. Может ли
вторым числом быть 65? Очевидно, нет. Разность чисел 87 и 65 рав­
на 22, а из чисел 22, 4, 3, 2 и 1 невозможно получить 3 с помощью
минусов. Достаточно проверить два варианта: 22 — 4 — 3 — 2 — 1
и 2 2 - 4 - 3 - 2 1 . В обоих случаях значение выражения не равно 3.
Значит, второй минус должен стоять между 6 и 5.
Рассуждая подобным образом, определяем положение третьего
минуса: он может быть только между 4 и 3. Место последнего ми­
нуса не вызывает сомнений. ОТВЕТ: 8 7 - 6 - 5 4 — 3 — 21.
Решение задачи 4. Первое взвешивание. Сравним 100-граммовый
и 200-граммовый слитки с 300-граммовым.
• Если весы находятся в равновесии, то слитки с маркировкой
100, и 200 и 300 граммов настоящие. Значит, фальшивый слиток —
500-граммовый. В этом случае нам удалось обойтись одним взвеши­
ванием.
• Теперь предположим, что одна из чаш перевешивает. Тогда
один из слитков с маркировкой 100, и 200 и 300 граммов фальши­
вые, а 500-граммовый слиток настоящий. Для определения фаль­
шивого слитка понадобится второе взвешивание.
322 Раздел 4. Решения

Второе взвешивание. Сравним 200-граммовый и 300-граммовый


слитки с 500-граммовым.
• Если весы находятся в равновесии, то слитки с маркировкой
200, и 300 и 500 граммов настоящие. Значит, фальшивый слиток—
100-граммовый.
• Что, если весы не уравновешены? Мы уже знаем, что 500-грам­
мовый слиток настоящий. Значит, фальшивым должен быть слиток
с маркировкой 200 или 300 граммов. Если чаша с 500-граммовым
слитком перевешивает, то фальшивое золото легче настоящего. Ес­
ли перевешивает другая чаша, то фальшивое золото тяжелее.
Сначала рассмотрим случай, когда фальшивое золото легче на­
стоящего. Чтобы определить фальшивый слиток, вспомним результат
предыдущего взвешивания. Тогда на одной чаше весов лежали два
слитка —100-граммовый (настоящий) и 200-граммовый, а на дру­
гой — 300-граммовый. Из двух подозрительных слитков — 200-грам­
мового и 300-граммового — фальшивый тот, который лежал на бо­
лее лёгкой чаше.
Теперь предположим, что фальшивое золото тяжелее настояще­
го. В этом случае фальшивый слиток —тот, который при первом
взвешивании лежал на более тяжёлой чаше.
Решение задачи 5. Когда жрец проходит между двумя горящи­
ми лампадами, они гаснут. Значит, горящих лампад становится на две
меньше. Когда он проходит между двумя лампадами, которые не горят,
горящих лампад становится на две больше. Когда он проходит между
двумя лампадами, одна из которых горит, а другая нет, количество
горящих лампад не изменяется. Значит, как бы ни двигался жрец, чёт­
ность количества горящих лампад не изменяется. Вначале горели две
лампады (чётное число). Поэтому количество горящих лампад всегда
остаётся чётным. Жрец не сможет зажечь все 13 лампад.
Занятие 19

Числовые ребусы I. Потерянные цифры

Решение задачи для разминки 2. Прибыль купца составила


2 рубля.
Решение задачи 1. Если придут 3 ведьмы или 5 ведьм, то Шмер-
лину надо купить 15 лягушек, так как 15 —наименьшее положи­
тельное целое число, которое делится и на 3, и на 5 (наименьшее
общее кратное этих чисел).
Если придут 4 ведьмы или 6 ведьм, то Шмерлину надо купить
12 лягушек, так как 12 —наименьшее общее кратное чисел 3 и 5.
Решение задачи 2. а) Джек взял с собой самородки стоимо­
стью 1, 2 и 4 доллара. Любое целое число от 1 до 7 можно составить
из этих трёх чисел: 3 = 2 + 1, 5 = 4 + 1, 6 = 4 + 2, 7 = 4 + 2 + 1.
б) Джеку надо взять с собой самородки стоимостью 1, 2, 4 и 8
долларов. Из пункта а) мы знаем, что любую сумму от 1 до 7 долла­
ров можно уплатить самородками стоимостью 1, 2 и 4 доллара. Те­
перь заметим, что любое число от 9 до 15 можно представить в виде
суммы числа 8 и какого-то числа от 1 до 7. Следовательно, с помо­
щью этих четырёх самородков можно уплатить любую сумму от 1
до 15 долларов.
Решение задачи 3. а) Цифрой сотен в верхнем числе может
быть только 9: иначе это слагаемое было бы равно самое большее
893 и сумма не могла бы быть больше 993. Зная первое слагаемое,
нетрудно восстановить цифры второго: 993 + 58 = 1051.
б) Заметим, что произведение цифры 9 и цифры единиц во вто­
ром множителе оканчивается цифрой 7. Значит, цифрой единиц во
втором множителе может быть только 3. Если бы второй множи­
тель начинался с цифры 2 или большей, то произведение этих двух
множителей было бы не меньше 29-20 = 580. Но произведение —
трёхзначное число с первой цифрой 3, т. е. не больше 399. Следова­
тельно, второй множитель равен 13.
в) Эта задача немного сложнее. Рассмотрим двузначное число
в третьей строке. Это число должно оканчиваться той же цифрой,
324 Раздел 4. Решения

что и окончательный результат, — цифрой 6. С другой стороны, оно


получается умножением первого множителя (двузначного числа)
на 8. Только три двузначных числа при умножении на 8 дают дву­
значное число: это 10, 11 и 12. Какое из них мы выберем? Произ­
ведение этого числа и 8 должно оканчиваться цифрой 6. Следова­
тельно, первый множитель равен 12. Зная первый множитель, легко
восстановить неизвестную цифру второго: это 2.
Решение задачи 4. а) Обозначим мешочки буквами А, В и С.
При первом взвешивании сравним А и В. Пусть, например, А легче
В. Каким может быть вес мешочка С? Он либо больше, чем вес А
и вес В, либо меньше их обоих, либо находится между ними. При
втором взвешивании положим на одну чашу весов С, а на другую —
более лёгкий из мешочков А и В (в данном случае А). Если С легче
А, то задача решена: С легче А и А легче В. Если С тяжелее А, то при
третьем взвешивании сравним С и В. Если С тяжелее В, то С — са­
мый тяжёлый мешочек. Если С легче В, то С имеет промежуточный
вес между А и В.
б) Обозначим мешочки буквами А, В, С и D. Сначала располо­
жим в порядке возрастания веса мешочки А, В и С. Для этого, как
показано в пункте а), нам понадобятся три взвешивания. Пусть, на­
пример, А < В < С. Осталось за два взвешивания определить поло­
жение мешочка D. Сначала сравним D с мешочком среднего веса —
в данном случае с В. Предположим, что D тяжелее. Теперь сравним
D и С. Если D легче С, то D имеет промежуточный вес между В и С.
Если D тяжелее С, то D — самый тяжёлый из всех мешочков. Если же
предположить, что D легче, чем В, то будем сравнивать D и А.
Решение задачи 5. Обозначим монеты так: Бг и Б2 — бронзо­
вые, Сг и С2 — серебряные, Зг и 3 2 — золотые.
При первом взвешивании сравним пару монет Сг и Бг с парой
С2 и 3 2 . Из двух монет Сг и С2 одна тяжёлая, а другая лёгкая. Поэто­
му в зависимости от веса Бг и 3 2 возможны три разных результата
взвешивания.
1. Сг и Бг легче, чем С2 и 3 2 . В этом случае мы можем утверждать,
что
— Сг легче, чем С2;
— Бг не тяжелее, чем 3 2 (возможны три варианта: Бг легче, чем 3 2 ;
Бг и 3 2 обе лёгкие; Бг и 3 2 обе тяжёлые).
Почему это так? Если бы монета Сг была тяжёлой, то на правой
чашке у нас было бы не больше чем лёгкая и тяжёлая монеты, и то-
Занятие 19. Числовые ребусы I. Потерянные цифры 325

гда левая чашка не могла бы быть легче правой. Значит, монета Сг


лёгкая. Если Бг тяжёлая, то 3 2 тяжёлая. Если Бг лёгкая, то 3 2 может
быть тяжёлой или лёгкой.
2. Сг и Бг тяжелее, чем С2 и 32. В этом случае мы можем утвер­
ждать, что
— Cj тяжелее, чем С2;
— 3 2 не тяжелее, чем Ба (возможны три варианта: Б2 тяжелее
чем 3 2 ; Бх и 3 2 обе лёгкие; Ба и 3 2 обе тяжёлые).
3. Сг и Бг весят столько же, сколько С2 и 3 2 . В этом случае либо
Бг лёгкая и 3 2 тяжёлая, либо наоборот.
При втором взвешивании возможны следующие результаты.
1. Если Сг и Бг легче, чем С2 и 32, то у нас три возможности для
пары Бг и 3 2 : лёгкая+лёгкая, лёгкая + тяжёлая, тяжёлая + тяжёлая,
а пара Сг и С2 точно лёгкая + тяжёлая. Поэтому сравним пару Ба и 3 2
с парой Сг и С2. Если они равны, то монета Ба лёгкая, а 3 2 — тяжёлая.
Если Ба и 3 2 вместе легче, чем Сг и С2, то Бг и 3 2 обе лёгкие. Если
Бг и 3 2 тяжелее, то они обе тяжёлые.
2. Если С1иБ1 тяжелее, чем С2 и 32, то мы рассуждаем так же,
как и в предыдущей ситуации.
3. Если Сг и Б1 весят столько же, сколько С2 и 32, то сравним
монеты Бг и 3 2 . Тогда мы узнаем, какая из них лёгкая (фальшивая)
и какая тяжёлая (настоящая). Например, если Бг тяжелее, то она
настоящая, а 3 2 фальшивая. Тогда Сг фальшивая, С2 настоящая.
Занятие 20

Числовые ребусы П.
Зашифрованные примеры

Решение задачи для разминки 1. Надо взять второй стакан


и перелить из него воду в пятый.
Решение задачи для разминки 2. Надо бросить мяч вертикаль­
но вверх.
Решение задачи 1. а) Сначала найдём цифру десятков во втором
множителе. Если бы это была цифра 2 или больше, то произведение
было бы не меньше 10-20 = 200. Но число в правой части —самое
большее 199. Значит, это цифра 1, и второй множитель равен 11.
Теперь определим цифру единиц в первом множителе. Для того
чтобы произведение первого множителя и 11 оканчивалось циф­
рой 1, первый множитель должен оканчиваться цифрой 1.
ОТВЕТ: 11-11 = 121.
б) Давайте перепишем этот пример по-другому: ***7 = *** + 8.
Тогда получается, что в правой части мы складываем трёхзначное
и однозначное число и получаем четырёхзначное число (в левой ча­
сти). Но сумма в правой части не может быть больше чем 999 + 8 =
= 1007. Значит, число в левой части обязательно равно 1007. Из
этого следует, что неизвестное число в правой части равно 999.
ОТВЕТ: 1007-999 = 8.
Решение задачи 2. Число 18 должно делиться на количество
зверят. Возможные варианты: 18, 9, 6, 3, 2 и 1.
Решение задачи 3. а) В этом примере сумма двух четырёхзнач­
ных чисел — шестизначное число. Но даже если сложить два самых
больших четырёхзначных числа, получится 2 • 9999 = 19 998 — пяти­
значное число. Следовательно, эта задача не имеет решения.
б) Заметим, что NICE —ICE = N000. Это четырёхзначное число!
Решение задачи 4. а) Заметим, что А= 1, так как сумма двух
трёхзначных чисел не может быть больше 1998. Теперь посмот­
рим на разряд сотен, где к неизвестному числу Е прибавляется 1
и, возможно, ещё 1 переносится из разряда десятков. В результа-
Занятие 20. Числовые ребусы II. Зашифрованные примеры 327

те получается самое меньшее 10 (так как есть перенос единицы


в разряд тысяч). Следовательно, Е может обозначать только 8 или 9.
Теперь быстрее всего просто проверить оба варианта. Подставив 8
вместо Е, мы не сможем прийти к ответу. А при Е = 9 он получается:
А=1,Е = 9,Н = 0.
б) Поскольку при сложении в разряде тысяч получается перенос,
0 = 1. Зная это, рассматриваем сложение в разряде сотен и прихо­
дим к выводу, что S = 2 или S = 3. Но в разряде единиц сумма А+А
оканчивается цифрой S, поэтому S —чётная цифра. Значит, S = 2.
Сумма А + А может оканчиваться цифрой 2 только в двух случаях:
А = 1 или А = 6. Но цифра 1 уже занята буквой О, поэтому А = 6.
Остальные цифры восстанавливаются без труда: 1 = 5, R = 8.
Решение задачи 5. При умножении двузначного первого мно­
жителя на 8 получается двузначное число (записанное в четвёртой
строке). Какие двузначные числа при умножении на 8 дают двузнач­
ное число? Только 10, 11 и 12. Какое из них может быть первым
множителем?
Только 12, и вот почему. При умножении этого же числа на по­
следнюю цифру второго множителя получается трёхзначное число.
Но если умножить 10 или 11 даже на 9 —самое большое однознач­
ное число, — то получится двузначное число. Итак, мы знаем пер­
вый множитель: это 12.
Заодно мы узнали последнюю цифру второго множителя: един­
ственная цифра, дающая при умножении на 12 трёхзначное чис­
ло, —это 9. Второй множитель равен 89.
Решение задачи 6. Сумма одинаковых слагаемых — это произ­
ведение. Надо найти наибольшее возможное значение второго мно­
жителя (количество «дубов»).
Сначала заметим, что число РОЩА не может быть больше 9876
(наибольшее четырёхзначное число, в котором все цифры разные).
Точно так же число ДУБ не может быть меньше 102 (наименьшее
трёхзначное число с разными цифрами). Разделив первое число на
второе, получаем 96,8. Следовательно, второй множитель (количе­
ство «дубов») не может быть больше 96.
Но оказывается, что 96 тоже не подходит. Почему? Если «дубов»
96, число ДУБ может быть равно только 102, 103 или 104, потому
что 105 • 96 —уже пятизначное число. Проверим каждый из этих ва­
риантов: 102-96 = 9792, 103-96 = 9888, 104-96 = 9984. Ни один из
них не подходит, так как в числе РОЩА нет повторяющихся цифр.
328 Раздел 4. Решения

Убедившись, что «дубов» не может быть 96, попробуем число 95.


В этом случае число ДУБ может быть равно 102, 103, 104 или 105.
Найдём соответствующие значения числа РОЩА: 102 • 95 = 9690,
103 • 95 = 9785, 104 • 95 = 9880 и 105 • 95 = 9975. Только одно из
полученных значений (9785) состоит из разных цифр, причём все
они отличаются и от цифр числа ДУБ. Итак, дубов может быть 95:
в этом случае ДУБ = 103 и РОЩА = 9785.
Решение задачи 7. Приведём схему решения. Легко видеть, что
М = 1 и О = 0. Рассмотрев сложение в разряде сотен, можно опреде­
лить, что N = Е + 1 , а переноса из разряда сотен нет. Следовательно,
S = 9. Теперь перейдём к разряду десятков. Зная, что N = Е + 1, a R
обозначает не 9, делаем вывод: N = 8, и есть перенос из столбца
единиц. Теперь попробуем угадать цифры D, Е и Y. Поскольку сумма
цифр D и Е больше 9, а цифры Е и N соседние, возможен только один
вариант: D = 7, Е = 5, N = 6, Y=2. ОТВЕТ: 9567+1085 = 10652.
Занятие 21

Математическая олимпиада II

Решение задачи 1. См. рисунок.

Решение задачи 2. Красных шаров у Макса не больше двух.


(Иначе белых шаров было бы не меньше 2 • 3 = 18, а всего шаров
было бы не меньше 3 + 18 = 21.) Итак, у нас только три варианта:
красных шаров может быть 0, 1 или 2.
• Если у Макса 0 красных шаров, то у него 0 белых шаров. Тогда
чёрных шаров 20. Этого не может быть: известно, что чёрных шаров
меньше, чем белых.
• Если у Макса 1 красный шар, то у него 6 белых шаров. Тогда
чёрных шаров 20 — 6 — 1 = 13. Это снова противоречит условию, что
чёрных шаров меньше, чем белых.
• Если у Макса 2 красных шара, то у него 12 белых шаров. Тогда
чёрных шаров 20 - 12 — 6 = 6. Это верный ответ, так как 6 меньше,
чем 12.
Решение задачи 3. См. рисунок.

Решение задачи 4. Эту задачу можно решить с помощью урав­


нения. Но большинство пятиклассников ещё не набили руку в со­
ставлении уравнений. Поэтому мы прибегаем к логическим рассуж­
дениям и рисункам. В действительности наше решение представля­
ет собой то же уравнение, только разобранное подробно и наглядно.
330 Раздел 4. Решения

Примем за единицу длины расстояние, которое Саша проходит


за минуту. Назовём момент времени, когда Тима вышел из автобу­
са, «моментом М», а точку в пространстве, где он впервые увидел
Сашу из окна, — «точкой Z».
На каком расстоянии от точки Z были мальчики в момент М?
Известно, что Тима бежал до точки Z четыре минуты и что он бежит
в два раза быстрее, чем идёт Саша. Следовательно, он вышел из
автобуса на расстоянии 8 единиц от точки Z. Саша в момент выхода
Тимы из автобуса был на расстоянии одной единицы от точки Z
(в противоположном направлении), так как Тима вышел через ми­
нуту после того, как увидел Сашу. Итак, в момент М ситуация была
такой (единицы показаны отрезками):
Саша Тима

Теперь мы знаем, что в момент М Тиму и Сашу разделяло 9 еди­


ниц. Тима двигается вдвое быстрее Саши; значит, к моменту их
встречи он преодолеет вдвое большее расстояние:

расстояния

расстояние
Таким образом, 9 единиц —это половина расстояния, которое
Тиме придётся бежать, чтобы догнать Сашу. К моменту встречи
Саша пройдёт 9 единиц, а Тима пробежит 18 единиц. Это значит,
что Тима догонит Сашу через 9 минут, если считать от момента,
когда Тима добежал до точки Z, и 13 минут с того момента, когда
он вышел из автобуса.
Решение задачи 5. Эта задача опять-таки решается либо урав­
нением, либо с помощью рассуждений и рисунков. Мы пользуемся
вторым способом.
Попробуем представить вес каждого животного на схеме. Жира-
фик весит 100 килограммов (первая строка схемы). Бегемотик ве­
сит 100 килограммов и ещё половину веса слонёнка. Теперь изобра­
зим вес слонёнка. Он равен сумме весов жирафика (100 кг) и беге-
мотика (100 кг и половина веса слонёнка). Последняя строка схемы
приводит нас к решению. Поскольку слонёнок весит 200 кг и ещё
Занятие 21. Математическая олимпиада II 331

половину собственного веса, половина его веса равна 200 кг. Зна­
чит, слонёнок весит 400 кг, а бегемотик —300 кг.

жирафик бегемотик

Решение задачи 6. а) Сначала выберем по­


ложение чёрной ладьи. Её можно поставить на
любую из 64 клеток доски. Теперь посмотрим,
куда можно поставить белую ладью. В любой
позиции чёрная ладья не позволяет белой ла­
дье стоять на пятнадцати клетках (на всех клет­
ках своей горизонтали и своей вертикали). Зна­
чит, где бы ни находилась чёрная ладья, для белой ладьи есть
64 —15 = 49 вариантов расположения. ОТВЕТ: 64 • 49.
б) Как изменится ответ к предыдущей задаче, если ладьи одно­
го цвета? Сначала рассмотрим какое-то конкретное положение ла­
дей. Пусть, например, они занимают угловые клетки: левую ниж­
нюю и правую верхнюю. В этом случае, если ладьи разных цветов,
их можно расставить двумя способами: белую ладью можно поста­
вить либо в левый нижний угол, либо в правый верхний. Если же
ладьи одного цвета, то они неразличимы, и поэтому в таком поло­
жении есть только один вариант их расстановки. Этот конкретный
пример показывает, что любому способу расставить пару одноцвет­
ных ладей соответствуют два способа расставить пару разноцвет­
ных ладей. Поэтому ответ к задаче б) равен ответу к задаче а), де­
лённому на два: 64 • 49:2.
Решение задачи 7. Рассмотрим клетку в левом нижнем углу
таблицы. Она находится в одном столбце с цифрами 2 и 3, в одной
строке с цифрой 4 и на одной диагонали с цифрой 5. Значит, в этой
клетке могла стоять только цифра 1.
Записав в левой нижней клетке цифру 1, можно определить циф­
ру в центральной клетке. Эта клетка находится на пересечении двух
диагоналей, а на этих диагоналях есть цифры 1, 3, 4 и 5. Значит,
в центральной клетке была написана цифра 2.
332 Раздел 4. Решения

Решение задачи 8. На левом рисунке показан разрез, на пра­


вом — новый флаг.

Решение задачи 9. а) Нетрудно придумать последовательность


ударов, после которых останется только одна голова.
б) Докажем, что если у Змея Горыныча осталась одна голова, то
она обязательно нормальная.
Удары меча, отрубающие одну голову, рассматривать не будем:
они ничего не изменяют. Если одним ударом отрублены две головы,
то возможны три варианта. Они приведены в следующей таблице.

Отрубленные
Изменение числа Изменение числа
Выросшие
головы
нормальных сумасшедших
головы
голов голов
2 нормальные, 0 нормальных,
О сумасшедших 1 сумасшедшая -2 +1
0 нормальных, 0 нормальных,
2 сумасшедшие 1 сумасшедшая 0 -1
1 нормальная, 1 нормальная,
1 сумасшедшая 0 сумасшедших 0 -1

Из таблицы видно, что при каждом ударе меча количество нор­


мальных голов либо не изменяется, либо уменьшается на 2. Поэто­
му оно всегда остаётся нечётным и не может стать равным нулю.
Значит, если у Змея Горыныча осталась только одна голова, то она
обязательно нормальная.
Решение задачи 10. Сначала подсчитаем общее количество бли­
нов, поглощённых братьями за три дня: 25 4- 37 + 30 = 92. Что озна­
чает это число? Это количество блинов, съеденных Ваней в поне­
дельник и в среду, Даней в понедельник и во вторник и Колей во
вторник и в среду. Поскольку каждый мальчик всегда съедает од­
но и то же количество блинов, это число составляется из удвоен-
Занятие 21. Математическая олимпиада II 333

ной порции Вани, удвоенной порции Дани и удвоенной порции Ко­


ли. Значит, оно в два раза больше того количества блинов, кото­
рое все трое могут осилить за один раз. В воскресенье они съедят
92:2 = 46 блинов.

ДОПОЛНИТЕЛЬНЫЕ ЗАДАЧИ

Решение задачи 11. Эту задачу можно решить разными спосо­


бами. Приведём два из них.
Первый способ заключается в том, чтобы рассмотреть конкрет­
ную последовательность изготовления Дедов Морозов. Например,
предположим, что Алина и её мама разделили все плитки на три
группы по три плитки в каждой. Из каждой группы получится три
Деда Мороза, причём останется столько же шоколада, сколько со­
держится в одной плитке. Итак, теперь у Алины и её мамы девять
Дедов Морозов и ещё столько же шоколада, сколько в трёх плит­
ках. Из такого остатка можно сделать трёх Дедов Морозов, и ещё
останется столько же шоколада, сколько в одной плитке. Из этого
нового остатка получится один Дед Мороз, и сколько-то шоколада
опять останется. Хватит ли этого последнего остатка, чтобы сделать
ещё одного Деда Мороза?
Чтобы узнать это, придётся заняться подсчётом. В трёх плитках
столько же шоколада, сколько в трёх Дедах Морозах и одной плитке.
2
Значит, на одного Деда Мороза затрачивается ~ плитки. Поэтому
последний остаток — это « плитки, и из него не получится Дед Мороз.
Итак, Алина с мамой смогут сделать 9 + 3 + 1 = 13 Дедов Морозов.
Таким же образом можно решить задачу про 14 плиток. В этом
случае получится 21 Дед Мороз, а шоколада не останется.
Второй способ. Сначала, как и при решении первым способом,
рассчитаем количество шоколада, из которого получается один Дед
2
Мороз: это -z плитки. Когда Алина с мамой закончат работу, весь
шоколад будет превращен в Дедов Морозов (кроме, возможно, неко­
торого остатка, которого не хватит на целого Деда Мороза). Поэто­
му мы решим задачу, если подсчитаем, сколько будущих Дедов Мо­
розов содержится в исходных плитках шоколада. Поскольку
334 Раздел 4. Решения

из 9 плиток получится 13 Дедов Морозов. Из 14 плиток получится


21 Дед Мороз, так как
14 = 21-§.
Решение задачи 12. Сначала рассмотрим кошек. Заметим, что
если бы какие-то три кошки сидели подряд, у средней из них не бы­
ло бы соседа-кота. Но нам известно, что каждая кошка сидит рядом
с котом. Поэтому подряд могут сидеть самое большее две кошки.
Следовательно, 19 кошек образуют не меньше 10 групп, между
которыми не меньше 9 промежутков, занятых котами. Поскольку
котов 10, а промежутков между кошками, где они могут сидеть, не
меньше 9, подряд могут сидеть самое большее два кота.
Значит, кот не может сидеть между двумя котами, и если это не
крайний кот в ряду, то среди его соседей должна быть хотя бы одна
кошка. Теперь предположим, что кот крайний. Если бы рядом с ним
сидел кот, то два кота сидели бы с краю и для заполнения девяти
промежутков между кошками оставалось бы только восемь котов.
Значит, крайний кот тоже должен сидеть рядом с кошкой.
Решение задачи 13. Нет, не может. Чтобы доказать это, раскра­
сим кубики 1 х 1 х 1 в порядке, аналогичном шахматному. Эта рас­
краска обладает такими свойствами.
• 14 кубиков окрашены в один цвет (назовём его красным),
а 13 кубиков —в другой (назовём его синим). Все угловые куби­
ки большого куба — красные, а центральный кубик (с крючком) —
синий.
• Кубики, имеющие общую грань, окрашены в разные цвета. По­
этому после синего кубика мышка всегда ест красный, а после крас­
ного — синий.
• Поскольку съесть центральный (синий) кубик нельзя, мышка
должна съесть 14 красных и 12 синих кубиков. Но упорядочить
14 красных и 12 синих предметов с чередованием цветов невоз­
можно.
Занятие 22

Делимость I. Определение и свойства

Решение задачи для разминки 1. Это задача-шутка. В самолё­


тах не бывает стоп-кранов. Как вы представляете себе остановку
самолёта в воздухе?
Решение задачи для разминки 2. Скорее всего, дети придума­
ют множество неверных ответов: на изготовление круглой крышки
уходит меньше металла, упавшему в люк человеку проще ухватить­
ся за края круглого отверстия и тому подобное. Выслушав такие от­
веты, предложите пару подсказок.
Подсказка 1. Почему кастрюли круглые? Вы когда-нибудь заду­
мывались об этом? «Чтобы еда не застревала в углах» — это, конеч­
но, разумное объяснение. Но, может быть, есть и другие причины?
Подсказка 2. Это скорее не подсказка, а наглядная демонстрация.
Поставьте на стол кастрюлю и квадратный контейнер. Поверните
крышку вертикально (перпендикулярно к поверхности стола) и при­
ставьте её сверху к кастрюле так, чтобы она касалась кастрюли в двух
местах. Теперь начните вращать крышку вокруг вертикальной оси.
Обычно в этот момент до детей «доходит». Кто-нибудь обязательно
закричит, что круглая крышка не может упасть в кастрюлю.
ОТВЕТ. Круглая крышка, как бы мы её ни поворачивали, не мо­
жет упасть в круглое отверстие люка. (Благодаря этому те, кто рабо­
тает внизу, могут чувствовать себя гораздо спокойнее!) А вот квад­
ратная крышка вполне может упасть в квадратное отверстие того
же размера.
Решение задачи 1. См. рисунок.
336 Раздел 4. Решения

Решение задачи 2. Надо вынуть одну монету из сундука с над­


писью «Золото и серебро». Как по этой монете определить содержи­
мое всех трёх сундуков?
Предположим, что эта монета оказалась золотой. Поскольку в сун­
дуке с надписью «Золото и серебро» не может быть смесь золотых
и серебряных монет, в нём только золотые монеты. Теперь рассмот­
рим сундук с надписью «Серебро». В нём могут быть либо только
золотые монеты, либо золотые и серебряные. Но мы уже знаем, что
золотые монеты без серебряных находятся в сундуке «Золото и се­
ребро». Значит, в сундуке «Серебро» смесь монет. Для сундука «Зо­
лото» остаётся только один вариант — серебряные монеты.
Такое же рассуждение приводит к ответу и в том случае, если
монета оказалась серебряной.
Решение задачи 3. Девять делителей: 1,2, 3, 4, 6, 9,12,18, 36.
Решение задачи 4. Таких чисел очень много. Например, чис­
ло 2 9 с делителями 1, 2, 2 2 ,..., 2 9 .
Решение задачи 5. а) Каждое четвёртое целое число делится
на 4. Другими словами, между двумя соседними числами, делящи­
мися на четыре, всегда ровно три числа. Поэтому четыре числа, не
делящиеся на 4, не могут идти подряд.
б) Каждое пятое целое число делится на 5. Другими словами,
между двумя соседними числами, делящимися на пять, всегда ровно
четыре числа. Поэтому пять чисел, не делящихся на 5, не могут идти
подряд.
Решение задачи 6. а) Докажем, что мистер X—рыцарь. Обо­
значим два числа буквами: А и В. Поскольку оба числа делятся
на 7, каждое из них равно произведению числа 7 и какого-то це­
лого числа: А = 7 • X и В —7 -Y. Рассмотрим разность этих чисел:
А — В = 7-X — 7-Y = 7- (X — У). Эта разность равна произведению
числа 7 и целого числа X — Y. Значит, она делится на 7.
б) Возьмём числа 15 и 8. Ни одно из них не делится на 7, но их
разность (которая равна 7) делится на 7. Следовательно, мистер Y—
лжец.
в) Докажем, что леди Z — рыцарь. Обозначим два числа буквами:
А и В. Поскольку оба числа делятся на С, каждое из них равно про­
изведению числа С и какого-то целого числа: А = С-X и В = С-Y.
Рассмотрим разность этих чисел: А — В = С • X — С -Y = С • (X - У ) .
Эта разность равна произведению числа С и целого числа X - Y.
Значит, она делится на С.
Занятие 22. Делимость I. Определение и свойства 337

Решение задачи 7. Чтобы объяснить решение этой задачи, луч­


ше начать с конкретного примера. Перечислим все делители чис­
ла 36: это 1,2,3,4, 6, 9,12,18, 36. Теперь заметим, что если разде­
лить число 36 на любой из его делителей, то снова получится де­
литель числа 36. Чем больше первый делитель, тем меньше второй.
Таким образом, делители числа 36 можно разбить на пары. Начнём
с наименьшего и наибольшего делителей: 1 и 36. Наименьший де­
литель после 1 и наибольший до 36 тоже образуют пару: 2 и 18.
Следующая пара: 3 и 12, и т.д.

В каждой паре делители дополняют друг друга: их произведение


равно 36. Но какой делитель будет дополнительным к делителю 6?
Само число 6: ведь 6 • 6 = 36! Значит, у делителя 6 нет пары. Итак,
все делители числа 36, кроме одного, можно разбить на пары. Зна­
чит, количество делителей нечётное.
Это же рассуждение применимо к делителям любого числа N. Для
любого делителя числа N есть делитель, дополнительный к нему, —
число, которое при умножении на него даёт N.
Если N не является точным квадратом, то делитель никогда не
совпадает с дополнительным к нему делителем (почему?). Значит,
у такого числа N чётное число делителей.
Если же N — полный квадрат, у него есть один особый делитель:
он совпадает с дополнительным к нему делителем. (Это, конечно,
квадратный корень из N, который при умножении на себя даёт N.)
Значит, если число N — полный квадрат, то у него нечётное число
делителей.
Решение задачи 8. Гномику исполняется х лет в году 1122 + х.
Он получает в подарок алмаз, если 1122 + х делится на х. Но это
происходит в те самые годы, когда 1122 делится на х: ведь х все­
гда делится на х. Поэтому наибольшее возможное значение х —
это 1122.
Решение задачи 9. Мы знаем, что Стае потратил на несколь­
ко леденцов все свои деньги. Поэтому задача будет решена, если
мы докажем, что разность между деньгами Тараса и деньгами Стаса
можно без остатка потратить на леденцы.
338 Раздел 4. Решения

У Тараса больше денег, чем у Стаса, на 11 сугриков и 11 тугри­


ков. 11 сугриков —это стоимость одного леденца. Поскольку каж­
дый тугрик равен целому числу сугриков, 11 тугриков в целое число
раз больше, чем 11 сугриков. Значит, это стоимость нескольких ле­
денцов.
Решение задачи 10. Поездка туда и обратно занимает 6 часов;
попробуем вычислить, какую часть этого времени автобус подни­
мается, а какую спускается. На каждом участке дороги автобус один
раз поднимается и один раз спускается: если на пути из Бел вью в Хи-
лвью на каком-то участке он поднимается, то на обратном пути на
этом же участке он спускается, и наоборот. Автобус поднимается
в два раза медленнее, чем спускается. Значит, на каждом участке
дороги он затрачивает на подъём в два раза больше времени, чем на
спуск. Если сложить время подъёма на всех участках, оно получится
в два раза больше, чем время спуска на всех участках. Значит, из
6 часов всей поездки автобус 4 часа поднимается и 2 часа спускает­
ся. Теперь нетрудно вычислить расстояние, которое он проезжает:
50 • 4 +100 • 2 = 400 км. Значит, между Белвью и Хилвью 200 км.
Занятие 23

Делимость П. Простые числа


и разложение на простые множители

Решение задачи для разминки 1. В первой группе — цифры, со­


стоящие только из прямых линий. Во второй группе — цифры с кри­
выми линиями.
Решение задачи для разминки 2. Цифра 5 рядом с её отраже­
нием.
Решение задачи 1. Это число делится на 2.
Оно не делится на 4, так как для этого в наборе простых множи­
телей должно быть хотя бы два числа 2 (4 = 2-2).
Оно делится на 14, так как 14 = 2-7, а среди простых множите­
лей нашего числа есть и 2, и 7.
Оно не делится на 12, так как для этого в наборе простых мно­
жителей должно быть хотя бы два числа 2 (12 = 2-2-3).
Решение задачи 2. а) Не делится. В наборе простых множите­
лей второго числа есть два числа 3, а в наборе простых множителей
первого —только одно.
б) Произведение этих чисел равно 2 3 • З 3 • 75 • 13.
Это произведение делится на 8, так как в наборе его простых
множителей есть три числа 2, а 8 = 2 • 2 • 2.
Произведение делится на 36, так как в наборе его простых мно­
жителей есть два числа 2 и два числа 3, а 36 = 2 • 2 • 3 • 3.
Произведение делится на 27, так как в наборе его простых мно­
жителей есть три числа 3, а 27 = 3 • 3 - 3.
Произведение не делится на 16, так как в наборе его простых
множителей только три числа 2, а для делимости на 16 их нужно
четыре (16 = 2-2-2-2).
Произведение делится на 56, так как в наборе его простых мно­
жителей есть три числа 2 и одно число 7, а 56 = 2 • 2 • 2 • 7.
Решение задачи 3. Оба числа чётные. Простое число, большее 2,
всегда нечётно. Если прибавить к нему 1 или вычесть из него 1, то
получится чётное число.
340 Раздел 4. Решения

Решение задачи 4. а) Мы уже знаем, что числа р - 1 и р + 1


чётные. Каждое второе чётное число делится на 4. Значит, из двух
идущих подряд чётных чисел р — 1 и р + 1 одно делится на 4.
б) Утверждение неверно. В этом можно убедиться на примере:
р = 5 или р = 13.
Решение задачи 5. Мистер X — рыцарь. Если число делится на 3,
то в наборе его простых множителей есть 3. Если оно к тому же
делится на 4, то в наборе его простых множителей есть два числа 2.
Значит, число делится на 2 • 2 • 3 = 12.
Мистер Y—лжец. Например, 12 делится на 6 и на 4, но не делит­
ся на 24.
Решение задачи 6. Сначала предположим, что на первом сундуке
неверная надпись. Тогда верна надпись на втором сундуке: в одном
из сундуков золото, а в другом — серебро. Поскольку первая надпись
неверна, золото лежит во втором сундуке, а серебро — в первом.
Теперь надо проверить другую возможность: на первом сундуке
верная надпись, а на втором — неверная. Согласно первой надпи­
си, в первом сундуке лежит золото, а во втором — серебро. Но тогда
вторая надпись тоже верна. Это невозможно: вторая надпись не мо­
жет быть верной и неверной одновременно. Значит, первая надпись
точно неверна.
Решение задачи 7. Проще всего найти делители числа 111 ме­
тодом проб и ошибок, проверяя по очереди все простые числа на­
чиная с 3. Уже на числе 3 обнаруживается делимость: 111 = 3 • 37.
Осталось проверить, что 37 — простое число. ОТВЕТ: 111 = 3-37.
Как найти делители числа 1111? Можно и в этом случае прове­
рять делимость на простые числа начиная с 3. Однако число 1111
выглядит как делящееся на 11. Поэтому проверку делимости нач­
нём с 11. Получаем 1111 = 11-101. Затем методом проб и ошибок
проверяем, что 101 —простое число. ОТВЕТ: 1111 = 11-101.
Для преподавателей. Объясните (или напомните) детям, что при проверке
делимости числа на простые числа можно ограничиться теми простыми
числами, квадрат которых не больше проверяемого числа. (Почему?)

Для числа 1001 снова воспользуемся методом проб и ошибок.


Наименьшее простое число, на которое делится 1001, —это 7. Ре­
зультат этого деления 143 делится на 11. ОТВЕТ: 1001 = 7-11-13.
Решение задачи 8. Разложим 2002 на простые множители, вос­
пользовавшись разложением числа 1001 из предыдущей задачи:
Занятие 23. Делимость II 341

2002 = 2 • 7 • 11 • 13. Значит, числа ВАО, ВА и В состоят из этих четы­


рёх множителей. Покажем, что В равно 1. Действительно, если бы В
было больше 1 (т. е. не меньше 2), то произведение ВАО • ВА- В было
бы не меньше 200 • 20 • 2 = 8000. Но это произведение равно 2002.
Следовательно, четыре простых множителя 2, 7, 11 и 13 вместе
составляют числа ВАО и ВА. Кроме того, ВА не делится на 11; зна­
чит, 11 относится к простым множителям числа ВАО.
К какому набору простых множителей — ВАО или ВА —относит­
ся 13? Если 13 — простой множитель числа ВАО, то и 7, и 2 должны
быть простыми множителями числа ВА (иначе ВА не было бы дву­
значным числом). Если 13 —простой множитель числа ВА, то и 7,
и 2 должны быть простыми множителями числа ВАО (иначе ВАО
не было бы трёхзначным числом). Итак, либо ВАО = 11 • 13 = 143
и ВА = 7 • 2 = 14, либо ВАО = 11 • 2 • 7 = 154 и ВА = 13. Поскольку
во втором случае букве А соответствуют разные цифры, остаётся
первый случай: ВАО = 143, ВА= 14, В = 1.
Решение задачи 9. На рисунке показа­
на основная идея решения. Сначала пре­
вратим лист бумаги в такую фигуру, кото­
рую можно развернуть в длинную полоску.
Для этого сделаем разрезы, показанные на
рисунке сплошными линиями. Чем больше
таких разрезов (и чем меньше расстояние между ними), тем длин­
нее получится полоска. Теперь сделаем длинный разрез посередине
полоски (на рисунке он показан штриховой линией). Этот разрез
будет проходить почти по всей длине полоски. Если сделать полоску
очень длинной, то отверстие получится достаточным для прохожде­
ния слона.
Решение задачи 10. Разделим всех жителей острова на две груп­
пы в зависимости от их высказывания в ходе опроса. Заметим, что
каждая группа состоит либо только из рыцарей, либо только из лже­
цов. Рассмотрим все возможные варианты состава групп.
Люди в группе 1 Люди в группе 2
Вариант 1 Рыцари Рыцари
Вариант 2 Рыцари Лжецы
Вариант 3 Лжецы Рыцари
Вариант 4 Лжецы Лжецы
342 Раздел 4. Решения

В каждом из четырёх случаев попробуем сделать вывод о числен­


ности населения острова.
Вариант 1. Так как обе группы состоят из рыцарей, на острове
О лжецов (чётное число). Согласно утверждению рыцарей из груп­
пы 1, число рыцарей четно. Так как на острове 0 лжецов, общее чис­
ло жителей тоже четно. (Заметим, что, так как рыцари не могут
лгать, в группе 2 должно быть 0 человек.)
Вариант 2. В этом случае все рыцари относятся к группе 1, а все
лжецы —к группе 2. Поскольку утверждение группы 1 верно, число
рыцарей четно. Поскольку утверждение группы 2 неверно, число
лжецов также четно. Значит, на острове чётное число жителей.
Вариант 3. В этом случае все лжецы относятся к группе 1, а все
рыцари —к группе 2. Поскольку утверждение группы 1 неверно,
число рыцарей нечётно. Поскольку утверждение группы 2 верно,
число лжецов также нечётно. Значит, на острове чётное число жи­
телей.
Вариант 4. В этом случае обе группы состоят из лжецов; значит,
на острове нет рыцарей. Поскольку утверждение группы 2 неверно,
число лжецов (всех жителей острова) четно. (Заметим, что, так как
лжецы не могут говорить правду, группа 1 должна быть пуста.)
Занятие 24

Математический аукцион III

Решение задачи для разминки 1. У этой задачи много решений.


Например, положим один шарик в первый стаканчик, 6 во второй
и 3 в последний. Затем поставим первый стаканчик во второй. Теперь
в первом стаканчике один шарик, во втором — 7, а в третьем — 3.
Решение задачи для разминки 2. У Пети участок в два раза
больше, чем у Васи.
Решение задачи 1. 1, 2 и 3.
Решение задачи 2. Пожалуй, самое простое решение: все семь
чисел равны 1.
Решение задачи 3. Разложим 555 на простые множители. Мно­
житель 5 легко угадать: 555 = 5-111. Разложение числа 111 было
в задаче из предыдущей подборки: 111 = 3-37. Итак, 555 = 3-5-37.
Разложение числа 2222 начнём с множителя 2. Запишем 2222 =
= 2-1111. Разложение числа 1111 было в задаче из предыдущей под­
борки: 1111 = 11 • 101. Итак, 2222 = 2 • 11 • 101.
Посмотрев на число 2277, можно предположить, что оно делится
на 11. Действительно, запишем равенство 2277 = 2200-1-77 и за­
метим, что каждое слагаемое делится на 11. Теперь разделим 2277
на 11 и получим 2222 = 11 - 207. Методом проб и ошибок найдём
разложение 207 = 3 • 69 = 3 • 3 • 23. Итак, 2277 = З2 • 11 • 23.
Наконец, 7007, очевидно, делится на 7. Запишем 7007 = 7-1001.
Разложение числа 1001 было в задаче из предыдущей подборки:
1001 = 7-11-13. Итак, 7007 = 72 • 11 • 13.
Решение задачи 4. Это простая задача по комбинаторике. У нас
восемь пропусков, и для каждого из них—два варианта заполнения:
0 и 1. ОТВЕТ: 2 8 .
Решение задачи 5. Наша промежуточная цель —налить 30 грам­
мов зелья в 90-граммовый флакон. Мы пойдём окольным путём.
• Наполним 40-граммовый флакон зельем из кувшина и выльем
из него всё содержимое в 90-граммовый флакон. Сделаем то же са­
мое ещё раз. Теперь в 90-граммовом флаконе 80 граммов зелья.
344 Раздел 4. Решения

• Наполним 40-граммовый флакон зельем из кувшина, а затем


заполним из него 90-граммовый флакон до краёв (там оставалось
места на 10 граммов). В 40-граммовом флаконе остаётся 30 грам­
мов зелья. Выльем обратно в кувшин всё зелье из 90-граммового
флакона и перельём в него 30 граммов из 40-граммового флакона.
Теперь сходным образом сделаем так, что в 90-граммовом кув­
шине окажется 60 граммов зелья.
• Наполним 40-граммовый флакон зельем из кувшина и выльем
из него всё содержимое в 90-граммовый флакон. Теперь в 90-грам­
мовом флаконе 70 граммов зелья.
• Наполним 40-граммовый флакон зельем из кувшина, а затем
заполним из него 90-граммовый флакон до краёв (там оставалось
места на 20 граммов). В 40-граммовом флаконе остаётся 20 грам­
мов зелья. Выльем обратно в кувшин всё зелье из 90-граммового
флакона и перельём в него 20 граммов из 40-граммового флакона.
• Наконец, снова наполним 40-граммовый флакон зельем из кув­
шина и выльем из него всё содержимое в 90-граммовый флакон.
Теперь в 90-граммовом флаконе 60 граммов зелья.
Решение задачи 6. Сначала заметим, что Н может обозначать
только 1, так как четырёхзначное число получилось в результате
сложения двух трёхзначных чисел. Теперь рассмотрим сложение
в разряде десятков. Сумма двух цифр А не может оканчиваться циф­
рой 1; значит, должен быть перенос из разряда единиц. Можно так­
же заключить, что сумма двух цифр А оканчивается цифрой 0. Зна­
чит, А—либо 0, либо 5.
Если А равно 0, то Т должно быть равно 5 (это видно из сложения
в разряде сотен). Но тогда из сложения в разряде единиц следует,
что D также равно 5. Это невозможно, так как разным буквам со­
ответствуют разные цифры. Значит, А —не 0, а 5. При сложении
в разряде десятков 1 переносится в разряд сотен. Значит, сумма двух
цифр Т оканчивается цифрой 4. Итак, Т—либо 2, либо 7. Но Т не мо­
жет быть равно 2, иначе в сумме не получилось бы четырёхзначное
число. Значит, Т равно 7, a D равно 8.
Решение задачи 7. Из решения предыдущих задач мы знаем,
что 111 = 3-37. Значит, 111 делится на 37.
Число, состоящее из 99 единиц, можно представить в виде суммы
таких чисел: 111,111000,111000000,..., 1110...0. Каждое из этих
чисел делится на 37. Значит, и их сумма делится на 37.
Занятие 25

Делимость III. Признаки делимости

Решение задачи для разминки 1. Эта задача сформулирована


нечётко. Поэтому желательно, чтобы участники не делали поспеш­
ных выводов, а сначала задавали уточняющие вопросы. Вот приме­
ры хороших вопросов.
• Пропорционален ли вес монеты её стоимости? (Да. 100-рублё­
вая монета весит в два раза больше 50-рублевой.)
• Имеют ли все монеты одинаковую форму? (Да. Все они круг­
лые.)
Решение. Стоимость золота в одном и том же объёме пример­
но одинакова в обоих вагонах. Но в первом вагоне золото зани­
мает вдвое больший объём. Значит, вагон, до краёв загруженный
50-рублевыми монетами, стоит дороже вагона, наполовину загру­
женного 100-рублевыми монетами.
Решение задачи для разминки 2. Во время прилива и отлива
корабль поднимается и опускается вместе с водой. Поэтому вторая
снизу перекладина всегда остаётся на её поверхности.
Решение задачи 1. См. рисунок.

Решение задачи 2. Подсчитаем сумму цифр этого числа: 1 + 2 = 3 .


Поскольку 3 делится на 3, само число тоже делится на 3.
Решение задачи 3. Ответ на первый вопрос положительный.
Например, 444 делится на 3. Ответ на второй вопрос отрицатель­
ный. Число, состоящее из одних четвёрок, чётное. Значит, на него
могут делиться только чётные числа. На число, состоящее из одних
троек, нечётное.
346 Раздел 4. Решения

Решение задачи 4. Поскольку код делится на 4, число, образо­


ванное его двумя последними цифрами, тоже делится на 4 (признак
делимости на 4). Среди четырёх двузначных чисел, состоящих из
двоек и троек (22, 23,32, 33), только 32 делится на 4. Следователь­
но, код оканчивается на 32. Двоек в этом семизначном коде боль­
ше, чем троек; значит, двоек не меньше четырёх. Итак, код может
состоять из четырёх двоек и трёх троек, пяти двоек и двух троек или
шести двоек и одной тройки. Сумма цифр такого числа равна соот­
ветственно 17, 16 или 15. Из этих чисел только последнее делится
на 3. Значит, код состоит из шести двоек и одной тройки. Поскольку
код оканчивается на 32, возможен только один вариант: 2222232.
Решение задачи 5. Число АВ равно 10-А + В, а число ВА равно
10 • В + А. Следовательно, их разность равна
10-А + В - 1 0 - В - А = 1 0 - ( А - В ) + В - А =
= 10-(А-В)-(А-В)=9-(А-В).
Значит, разность этих чисел делится на 9. Но 7 не делится на 9.
Поэтому разность не может быть равна 7.
Решение задачи 6. Сначала посмотрим, сколько двоек в каждом
из множителей нашего произведения. У чисел 2, 6, 10 и 14 по одно­
му простому множителю 2. У чисел 4 и 12 по два простых множи­
теля 2. У числа 8 их три, а у числа 16 —четыре. Теперь количество
двоек в наборе простых множителей числа 16! можно подсчитать:
4-1 + 2-2 + 3 + 4=15.
Решение задачи 7. Рассмотрим три случая.
• Сначала предположим, что великан выбрал две красные шля­
пы и одну чёрную. Когда зажжётся свет, гном в чёрной шляпе уви­
дит двух своих друзей в красных шляпах. Поскольку он знает, что
красных шляп всего две, он будет уверен, что на нём чёрная шляпа.
Когда великан в первый раз взмахнёт рукой, этот гном сможет запи­
сать цвет своей шляпы. Тогда его друзья поймут, что на них красные
шляпы: иначе первый гном не смог бы сразу определить цвет своей
шляпы. При втором взмахе они оба запишут цвет своих шляп.
• Теперь допустим, что великан выбрал одну красную шляпу
и две чёрные. Когда зажжётся свет, гном в красной шляпе увидит
двух своих друзей в чёрных шляпах. Он сделает вывод, что его шля­
па может быть как красной, так и чёрной. Каждый из двух гномов
в чёрных шляпах увидит одного друга в красной шляпе, а другого —
в чёрной. Итак, у всех трёх гномов недостаточно информации, что-
Занятие 25. Делимость III. Признаки делимости 347

бы определить цвет своих шляп. Поэтому при первом взмахе они


ничего не напишут. Однако сам факт отсутствия ответа даст гно­
мам в чёрных шляпах важную информацию. Каждый из них будет
рассуждать так: «Если бы на мне была красная шляпа, гном в чёрной
шляпе увидел бы две красные шляпы. Тогда он сразу определил бы
цвет своей шляпы и записал бы его. Но он этого не сделал. Значит,
на мне чёрная шляпа». Итак, при втором взмахе оба гнома в чёрных
шляпах правильно запишут цвет своих шляп. Гном в красной шляпе
поймёт: они смогли дать правильный ответ потому, что его шляпа
красная. При третьем взмахе он запишет свой цвет.
• Пусть, наконец, великан выбрал три чёрные шляпы. Когда за­
жжётся свет, каждый гном увидит двух своих друзей в чёрных шля­
пах. Каждый гном поймёт: если его шляпа красная, оба друга в чёр­
ных шляпах определят цвет своих шляп при втором взмахе. Но на
самом деле при втором взмахе никто ничего не напишет. Тогда каж­
дый гном сделает вывод, что на нём чёрная шляпа. При третьем
взмахе все три гнома запишут цвет своих шляп.
Решение задачи 8. Произведение всех чисел от 1 до 100, оче­
видно, делится на 9. По признаку делимости сумма его цифр также
делится на 9. Следовательно, сумма цифр этой суммы тоже делится
на 9, и т.д.: все числа, которые получит Робби, будут делиться на 9.
Поскольку эти числа становятся всё меньше и меньше, рано или
поздно Робби дойдёт до однозначного числа, которое делится на 9.
Очевидно, что это число —не 0; значит, это 9.
Решение задачи 9. Для детей этого возраста задача, вероятно,
окажется трудной, поэтому начнём её разбор с конкретного приме­
ра. Пусть первое число равно 75483. Его можно записать так:
7-10000 + 5-1000 + 4-100 + 8-10 + 3.
Если записать цифры этого числа в обратном порядке, получит­
ся 38457. Это число равно такому выражению:
3-10000 + 8-1000 + 4-100 + 5-10 + 7.
Каждая цифра исходного числа вносит определённый вклад в раз­
ность двух чисел. Вычислим вклад каждой цифры.
Вклад цифры 7:
7 - 1 0 0 0 0 - 7 = 7 - ( 1 0 0 0 0 - 1 ) = 7-9999.
Вклад цифры 5:
5 - 1 0 0 0 - 5 - 1 0 = 5 - ( 1 0 0 0 - 1 0 ) = 5-990.
348 Раздел 4. Решения

Вклад цифры 4 нулевой:


4-100-4-100 = 0.
Вклад цифры 8:
8-10-8-1000 = 8-(10-1000) = -8-990.
Вклад цифры 3:
3-3-10000 = 3-(1-10000) = -3-9999.
Все эти числа делятся на 9. Разность двух исходных чисел равна
сумме этих вкладов. Значит, она также делится на 9.
Такое же рассуждение подходит для разности любых двух чисел,
одно из которых получается из другого обращением порядка цифр.
Каждая цифра исходного числа «создаёт» некоторую часть разности,
и эта часть делится на 9. Разность равна сумме таких частей и, зна­
чит, тоже делится на 9.
Занятие 26

Делимость IV. Взаимно простые числа

Решение задачи для разминки 1. Цифры выписаны в алфавит­


ном порядке. Следующая цифра 3.
Решение задачи для разминки 2. Играет в шахматы.
Решение задачи 1. У этой задачи много реше­
ний. Все они основаны на одной идее: ряды не
обязательно параллельны. Ведь в условии ничего
не сказано о расположении рядов. Один из воз­
можных ответов показан на рисунке.
Решение задачи 2. Нет. Ни одно правило не запрещает хобби-
там селиться отдельно.
Решение задачи 3. а) Числа 2 и 9 взаимно просты. Значит, если
число делится на 2 и 9, то оно делится и на их произведение 18.
б) Нет. Например, 36 и 90 делятся на 6 и на 9, но не делятся
на 54.
в) Числа 3 и 8 взаимно просты. Значит, если число делится на 3
и 8, то оно делится и на их произведение 24.
г) Числа 21 и 10 взаимно просты. Значит, если число делится
на 21 и 10, то оно делится и на их произведение 210.
Решение задачи 4. Как связаны между собой количество нулей
в конце числа и его разложение на простые множители? Каждый
нуль в конце соответствует одному множителю 10, а каждый мно­
житель 10 раскладывается в произведение простых множителей 2
и 5. Значит, каждый нуль в конце числа соответствует одной па­
ре (2,5) в наборе простых множителей этого числа. Поэтому ответ
к задаче —это количество пар (2,5), которые получаются при раз­
ложении 25! на простые множители.
Заметим, что в наборе простых множителей числа 25! двоек го­
раздо больше, чем пятёрок. Поэтому количество нулей определяет­
ся количеством пятёрок. Подсчитаем количество простых множи­
телей 5 в каждом из чисел, составляющих произведение 25!. У чи­
сел 5, 10, 15 и 20 по одному простому множителю 5, а у числа 25 их
350 Раздел 4. Решения

два. Всего пятёрок 4 + 2 = 6, значит, число 25! оканчивается шестью


нулями.
Решение задачи 5. а) Число делится на 18, если оно делится на 2
и на 9. Число 10 000100 008 чётное; значит, оно делится на 2. Но это
число не делится на 9, так как сумма его цифр (которая равна 10)
не делится на 9.
б) Число делится на 12, если оно делится на 3 и на 4. Число
37000032 оканчивается на 32. Поскольку 32 делится на 4, чис­
ло 37000032 также делится на 4. Подсчитаем сумму цифр числа
37000032. Получим 3 + 7 + 3 + 2 = 15. Сумма цифр делится на 3;
значит, 37000032 делится на 3. Итак, 37000032 делится на 12.
в) Имеем 60 = 10-6 = 2-5-2-3 = 2-2-3-5.Числа4, 3 и 5 попарно
взаимно просты. Поэтому для проверки делимости на 60 достаточно
проверить делимость на 3, на 4 и на 5. Число 12 345 678 900 делится
на 4, так как две его последние цифры 00 образуют число, делящее­
ся на 4. Число 12 345 678 900 также делится на 5, поскольку оканчи­
вается цифрой 0. Наконец, число 12 345 678 900 делится на 3: сумма
его цифр 1 + 2 + 3 + 4 + 5 + 6 + 7 + 8 + 9 = 45 делится на 3. Значит,
12 345 678 900 делится на 60.
Решение задачи 6. Произведение этих двух чисел равно 100.
Значит, их можно составить из тех же простых множителей, что
и число 100.
Разложим 100 на простые множители: 100 = 10*10 = 2-2-5-5.
Следовательно, два числа, которые нужно найти, составляются из
двух двоек и двух пятёрок.
Заметим, что если среди простых множителей какого-нибудь
числа есть 2 и 5, то это число делится на 10. Значит, ни одно из
наших чисел не может одновременно содержать простые множите­
ли 2 и 5: каждое из них состоит либо только из двоек, либо только из
пятёрок. Поэтому возможен только один ответ: одно из чисел равно
2-2 = 4, другое равно 5 • 5 = 25.
Решение задачи 7. Произведение чисел А, ВВ и ВСВ равно 2222.
Значит, объединённый набор простых множителей чисел А, ВВ и
ВСВ — это набор простых множителей числа 2222. Число 2222 рас­
кладывается на простые множители так: 2222 = 2 • 11 • 101. Очевид­
но, простой множитель 101 относится к числу ВСВ: числа А и ВВ для
него слишком малы. Простой множитель 11 относится к числу ВВ:
число А для него слишком мало, а число ВСВ слишком мало сразу
для двух простых множителей 11 и 101.
Занятие 26. Делимость IV. Взаимно простые числа 351

Осталось определить принадлежность простого множителя 2.


Предположим, что он относится к числу ВСВ. Тогда ВСВ = 2 • 11 = 202
и, значит, В = 2. Но в таком случае ВВ • ВСВ больше 2222. Поэтому 2
не является простым множителем числа ВСВ. Таким же образом до­
казывается, что 2 не является простым множителем числа ВВ. Зна­
чит, 2 — простой множитель числа А. Итак, А = 2, ВВ = 11, ВСВ = 101.
Для преподавателей. Делимость 2222 на 11 — отличный пример примене­
ния «упрощённого» признака делимости на 11. Напомните детям: при
поиске простых делителей числа стоит посмотреть, не «бросается ли
в глаза» делимость на 11.
Решение задачи 8. Иван-Царевич налил царю Афрону обычной
воды. Поэтому вода из десятого колодца не вылечила, а отравила
царя.
Перед поединком Иван-Царевич выпил мёртвой воды из колод­
ца с номером, меньшим 10. Поэтому поднесённая Афроном вода из
десятого колодца послужила Ивану противоядием.
Занятие 27

Математические игры со стратегией I

Решение задачи для разминки 1. Выполнить этот приказ невоз­


можно. Если парикмахер побреется, то он нарушит приказ: ведь он
должен побрить только тех, кто не бреется сам. Если он не побреет­
ся, то он всё равно нарушит приказ: он должен побрить всех, кто не
бреется сам.
Решение задачи для разминки 2. Парикмахер —женщина.
Решение задачи 1. а) Второй поварёнок (Билли) выиграет, если
применит «симметричную» стратегию: каждый раз брать то же ко­
личество тарелок, что и первый, но с другого стола. После каждого
хода второго поварёнка восстанавливается симметрия: одинаковое
количество тарелок на каждом столе. Поэтому, сколько бы тарелок
ни взял первый поварёнок, второй всегда сможет унести столько же.
б) Первый поварёнок (Джимми) выиграет, если первым ходом
уберёт все 4 тарелки с третьего стола. После этого мы приходим
к условию первой задачи: два стола с одинаковым количеством та­
релок. Теперь Джимми должен использовать «симметричную» стра­
тегию—убирать то же количество тарелок, что и Билли, но с друго­
го стола.
Решение задачи 2. В этой игре также основная идея —симмет­
рия. Первым ходом Джон должен выпить поллитра грога из кувши­
на. После этого в бутылке и в кувшине станет одинаковое количе­
ство жидкости. Теперь Джон должен копировать ходы Роджера: от­
пивать столько же, но из другого сосуда. Тогда Джон своим ходом
всегда будет восстанавливать симметрию — уравнивать количество
жидкости в сосудах. Поэтому он выпьет последний глоток.
Решение задачи 3. Давайте разобьём числа на пары: 1 и 6, 2
и 7, 3 и 8, 4 и 9, 5 и 10. У второго игрока есть выигрышная стра­
тегия: какое бы число ни вычеркнул первый игрок, второй должен
убрать второе число из соответствующей пары. Например, если пер­
вый вычеркнул 3, то второй убирает 8. Тогда после каждого хода
второго игрока на доске остаются только пары чисел. Два последних
Занятие 27. Математические игры со стратегией I 353

оставшихся числа тоже будут парой. Так как разница чисел в паре
равна 5, второй игрок победит.
Решение задачи 4. а) Второй игрок (Тарас) имеет выигрышную
стратегию. Он должен использовать центральную симметрию: ка­
кую бы из своих фишек ни подвинул Стае, Тарас должен двигать
свою фишку в другом ряду на столько же клеточек. Тогда после каж­
дого хода Тараса будет восстанавливаться центральная симметрия
на доске. Таким образом, если Стае может сделать ход, то Тарас все­
гда может ответить.
б) Та же самая стратегия — симметрия относительно центра по­
лоски. (Вопрос про 2 • 80 добавлен потому, что дети часто решают
задачу про полоску 2 • 8 перебором вариантов.)

Для преподавателей. Дети часто предлагают неверную стратегию, кото­


рая использует идею осевой симметрии. Они говорят, что Тарас должен
двигать фишку в том же ряду, что и Стае, и на столько же клеточек.
Почему эта идея не работает? Потому что если Стае своим ходом пе­
реводит фишку на половину поля Тараса, то Тарас не может ответить.
(Рисунок иллюстрирует эту ситуацию.)

Решение задачи 5. Второй игрок выиграет, если будет отражать


ходы первого игрока относительно центра доски. Придерживаясь
этой стратегии, второй игрок каждым своим ходом восстанавли­
вает центральную симметрию. Поэтому если первый игрок может
найти свободное место для своей костяшки, то симметричное место
тоже свободно. Значит, последний ход обязательно останется за
вторым игроком.
Для преподавателей. Дети часто предлагают для этой игры две неверные
стратегии.
Одна ошибочная стратегия основана на том, что для полного покры­
тия 100-клеточной доски нужно 50 костяшек домино. Поскольку второй
354 Раздел 4. Решения

игрок делает все чётные ходы, он сделает и последний, 50-й ход. Ошиб­
ка заключается в предположении, что ходов всегда ровно 50. На самом
деле в ходе игры могут возникнуть отдельные пустые клетки, со всех
сторон окружённые костяшками. Поскольку использовать эти клетки
невозможно, игра закончится до того, как вся доска будет заполнена.
Другая неверная стратегия состоит в том, чтобы ставить костяш­
ки симметрично относительно вертикальной или горизонтальной оси
симметрии. Чтобы объяснить ошибочность этой стратегии, достаточно
продемонстрировать ситуацию, когда следовать ей невозможно. Дей­
ствительно, если первый игрок поставит костяшку так, чтобы она пере­
секала ось симметрии, то второй игрок не сможет сделать симметрич­
ный ход.
Решение задачи 6.
• У числа р 2 такие делители: 1, р и р 2 (всего 3).
• У числа р 5 такие делители: 1, р , р 2 , р 3 , р 4 и р 5 (всего 6).
• У числа рх такие делители: 1, р, р 2 , р 3 , ..., рх (всего х +1).
• У числа р • q такие делители: 1, р , q и р • q (всего 4).
• У числа р 2 • q такие делители: 1, р , р2, q, р - q и р 2 • q (всего 6).
• У числа рх -q такие делители: 1, р, р 2 , р 3 , ..., рх и д , p-q, p2-q,
p -q, ..., px -q (всего (x + l ) - 2 ) .
3

• Последний вопрос самый трудный. Но ответы на предыдущие


вопросы дают подсказку для поиска делителей числа рх • qy:

1, Р, Р2, Р 3 , •••> Рх, q> Р'Ъ Р2'Ъ Р3*<Ь •••>


•••> р х - ъ <i2> р - ч 2 , p2-<i2, p 3 - q 2 , •••> РХ'Ч2 и т. д.
Последняя группа делителей:

чу> р-<1у> р2-я.у> p 3 -q y > .... рх-я.у


(всего(х + 1)-(у + 1)).
Получить тот же ответ быстрее поможет комбинаторика. Каж­
дый делитель — это произведение числа р в какой-то степени и чис­
ла q в какой-то степени. Сколькими способами можно выбрать по­
казатели этих степеней? Для р возможных показателей х + 1 , а для
q их у + 1 . Значит, всего вариантов (х + 1 ) • (у + 1 ) .
Решение задачи 7. Что получится, если из числа с такими свой­
ствами вычесть 17? Число с суммой цифр 9, оканчивающееся на 00
и делящееся на 17. Как найти такое число? Поскольку числа 100
и 17 взаимно просты, можно убрать два нуля в конце: делимость
на 17 сохранится. Итак, мы свели задачу к более простой: найти
Занятие 27. Математические игры со стратегией I 355

число, делящееся на 17, с суммой цифр, равной 9. Для поиска такого


числа можно по очереди умножать 17 на все целые числа начиная
с 2 (2,3,4,...), до тех пор пока не получится число с суммой цифр 9.
Однако признак делимости на 9 позволяет существенно сократить
вычисления: ведь число с суммой цифр 9 должно делиться на 9. По­
скольку 17 и 9 —взаимно простые числа, достаточно умножать 17
на 9, 18, 27, ... (только на те числа, которые делятся на 9). Умно­
жив 17 на 9, получим 153. Это число удовлетворяет всем условиям.
Итак, окончательный ответ: 15 317.
Отметим, что это не единственно возможный ответ.
Занятие 28

Математические игры со стратегией II

Решение задачи для разминки 1. Стоящие часы показывают


правильное время каждые 12 часов.
Решение задачи для разминки 2. После выстрела все уцелев­
шие птицы улетели. Ответ: ни одной.
Решение задачи 1. Пусть с момента, когда отцу было 27 лет,
прошло Л: лет. Сейчас возраст отца и сына — соответственно 27 + х
и 3 + лг. Поскольку отец втрое старше сына, можно составить урав­
нение: 27 + х = 3-(3 + х).
Решив это уравнение, получим х = 9. Значит, сыну 12 лет.
Решение задачи 2. Докажем, что в столичном зоопарке нет гип­
попотама.
Предположим, что там есть гиппопотам. Тогда из утверждения 3
(если в зоопарке есть и гиппопотам, и жираф, то там есть носорог)
следует, что в столичном зоопарке есть носорог. Поэтому согласно
утверждению 1 (если в зоопарке есть и гиппопотамы, и носороги,
там нет жирафов) в этом зоопарке нет жирафа. Получилось проти­
воречие: ведь по условию задачи в столичном зоопарке есть жираф.
Значит, в этом зоопарке нет гиппопотама.
Теперь докажем, что в столичном зоопарке есть носорог. Для это­
го воспользуемся утверждением 2: в каждом зоопарке есть гиппопо­
там или носорог. Поскольку в столичном зоопарке нет гиппопотама,
там обязательно есть носорог.
Решение задачи 3. У каждого ученика либо не меньше двух жёл­
тых шаров, либо не меньше двух красных, но ни у кого нет двух жёл­
тых и двух красных одновременно. Поэтому во второй раз подняли
руку все, кто не поднял её в первый раз, и только они. Значит, всего
учеников 13 4-15 = 28. Теперь заметим, что у каждого ученика либо
есть шары разных цветов, либо все три шара одного цвета. Поэтому
тех, у кого все шары одного цвета, 28 - 17= 11.
Решение задачи 4. У первого игрока (Ильи) есть выигрышная
стратегия, основанная на симметрии.
Занятие 28. Математические игры со стратегией II 357

Первым ходом Илья должен взять четыре крекера с той тарелки,


на которой 31 крекер. Тогда на обеих тарелках будет одинаковое ко­
личество крекеров. Каждым следующим ходом Илья должен повто­
рять ходы Наташи: брать столько же крекеров, но с другой тарелки.
Решение задачи 5. У первого игрока (Кати) есть выигрышная
стратегия.
Первым ходом Катя берёт два шара. Остаётся 54 шара (заметим,
что это число делится на 9). Каждым последующим ходом Катя до­
полняет ход Лёши до девяти: 1—8, 2—7, 3—6 и т.д. После каждого
хода Кати количество оставшихся в куче шаров делится на 9, а по­
сле хода Лёши —не делится. Значит, 0 шаров останется после хода
Кати.
Решение задачи 6. Это псевдоигра. При каждом разламывании
куска шоколада количество кусков увеличивается на один. Поэто­
му после каждого хода Максима кусков будет чётное число, а после
каждого хода Олега —нечётное.
Игра закончится, когда кусков шоколада будет 60. Поскольку
60 —чётное число, выиграет Максим.
Решение задачи 7. Это псевдоигра. В выражении пять нечётных
и пять чётных чисел. Как бы ни расставляли между ними плюсы
и минусы, результат будет нечётным. Поэтому второй игрок (Юс­
тас) обязательно выиграет.
Решение задачи 8. Представим себе, что каждый солдат стоит
в круге красного или синего цвета, причём эти цвета чередуются.
(Например, все солдаты с нечётными номерами стоят в красных
кругах, а все солдаты с нечётными номерами — в синих.) Тогда лю­
бые два солдата, стоящие через одного, находятся в кругах одно­
го цвета. Когда они меняются местами, каждый из них переходит
в круг того же цвета. Значит, ни один солдат не может перейти
в круг другого цвета.
Теперь приведём пример расположения солдат, из которого невоз­
можно расставить их по росту. Пусть самый высокий солдат стоит
крайним слева, а второй по росту—на третьем месте слева. Если
бы солдаты были расставлены по росту, то эти два солдата стояли
бы рядом, т. е. в кругах разных цветов. Но первоначально они стоят
в кругах одного цвета и поэтому не могут оказаться в кругах разных
цветов. Значит, расставить солдат по росту невозможно.
Решение задачи 9. У второго игрока есть выигрышная страте­
гия. Приведём её описание.
358 Раздел 4. Решения

Будем считать, что 25—длина комнаты, а 5 —её ширина. Пусть


второй рабочий всегда кладёт свой ящик рядом с ящиком первого,
полностью исключая из игры поперечную полоску 5 x 2 . (Так как
общая ширина полоски — 5 метров, второй рабочий всегда найдёт
2 метра для своего ящика.) Таким образом, первый игрок всё время
должен выбирать для своего ящика новую поперечную полоску, а вто­
рой всегда имеет возможность ответить: поставить свой ящик вто­
рым в эту же полоску. Поэтому второй игрок обязательно выиграет.
Дети часто решают эту задачу неправильно — используют осе­
вую или центральную симметрию или их комбинацию. Для таких
решений легко указать последовательность ходов, после которой
невозможно сделать следующий ход, пользуясь выбранной страте­
гией. Например, для осевой симметрии невозможно сделать ответ­
ный ход, если ящик пересекает ось симметрии.
Занятие 29

Математическая олимпиада III

Решение задачи 1. Эту задачу можно решить уравнением или


с помощью рисунка.
Уравнение. Пусть медведь съел х крекеров. Тогда носорог съел
х + 1, гиппопотам съел х + 2, а слон съел х + 3. Всего животные
съели 4х+ 1 + 2 + 3 = 4*+ 6 крекеров. Получаем 4х + 6 = 2010. Сле­
довательно, 4х = 2010 — 6 = 2004, и х = 2004:4 = 501. Значит, слон
съел 504 крекера.
Эта задача сходна с примером 3 на с. 54, с тем отличием, что
подряд вдут не три, а четыре числа. Поэтому надо сделать такой же
рисунок, добавив ещё одну строку.
Решение задачи 2. Сначала заметим, что обе надписи утвер­
ждают одно и то же: за обеими дверями никого нет. Значит, либо
они обе истинны, либо обе ложны. Может ли эта надпись быть
истинной? Предположим, что за обеими дверями действительно
никого нет. Но король сказал: «Если за второй дверью никого нет,
то надпись на этой двери ложна». Значит, надпись ложна! Мы при­
шли к противоречию. Следовательно, наше предположение невер­
но, и надписи ложны.
Вспомним, что ещё сказал король о второй двери: «Если за ней
тигр, то надпись на ней правдива». Но мы уже знаем, что надпись
ложна: значит, за второй дверью нет тигра! Поэтому принц может
без всякого риска открыть вторую дверь.
Решение задачи 3. Поскольку 12 = 3-4, а числа 3 и 4 взаимно
просты, достаточно проверить делимость на 3 и на 4.
Начнём с делимости на 4. Последние две цифры числа
101112...1920
образуют число 20. Поскольку 20 делится на 4, всё число также де­
лится на 4.
Теперь проверим делимость на 3. Подсчитаем сумму цифр числа:
1 + 0 + 1 + 1 + ... + 1 + 9 + 2 + 0 = 10-1 + 1 + ... + 9 + 2 = 57.
Число 57 делится на 3. Значит, исходное число делится на 12.
360 Раздел 4. Решения

Решение задачи 4. См. рисунок.


Решение задачи 5. Попробуем опреде­
лить истинное положение рыбки в трёх клю­
чевых точках: там, где рыбка начинает дви­
жение, где делает поворот и где останавлива­
ется. Эти важные точки обозначим буквами
А, В, С. Верхний ряд картинок показывает,
как видят эти точки Макс и Белла, когда смотрят на боковую и на
переднюю грани.
Найдём точку А. Когда рыбка находится в этой точке аквариума,
Белле кажется, что она находится в точке Аг на передней грани.
(Второй ряд картинок, рисунок слева.) Значит, реально точка А на­
ходится где-то на ребре, начинающемся в точке Аг и идущем «на­
зад» по границе левой грани аквариума с верхней гранью. Максу
в этот же момент кажется, что рыбка находится в точке А2. Зна­
чит, точка А находится где-то на ребре, начинающемся в точке А2
и идущем по границе задней грани аквариума с верхней гранью.
Единственная точка, принадлежащая обоим этим рёбрам,—левая
верхняя задняя вершина аквариума. (На рисунке эта точка обозна­
чена буквой А в сером кружке.)
Найдём точку В. Когда рыбка находится в этой точке, Белла
видит её в середине правого переднего ребра. Значит, точка В на­
ходится на горизонтальной линии, проходящей посередине правой
грани аквариума. (Это линия, начинающаяся в точке В2 на средней
картинке второго ряда.) А из того, как рыбку в этот же момент ви­
дит Макс, следует, что точка В находится на горизонтальной линии,
проходящей посередине передней грани аквариума. (Это линия,
начинающаяся в точке В1 на средней картинке второго ряда.) Един­
ственная точка, принадлежащая обеим этим линиям, — середина
ребра, идущего по границе передней грани с правой. (На рисунке
эта точка обозначена буквой В в сером кружке.)
Найдём точку С. Из того, как точку С видит Белла, делаем вывод:
эта точка находится на ребре, идущем по границе левой и нижней
грани аквариума (ребро, начинающееся в точке Са). Зная, как точку
С видит Макс, определяем, что она находится на ребре, идущем по
границе задней и верхней граней (ребро, начинающееся в точке
С2). Единственная точка, принадлежащая обоим этим рёбрам, — ле­
вая нижняя задняя вершина аквариума. (На правой картинке в сред­
нем ряду эта точка обозначена буквой С в сером кружке.)
Занятие 29. Математическая олимпиада III 361

Теперь мы знаем эти точки и можем нарисовать траекторию рыб­


ки в трёхмерном представлении — это первая картинка нижнего ряда.

с точки зрения Беллы с точки зрения Макса

траектория рыбки вид сверху

Поэтому с точки зрения папы рыбка движется прямо по диагона­


ли (см. последний рисунок в нижнем ряду), проплывая её дважды —
туда и обратно.
Решение задачи 6. Сначала решим более простую задачу. Пред­
положим, что Винни-Пух дошёл до дома Кролика, посмотрел на ча­
сы и сразу пошёл обратно.
Как Винни-Пуху узнать, сколько времени занимает дорога до до­
ма Кролика? Ему надо посмотреть на свои часы с кукушкой два ра­
за: перед самым выходом из дома и сразу после возвращения. По­
скольку часы заведены, разность между их показаниями будет рав­
на времени, которое Винни-Пуха не было дома. За это время Винни-
Пух прошёл до дома Кролика и обратно; значит, эта разность в два
раза больше времени, которое занимает дорога между двумя дома­
ми в одну сторону.
Итак, Винни-Пух может определить, сколько времени занимает
дорога. Как ему установить правильное время после возвращения до­
мой? Для этого нужно запомнить время на часах Кролика и приба­
вить к нему время, проведённое в пути. Упрощённая задача решена.
Вернёмся к исходной задаче. Предположим, что Винни-Пух про­
вёл некоторое время в доме Кролика. В этом случае он также может
362 Раздел 4. Решения

вычислить время, проведённое в пути. Для этого ему нужно знать


два числа: время, которое его не было дома, и время, проведённое
у Кролика. Сравнив показания часов с кукушкой перед выходом
и после возвращения, Винни-Пух определит, сколько времени его
не было дома. Посмотрев на часы Кролика два раза —при входе
в дом Кролика и перед самым уходом, — Винни-Пух узнаёт, сколько
времени он провёл у Кролика.
Если вычесть время, проведённое у Кролика, из общего времени
отсутствия дома, получится время, проведённое в пути туда и обрат­
но. Значит, половина этой разности равна продолжительности пути
от одного дома к другому. Зная время, проведённое в пути, Винни-
Пух сможет правильно установить свои часы так же, как в упрощён­
ной задаче: он заметит точное время перед уходом из дома Кролика
и прибавит к нему время в пути.
Решение задачи 7. У первого игрока есть выигрышная страте­
гия. Первым ходом он бросает 75 монет в копилку, вмещающую
375 монет. Теперь во второй копилке столько же свободного места,
сколько и в первой. После этого первый игрок отражает ходы второ­
го: бросает столько же монет, сколько и второй игрок, но в другую
копилку. После каждого хода первого игрока в обеих копилках оста­
ётся один и тот же объём свободного места. Поэтому если второй
игрок может сделать ход, то первый игрок всегда сможет сделать
следующий ход.
Решение задачи 8. Так как число, сгенерированное компьюте­
ром, делится на 9, сумма его цифр (число Ильи) тоже делится на 9.
Значит, сумма цифр числа Ильи (число Макса) тоже делится на 9.
Аналогично Беллино число тоже делится на 9. Какое же это число?
Каждая из 100 цифр числа, сгенерированного компьютером, не
больше 9. Значит, число на доске не больше 9 • 100 = 900; поэтому
у него не больше трёх цифр. Тогда число в тетради Макса не может
быть больше чем 9 • 3 = 27. Значит, первая цифра числа Макса не
больше 2, а вторая не больше 9; поэтому сумма этих цифр —число
в тайном дневнике Беллы — не больше 11. Но мы знаем, что это чис­
ло делится на 9 и не может быть равным нулю. Значит, секретным
числом Беллы может быть только 9.
Решение задачи 9. Сначала подсчитаем все возможные спосо­
бы расположить костяшку горизонтально. В каждом таком положе­
нии левая половина костяшки закрывает какое-то поле на одной из
первых семи вертикалей доски (считая слева). Всего таких полей
Занятие 29. Математическая олимпиада III 363

7-8 = 56; значит, существует 56 способов расположить костяшку го­


ризонтально.
Точно так же подсчитаем способы расположить костяшку верти­
кально: их тоже 56. Значит, всего способов расположить костяш­
ку 112.
Решение задачи 10. Давайте назовём драконов А1, А2, ..., А9
и А10. Среди драконов А1, А2, A3, А4 есть два одного цвета. Допу­
стим, что это А1 и А2. (Если это не так, то мы можем их переиме­
новать.) Теперь посмотрим на драконов A3, А4, А5, А6. Среди них
тоже есть два какого-то одного цвета. Допустим, что это A3 и А4.
Аналогично из драконов А5, А6, А7, и А8 два (А5 и А6) будут одного
цвета. Из драконов А7, А8, А9 и А10 два (А7 и А8) тоже будут одного
цвета. Так что мы точно знаем, что есть четыре пары, каждая одного
цвета. Докажем, что у двух из этих пар цвет совпадает. Выберем
по дракону из каждой пары: А1, A3, А5 и А7. Этих драконов всего
четверо; значит, два из них одного цвета. Допустим, например, что
A3 и А7 оба красные. Но тогда напарники каждого из этих драконов
(А4 и А8) тоже красные. Мы нашли четырёх драконов одного цвета.

Д О П О Л Н И Т Е Л Ь Н Ы Е ЗАДАЧИ

Решение задачи 11. В уравнении 17* + 10у = 223 неизвестные


числа х и у целые. Найти их можно разными способами. Во-первых,
подойдёт метод проб и ошибок —проверка всех возможных зна­
чений х от 0 до 13. (Число х не может быть больше 13, так как
17 • 14 > 223.) Однако, немного подумав, можно обойтись без дли­
тельного перебора. Поскольку число 10у всегда оканчивается ну­
лём, число 17* должно оканчиваться цифрой 3. Значит, число 7х
тоже оканчивается цифрой 3. Это возможно только в том случае,
если х оканчивается цифрой 9. Но х — целое число между 0 и 13;
значит, х = 9 и у = (223 — 17 • 9): 10 = 7. Итак, мы нашли решение
и при этом доказали, что оно единственное.
Решение задачи 12. Имеем 120 = 10 • 12 = 2 • 5 • 4 • 3 = 3 • 5 • 8.
Поскольку числа 3, 5 и 8 попарно взаимно просты, достаточно до­
казать делимость на 3, 5 и 8.
Делимость на 3 следует из того, что среди любых трёх идущих
подряд целых чисел одно делится на 3. Делимость на 5 следует из
того, что среди любых пяти идущих целых подряд чисел одно делит­
ся на 5.
364 Раздел 4. Решения
Доказать делимость на 8 чуть сложнее. Заметим, что из любых
пяти идущих подряд целых чисел можно выбрать пару чётных чисел,
между которыми ровно одно число. В любой такой паре одно из
чисел обязательно делится на 4, а другое на 2. Значит, произведение
этих двух чисел делится на 8. Поэтому произведение всех пяти чисел
также делится на 8.
Теперь мы знаем, что произведение делится на 3, на 5 и на 8. Эти
три числа попарно взаимно просты; значит, произведение делится
на 3-5-8 = 120.
Решение задачи 13. Это усложнённый вариант соответствую­
щей задачи из основного списка. Ответ показан на рисунке:

Решение задачи 14. На двух половинках костяшки разное коли­


чество точек. Значит, если положить костяшку на доску, а затем по­
вернуть её на 180°, то эти два положения костяшки будут разными.
Поэтому каждому положению костяшки [6—6] соответствуют два
разных положения костяшки [6—3]. Ответ: 2-112 = 224.
Решение задачи 15. У первого игрока (Тима) есть выигрышная
стратегия. Первым ходом он проводит отрезок, соединяющий две
противоположные точки. Каждым последующим ходом он зеркаль­
но отражает ходы второго игрока относительно этого первого от­
резка. (Каждый проведённый им отрезок должен быть симметри­
чен отрезку, проведённому вторым игроком, относительно первого
отрезка.)
Решение задачи 16. Пусть АВ — число, записанное Антоном (А —
первая цифра, В —вторая). Когда Саша приписал цифру 6, полу­
чилось число 6АВ, которое можно выразить как 600 + АВ. Зная,
что это число в 9 раз больше исходного, составляем уравнение:
600 + АВ = 9 • АВ. Решая это уравнение, получаем АВ = 600:8 = 75.
Решение задачи 17. Сначала рассмотрим столбцы таблицы. По­
скольку в каждом столбце пять клеток, в нём либо больше тара­
канов (не меньше трёх), либо больше божьих коровок (не меньше
трёх). Всего в таблице пять столбцов: значит, хотя бы в трёх из них
преобладают насекомые какого-то определённого вида. (Предполо-
Занятие 29. Математическая олимпиада III 365

жим, что это тараканы.) Рассмотрим эти столбцы. Докажем, что из


них можно выбрать два столбца так, что на пересечениях этих двух
столбцов с какими-то двумя строками сидят только тараканы.
Возьмём первые два столбца из тех, где больше тараканов. В этих
двух столбцах сидят 10 насекомых. Разобьём их на пять пар: каждая
пара состоит из двух насекомых, сидящих в одной строке. Если хотя
бы две такие пары состоят только из тараканов, то задача решена.
Предположим, что это не так. Тогда в каждой паре есть хотя бы
один таракан. Почему? В этих пяти парах не меньше шести тарака­
нов (так как их не меньше трёх в каждом столбце). Если бы среди
этих пяти пар была пара божьих коровок, то в остальных четырёх
парах было бы не меньше шести тараканов. Тогда хотя бы в двух из
этих четырёх пар были бы только тараканы. Но мы рассматриваем
случай, когда таких двух пар нет. Значит, в этих двух столбцах есть
хотя бы по одному таракану в каждой строке.
Теперь рассмотрим третий столбец с преобладанием тараканов.
Выберем в этом столбце трёх тараканов. Согласно доказанному в пре­
дыдущем параграфе, каждый из них находится в одной строке с та­
раканом из первого или второго столбца. Другими словами, для
каждого из этих трёх тараканов есть две возможности: либо он
в одной строке с тараканом из первого столбца, либо в одной строке
с тараканом из второго. Одна из этих возможностей реализована
хотя бы для двух тараканов из трёх: каждый из этих двух тарака­
нов сидит в одной строке с тараканом из одного и того же столбца
(первого или второго). Итак, мы нашли два столбца и две строки,
на пересечениях которых сидят только тараканы. Задача решена.
Решение задачи 18. Предположим, что Света заполнила табли­
цу с соблюдением этого условия, а затем стёрла все числа в чет­
вёртом столбце (крайнем справа) и в четвёртой строке (нижней).
Остались числа в таблице 3 x 3 , занимающей левый верхний угол
исходной таблицы. Докажем, что все стёртые числа можно восста­
новить.
Сначала рассмотрим первую строку таблицы 4 x 4 . Первые три
клетки этой строки уже заполнены. Поскольку произведение чисел
в этой строке должно быть равно 1, четвёртое число нетрудно опре­
делить: это 1, если произведение первых трёх чисел равно 1, и —1,
если это произведение равно —1. Точно так же можно единствен­
ным образом восстановить последние числа во второй и в третьей
строке и в первых трёх столбцах.
366 Раздел 4. Решения

Осталось неизвестным число, стоящее на пересечении четвёр­


той строки и четвёртого столбца. Но мы знаем, что произведение
этого числа и первых трёх чисел в четвёртой строке равно 1. Вспом­
нив, как мы восстанавливали эти три числа, определим их произве­
дение: оно равно произведению всех девяти чисел в таблице 3 x 3 .
Поэтому по числам в таблице 3 x 3 можно определить число на пе­
ресечении четвёртой строки и четвёртого столбца: это 1, если про­
изведение чисел в таблице 3 x 3 равно 1, и —1, если это произведе­
ние равно —1.
Сделаем вывод: как бы ни была заполнена таблица 3 x 3 , осталь­
ные клетки таблицы 4 x 4 можно заполнить с соблюдением нашего
условия, причём единственным способом. Если таблицы 3 х 3 за­
полнены по-разному, то и соответствующие таблицы 4 x 4 будут за­
полнены по-разному. Значит, если подсчитать все способы заполне­
ния таблицы 3 x 3 числами +1 и —1, то получится ответ к задаче.
Это 2 9 .
Библиография
1. Генкин С. А., Итенберг КВ., Фомин Д. В. Ленинградские математиче­
ские кружки. Киров: АСА, 1994.
2. СпивакА. В. Математический кружок. 6—7 классы. М.: МЦНМО, 2010.
3. СпивакА.В. Тысяча и одна задача по математике. М.: Просвещение, 2010.
4. СпивакА. В. Математический праздник. М.: Бюро Квантум, 2004.
5. Шарыгин И. Ф., Ерганжиева Л. Н. Наглядная геометрия, 5—6 классы. М.:
Дрофа, 2009.
6. Козлова Е. Г. Сказки и подсказки. М.: МЦНМО, 2006.
7. Ященко И. В. Приглашение на математический праздник. М.: МЦНМО,
2009.
8. Шарыгин И. Ф. Уроки дедушки Гаврилы, или Развивающие каникулы.
М.: Дрофа, 2003.
9. Шарыгин И. Ф. Задачи на смекалку, 5—6 классы. М.: Просвещение, 2010.
10. Медников Л. Э. Чётность. М.: МЦНМО, 2009.
11. Чулков П. В. Арифметические задачи. М.: МЦНМО, 2009.
12. Кноп К. А. Взвешивания и алгоритмы. М.: МЦНМО, 2010.
13. Сбигнев К. А. Делимость и простые числа. М.: МЦНМО, 2012.
14. Раскина И. В., Шнолъ Д. Э. Логические задачи. М.: МЦНМО, 2014.
15. Савин А. П. Занимательные математические задачи. М.: ACT, 1995.
16. Бугаенко В. О. Турниры им. Ломоносова. М.: МЦНМО, 1998.
17. Смаллиан Р. Принцесса или тигр. М.: Мир, 1985.
18. Смаллиан Р. Загадка Шахразады и другие удивительные древние и со­
временные головоломки. М.: ЛОРИ, 2014.
19. Dorichenko S. A Moscow Math Circle: Week-by-Week Problem Sets (MSRI
Mathematical Circles Library) American Mathematical Society, 2012.
20. Gardner M. Mental Magic. Dover Publications, 1986.
21. Gardner M. Entertaining Mathematical Puzzles.
22. Patrick D. Introduction to Counting & Probability. The Art of Problem
Solving 2007.
23. Vandervelde S. Circle in a Box. American Mathematical Society, 2012.
24. Dudeney H. E. The Strand Magazine. 1924. Vol. 68.
368 Литература

25. Московский центр непрерывного математического образования. Ин­


тернет-проект «Задачи», http: //problems. ru/.
26. Математический Гуру. Материалы кружка А.Н.Савина, http://www.
mathguru.ru/circle/.
27. Центр дополнительного образования одаренных школьников. Киров,
http: //www. cdoosh. ru. Математическое домино.
28. Davis Т. Mathematical Circles Topics,
http://www.geometer.org/mathcircles/.
29. Garlikov R. The Socratic Method: Teaching by Asking, www.garlikov.com/
Soc/Meth.html.

Вам также может понравиться